Sorry

This feed does not validate.

In addition, interoperability with the widest range of feed readers could be improved by implementing the following recommendations.

Source: https://medical-phd.blogspot.com/feeds/posts/default

  1. <?xml version='1.0' encoding='UTF-8'?><?xml-stylesheet href="http://www.blogger.com/styles/atom.css" type="text/css"?><feed xmlns='http://www.w3.org/2005/Atom' xmlns:openSearch='http://a9.com/-/spec/opensearchrss/1.0/' xmlns:blogger='http://schemas.google.com/blogger/2008' xmlns:georss='http://www.georss.org/georss' xmlns:gd="http://schemas.google.com/g/2005" xmlns:thr='http://purl.org/syndication/thread/1.0'><id>tag:blogger.com,1999:blog-3302710210518459416</id><updated>2024-03-12T17:53:35.021-07:00</updated><category term="Case Files"/><category term="Family Medicine Case File"/><category term="Internal Medicine Case File"/><category term="Emergency Medicine Case File"/><category term="Anatomy"/><category term="Obstetrics and Gynecology Case File"/><category term="Anesthesiology"/><category term="Biochemistry"/><category term="Neurology Case File"/><category term="Microbiology Case File"/><category term="Neuroscience Case File"/><category term="High-Risk Obstetrics Case File"/><category term="Critical Care Case File"/><category term="Gynecologic Surgery Case File"/><category term="Orthopaedic Surgery Case File"/><category term="Cardiology"/><category term="Anatomy Case Files"/><category term="Anesthesiology Case Files"/><category term="Neurology"/><title type='text'>Medical phd</title><subtitle type='html'>Highly skilled and experienced medical staff, advance, medical education, health information and research. Comprehensive information on the study of medicine.</subtitle><link rel='http://schemas.google.com/g/2005#feed' type='application/atom+xml' href='https://medical-phd.blogspot.com/feeds/posts/default'/><link rel='self' type='application/atom+xml' href='https://www.blogger.com/feeds/3302710210518459416/posts/default?redirect=false'/><link rel='alternate' type='text/html' href='https://medical-phd.blogspot.com/'/><link rel='hub' href='http://pubsubhubbub.appspot.com/'/><link rel='next' type='application/atom+xml' href='https://www.blogger.com/feeds/3302710210518459416/posts/default?start-index=26&amp;max-results=25&amp;redirect=false'/><author><name>Medical Group</name><uri>http://www.blogger.com/profile/01780656800578397393</uri><email>noreply@blogger.com</email><gd:image rel='http://schemas.google.com/g/2005#thumbnail' width='32' height='32' src='//blogger.googleusercontent.com/img/b/R29vZ2xl/AVvXsEiMuROkBDv6yw4SmxGqRA5BPQhOXSwFld-Gufmy4kUqfpoAGQRl7cv-3naew6wjAnCjRFSe_FKX8z7CdE6cFKxxBpnqBuwUTSr-15xd3mFGcr_wJOX2y73G_zJCGND98Q/s220/Final-logo.jpg'/></author><generator version='7.00' uri='http://www.blogger.com'>Blogger</generator><openSearch:totalResults>748</openSearch:totalResults><openSearch:startIndex>1</openSearch:startIndex><openSearch:itemsPerPage>25</openSearch:itemsPerPage><entry><id>tag:blogger.com,1999:blog-3302710210518459416.post-3004634123698531224</id><published>2022-03-31T23:42:00.003-07:00</published><updated>2022-03-31T23:42:26.533-07:00</updated><category scheme="http://www.blogger.com/atom/ns#" term="Case Files"/><category scheme="http://www.blogger.com/atom/ns#" term="Orthopaedic Surgery Case File"/><title type='text'>Hip Osteoarthritis Case File</title><content type='html'>&lt;div class=&quot;scrollbox&quot;&gt;
  2.  
  3. &lt;div&gt;&lt;b&gt;&lt;span style=&quot;font-size: large;&quot;&gt;Hip Osteoarthritis Case File&lt;/span&gt;&lt;/b&gt;&lt;/div&gt;&lt;div&gt;Eugene C. Toy, MD, Andrew J. Rosenbaum, MD, Timothy T. Roberts, MD, Joshua S. Dines, MD&lt;/div&gt;&lt;div&gt;&lt;br /&gt;&lt;/div&gt;&lt;div&gt;&lt;b&gt;&lt;span style=&quot;color: #017087; font-size: medium;&quot;&gt;CASE 40&lt;/span&gt;&lt;/b&gt;&lt;/div&gt;&lt;div&gt;&lt;div&gt;A 65-year-old woman presents to her primary care physician with complaints of right “groin” pain that radiates to the buttock. She has noticed some mild pain in her right hip for the past 2 years, but has had severe pain over the last 4 months. She describes the pain as being “4/10” but at times can be an “8/10.” The quality of the pain is a dull ache, and she denies any weakness, numbness, or tingling in her leg below the knee; however, the pain recently has kept her from getting sleep at night. She is currently using a cane for ambulation around her community and has to rest every 4 to 5 blocks because of pain. She also notes difficulty putting on her shoes and socks and getting in and out of the car. Her symptoms are refractory to ibuprofen and acetaminophen. She denies any recent trauma, infections, or dental work. Vital signs are within normal limits. When she ambulates, she leans her body to the right side during stance phase while maintaining a level pelvis. When she is lying flat on her back, she is able to flex her right hip to 90 degrees but has severe pain past this point. She has limited abduction of her right hip compared with the left. She has 0 degrees of right hip internal rotation and 20 degrees of external rotation. When holding her left knee to her chest and extending the right hip, the right posterior thigh does not fully lie on the exam table. She has a negative straight leg raise bilaterally and is neurovascularly intact in both lower extremities.&lt;/div&gt;&lt;div&gt;&lt;br /&gt;&lt;/div&gt;&lt;div&gt;&lt;span style=&quot;color: #017087;&quot;&gt;►&lt;/span&gt;&amp;nbsp;What is the most likely diagnosis?&lt;/div&gt;&lt;div&gt;&lt;span style=&quot;color: #017087;&quot;&gt;►&lt;/span&gt;&amp;nbsp;What is your next diagnostic step?&lt;/div&gt;&lt;div&gt;&lt;span style=&quot;color: #017087;&quot;&gt;►&lt;/span&gt;&amp;nbsp;What are the nonoperative treatment options?&lt;/div&gt;&lt;div&gt;&lt;span style=&quot;color: #017087;&quot;&gt;►&lt;/span&gt;&amp;nbsp;What are the surgical treatment options?&lt;/div&gt;&lt;/div&gt;&lt;div&gt;&lt;br /&gt;&lt;/div&gt;&lt;div&gt;&lt;br /&gt;&lt;/div&gt;&lt;div&gt;&lt;div&gt;&lt;b&gt;&lt;span style=&quot;font-size: medium;&quot;&gt;ANSWER TO CASE 40:&lt;/span&gt;&lt;/b&gt;&lt;/div&gt;&lt;div&gt;&lt;span style=&quot;background-color: #daebf0; font-size: medium;&quot;&gt;&lt;span&gt;&lt;span style=&quot;background-color: transparent;&quot;&gt;&lt;span style=&quot;color: #017087;&quot;&gt;&lt;b&gt;Hip Osteoarthritis&lt;/b&gt;&lt;/span&gt;&lt;/span&gt;&lt;b&gt;&amp;nbsp;&amp;nbsp;&lt;/b&gt;&lt;/span&gt;&lt;span style=&quot;font-weight: bold;&quot;&gt;&amp;nbsp;&amp;nbsp; &amp;nbsp;&lt;/span&gt;&lt;span style=&quot;font-weight: bold;&quot;&gt;&amp;nbsp;&amp;nbsp; &amp;nbsp;&lt;/span&gt;&lt;span style=&quot;font-weight: bold;&quot;&gt;&amp;nbsp;&amp;nbsp; &amp;nbsp;&lt;/span&gt;&lt;span style=&quot;font-weight: bold;&quot;&gt;&amp;nbsp;&amp;nbsp; &amp;nbsp;&lt;/span&gt;&lt;span style=&quot;font-weight: bold;&quot;&gt;&amp;nbsp; &amp;nbsp;&lt;/span&gt;&lt;/span&gt;&lt;/div&gt;&lt;/div&gt;&lt;div&gt;&lt;br /&gt;&lt;/div&gt;&lt;div&gt;&lt;div&gt;&lt;i&gt;Summary&lt;/i&gt;: A 65-year-old woman presents with a chief complaint of right “groin” pain that radiates to the buttock. She is having difficulty performing activities of daily living and sleeping, and her pain is not relieved by anti-inflammatory medications. She has not had a traumatic event or recent infections. On physical exam, she has painful and limited range of motion of her hip, with a flexion contracture.&lt;/div&gt;&lt;div&gt;&lt;ul style=&quot;text-align: left;&quot;&gt;&lt;li&gt;&lt;b&gt;Most likely diagnosis:&lt;/b&gt; Right hip osteoarthritis.&lt;/li&gt;&lt;li&gt;&lt;b&gt;Next diagnostic step:&lt;/b&gt; Anteroposterior (AP) pelvis radiograph and AP and lateral radiographs of the right hip ( Figures 40–1 and 40–2 ).&lt;/li&gt;&lt;li&gt;&lt;b&gt;Nonoperative treatment options:&lt;/b&gt; Nonsteroidal anti-inflammatory drugs (NSAIDs), physical therapy (eg, stretching and range-of-motion exercises, modalities, pool therapy), and corticosteroid injections.&lt;/li&gt;&lt;li&gt;&lt;b&gt;Surgical treatment option(s):&lt;/b&gt; Total hip replacement.&lt;/li&gt;&lt;/ul&gt;&lt;/div&gt;&lt;/div&gt;&lt;div&gt;&lt;br /&gt;&lt;/div&gt;&lt;div&gt;&lt;br /&gt;&lt;/div&gt;&lt;div&gt;&lt;div&gt;&lt;b&gt;&lt;span style=&quot;color: #017087; font-size: medium;&quot;&gt;ANALYSIS&lt;/span&gt;&lt;/b&gt;&lt;/div&gt;&lt;div&gt;&lt;b&gt;&lt;span style=&quot;font-size: medium;&quot;&gt;&lt;i&gt;Objectives&lt;/i&gt;&lt;/span&gt;&lt;/b&gt;&lt;/div&gt;&lt;/div&gt;&lt;div&gt;&lt;div&gt;&lt;ol style=&quot;text-align: left;&quot;&gt;&lt;li&gt;Understand the etiologies of arthritis.&lt;/li&gt;&lt;li&gt;Be familiar with the treatment options for hip arthritis.&lt;/li&gt;&lt;li&gt;Understand the indications for total hip arthroplasty.&lt;/li&gt;&lt;/ol&gt;&lt;/div&gt;&lt;/div&gt;&lt;div class=&quot;separator&quot; style=&quot;clear: both; text-align: center;&quot;&gt;&lt;br /&gt;&lt;/div&gt;&lt;div class=&quot;separator&quot; style=&quot;clear: both; text-align: center;&quot;&gt;&lt;a href=&quot;https://blogger.googleusercontent.com/img/b/R29vZ2xl/AVvXsEiIKcCAYy8fEiprG9Q9Cj8bA98NVe_nPkvaQIRFHfGEfN7OGjQbcF7Il2vacT1PrAeIQNP4M8ljF7MSBuN60JwzW3s0j3jzotH8mhUTaG3DsSrSJ6MI_8bjUtSsrlMEsT3w5Mc4MpwREwoYm77rDV3vCRb1U2L7ZiOU1wr-D36mPG3FCw9R_A8n6tLu1g/s500/AP-pelvis-radiograph.jpg&quot; style=&quot;margin-left: 1em; margin-right: 1em;&quot;&gt;&lt;img alt=&quot;AP pelvis radiograph&quot; border=&quot;0&quot; data-original-height=&quot;360&quot; data-original-width=&quot;500&quot; height=&quot;460&quot; src=&quot;https://blogger.googleusercontent.com/img/b/R29vZ2xl/AVvXsEiIKcCAYy8fEiprG9Q9Cj8bA98NVe_nPkvaQIRFHfGEfN7OGjQbcF7Il2vacT1PrAeIQNP4M8ljF7MSBuN60JwzW3s0j3jzotH8mhUTaG3DsSrSJ6MI_8bjUtSsrlMEsT3w5Mc4MpwREwoYm77rDV3vCRb1U2L7ZiOU1wr-D36mPG3FCw9R_A8n6tLu1g/w640-h460/AP-pelvis-radiograph.jpg&quot; title=&quot;AP pelvis radiograph&quot; width=&quot;640&quot; /&gt;&lt;/a&gt;&lt;/div&gt;&lt;div&gt;&lt;b&gt;&lt;span style=&quot;color: #017087;&quot;&gt;Figure 40–1.&lt;/span&gt;&lt;/b&gt; AP pelvis radiograph demonstrating osteoarthritis in the right hip.&lt;/div&gt;&lt;div&gt;&lt;br /&gt;&lt;/div&gt;&lt;div class=&quot;separator&quot; style=&quot;clear: both; text-align: center;&quot;&gt;&lt;a href=&quot;https://blogger.googleusercontent.com/img/b/R29vZ2xl/AVvXsEh4FMes8_CQ3OJrnyaaTl8jXxB2U1-zzkLKujncUdUo-3_y-3sGEMbOM3B2_kN1Ng-X1DWrJpfRP3KAqjXNEyCX8miYSOVFLuWPPRRSOm8JoQjSH6eHy4JA15y3GbwQPBX5p_cEhbqkv5SLWpBIP4Fj111OK2EIiuiz7r5KwjmIxwaollobeOEUI-xjOQ/s500/Frog-leg-lateral-radiograph.jpg&quot; style=&quot;margin-left: 1em; margin-right: 1em;&quot;&gt;&lt;img alt=&quot;Frog-leg lateral radiograph&quot; border=&quot;0&quot; data-original-height=&quot;428&quot; data-original-width=&quot;500&quot; height=&quot;548&quot; src=&quot;https://blogger.googleusercontent.com/img/b/R29vZ2xl/AVvXsEh4FMes8_CQ3OJrnyaaTl8jXxB2U1-zzkLKujncUdUo-3_y-3sGEMbOM3B2_kN1Ng-X1DWrJpfRP3KAqjXNEyCX8miYSOVFLuWPPRRSOm8JoQjSH6eHy4JA15y3GbwQPBX5p_cEhbqkv5SLWpBIP4Fj111OK2EIiuiz7r5KwjmIxwaollobeOEUI-xjOQ/w640-h548/Frog-leg-lateral-radiograph.jpg&quot; title=&quot;Frog-leg lateral radiograph&quot; width=&quot;640&quot; /&gt;&lt;/a&gt;&lt;/div&gt;&lt;div&gt;&lt;b&gt;&lt;span style=&quot;color: #017087;&quot;&gt;Figure 40–2.&lt;/span&gt;&lt;/b&gt; Frog-leg lateral radiograph of the right hip demonstrating osteoarthritic changes.&lt;/div&gt;&lt;div&gt;&lt;br /&gt;&lt;/div&gt;&lt;div&gt;&lt;br /&gt;&lt;/div&gt;&lt;div&gt;&lt;div&gt;&lt;b&gt;&lt;i&gt;&lt;span style=&quot;font-size: medium;&quot;&gt;Considerations&lt;/span&gt;&lt;/i&gt;&lt;/b&gt;&lt;/div&gt;&lt;div&gt;&lt;br /&gt;&lt;/div&gt;&lt;div&gt;This 65-year-old patient presents with &lt;b&gt;“groin” pain,&lt;/b&gt; which is the typical location of pain that is coming from the hip joint. Patients often complain additionally of&amp;nbsp;&lt;b&gt;radiating pain to the buttock, lateral thigh, or knee.&lt;/b&gt; Sometimes patients who have hip arthritis will actually present with “knee pain.” This is called referred pain and is thought to be caused by irritation of the obturator nerve or, less commonly, the femoral nerve. “Hip pain” can also be due to problems in the spine and pelvis. On physical exam, &lt;b&gt;pain with resisted straight leg raise and resisted hip flexion and groin&amp;nbsp;&lt;/b&gt;&lt;b&gt;pain with hip flexion and internal rotation&lt;/b&gt; are more commonly seen in hip disease and less commonly with low back disease. This patient also had &lt;b&gt;limitations in hip&amp;nbsp;&lt;/b&gt;&lt;b&gt;range of motion,&lt;/b&gt; which is common with end-stage osteoarthritis. Although pain in the &lt;b&gt;hip can radiate&lt;/b&gt; &lt;i&gt;down&lt;/i&gt; &lt;b&gt;to the knee, lumbar spine disease (eg, sciatica) will often&amp;nbsp;&lt;/b&gt;&lt;b&gt;radiate &lt;/b&gt;&lt;i&gt;below&lt;/i&gt;&lt;b&gt; the knee and into the foot. &lt;/b&gt;If the diagnosis is unclear or if the patient has both hip and lumbar spine pathology, a &lt;b&gt;diagnostic injection of anesthetic and&amp;nbsp;&lt;/b&gt;&lt;b&gt;corticosteroids&lt;/b&gt; into the hip joint can be performed to determine whether the pain is coming primarily from the hip. If the pain is alleviated with the injection, the pain is likely coming from the hip. It is important to obtain a quality &lt;b&gt;AP pelvis radiograph&amp;nbsp;&lt;/b&gt;&lt;b&gt;as well as AP and lateral radiographs&lt;/b&gt; of the affected hip when there is concern for arthritis ( Figures 40–1 and 40–2 ). If the patient complains of radiating pain, AP and lateral radiographs of the lumbar spine and/or weightbearing AP/lateral/Merchant x-rays of the knee may be required.&lt;/div&gt;&lt;/div&gt;&lt;div&gt;&lt;br /&gt;&lt;/div&gt;&lt;div&gt;&lt;br /&gt;&lt;/div&gt;&lt;div&gt;&lt;div&gt;&lt;div&gt;&lt;b&gt;&lt;span style=&quot;font-size: medium;&quot;&gt;APPROACH TO:&lt;/span&gt;&lt;/b&gt;&lt;/div&gt;&lt;div&gt;&lt;span style=&quot;background-color: #daebf0; color: #017087; font-size: medium;&quot;&gt;&lt;span&gt;&lt;span style=&quot;background-color: transparent;&quot;&gt;&lt;b&gt;Hip Osteoarthritis&lt;/b&gt;&lt;/span&gt;&lt;b&gt;&amp;nbsp; &amp;nbsp;&amp;nbsp;&lt;/b&gt;&lt;/span&gt;&lt;span style=&quot;font-weight: bold;&quot;&gt;&amp;nbsp;&amp;nbsp; &amp;nbsp;&lt;/span&gt;&lt;span style=&quot;font-weight: bold;&quot;&gt;&amp;nbsp;&amp;nbsp; &amp;nbsp;&lt;/span&gt;&lt;span style=&quot;font-weight: bold;&quot;&gt;&amp;nbsp;&amp;nbsp; &amp;nbsp;&lt;/span&gt;&lt;span style=&quot;font-weight: bold;&quot;&gt;&amp;nbsp;&amp;nbsp; &amp;nbsp;&lt;/span&gt;&lt;span style=&quot;font-weight: bold;&quot;&gt;&amp;nbsp;&amp;nbsp; &amp;nbsp;&lt;/span&gt;&lt;span style=&quot;font-weight: bold;&quot;&gt;&amp;nbsp;&amp;nbsp; &amp;nbsp;&lt;/span&gt;&lt;span style=&quot;font-weight: bold;&quot;&gt;&amp;nbsp;&amp;nbsp;&amp;nbsp;&lt;/span&gt;&lt;span style=&quot;font-weight: bold;&quot;&gt;&amp;nbsp;&lt;/span&gt;&lt;/span&gt;&lt;/div&gt;&lt;/div&gt;&lt;div&gt;&lt;br /&gt;&lt;/div&gt;&lt;div&gt;&lt;b&gt;&lt;span style=&quot;color: #017087; font-size: medium;&quot;&gt;DEFINITIONS&lt;/span&gt;&lt;/b&gt;&lt;/div&gt;&lt;/div&gt;&lt;div&gt;&lt;br /&gt;&lt;/div&gt;&lt;div&gt;&lt;div&gt;&lt;b&gt;OSTEOARTHRITIS:&lt;/b&gt; Progressive, degenerative disorder of the joints caused by a gradual loss of cartilage and bone resulting in pain, stiffness, and bony overgrowth&lt;/div&gt;&lt;div&gt;&lt;b&gt;&lt;br /&gt;&lt;/b&gt;&lt;/div&gt;&lt;div&gt;&lt;b&gt;REFERRED PAIN:&lt;/b&gt; Pain perceived in a body location other than the location of pathology&lt;/div&gt;&lt;/div&gt;&lt;div&gt;&lt;br /&gt;&lt;/div&gt;&lt;div&gt;&lt;br /&gt;&lt;/div&gt;&lt;div&gt;&lt;b&gt;&lt;span style=&quot;color: #017087; font-size: medium;&quot;&gt;CLINICAL APPROACH&lt;/span&gt;&lt;/b&gt;&lt;/div&gt;&lt;div&gt;&lt;br /&gt;&lt;/div&gt;&lt;div&gt;&lt;div&gt;&lt;i&gt;&lt;span style=&quot;font-size: medium;&quot;&gt;&lt;b&gt;Etiologies&lt;/b&gt;&lt;/span&gt;&lt;/i&gt;&lt;/div&gt;&lt;div&gt;&lt;br /&gt;&lt;/div&gt;&lt;div&gt;&lt;b&gt;Osteoarthritis is the most common&lt;/b&gt; form of end-stage arthritis in the community; other types of arthritis include &lt;b&gt;juvenile rheumatoid arthritis, rheumatoid arthritis,&amp;nbsp;&lt;/b&gt;&lt;b&gt;posttraumatic arthritis, inflammatory arthritis (eg, systemic lupus erythematosus),&amp;nbsp;&lt;/b&gt;&lt;b&gt;secondary arthritis from avascular necrosis, and septic arthritis.&lt;/b&gt; It is important to determine the etiology of the underlying arthritis, as this may affect surgical decision making and postoperative management. Radiographs are often helpful in determining the etiology of arthritis. &lt;b&gt;Osteoarthritis&lt;/b&gt; will show evidence of &lt;b&gt;asymmetric&amp;nbsp;&lt;/b&gt;&lt;b&gt;joint space narrowing, osteophyte formation, subchondral sclerosis, and&amp;nbsp;&lt;/b&gt;&lt;b&gt;subchondral cysts, &lt;/b&gt;whereas&lt;b&gt; rheumatoid arthritis &lt;/b&gt;will demonstrate&lt;b&gt; symmetric joint&amp;nbsp;&lt;/b&gt;&lt;b&gt;space narrowing, bony erosions, and osteopenia.&lt;/b&gt; Osteoarthritis of the hip may also occur secondary to previous traumatic injuries, developmental dysplasia of the hip, osteonecrosis of the hip, and femoroacetabular impingement.&lt;/div&gt;&lt;/div&gt;&lt;div&gt;&lt;br /&gt;&lt;/div&gt;&lt;div&gt;&lt;br /&gt;&lt;/div&gt;&lt;div&gt;&lt;div&gt;&lt;b&gt;&lt;i&gt;&lt;span style=&quot;font-size: medium;&quot;&gt;Clinical Presentation&lt;/span&gt;&lt;/i&gt;&lt;/b&gt;&lt;/div&gt;&lt;div&gt;&lt;br /&gt;&lt;/div&gt;&lt;div&gt;The majority of patients who present with hip pain will not require surgery. As with any painful condition, when taking a patient’s history, it is important to ask (1) location of the pain, (2) whether or not the pain radiates to another location, (3) alleviating or aggravating factors, (4) what the patient has tried to alleviate the pain, (5) quality of the pain, (6) quantity of the pain, (7) duration of the pain, (8) when the pain occurs, and (9) associated symptoms, including numbness, weakness, fevers, weight loss, involvement of other joints, and so forth.&lt;/div&gt;&lt;/div&gt;&lt;div&gt;&lt;br /&gt;&lt;/div&gt;&lt;div&gt;&lt;div&gt;&lt;span&gt;&amp;nbsp;&amp;nbsp; &amp;nbsp;&lt;/span&gt;As discussed, patients with arthritis (particularly osteoarthritis) present with groin pain that can radiate to the buttock, lateral proximal femur, thigh, or knee. Patients often describe the pain as a &lt;b&gt;constant, dull ache&lt;/b&gt; that &lt;b&gt;worsens with physical&amp;nbsp;&lt;/b&gt;&lt;b&gt;activity&lt;/b&gt; and is relieved, at least partially, with rest. Patients often tolerate the pain, taking anti-inflammatories or acetaminophen for many years. Often, what compels patients to finally seek treatment is restricted motion that &lt;b&gt;prevents them from&amp;nbsp;&lt;/b&gt;&lt;b&gt;performing activities of daily living,&lt;/b&gt; including dressing and driving, or &lt;b&gt;pain that&amp;nbsp;&lt;/b&gt;&lt;b&gt;interferes with their ability to sleep.&lt;/b&gt;&lt;/div&gt;&lt;div&gt;&lt;br /&gt;&lt;/div&gt;&lt;div&gt;&lt;span&gt;&amp;nbsp;&amp;nbsp; &amp;nbsp;&lt;/span&gt;On physical exam, they often have relatively normal vital signs (unless they have an underlying medical condition) and ambulate with a &lt;b&gt;coxalgic gait. &lt;/b&gt;A coxalgic gait is when the patient&lt;b&gt; lurches his or her torso to the affected side but keeps&amp;nbsp;&lt;/b&gt;&lt;b&gt;the pelvis level.&lt;/b&gt; This is different from a Trendelenburg gait (a sign of hip abductor weakness), in which the patient leans his or her torso to the affected side but the pelvis is not level and is different from the antalgic gait seen with knee pathology, in which the patient limits the amount of time he or she weight bears on the affected knee during stance phase. As a result of loss of cartilage or bone, patients may have a &lt;b&gt;limb length discrepancy,&lt;/b&gt; which may also cause an abnormal gait. With the patient supine, patients often have &lt;b&gt;painful, restricted range of motion. &lt;/b&gt;A&lt;b&gt; Thomas test,&amp;nbsp;&lt;/b&gt;in which the patient holds his or her unaffected knee to the chest and attempts to extend the affected hip, can help the clinician to determine whether the patient also has a &lt;b&gt;hip flexion contracture.&lt;/b&gt; Patients with hip pathology also have &lt;b&gt;pain with&amp;nbsp;&lt;/b&gt;&lt;b&gt;resisted hip flexion and resisted straight leg raise.&lt;/b&gt; Radiographic evaluation of a painful hip includes a quality &lt;b&gt;AP pelvis&lt;/b&gt; radiograph ( Figure 40–1 ) and an &lt;b&gt;AP and&amp;nbsp;&lt;/b&gt;&lt;b&gt;lateral radiograph&lt;/b&gt; ( Figure 40–2 ) of the affected hip. Magnetic resonance imaging (MRI) is rarely required to make the diagnosis of arthritis, but may be beneficial in eliminating other causes of hip pain or to diagnose early cartilage loss.&lt;/div&gt;&lt;/div&gt;&lt;div&gt;&lt;br /&gt;&lt;/div&gt;&lt;div&gt;&lt;br /&gt;&lt;/div&gt;&lt;div&gt;&lt;b style=&quot;color: #017087;&quot;&gt;&lt;span style=&quot;font-size: medium;&quot;&gt;TREATMENT&lt;/span&gt;&lt;/b&gt;&lt;/div&gt;&lt;div&gt;&lt;br /&gt;&lt;/div&gt;&lt;div&gt;&lt;div&gt;Total hip arthroplasty is one of the most successful procedures in medicine. Indications for total hip replacement in general terms are &lt;b&gt;debilitating, end-stage arthritis&amp;nbsp;&lt;/b&gt;&lt;b&gt;of the hip that limits activities of daily living.&lt;/b&gt; Orthopaedic surgeons generally evaluate pain, success (or lack of success) of nonoperative interventions, baseline function, mobility, and radiographic evidence of joint space damage in their decision to recommend surgery. In addition, patients must have &lt;b&gt;failed nonoperative treatment&amp;nbsp;&lt;/b&gt;&lt;b&gt;interventions,&lt;/b&gt; including physical therapy (eg, stretching and range-of-motion exercises, modalities, pool therapy), nonsteroidal anti-inflammatories, and intraarticular corticosteroid injections.&lt;/div&gt;&lt;/div&gt;&lt;div&gt;&lt;br /&gt;&lt;/div&gt;&lt;div&gt;&lt;div&gt;&lt;span&gt;&amp;nbsp;&amp;nbsp; &amp;nbsp;&lt;/span&gt;A variety of implants and bearing surfaces are currently available. The most common current bearing options include metal on metal, metal on polyethylene, ceramic on ceramic, ceramic on-polyethylene, and Oxinium on polyethylene. With each of the previously mentioned types of bearings, the femoral head is former and the acetabular liner is latter. There are advantages and disadvantages to each type of bearing surface. In older adults, the most commonly used bearing surface is metal on polyethylene. In addition, both the acetabular and femoral components can be cementless or cemented, depending on the patient’s age and bone quality and surgeon preference.&lt;/div&gt;&lt;/div&gt;&lt;div&gt;&lt;br /&gt;&lt;/div&gt;&lt;div&gt;&lt;br /&gt;&lt;/div&gt;&lt;div&gt;&lt;b&gt;&lt;span style=&quot;color: #017087; font-size: medium;&quot;&gt;&lt;u&gt;COMPREHENSION QUESTIONS&lt;/u&gt;&lt;/span&gt;&lt;/b&gt;&lt;/div&gt;&lt;div&gt;&lt;br /&gt;&lt;/div&gt;&lt;div&gt;&lt;div&gt;40.1 A 58-year-old golfer presents to your office with complaints of progressive right groin pain, 2 years in duration. Lately he has been having difficulty walking between holes on the golf course. On examination, he is overweight, exhibits an antalgic gait, and has markedly limited range of hip motion. Which of the following choices will provide the most information to the orthopaedic surgeon regarding the presence of degenerative joint disease?&lt;/div&gt;&lt;/div&gt;&lt;blockquote style=&quot;border: none; margin: 0px 0px 0px 40px; padding: 0px; text-align: left;&quot;&gt;&lt;div&gt;&lt;div&gt;A. AP pelvis plain x-ray&lt;/div&gt;&lt;/div&gt;&lt;div&gt;&lt;div&gt;B. CT scan of the pelvis&lt;/div&gt;&lt;/div&gt;&lt;div&gt;&lt;div&gt;C. AP pelvis, AP, and lateral plain radiographs of the affected hip&lt;/div&gt;&lt;/div&gt;&lt;div&gt;&lt;div&gt;D. AP and lateral comparison plain x-rays of the unaffected hip&lt;/div&gt;&lt;/div&gt;&lt;div&gt;&lt;div&gt;E. MRI of the pelvis&lt;/div&gt;&lt;/div&gt;&lt;/blockquote&gt;&lt;div&gt;&lt;div&gt;&lt;br /&gt;&lt;/div&gt;&lt;div&gt;40.2 A 68-year-old male active tennis player presents to your practice with a 4-year history of left groin pain. His pain has been worsening over the last year, and he is now unable to participate in the local singles league. He has moderate relief with NSAIDs and acetaminophen in the past, but states these are no longer fully effective. An orthopaedic spine surgeon has evaluated him in the recent past for degenerative disc disease but only treated him with NSAIDs and prednisone for what he recalls as a “slipped disc.” On exam, the patient has a limited range of hip motion and pain with internal rotation. Straight leg raise above 70 degrees causes pain to radiate down his leg. X-rays show mild degenerative changes in his bilateral hips. What is the next best step in management of this patient?&lt;/div&gt;&lt;/div&gt;&lt;blockquote style=&quot;border: none; margin: 0px 0px 0px 40px; padding: 0px; text-align: left;&quot;&gt;&lt;div&gt;&lt;div&gt;A. Left total hip arthroplasty&lt;/div&gt;&lt;/div&gt;&lt;div&gt;&lt;div&gt;B. Left hip hemiarthroplasty&lt;/div&gt;&lt;/div&gt;&lt;div&gt;&lt;div&gt;C. Immediate referral back to his spine surgeon for further workup&lt;/div&gt;&lt;/div&gt;&lt;div&gt;&lt;div&gt;D. Intraarticular corticosteroid injection&lt;/div&gt;&lt;/div&gt;&lt;div&gt;&lt;div&gt;E. Continue NSAIDs and acetaminophen, recommend physical therapy, and add a low-dose narcotic for daily pains&lt;/div&gt;&lt;/div&gt;&lt;/blockquote&gt;&lt;div&gt;&lt;div&gt;&lt;br /&gt;&lt;/div&gt;&lt;div&gt;40.3 A 63-year-old obese woman is referred to your office for evaluation for arthritis, diagnosed on x-rays ordered by her primary care physician. The patient complains that she gets left hip pain when power-walking with her dog. She states she has an aunt who had relief of chronic pain from a hip replacement last year and wants you to perform the same “miracle operation.” A careful history, physical exam, and appropriate imaging confirm the diagnosis of left hip osteoarthritis. Which of the following is not an appropriate first-line treatment?&lt;/div&gt;&lt;/div&gt;&lt;blockquote style=&quot;border: none; margin: 0px 0px 0px 40px; padding: 0px; text-align: left;&quot;&gt;&lt;div&gt;&lt;div&gt;A. Activity modification&lt;/div&gt;&lt;/div&gt;&lt;div&gt;&lt;div&gt;B. Weight loss&lt;/div&gt;&lt;/div&gt;&lt;div&gt;&lt;div&gt;C. Physical therapy for quadriceps strengthening&lt;/div&gt;&lt;/div&gt;&lt;div&gt;&lt;div&gt;D. Intraarticular steroid injection&lt;/div&gt;&lt;/div&gt;&lt;div&gt;&lt;div&gt;E. Total hip arthroplasty&lt;/div&gt;&lt;/div&gt;&lt;/blockquote&gt;&lt;div&gt;&lt;br /&gt;&lt;/div&gt;&lt;div&gt;&lt;br /&gt;&lt;/div&gt;&lt;div&gt;&lt;b&gt;&lt;span style=&quot;color: #017087; font-size: medium;&quot;&gt;&lt;u&gt;ANSWERS&lt;/u&gt;&lt;/span&gt;&lt;/b&gt;&lt;/div&gt;&lt;div&gt;&lt;br /&gt;&lt;/div&gt;&lt;div&gt;&lt;div&gt;40.1 &lt;b&gt;C.&lt;/b&gt; AP pelvis, AP hip, and frog-leg lateral radiographs are the standard films to obtain of the hip when evaluating a patient for degenerative joint disease. CT scans and MRI are typically unnecessary in the setting of osteoarthritis.&lt;/div&gt;&lt;div&gt;&lt;br /&gt;&lt;/div&gt;&lt;div&gt;40.2 &lt;b&gt;D.&lt;/b&gt; It is unclear by this question whether his symptoms are from osteoarthritis or spinal pathology. In these situations, intraarticular corticosteroid injections may play a diagnostic role in addition to being therapeutic. The simultaneous injection of lidocaine or similar local anesthetics should provide near-immediate relief of symptoms if his pain is arthritic in nature. Although referral and/or consultation with a spine surgeon is appropriate, this patient may well have symptomatic osteoarthritis of the hip and should not be simply sent to see someone else. Although NSAIDs, acetaminophen, and physical therapy play an important role in his therapy, they do not necessarily provide the diagnostic evidence needed to determine the source of his pain; secondary, chronic narcotics should almost never be used in the treatment of degenerative joint disease. Of course, total hip arthroplasty or hemiarthroplasty should not be performed until the surgeon is certain of the underlying diagnosis and has exhausted nonoperative treatments when effective.&lt;/div&gt;&lt;div&gt;&lt;br /&gt;&lt;/div&gt;&lt;div&gt;40.3 &lt;b&gt;E.&lt;/b&gt; Activity modification, physical therapy (especially that directed at quadriceps strengthening), weight loss, and intraarticular steroids are all first-line nonoperative treatments for osteoarthritis. Total hip arthroplasty is reserved for patients who have failed conservative measures and whose arthritis is adversely affecting their ability to accomplish activities of daily living.&lt;/div&gt;&lt;/div&gt;&lt;div&gt;&lt;br /&gt;&lt;/div&gt;&lt;div&gt;&lt;b style=&quot;background-color: #4093a5;&quot;&gt;&amp;nbsp; &amp;nbsp;&lt;span style=&quot;font-size: medium;&quot;&gt;&amp;nbsp;&lt;/span&gt;&lt;span style=&quot;color: white;&quot;&gt;&lt;span style=&quot;font-size: medium;&quot;&gt;CLINICAL PEARLS&lt;/span&gt;&lt;span&gt;&amp;nbsp; &amp;nbsp;&amp;nbsp;&lt;/span&gt;&lt;/span&gt;&lt;/b&gt;&lt;/div&gt;&lt;div&gt;&lt;table border=&quot;1&quot; cellpadding=&quot;0&quot; cellspacing=&quot;0&quot; class=&quot;MsoTableGrid&quot; style=&quot;border-collapse: collapse; border: none; mso-border-alt: solid windowtext .5pt; mso-padding-alt: 0in 5.4pt 0in 5.4pt; mso-yfti-tbllook: 1184;&quot;&gt;
  4. &lt;tbody&gt;&lt;tr style=&quot;height: 19.15pt; mso-yfti-firstrow: yes; mso-yfti-irow: 0; mso-yfti-lastrow: yes;&quot;&gt;
  5.  &lt;td style=&quot;background: rgb(221, 237, 239); border-left: solid #017087 1.0pt; border: 1pt solid rgb(64, 147, 165); height: 19.15pt; padding: 0in 5.4pt; width: 433.7pt;&quot; valign=&quot;top&quot; width=&quot;578&quot;&gt;
  6.  &lt;div style=&quot;line-height: normal; margin-bottom: 0in;&quot;&gt;&lt;span style=&quot;color: #017087;&quot;&gt;&lt;br /&gt;&lt;/span&gt;&lt;/div&gt;&lt;div style=&quot;line-height: normal; margin-bottom: 0in;&quot;&gt;&lt;span style=&quot;color: #017087;&quot;&gt;►&lt;/span&gt;&amp;nbsp;The most common cause of end-stage hip arthritis in the United States is osteoarthritis.&lt;/div&gt;&lt;div style=&quot;line-height: normal; margin-bottom: 0in;&quot;&gt;&lt;span style=&quot;color: #017087;&quot;&gt;&lt;br /&gt;&lt;/span&gt;&lt;/div&gt;&lt;div style=&quot;line-height: normal; margin-bottom: 0in;&quot;&gt;&lt;span style=&quot;color: #017087;&quot;&gt;►&lt;/span&gt;&amp;nbsp;Most patients with intraarticular hip pathology present with groin pain that may or may not radiate to other locations around the hip joint or the knee.&lt;/div&gt;&lt;div style=&quot;line-height: normal; margin-bottom: 0in;&quot;&gt;&lt;span style=&quot;color: #017087;&quot;&gt;&lt;br /&gt;&lt;/span&gt;&lt;/div&gt;&lt;div style=&quot;line-height: normal; margin-bottom: 0in;&quot;&gt;&lt;span style=&quot;color: #017087;&quot;&gt;►&lt;/span&gt;&amp;nbsp;If it is unclear whether or not a patient is having hip, back, or pelvis pain, a diagnostic intraarticular injection of anesthetic and corticosteroids may help elicit the source of the underlying pain.&lt;/div&gt;&lt;div style=&quot;line-height: normal; margin-bottom: 0in;&quot;&gt;&lt;span style=&quot;color: #017087;&quot;&gt;&lt;br /&gt;&lt;/span&gt;&lt;/div&gt;&lt;div style=&quot;line-height: normal; margin-bottom: 0in;&quot;&gt;&lt;span style=&quot;color: #017087;&quot;&gt;►&lt;/span&gt;&amp;nbsp;Standard radiographic imaging of patient with hip arthritis includes an AP pelvis radiograph and an AP and lateral radiograph of the affected hip.&lt;/div&gt;&lt;div style=&quot;line-height: normal; margin-bottom: 0in;&quot;&gt;&lt;span style=&quot;color: #017087;&quot;&gt;&lt;br /&gt;&lt;/span&gt;&lt;/div&gt;&lt;div style=&quot;line-height: normal; margin-bottom: 0in;&quot;&gt;&lt;span style=&quot;color: #017087;&quot;&gt;►&lt;/span&gt;&amp;nbsp;First-line therapies for degenerative disease of the hip are almost always nonoperative in nature. They include interventions such as physical therapy (eg, stretching and range-of-motion exercises, modalities, pool therapy), nonsteroidal anti-inflammatories, and intraarticular corticosteroid injections.&lt;/div&gt;&lt;div style=&quot;line-height: normal; margin-bottom: 0in;&quot;&gt;&lt;br /&gt;&lt;/div&gt;
  7.  &lt;/td&gt;
  8. &lt;/tr&gt;
  9. &lt;/tbody&gt;&lt;/table&gt;&lt;br /&gt;&lt;/div&gt;
  10.  
  11. &lt;details close=&quot;&quot;&gt;
  12.  &lt;summary&gt;&lt;b&gt;&lt;span style=&quot;color: #00788b;&quot;&gt;REFERENCES&lt;/span&gt;&lt;/b&gt;&lt;/summary&gt;
  13.  &lt;p&gt;Hochberg MC, et al. Guidelines for the medical management of osteoarthritis. Part I. Osteoarthritis of
  14. the hip. American College of Rheumatology. Arthritis Rheum . 1995;38:1535-1540.&amp;nbsp;&lt;/p&gt;&lt;p&gt;Huo MH, et al. What’s new in total hip arthroplasty. J Bone Joint Surg Am . 2011;92:2959-2972.&amp;nbsp;&lt;/p&gt;&lt;p&gt;Pluot E, et al. Hip arthroplasty. Part 1: prosthesis terminology and classification. Clin Radiol. 2009;64:
  15. 954-960.&lt;/p&gt;
  16. &lt;/details&gt;&lt;/div&gt;</content><link rel='replies' type='application/atom+xml' href='https://medical-phd.blogspot.com/feeds/3004634123698531224/comments/default' title='Post Comments'/><link rel='replies' type='text/html' href='https://medical-phd.blogspot.com/2022/03/hip-osteoarthritis-case-file.html#comment-form' title='0 Comments'/><link rel='edit' type='application/atom+xml' href='https://www.blogger.com/feeds/3302710210518459416/posts/default/3004634123698531224'/><link rel='self' type='application/atom+xml' href='https://www.blogger.com/feeds/3302710210518459416/posts/default/3004634123698531224'/><link rel='alternate' type='text/html' href='https://medical-phd.blogspot.com/2022/03/hip-osteoarthritis-case-file.html' title='Hip Osteoarthritis Case File'/><author><name>Medical Group</name><uri>http://www.blogger.com/profile/01780656800578397393</uri><email>noreply@blogger.com</email><gd:image rel='http://schemas.google.com/g/2005#thumbnail' width='32' height='32' src='//blogger.googleusercontent.com/img/b/R29vZ2xl/AVvXsEiMuROkBDv6yw4SmxGqRA5BPQhOXSwFld-Gufmy4kUqfpoAGQRl7cv-3naew6wjAnCjRFSe_FKX8z7CdE6cFKxxBpnqBuwUTSr-15xd3mFGcr_wJOX2y73G_zJCGND98Q/s220/Final-logo.jpg'/></author><media:thumbnail xmlns:media="http://search.yahoo.com/mrss/" url="https://blogger.googleusercontent.com/img/b/R29vZ2xl/AVvXsEiIKcCAYy8fEiprG9Q9Cj8bA98NVe_nPkvaQIRFHfGEfN7OGjQbcF7Il2vacT1PrAeIQNP4M8ljF7MSBuN60JwzW3s0j3jzotH8mhUTaG3DsSrSJ6MI_8bjUtSsrlMEsT3w5Mc4MpwREwoYm77rDV3vCRb1U2L7ZiOU1wr-D36mPG3FCw9R_A8n6tLu1g/s72-w640-h460-c/AP-pelvis-radiograph.jpg" height="72" width="72"/><thr:total>0</thr:total><georss:featurename>United States</georss:featurename><georss:point>37.09024 -95.712891</georss:point><georss:box>8.780006163821156 -130.869141 65.400473836178847 -60.556641</georss:box></entry><entry><id>tag:blogger.com,1999:blog-3302710210518459416.post-1478683141063029018</id><published>2022-03-31T23:13:00.002-07:00</published><updated>2022-03-31T23:13:14.663-07:00</updated><category scheme="http://www.blogger.com/atom/ns#" term="Case Files"/><category scheme="http://www.blogger.com/atom/ns#" term="Orthopaedic Surgery Case File"/><title type='text'>Total Knee Arthroplasty Case File</title><content type='html'>&lt;div class=&quot;scrollbox&quot;&gt;
  17.  
  18. &lt;div style=&quot;text-align: left;&quot;&gt;&lt;b&gt;&lt;span style=&quot;font-size: large;&quot;&gt;Total Knee Arthroplasty Case File&lt;/span&gt;&lt;/b&gt;&lt;/div&gt;&lt;div&gt;Eugene C. Toy, MD, Andrew J. Rosenbaum, MD, Timothy T. Roberts, MD, Joshua S. Dines, MD&lt;/div&gt;&lt;div&gt;&lt;br /&gt;&lt;/div&gt;&lt;div&gt;&lt;b&gt;&lt;span style=&quot;color: #017087; font-size: medium;&quot;&gt;CASE 39&lt;/span&gt;&lt;/b&gt;&lt;/div&gt;&lt;div&gt;&lt;div&gt;A 73-year-old man is seen in the outpatient clinic with complaints of right knee pain, 6 months in duration. He denies recent trauma and is otherwise without significant medical problems. He states that over the past several years, he has had a progressive “dull ache” in his knee. The near constant pain wakes him at least 3 times per week. Last month, he started using a golf cart instead of walking between holes and now borrows a friend’s cane when walking long distances. He states that his right leg feels “unstable.” He denies fevers, weight loss, and recent illnesses, and he takes only acetaminophen as needed for pain relief. Family history is unremarkable. On physical exam of his right knee, there is no obvious effusion, erythema, or drainage. He has joint line tenderness, most prominent on the lateral aspect of his knee, and he exhibits a slight genu valgum. There is crepitus with range of motion. A 5-degree flexion contracture is noted when he is fully extended, and he can only flex the knee to 100 degrees. He is neurovascularly and ligamentously intact. He has brought an x-ray of his right knee, taken last month at his primary care doctor’s office ( Figure 39–1 ).&lt;/div&gt;&lt;div&gt;&lt;br /&gt;&lt;/div&gt;&lt;div&gt;&lt;span style=&quot;color: #017087;&quot;&gt;►&lt;/span&gt;&amp;nbsp;What is the most likely diagnosis?&lt;/div&gt;&lt;div&gt;&lt;span style=&quot;color: #017087;&quot;&gt;►&lt;/span&gt;&amp;nbsp;What are the next steps in the workup for this condition?&lt;/div&gt;&lt;div&gt;&lt;span style=&quot;color: #017087;&quot;&gt;►&lt;/span&gt;&amp;nbsp;What are the next steps in the management of this condition?&lt;/div&gt;&lt;/div&gt;&lt;div&gt;&lt;br /&gt;&lt;/div&gt;&lt;div class=&quot;separator&quot; style=&quot;clear: both; text-align: center;&quot;&gt;&lt;a href=&quot;https://blogger.googleusercontent.com/img/b/R29vZ2xl/AVvXsEj90Na5B280mvyktstkFH1p35EKF4w_jzrBQwu43XrVnwGNqWQOzRVFMIk0tFNye8TINEWgIIbpz5-iOCBCA5OCpvfUcnX3S26-VP9LoHxUlXGibDMb1W084LYXPkmgpC5MEd5SKxlIx-9k5NaPckFoEcsVwRbmjcuy097OWiSlgKQsUzRx4Qamffj9-Q/s500/AP-radiograph-of-the-right-knee.jpg&quot; style=&quot;margin-left: 1em; margin-right: 1em;&quot;&gt;&lt;img alt=&quot;AP radiograph of the right knee&quot; border=&quot;0&quot; data-original-height=&quot;500&quot; data-original-width=&quot;227&quot; height=&quot;640&quot; src=&quot;https://blogger.googleusercontent.com/img/b/R29vZ2xl/AVvXsEj90Na5B280mvyktstkFH1p35EKF4w_jzrBQwu43XrVnwGNqWQOzRVFMIk0tFNye8TINEWgIIbpz5-iOCBCA5OCpvfUcnX3S26-VP9LoHxUlXGibDMb1W084LYXPkmgpC5MEd5SKxlIx-9k5NaPckFoEcsVwRbmjcuy097OWiSlgKQsUzRx4Qamffj9-Q/w290-h640/AP-radiograph-of-the-right-knee.jpg&quot; title=&quot;AP radiograph of the right knee&quot; width=&quot;290&quot; /&gt;&lt;/a&gt;&lt;/div&gt;&lt;div&gt;&lt;b&gt;&lt;span style=&quot;color: #017087;&quot;&gt;Figure 39–1.&lt;/span&gt;&lt;/b&gt; AP radiograph of the right knee.&lt;/div&gt;&lt;div&gt;&lt;br /&gt;&lt;/div&gt;&lt;div&gt;&lt;br /&gt;&lt;/div&gt;&lt;div&gt;&lt;div&gt;&lt;b&gt;&lt;span style=&quot;font-size: medium;&quot;&gt;ANSWER TO CASE 39:&lt;/span&gt;&lt;/b&gt;&lt;/div&gt;&lt;div&gt;&lt;span style=&quot;background-color: #daebf0; font-size: medium;&quot;&gt;&lt;span&gt;&lt;span style=&quot;background-color: transparent;&quot;&gt;&lt;span style=&quot;color: #017087;&quot;&gt;&lt;b&gt;Total Knee Arthroplasty&lt;/b&gt;&lt;/span&gt;&lt;/span&gt;&lt;b&gt;&amp;nbsp; &amp;nbsp;&amp;nbsp;&lt;/b&gt;&lt;/span&gt;&lt;span style=&quot;font-weight: bold;&quot;&gt;&amp;nbsp;&amp;nbsp; &amp;nbsp;&lt;/span&gt;&lt;span style=&quot;font-weight: bold;&quot;&gt;&amp;nbsp;&amp;nbsp; &amp;nbsp;&lt;/span&gt;&lt;span style=&quot;font-weight: bold;&quot;&gt;&amp;nbsp;&amp;nbsp; &amp;nbsp;&lt;/span&gt;&lt;span style=&quot;font-weight: bold;&quot;&gt;&amp;nbsp; &amp;nbsp;&lt;/span&gt;&lt;/span&gt;&lt;/div&gt;&lt;/div&gt;&lt;div&gt;&lt;br /&gt;&lt;/div&gt;&lt;div&gt;&lt;div&gt;&lt;i&gt;Summary&lt;/i&gt;: A relatively healthy 73-year-old man presents with progressive right knee pain. His pain is severe enough that it has caused him to change his daily activities, has caused him to start using a cane, and wakes him at night. Exam reveals crepitus, lateral joint line tenderness, and a limited range of motion. He denies a history of traumatic injury and exhibits neither signs nor symptoms of infection.&lt;/div&gt;&lt;div&gt;&lt;ul style=&quot;text-align: left;&quot;&gt;&lt;li&gt;&lt;b&gt;Most likely diagnosis:&lt;/b&gt; Osteoarthritis (degenerative joint disease) of the right knee.&lt;/li&gt;&lt;li&gt;&lt;b&gt;Next diagnostic test:&lt;/b&gt; Obtain plain radiographs of bilateral knees, including weightbearing posteroanterior and lateral views in approximately 45 degrees of flexion, a tunnel or notch view, and Merchant (sunrise) views of the patellofemoral joint.&lt;/li&gt;&lt;li&gt;&lt;b&gt;Next step in therapy:&lt;/b&gt; Conservative therapy including nonsteroidal antiinflammatory drugs (NSAIDs; ibuprofen, naproxen, diclofenac, or selective COX-2 inhibitors); physical therapy is sometimes recommended to help strengthen the muscles around the knee, including the hamstrings, gastrocsoleus complex, and quadriceps. Assistive devices such as canes, crutches, and walkers can be used as well to offset some of the joint reactive forces stressing the knee.&lt;/li&gt;&lt;/ul&gt;&lt;/div&gt;&lt;/div&gt;&lt;div&gt;&lt;br /&gt;&lt;/div&gt;&lt;div&gt;&lt;br /&gt;&lt;/div&gt;&lt;div&gt;&lt;div&gt;&lt;b&gt;&lt;span style=&quot;color: #017087; font-size: medium;&quot;&gt;ANALYSIS&lt;/span&gt;&lt;/b&gt;&lt;/div&gt;&lt;div&gt;&lt;b&gt;&lt;span style=&quot;font-size: medium;&quot;&gt;&lt;i&gt;Objectives&lt;/i&gt;&lt;/span&gt;&lt;/b&gt;&lt;/div&gt;&lt;/div&gt;&lt;div&gt;&lt;div&gt;&lt;ol style=&quot;text-align: left;&quot;&gt;&lt;li&gt;Recognize the presentation of DJD of the knee and understand the salient aspects of patient history necessary to guide treatment.&lt;/li&gt;&lt;li&gt;Develop a standard treatment algorithm for DJD of the knee.&lt;/li&gt;&lt;li&gt;Understand the surgical options available for the treatment of knee arthritis.&lt;/li&gt;&lt;li&gt;Understand common potential complications after total knee arthroplasty.&lt;/li&gt;&lt;/ol&gt;&lt;/div&gt;&lt;/div&gt;&lt;div&gt;&lt;br /&gt;&lt;/div&gt;&lt;div&gt;&lt;br /&gt;&lt;/div&gt;&lt;div&gt;&lt;div&gt;&lt;b&gt;&lt;i&gt;&lt;span style=&quot;font-size: medium;&quot;&gt;Considerations&lt;/span&gt;&lt;/i&gt;&lt;/b&gt;&lt;/div&gt;&lt;div&gt;&lt;br /&gt;&lt;/div&gt;&lt;div&gt;There are several differential diagnoses that must be considered in any patient with nonacute traumatic joint pain. Common etiologies include osteoarthritis, also referred to as degenerative joint disease (DJD), inflammatory arthritis, osteonecrosis, and posttraumatic arthritis. Given this patient’s atraumatic history and his lack of fevers, joint erythema, or other systemic inflammatory symptomatology, osteoarthritis is the most likely diagnosis. Specifics for workup of these alternate diagnoses are considered later in this chapter. This 73-year-old male presents with signs, symptoms, and radiographic findings consistent with right knee osteoarthritis, specifically the lateral compartment. In a patient who has had no previous intervention to address his symptoms, &lt;b&gt;the first priority is pain relief.&lt;/b&gt; There are many options for pain relief, including acetaminophen, NSAIDs, corticosteroid injections, and narcotics. Ideally, narcotics are reserved for short-term relief in individuals with intractable pain, refractory to all other modalities. Following analgesia recommendations, attempts should be made to address pathologic changes in the mechanics of knee. The available treatment options are bracing, physical therapy, and assistive devices for ambulation. Other conservative modalities include intraarticular viscosupplementation with hyaluronic acid analogues or derivatives (Synvisc ®) and corticosteroid injections.&lt;/div&gt;&lt;/div&gt;&lt;div&gt;&lt;br /&gt;&lt;/div&gt;&lt;div&gt;&lt;br /&gt;&lt;/div&gt;&lt;div&gt;&lt;div&gt;&lt;div&gt;&lt;div&gt;&lt;b&gt;&lt;span style=&quot;font-size: medium;&quot;&gt;APPROACH TO:&lt;/span&gt;&lt;/b&gt;&lt;/div&gt;&lt;div&gt;&lt;span style=&quot;background-color: #daebf0; color: #017087; font-size: medium;&quot;&gt;&lt;span&gt;&lt;span style=&quot;background-color: transparent;&quot;&gt;&lt;b&gt;Osteoarthritis of the Knee&lt;/b&gt;&lt;/span&gt;&lt;b&gt;&amp;nbsp;&amp;nbsp;&lt;/b&gt;&lt;/span&gt;&lt;span style=&quot;font-weight: bold;&quot;&gt;&amp;nbsp;&amp;nbsp; &amp;nbsp;&lt;/span&gt;&lt;span style=&quot;font-weight: bold;&quot;&gt;&amp;nbsp;&amp;nbsp; &amp;nbsp;&lt;/span&gt;&lt;span style=&quot;font-weight: bold;&quot;&gt;&amp;nbsp;&amp;nbsp; &amp;nbsp;&lt;/span&gt;&lt;span style=&quot;font-weight: bold;&quot;&gt;&amp;nbsp;&amp;nbsp; &amp;nbsp;&lt;/span&gt;&lt;span style=&quot;font-weight: bold;&quot;&gt;&amp;nbsp;&amp;nbsp; &amp;nbsp;&lt;/span&gt;&lt;span style=&quot;font-weight: bold;&quot;&gt;&amp;nbsp;&amp;nbsp; &amp;nbsp;&lt;/span&gt;&lt;span style=&quot;font-weight: bold;&quot;&gt;&amp;nbsp;&amp;nbsp;&amp;nbsp;&lt;/span&gt;&lt;span style=&quot;font-weight: bold;&quot;&gt;&amp;nbsp;&lt;/span&gt;&lt;/span&gt;&lt;/div&gt;&lt;/div&gt;&lt;div&gt;&lt;br /&gt;&lt;/div&gt;&lt;div&gt;&lt;b&gt;&lt;span style=&quot;color: #017087; font-size: medium;&quot;&gt;DEFINITIONS&lt;/span&gt;&lt;/b&gt;&lt;/div&gt;&lt;/div&gt;&lt;div&gt;&lt;br /&gt;&lt;/div&gt;&lt;/div&gt;&lt;div&gt;&lt;div&gt;&lt;b&gt;ARTHROPLASTY:&lt;/b&gt; The surgical replacement and reconstruction of a functional joint.&lt;/div&gt;&lt;div&gt;&lt;b&gt;&lt;br /&gt;&lt;/b&gt;&lt;/div&gt;&lt;div&gt;&lt;b&gt;OSTEONECROSIS:&lt;/b&gt; The death of bone tissue secondary to impaired or disrupted blood supply, often as the result of trauma or disease. Osteonecrosis is marked by severe, localized pain and by structurally weakened bone that may flatten and collapse. This process is also known as aseptic or avascular necrosis (AVN).&lt;/div&gt;&lt;div&gt;&lt;b&gt;&lt;br /&gt;&lt;/b&gt;&lt;/div&gt;&lt;div&gt;&lt;b&gt;POLYMETHYLMETHACRYLATE (PMMA): &lt;/b&gt;A synthetic polymer used commonly as cement for the implantation of knee and hip prostheses&lt;/div&gt;&lt;/div&gt;&lt;div&gt;&lt;br /&gt;&lt;/div&gt;&lt;div&gt;&lt;br /&gt;&lt;/div&gt;&lt;div&gt;&lt;b&gt;&lt;span style=&quot;color: #017087; font-size: medium;&quot;&gt;CLINICAL APPROACH&lt;/span&gt;&lt;/b&gt;&lt;/div&gt;&lt;div&gt;&lt;br /&gt;&lt;/div&gt;&lt;div&gt;&lt;div&gt;&lt;b&gt;&lt;i&gt;&lt;span style=&quot;font-size: medium;&quot;&gt;Differential Diagnosis&lt;/span&gt;&lt;/i&gt;&lt;/b&gt;&lt;/div&gt;&lt;div&gt;&lt;br /&gt;&lt;/div&gt;&lt;div&gt;As discussed, there are several causes of knee pain that must be considered before the diagnosis of osteoarthritis is made. Like DJD, &lt;b&gt;inflammatory arthropathies&amp;nbsp;&lt;/b&gt;&lt;b&gt;such as gout, chronic septic arthritis, and rheumatologic processes may present&amp;nbsp;&lt;/b&gt;&lt;b&gt;with insidious, atraumatic knee pain.&lt;/b&gt; Several conditions in this wide spectrum of inflammatory diseases are discussed in greater depth elsewhere in this text. As a general rule, however, if there is suspicion for underlying infectious, rheumatologic, or crystalline process, further workup should be performed and includes obtaining blood and synovial fluid samples. Although the specific laboratory tests to be ordered are chosen on a case-by-case basis, general studies typically always obtained include a complete blood count (CBC), erythrocyte sedimentation rate (ESR), C-reactive protein (CRP), and synovial fluid gram stain, crystal analysis, cell count and aerobic and anaerobic cultures.&lt;/div&gt;&lt;div&gt;&lt;b&gt;&lt;br /&gt;&lt;/b&gt;&lt;/div&gt;&lt;div&gt;&lt;b&gt;&lt;span&gt;&amp;nbsp;&amp;nbsp; &amp;nbsp;&lt;/span&gt;Avascular necrosis&lt;/b&gt; of the knee can also be a cause of this gentleman’s pain. There are several etiologies of this condition. First, spontaneous osteonecrosis of the knee, an idiopathic condition that commonly affects women older than 55 years, is typically unilateral and affects only 1 condyle. The other causes include excessive alcohol use, sickle cell anemia, trauma, or corticosteroid use and usually affect patients younger than 45 years. Both conditions may present with slow-onset knee pain, sometimes worst at night, and typically affect the medial compartment. As with DJD, these conditions may respond to arthroplasty when conservative treatments fail.&lt;/div&gt;&lt;div&gt;&lt;br /&gt;&lt;/div&gt;&lt;div&gt;&lt;span&gt;&amp;nbsp;&amp;nbsp; &amp;nbsp;&lt;/span&gt;Finally, &lt;b&gt;posttraumatic arthritis&lt;/b&gt; is an accelerated degenerative process of the knee that results from previous damage to the knee’s articular surfaces. It is managed in much the same way as DJD, with conservative measures first and arthroplasty for end-stage disease.&lt;/div&gt;&lt;/div&gt;&lt;div&gt;&lt;br /&gt;&lt;/div&gt;&lt;div&gt;&lt;br /&gt;&lt;/div&gt;&lt;div&gt;&lt;span style=&quot;color: #017087; font-size: medium;&quot;&gt;&lt;b&gt;TREATMENT OF OSTEOARTHRITIS&lt;/b&gt;&lt;/span&gt;&lt;/div&gt;&lt;div&gt;&lt;br /&gt;&lt;/div&gt;&lt;div&gt;&lt;div&gt;&lt;b&gt;&lt;i&gt;&lt;span style=&quot;font-size: medium;&quot;&gt;Nonoperative (Conservative) Treatment&lt;/span&gt;&lt;/i&gt;&lt;/b&gt;&lt;/div&gt;&lt;div&gt;&lt;br /&gt;&lt;/div&gt;&lt;div&gt;Despite the surgical focus of this text, it must be stressed that the&lt;b&gt; initial management&amp;nbsp;&lt;/b&gt;&lt;b&gt;of DJD is almost always nonoperative.&lt;/b&gt; The conservative interventions described previously are usually successful in attenuating patients’ DJD symptoms, at least in the short term. &lt;b&gt;How long, exactly, DJD can be managed nonoperatively&amp;nbsp;&lt;/b&gt;&lt;b&gt;varies greatly between individuals.&lt;/b&gt; Several studies show significant short-term pain relief and functional improvement using corticosteroid injections and viscosupplementation, but results, again, are unpredictable.&lt;/div&gt;&lt;/div&gt;&lt;div&gt;&lt;br /&gt;&lt;/div&gt;&lt;div&gt;&lt;div&gt;&lt;span&gt;&amp;nbsp;&amp;nbsp; &amp;nbsp;&lt;/span&gt;It is important to recognize that none of these aforementioned treatments are a cure for DJD. Therefore, when discussing the natural history of osteoarthritis with patients, it is important to explain that it will progress as long as they partake in weightbearing activities. Because &lt;b&gt;the lifespan of a prosthetic joint is finite&lt;/b&gt; —and most prone to wear in younger, active individuals—arthroplasty interventions are generally delayed until the patient can no longer tolerate conservative measures.&lt;/div&gt;&lt;/div&gt;&lt;div&gt;&lt;br /&gt;&lt;/div&gt;&lt;div&gt;&lt;br /&gt;&lt;/div&gt;&lt;div&gt;&lt;div&gt;&lt;b&gt;&lt;i&gt;&lt;span style=&quot;font-size: medium;&quot;&gt;Operative Treatment&lt;/span&gt;&lt;/i&gt;&lt;/b&gt;&lt;/div&gt;&lt;div&gt;&lt;br /&gt;&lt;/div&gt;&lt;div&gt;Once a patient has failed all conservative therapy, surgery may be indicated. There are several surgical options available and include total knee arthroplasty, osteotomies, arthroscopic interventions, and unicompartmental arthroplasty.&lt;/div&gt;&lt;/div&gt;&lt;div&gt;&lt;br /&gt;&lt;/div&gt;&lt;div&gt;&lt;br /&gt;&lt;/div&gt;&lt;div&gt;&lt;div&gt;&lt;b&gt;&lt;i&gt;&lt;span style=&quot;font-size: medium;&quot;&gt;Arthroscopy and Osteotomies&lt;/span&gt;&lt;/i&gt;&lt;/b&gt;&lt;/div&gt;&lt;div&gt;&lt;br /&gt;&lt;/div&gt;&lt;div&gt;Arthroscopic interventions are reserved for young patients with mild, localized disease in the setting of mechanical symptoms (ie, joint clicking, locking, or instability). Arthroscopic treatments are guided at debridement, chondroplasty, and removal of chondral loose bodies commonly found in osteoarthritic disease. Arthroscopy also plays a diagnostic role in determining the severity of arthritic disease, as some patients with advanced, debilitating arthritis may have deceptively benignappearing x-rays.&lt;/div&gt;&lt;/div&gt;&lt;div&gt;&lt;br /&gt;&lt;/div&gt;&lt;div&gt;&lt;div&gt;&lt;span&gt;&amp;nbsp;&amp;nbsp; &amp;nbsp;&lt;/span&gt;Another surgical option that does not involve prostheses are osteotomies, or wedge-shaped bone cuts, that alter the alignment of the knee joint. When patients present with disproportionately worn compartments, the alignment of the knee can be altered to divert weightbearing pressures from the affected side. Ideal patients for this procedure are younger than 50 years, with isolated unicompartmental disease and a normal range of motion. They should also have competent ligaments, active flexion beyond 90 degrees, and minimal evidence of a flexion contracture. A varus or valgus osteotomy can be used to offload the lateral or medial compartments, respectively. In general, varus-producing osteotomies are performed on the femur, and valgus-producing osteotomies are performed on the tibia.&lt;/div&gt;&lt;/div&gt;&lt;div&gt;&lt;br /&gt;&lt;/div&gt;&lt;div&gt;&lt;br /&gt;&lt;/div&gt;&lt;div&gt;&lt;div&gt;&lt;b&gt;&lt;i&gt;&lt;span style=&quot;font-size: medium;&quot;&gt;Arthroplasty&lt;/span&gt;&lt;/i&gt;&lt;/b&gt;&lt;/div&gt;&lt;div&gt;&lt;br /&gt;&lt;/div&gt;&lt;div&gt;Before discussing total joint arthroplasty, it is worthy to mention the increasing popularity of unicompartmental knee arthroplasty (UKA). This procedure is restricted to patients with unicompartmental noninflammatory osteoarthritis, deformities of less than 10 degrees (varus or valgus), and an intact anterior cruciate ligament. Medial compartment replacement is by far the most commonly performed UKA, as the prevalence of medial-sided arthritis far exceeds that of isolated lateral disease. In the correct patient population, advantages of UKA over total joint replacement include faster recovery, less rehabilitation, and smaller incisions.&lt;/div&gt;&lt;/div&gt;&lt;div&gt;&lt;br /&gt;&lt;/div&gt;&lt;div&gt;&lt;div&gt;&lt;span&gt;&amp;nbsp;&amp;nbsp; &amp;nbsp;&lt;/span&gt;The most commonly performed procedure for knee arthritis in the United States is total knee arthroplasty (TKA). Typically, patients are older than 50 years, have disease in all 3 compartments, and have experienced changes in their activity levels secondary to pain. TKA involves a resurfacing procedure of the distal femur and proximal tibia and typically the undersurface of the patella. &lt;b&gt;The goals are to&amp;nbsp;&lt;/b&gt;&lt;b&gt;reconstruct a stable knee with a functional range of motion, to restore knee alignment&amp;nbsp;&lt;/b&gt;&lt;b&gt;and smooth patella tracking, and to relieve pain.&lt;/b&gt; There are a great variety of TKA prosthetic designs whose nuances extend beyond the scope of this discussion. Most modern TKA systems, however, share these same goals, and indeed there is no definitive evidence that supports the superiority of one modern TKA system over another.&lt;/div&gt;&lt;/div&gt;&lt;div&gt;&lt;br /&gt;&lt;/div&gt;&lt;div&gt;&lt;br /&gt;&lt;/div&gt;&lt;div&gt;&lt;div&gt;&lt;b&gt;&lt;i&gt;&lt;span style=&quot;font-size: medium;&quot;&gt;Complications&lt;/span&gt;&lt;/i&gt;&lt;/b&gt;&lt;/div&gt;&lt;div&gt;&lt;br /&gt;&lt;/div&gt;&lt;div&gt;The common complications that are associated with TKA include infection, deep vein thrombosis, aseptic loosening, osteolysis, periprosthetic fracture, failure of the extensor mechanism, and postoperative stiffness or arthrofibrosis.&lt;/div&gt;&lt;/div&gt;&lt;div&gt;&lt;br /&gt;&lt;/div&gt;&lt;div&gt;&lt;div&gt;&lt;span&gt;&amp;nbsp;&amp;nbsp; &amp;nbsp;&lt;/span&gt;Infections in an artificial joint, although rare (&amp;lt; 1%), are one of the most devastating complications of TKA. Periprosthetic joint infections (PJIs) can present in a multitude of ways. While some patients present with acute pain, swelling, and redness, others have a more insidious onset of symptoms. In general, any patient with a TKA and new-onset pain should be evaluated for infection, amongst other causes of pain. Diagnosis can be challenging and relies on physical exam findings, laboratory studies (ESR, CRP), and synovial fluid analysis. Although a positive gram stain and/or cultures make the diagnosis easy, many times these tests are negative. When this occurs, many believe that a synovial fluid white blood cell count of greater than 1100 cells/mL and a polymorphonuclear level of greater than 64% is consistent with a PJI, regardless of gram stain and culture findings. The management of PJI is challenging. In general, acute infections (defined as those of less than 3 weeks duration) are treated with a surgical irrigation and debridement and polyethylene exchange. Unfortunately, a PJI of greater than 3 weeks duration requires a complete explantation of the total knee system, placement of an antibiotic-eluting spacer where the total knee was, and 4 to 6 weeks of intravenous antibiotics. After this, a replantation of the total knee may be performed.&lt;/div&gt;&lt;/div&gt;&lt;div&gt;&lt;br /&gt;&lt;/div&gt;&lt;div&gt;&lt;div&gt;&lt;span&gt;&amp;nbsp;&amp;nbsp; &amp;nbsp;&lt;/span&gt;Finally, patients undergoing total joint replacements are notoriously at risk for deep vein thrombosis (DVT). DVTs occur in up to 5% to 15% of patients undergoing arthroplasty and are considered the most common complication of TKA. Most surgeons and hospitals go to considerable lengths to reduce DVT rates, including frequent mobilization postoperatively, placement of sequential compression stockings and compression stockings, and the use of anticoagulation medications such as postoperative heparin, low-molecular-weight heparin, warfarin, aspirin, and several other drugs. Unfortunately, despite significant efforts, the rates of DVT in arthroplasty patients remain relatively high.&lt;/div&gt;&lt;/div&gt;&lt;div&gt;&lt;br /&gt;&lt;/div&gt;&lt;div&gt;&lt;br /&gt;&lt;/div&gt;&lt;div&gt;&lt;b&gt;&lt;span style=&quot;color: #017087; font-size: medium;&quot;&gt;&lt;u&gt;COMPREHENSION QUESTIONS&lt;/u&gt;&lt;/span&gt;&lt;/b&gt;&lt;/div&gt;&lt;div&gt;&lt;br /&gt;&lt;/div&gt;&lt;div&gt;&lt;div&gt;39.1 A 55-year-old construction worker presents to your office with complaints of right knee pain, several years in duration. Examination of the right knee is notable for a slight flexion contraction, painful range of motion (ROM), and palpable crepitus during ROM. The left knee by comparison has a greater, albeit limited ROM that is not painful. Standing radiographs show significant medial and lateral compartmental joint space narrowing with sclerotic and osteophytic changes in bilateral knees. He has not yet been treated for his condition. What is the best next step in the management of this patient?&lt;/div&gt;&lt;/div&gt;&lt;blockquote style=&quot;border: none; margin: 0px 0px 0px 40px; padding: 0px; text-align: left;&quot;&gt;&lt;div&gt;&lt;div&gt;A. Right total knee arthroplasty&lt;/div&gt;&lt;/div&gt;&lt;div&gt;&lt;div&gt;B. Right knee medial unicompartmental arthroplasty&lt;/div&gt;&lt;/div&gt;&lt;div&gt;&lt;div&gt;C. Right knee lateral unicompartmental arthroplasty&lt;/div&gt;&lt;/div&gt;&lt;div&gt;&lt;div&gt;D. NSAIDs and acetaminophen, recommend physical therapy&lt;/div&gt;&lt;/div&gt;&lt;div&gt;&lt;div&gt;E. Bilateral knee total arthroplasties&lt;/div&gt;&lt;/div&gt;&lt;/blockquote&gt;&lt;div&gt;&lt;div&gt;&lt;br /&gt;&lt;/div&gt;&lt;div&gt;39.2 A 52-year-old woman with a history of rheumatoid arthritis is referred to your clinic with right knee pain. For the past 3 years, her rheumatoid arthritis has been well controlled with a disease-modifying anti-rheumatic drug (DMARD) regimen; however, she continues to have persistent knee pain. On exam you note a comparatively varus right knee with a painful and limited ROM. She is ligamentously intact and is tender to palpation only on the anteromedial aspect of her knee. Radiographs show significant narrowing of the right knee medial compartment, with relative sparing of the patellofemoral and lateral compartments. There are sclerotic and cystic changes on the medial aspect of her knee only. Having failed nonoperative interventions, which of the following is the most appropriate surgical intervention for this patient?&lt;/div&gt;&lt;/div&gt;&lt;blockquote style=&quot;border: none; margin: 0px 0px 0px 40px; padding: 0px; text-align: left;&quot;&gt;&lt;div&gt;&lt;div&gt;A. Right total knee arthroplasty&lt;/div&gt;&lt;/div&gt;&lt;div&gt;&lt;div&gt;B. Right knee lateral unicompartmental arthroplasty&lt;/div&gt;&lt;/div&gt;&lt;div&gt;&lt;div&gt;C. Right knee medial unicompartmental arthroplasty&lt;/div&gt;&lt;/div&gt;&lt;div&gt;&lt;div&gt;D. Right knee corrective osteotomy&lt;/div&gt;&lt;/div&gt;&lt;/blockquote&gt;&lt;div&gt;&lt;br /&gt;&lt;/div&gt;&lt;div&gt;&lt;div&gt;39.3 A 73-year-old woman is on postoperative day 3 for a total knee replacement. The procedure was without complication, but as a result of postoperative pain, she has been slow to mobilize. You are called to evaluate the patient for increasing tachycardia (heart rate 115 beats/minute) and sudden-onset tachypnea (respiratory rate 30 breaths/min). Blood pressure and temperature are within normal limits. She has received a dose of intravenous morphine 30 minutes before your evaluation and currently states her pain is “only a 3/10.” Electrocardiogram, cardiac enzymes, and chest x-ray are obtained and demonstrate no acute changes or abnormalities. She is a smoker and has a history of breast cancer. Which of the following is the most likely diagnosis?&lt;/div&gt;&lt;/div&gt;&lt;blockquote style=&quot;border: none; margin: 0px 0px 0px 40px; padding: 0px; text-align: left;&quot;&gt;&lt;div&gt;&lt;div&gt;A. Acute myocardial infarction&lt;/div&gt;&lt;/div&gt;&lt;div&gt;&lt;div&gt;B. Pneumonia&lt;/div&gt;&lt;/div&gt;&lt;div&gt;&lt;div&gt;C. Pulmonary embolism&lt;/div&gt;&lt;/div&gt;&lt;div&gt;&lt;div&gt;D. Fat embolism&lt;/div&gt;&lt;/div&gt;&lt;div&gt;&lt;div&gt;E. Blood loss anemia with secondary hypovolemia&lt;/div&gt;&lt;/div&gt;&lt;/blockquote&gt;&lt;div&gt;&lt;br /&gt;&lt;/div&gt;&lt;div&gt;&lt;br /&gt;&lt;/div&gt;&lt;div&gt;&lt;b&gt;&lt;span style=&quot;color: #017087; font-size: medium;&quot;&gt;&lt;u&gt;ANSWERS&lt;/u&gt;&lt;/span&gt;&lt;/b&gt;&lt;/div&gt;&lt;div&gt;&lt;br /&gt;&lt;/div&gt;&lt;div&gt;&lt;div&gt;39.1 &lt;b&gt;D.&lt;/b&gt; Nonoperative measures are almost always trialed before arthroplasty is performed for patients with DJD. Although this patient has radiographic degenerative changes, he is asymptomatic on the left, and thus bilateral TKA would be contraindicated. The presence of bilateral compartment disease is a contraindication to unicompartmental arthroplasty should the patient fail conservative treatment.&lt;/div&gt;&lt;div&gt;&lt;br /&gt;&lt;/div&gt;&lt;div&gt;39.2 &lt;b&gt;A.&lt;/b&gt; Although this patient’s arthritis is confined to the medial space, the presence of rheumatoid arthritis is a contraindication to unicompartmental replacement. Corrective osteotomies may correct the patient’s varus alignment but do nothing to address the underlying joint pathology.&lt;/div&gt;&lt;div&gt;&lt;br /&gt;&lt;/div&gt;&lt;div&gt;39.3 &lt;b&gt;C.&lt;/b&gt; Pulmonary emboli (PE) are an unfortunate and not too uncommon complication of knee or hip arthroplasty. Most occur second to embolized DVTs, which result from immobilizations, surgical trauma, insufficient postoperative thrombus prophylaxis, and underlying risk factors, such as this patient’s history of tobacco use and malignancy. The next step should be a CT angiogram of the chest (the diagnostic gold standard) or a ventilation perfusion scan. Although pneumonia and acute myocardial infarction may account for her symptoms, they are less likely given the normal chest x-ray and cardiac enzymes/electrocardiogram changes, respectively. Blood loss anemia with subsequent hypovolemic shock is also common; however, it is less likely given her normal blood pressure. Fat embolism may occur in the setting of joint replacement procedures, but more often occurs intraoperatively or immediately postoperatively with placement of intramedullary components. Remember that PEs rarely cause electrocardiogram changes and are not typically diagnosable on regular chest x-rays.&lt;/div&gt;&lt;/div&gt;&lt;div&gt;&lt;br /&gt;&lt;/div&gt;&lt;div&gt;&lt;b style=&quot;background-color: #4093a5;&quot;&gt;&amp;nbsp; &amp;nbsp;&lt;span style=&quot;font-size: medium;&quot;&gt;&amp;nbsp;&lt;/span&gt;&lt;span style=&quot;color: white;&quot;&gt;&lt;span style=&quot;font-size: medium;&quot;&gt;CLINICAL PEARLS&lt;/span&gt;&lt;span&gt;&amp;nbsp; &amp;nbsp;&amp;nbsp;&lt;/span&gt;&lt;/span&gt;&lt;/b&gt;&lt;/div&gt;&lt;div&gt;&lt;table border=&quot;1&quot; cellpadding=&quot;0&quot; cellspacing=&quot;0&quot; class=&quot;MsoTableGrid&quot; style=&quot;border-collapse: collapse; border: none; mso-border-alt: solid windowtext .5pt; mso-padding-alt: 0in 5.4pt 0in 5.4pt; mso-yfti-tbllook: 1184;&quot;&gt;
  19. &lt;tbody&gt;&lt;tr style=&quot;height: 19.15pt; mso-yfti-firstrow: yes; mso-yfti-irow: 0; mso-yfti-lastrow: yes;&quot;&gt;
  20.  &lt;td style=&quot;background: rgb(221, 237, 239); border-left: solid #017087 1.0pt; border: 1pt solid rgb(64, 147, 165); height: 19.15pt; padding: 0in 5.4pt; width: 433.7pt;&quot; valign=&quot;top&quot; width=&quot;578&quot;&gt;
  21.  &lt;div style=&quot;line-height: normal; margin-bottom: 0in;&quot;&gt;&lt;span style=&quot;color: #017087;&quot;&gt;&lt;br /&gt;&lt;/span&gt;&lt;/div&gt;&lt;div style=&quot;line-height: normal; margin-bottom: 0in;&quot;&gt;&lt;span style=&quot;color: #017087;&quot;&gt;►&lt;/span&gt;&amp;nbsp;Standard radiographic imaging of patient with DJD of the knee includes standing AP radiographs of bilateral knees, extension- and flexion-lateral radiographs of the affected knee, and a Merchant-view radiograph.&lt;/div&gt;&lt;div style=&quot;line-height: normal; margin-bottom: 0in;&quot;&gt;&lt;span style=&quot;color: #017087;&quot;&gt;&lt;br /&gt;&lt;/span&gt;&lt;/div&gt;&lt;div style=&quot;line-height: normal; margin-bottom: 0in;&quot;&gt;&lt;span style=&quot;color: #017087;&quot;&gt;►&lt;/span&gt;&amp;nbsp;First-line therapies for DJD of the knee are almost always nonoperative in nature. They include interventions such as physical therapy, nonsteroidal anti-inflammatories, and intraarticular corticosteroid injections.&lt;/div&gt;&lt;div style=&quot;line-height: normal; margin-bottom: 0in;&quot;&gt;&lt;span style=&quot;color: #017087;&quot;&gt;&lt;br /&gt;&lt;/span&gt;&lt;/div&gt;&lt;div style=&quot;line-height: normal; margin-bottom: 0in;&quot;&gt;&lt;span style=&quot;color: #017087;&quot;&gt;►&lt;/span&gt;&amp;nbsp;Total knee arthroplasty is the treatment of choice for end-stage osteoarthritis in patients who have failed conservative measures.&lt;/div&gt;&lt;div style=&quot;line-height: normal; margin-bottom: 0in;&quot;&gt;&lt;span style=&quot;color: #017087;&quot;&gt;&lt;br /&gt;&lt;/span&gt;&lt;/div&gt;&lt;div style=&quot;line-height: normal; margin-bottom: 0in;&quot;&gt;&lt;span style=&quot;color: #017087;&quot;&gt;►&lt;/span&gt;&amp;nbsp;A synovial fluid white blood cell count of greater than 1100 cells/mL and a polymorphonuclear level of greater than 64% is concerning for a PJI.&lt;/div&gt;&lt;div style=&quot;line-height: normal; margin-bottom: 0in;&quot;&gt;&lt;br /&gt;&lt;/div&gt;
  22.  &lt;/td&gt;
  23. &lt;/tr&gt;
  24. &lt;/tbody&gt;&lt;/table&gt;&lt;br /&gt;&lt;/div&gt;
  25.  
  26. &lt;details close=&quot;&quot;&gt;
  27.  &lt;summary&gt;&lt;b&gt;&lt;span style=&quot;color: #00788b;&quot;&gt;REFERENCES&lt;/span&gt;&lt;/b&gt;&lt;/summary&gt;
  28.  &lt;p&gt;Flynn JM, ed. Knee reconstruction and replacement. In: Orthopaedic Knowledge Update: Ten . Rosemont,
  29. IL: American Academy of Orthopaedic Surgeons; 2011:469-478.&amp;nbsp;&lt;/p&gt;&lt;p&gt;Miller MD, ed. Adult reconstruction. In: Review of Orthopaedics . 5th ed. Philadelphia: Saunders Elsevier; 2008:
  30. 306-358.&lt;/p&gt;
  31. &lt;/details&gt;&lt;/div&gt;</content><link rel='replies' type='application/atom+xml' href='https://medical-phd.blogspot.com/feeds/1478683141063029018/comments/default' title='Post Comments'/><link rel='replies' type='text/html' href='https://medical-phd.blogspot.com/2022/03/total-knee-arthroplasty-case-file.html#comment-form' title='0 Comments'/><link rel='edit' type='application/atom+xml' href='https://www.blogger.com/feeds/3302710210518459416/posts/default/1478683141063029018'/><link rel='self' type='application/atom+xml' href='https://www.blogger.com/feeds/3302710210518459416/posts/default/1478683141063029018'/><link rel='alternate' type='text/html' href='https://medical-phd.blogspot.com/2022/03/total-knee-arthroplasty-case-file.html' title='Total Knee Arthroplasty Case File'/><author><name>Medical Group</name><uri>http://www.blogger.com/profile/01780656800578397393</uri><email>noreply@blogger.com</email><gd:image rel='http://schemas.google.com/g/2005#thumbnail' width='32' height='32' src='//blogger.googleusercontent.com/img/b/R29vZ2xl/AVvXsEiMuROkBDv6yw4SmxGqRA5BPQhOXSwFld-Gufmy4kUqfpoAGQRl7cv-3naew6wjAnCjRFSe_FKX8z7CdE6cFKxxBpnqBuwUTSr-15xd3mFGcr_wJOX2y73G_zJCGND98Q/s220/Final-logo.jpg'/></author><media:thumbnail xmlns:media="http://search.yahoo.com/mrss/" url="https://blogger.googleusercontent.com/img/b/R29vZ2xl/AVvXsEj90Na5B280mvyktstkFH1p35EKF4w_jzrBQwu43XrVnwGNqWQOzRVFMIk0tFNye8TINEWgIIbpz5-iOCBCA5OCpvfUcnX3S26-VP9LoHxUlXGibDMb1W084LYXPkmgpC5MEd5SKxlIx-9k5NaPckFoEcsVwRbmjcuy097OWiSlgKQsUzRx4Qamffj9-Q/s72-w290-h640-c/AP-radiograph-of-the-right-knee.jpg" height="72" width="72"/><thr:total>0</thr:total><georss:featurename>United States</georss:featurename><georss:point>37.09024 -95.712891</georss:point><georss:box>8.780006163821156 -130.869141 65.400473836178847 -60.556641</georss:box></entry><entry><id>tag:blogger.com,1999:blog-3302710210518459416.post-2159250416480394026</id><published>2022-03-31T22:22:00.008-07:00</published><updated>2022-03-31T22:22:50.773-07:00</updated><category scheme="http://www.blogger.com/atom/ns#" term="Case Files"/><category scheme="http://www.blogger.com/atom/ns#" term="Orthopaedic Surgery Case File"/><title type='text'>Osteosarcoma Case File</title><content type='html'>&lt;div class=&quot;scrollbox&quot;&gt;
  32.  
  33. &lt;div style=&quot;text-align: left;&quot;&gt;&lt;b&gt;&lt;span style=&quot;font-size: large;&quot;&gt;Osteosarcoma Case File&lt;/span&gt;&lt;/b&gt;&lt;/div&gt;&lt;div&gt;Eugene C. Toy, MD, Andrew J. Rosenbaum, MD, Timothy T. Roberts, MD, Joshua S. Dines, MD&lt;/div&gt;&lt;div&gt;&lt;br /&gt;&lt;/div&gt;&lt;div&gt;&lt;b&gt;&lt;span style=&quot;color: #017087; font-size: medium;&quot;&gt;CASE 38&lt;/span&gt;&lt;/b&gt;&lt;/div&gt;&lt;div&gt;&lt;div&gt;A 19-year-old woman presents to the office complaining of dull right thigh pain for the past 2 months. She denies any trauma or inciting events over that time. The pain was initially present only while walking and after other physical activities but has progressed and is present at rest. It now disrupts her sleep at night. She is otherwise healthy and denies any fevers, chills, weight loss, or night sweats. On physical exam, her right lower extremity is neurovascularly intact. Her knee has full range of motion, and there is no warmth or erythema. However, you do note mild swelling of her distal thigh and on palpation appreciate a mass in that region. An anteroposterior (AP) radiograph of the distal femur is shown in Figure 38–1 .&lt;/div&gt;&lt;/div&gt;&lt;div&gt;&lt;br /&gt;&lt;/div&gt;&lt;div class=&quot;separator&quot; style=&quot;clear: both; text-align: center;&quot;&gt;&lt;a href=&quot;https://blogger.googleusercontent.com/img/b/R29vZ2xl/AVvXsEjfeOgM38nd88M3Qj0xx5nVYN74GgsYp6scv-TSV-6bYqZlSceowy-TYT6sjcg_cKLo7IXGWzL4jF4wuW0xVFLIrWDXZmyXDHnz3wp3DtsRMFkPrMntFN7blpMuBtk6c87gwcPu91i_OcghGSzZigxjdcy4SCVlikFIfCLNe0CZjF_p8CHSHX1nb650KQ/s445/AP-view-of-the-distal-femur.jpg&quot; style=&quot;margin-left: 1em; margin-right: 1em;&quot;&gt;&lt;img alt=&quot;AP view of the distal femur&quot; border=&quot;0&quot; data-original-height=&quot;445&quot; data-original-width=&quot;325&quot; height=&quot;640&quot; src=&quot;https://blogger.googleusercontent.com/img/b/R29vZ2xl/AVvXsEjfeOgM38nd88M3Qj0xx5nVYN74GgsYp6scv-TSV-6bYqZlSceowy-TYT6sjcg_cKLo7IXGWzL4jF4wuW0xVFLIrWDXZmyXDHnz3wp3DtsRMFkPrMntFN7blpMuBtk6c87gwcPu91i_OcghGSzZigxjdcy4SCVlikFIfCLNe0CZjF_p8CHSHX1nb650KQ/w468-h640/AP-view-of-the-distal-femur.jpg&quot; title=&quot;AP view of the distal femur&quot; width=&quot;468&quot; /&gt;&lt;/a&gt;&lt;/div&gt;&lt;div&gt;&lt;div&gt;&lt;b&gt;&lt;span style=&quot;color: #017087;&quot;&gt;Figure 38–1.&lt;/span&gt;&lt;/b&gt; AP view of the distal femur. Many of the radiographic features of this osteosarcoma&lt;/div&gt;&lt;div&gt;mark it as a malignant tumor. The abnormal area of mottled lucent and sclerotic tumor in the&lt;/div&gt;&lt;div&gt;metaphysis fades gradually into the shadows of surrounding normal bone. It is difficult to see where&lt;/div&gt;&lt;div&gt;the tumor begins and ends; there is a large soft-tissue mass adjacent to the bone (M). The periosteum&lt;/div&gt;&lt;div&gt;has been unable to maintain a shell of mineralized new bone around this mass. The sclerotic&lt;/div&gt;&lt;div&gt;areas within the bone and the mineralized portions of the soft-tissue mass both have a relatively&lt;/div&gt;&lt;div&gt;amorphous, smudged appearance that is seen with calcified osteoid matrix. (Reproduced, with&lt;/div&gt;&lt;div&gt;permission, from Chen MYM, Pope TL, Ott DJ. &lt;i&gt;Basic Radiology&lt;/i&gt;. 2nd ed. New York, NY: McGraw-Hill; 2011:Fig. 6-24.)&lt;/div&gt;&lt;/div&gt;&lt;div&gt;&lt;div&gt;&lt;br /&gt;&lt;/div&gt;&lt;div&gt;&lt;span style=&quot;color: #017087;&quot;&gt;►&lt;/span&gt;&amp;nbsp;What is the most likely diagnosis?&lt;/div&gt;&lt;div&gt;&lt;span style=&quot;color: #017087;&quot;&gt;►&lt;/span&gt;&amp;nbsp;What are the next steps in the workup?&lt;/div&gt;&lt;div&gt;&lt;span style=&quot;color: #017087;&quot;&gt;►&lt;/span&gt;&amp;nbsp;What is the treatment approach?&lt;/div&gt;&lt;/div&gt;&lt;div&gt;&lt;br /&gt;&lt;/div&gt;&lt;div&gt;&lt;br /&gt;&lt;/div&gt;&lt;div&gt;&lt;div&gt;&lt;b&gt;&lt;span style=&quot;font-size: medium;&quot;&gt;ANSWER TO CASE 38:&lt;/span&gt;&lt;/b&gt;&lt;/div&gt;&lt;div&gt;&lt;span style=&quot;background-color: #daebf0; font-size: medium;&quot;&gt;&lt;span&gt;&lt;span style=&quot;background-color: transparent;&quot;&gt;&lt;span style=&quot;color: #017087;&quot;&gt;&lt;b&gt;Osteosarcoma&lt;/b&gt;&lt;/span&gt;&lt;/span&gt;&lt;b&gt;&amp;nbsp;&amp;nbsp;&lt;/b&gt;&lt;/span&gt;&lt;span style=&quot;font-weight: bold;&quot;&gt;&amp;nbsp;&amp;nbsp; &amp;nbsp;&lt;/span&gt;&lt;span style=&quot;font-weight: bold;&quot;&gt;&amp;nbsp;&amp;nbsp; &amp;nbsp;&lt;/span&gt;&lt;span style=&quot;font-weight: bold;&quot;&gt;&amp;nbsp;&amp;nbsp; &amp;nbsp;&lt;/span&gt;&lt;span style=&quot;font-weight: bold;&quot;&gt;&amp;nbsp;&amp;nbsp; &amp;nbsp;&lt;/span&gt;&lt;span style=&quot;font-weight: bold;&quot;&gt;&amp;nbsp; &amp;nbsp;&lt;/span&gt;&lt;/span&gt;&lt;/div&gt;&lt;/div&gt;&lt;div&gt;&lt;br /&gt;&lt;/div&gt;&lt;div&gt;&lt;div&gt;&lt;i&gt;Summary&lt;/i&gt;: This is a 19-year-old otherwise healthy woman who presents with a 2-month history of dull right thigh pain that was initially activity dependent but has progressed and is now most severe at rest, especially at night. She denies any fevers, chills, night sweats, or recent weight loss. A mass in her distal thigh is palpated on examination. An AP plain radiograph illustrates a soft-tissue mass adjacent to a mottled lucent and sclerotic lesion in the metaphysis of her right femur.&lt;/div&gt;&lt;div&gt;&lt;ul style=&quot;text-align: left;&quot;&gt;&lt;li&gt;&lt;b&gt;Most likely diagnosis:&lt;/b&gt; Pathologic bone tumor, specifically osteosarcoma.&lt;/li&gt;&lt;li&gt;&lt;b&gt;Next steps in workup:&lt;/b&gt; Magnetic resonance imaging (MRI) of the right lower extremity, tissue biopsy, computed tomography (CT) scan of the chest, and a bone scan.&lt;/li&gt;&lt;li&gt;&lt;b&gt;Treatment approach:&lt;/b&gt; Multidisciplinary approach ultimately dependent on tumor grade and type of osteosarcoma.&lt;/li&gt;&lt;/ul&gt;&lt;/div&gt;&lt;/div&gt;&lt;div&gt;&lt;br /&gt;&lt;/div&gt;&lt;div&gt;&lt;br /&gt;&lt;/div&gt;&lt;div&gt;&lt;div&gt;&lt;b&gt;&lt;span style=&quot;color: #017087; font-size: medium;&quot;&gt;ANALYSIS&lt;/span&gt;&lt;/b&gt;&lt;/div&gt;&lt;div&gt;&lt;b&gt;&lt;span style=&quot;font-size: medium;&quot;&gt;&lt;i&gt;Objectives&lt;/i&gt;&lt;/span&gt;&lt;/b&gt;&lt;/div&gt;&lt;/div&gt;&lt;div&gt;&lt;div&gt;&lt;ol style=&quot;text-align: left;&quot;&gt;&lt;li&gt;Understand the important history, physical exam, and radiographic findings consistent with a diagnosis of osteosarcoma.&lt;/li&gt;&lt;li&gt;Know the essential components of the workup for a musculoskeletal lesion, specifically an osteosarcoma.&lt;/li&gt;&lt;li&gt;Appreciate the need for a multidisciplinary approach to treatment and how the Musculoskeletal Tumor Society Staging System (MSTS, Enneking) influences it.&lt;/li&gt;&lt;/ol&gt;&lt;/div&gt;&lt;/div&gt;&lt;div&gt;&lt;br /&gt;&lt;/div&gt;&lt;div&gt;&lt;br /&gt;&lt;/div&gt;&lt;div&gt;&lt;div&gt;&lt;b&gt;&lt;i&gt;&lt;span style=&quot;font-size: medium;&quot;&gt;Considerations&lt;/span&gt;&lt;/i&gt;&lt;/b&gt;&lt;/div&gt;&lt;div&gt;&lt;br /&gt;&lt;/div&gt;&lt;div&gt;There are many important clinical, radiographic, and pathologic findings that will aid the orthopaedist in establishing the proper diagnosis in the setting of a likely musculoskeletal tumor, as in this case. Diagnosis begins with the history and physical exam. Concerning findings in this patient include her progression of pain symptoms, to the point where it awakens her from sleep, and the presence of a palpable mass. Age is another important consideration, especially for establishing an appropriate differential diagnosis. Furthermore, some tumors have a sex predilection; osteosarcoma is more common in males. Laboratory tests are not typically diagnostic for bone tumors, but may be helpful once a diagnosis is made. Imaging must begin with at least 2 plain radiographic views of the lesion. An MRI is typically obtained after and has become the imaging modality of choice for most musculoskeletal tumors. Nuclear imaging and CT are also performed for evaluation of metastases, staging, and surveillance. Biopsy of the lesion allows for histologic analysis, which determines tumor grade and significantly impacts treatment options and ultimate prognosis. After this workup, a multidisciplinary treatment plan can be developed in conjunction with both the oncologist and orthopaedic surgeon.&lt;/div&gt;&lt;/div&gt;&lt;div&gt;&lt;br /&gt;&lt;/div&gt;&lt;div&gt;&lt;br /&gt;&lt;/div&gt;&lt;div&gt;&lt;div&gt;&lt;div&gt;&lt;b&gt;&lt;span style=&quot;font-size: medium;&quot;&gt;APPROACH TO:&lt;/span&gt;&lt;/b&gt;&lt;/div&gt;&lt;div&gt;&lt;span style=&quot;background-color: #daebf0; color: #017087; font-size: medium;&quot;&gt;&lt;span&gt;&lt;span style=&quot;background-color: transparent;&quot;&gt;&lt;b&gt;Osteosarcoma&lt;/b&gt;&lt;/span&gt;&lt;b&gt;&amp;nbsp; &amp;nbsp;&amp;nbsp;&lt;/b&gt;&lt;/span&gt;&lt;span style=&quot;font-weight: bold;&quot;&gt;&amp;nbsp;&amp;nbsp; &amp;nbsp;&lt;/span&gt;&lt;span style=&quot;font-weight: bold;&quot;&gt;&amp;nbsp;&amp;nbsp; &amp;nbsp;&lt;/span&gt;&lt;span style=&quot;font-weight: bold;&quot;&gt;&amp;nbsp;&amp;nbsp; &amp;nbsp;&lt;/span&gt;&lt;span style=&quot;font-weight: bold;&quot;&gt;&amp;nbsp;&amp;nbsp; &amp;nbsp;&lt;/span&gt;&lt;span style=&quot;font-weight: bold;&quot;&gt;&amp;nbsp;&amp;nbsp; &amp;nbsp;&lt;/span&gt;&lt;span style=&quot;font-weight: bold;&quot;&gt;&amp;nbsp;&amp;nbsp; &amp;nbsp;&lt;/span&gt;&lt;span style=&quot;font-weight: bold;&quot;&gt;&amp;nbsp;&amp;nbsp;&amp;nbsp;&lt;/span&gt;&lt;span style=&quot;font-weight: bold;&quot;&gt;&amp;nbsp;&lt;/span&gt;&lt;/span&gt;&lt;/div&gt;&lt;/div&gt;&lt;div&gt;&lt;br /&gt;&lt;/div&gt;&lt;div&gt;&lt;b&gt;&lt;span style=&quot;color: #017087; font-size: medium;&quot;&gt;DEFINITIONS&lt;/span&gt;&lt;/b&gt;&lt;/div&gt;&lt;/div&gt;&lt;div&gt;&lt;br /&gt;&lt;/div&gt;&lt;div&gt;&lt;div&gt;&lt;b&gt;CODMAN TRIANGLE:&lt;/b&gt; A radiographic finding of new subperiosteal bone created when a lesion elevates the periosteum away from bone. Although it can appear triangular, it is often a pseudotriangle on plain radiographs with a 2-sided appearance owing to tumor growing at a faster rate than the periosteum can expand.&lt;/div&gt;&lt;div&gt;&lt;b&gt;&lt;br /&gt;&lt;/b&gt;&lt;/div&gt;&lt;div&gt;&lt;b&gt;MSTS (ENNEKING) STAGING SYSTEM:&lt;/b&gt; Most recognized staging system for malignant bone tumors. It is based on tumor grade (low, high), extent (intracompartmental, extracompartmental), and the presence or absence of metastases. In general, low-grade, localized tumors are stage I; high-grade, localized tumors are stage II; and metastatic tumors (regardless of grade) are stage III. Each stage is further categorized into type a and b subsets, representative of intra- versus extracompartmental disease, respectively.&lt;/div&gt;&lt;div&gt;&lt;b&gt;&lt;br /&gt;&lt;/b&gt;&lt;/div&gt;&lt;div&gt;&lt;b&gt;LIMB-SALVAGE SURGERY:&lt;/b&gt; A constellation of surgical procedures designed to achieve removal of a malignant tumor and reconstruction of a limb with an acceptable oncologic, functional, and cosmetic result.&lt;/div&gt;&lt;/div&gt;&lt;div&gt;&lt;br /&gt;&lt;/div&gt;&lt;div&gt;&lt;br /&gt;&lt;/div&gt;&lt;div&gt;&lt;b&gt;&lt;span style=&quot;color: #017087; font-size: medium;&quot;&gt;CLINICAL APPROACH&lt;/span&gt;&lt;/b&gt;&lt;/div&gt;&lt;div&gt;&lt;br /&gt;&lt;/div&gt;&lt;div&gt;&lt;div&gt;Osteosarcoma is the most common bone sarcoma, affecting approximately 560 children and adolescents annually in the United States. Its incidence peaks in the second decade of life and is most commonly seen at sites of rapid bone turnover, such as the distal femur, proximal tibia, proximal humerus, and proximal femur. Although most cases of osteosarcoma are sporadic, those that carry the &lt;b&gt;retinoblastoma&amp;nbsp;&lt;/b&gt;&lt;b&gt;tumor suppressor gene (&lt;i&gt;RB1&lt;/i&gt;)&lt;/b&gt; or who have &lt;b&gt;Li-Fraumeni syndrome&amp;nbsp;&lt;/b&gt;&lt;b&gt;(p53 gene)&lt;/b&gt; are predisposed.&lt;/div&gt;&lt;/div&gt;&lt;div&gt;&lt;br /&gt;&lt;/div&gt;&lt;div&gt;&lt;div&gt;&lt;span&gt;&amp;nbsp;&amp;nbsp; &amp;nbsp;&lt;/span&gt;There are multiple subtypes of osteosarcoma, including low-grade, high-grade, and telangiectatic intramedullary variants, as well as parosteal, periosteal, and highgrade surface types. This discussion pertains to the classic, most common subtype, which accounts for approximately 80% of all osteosarcomas, the high-grade intramedullary osteosarcoma.&lt;/div&gt;&lt;/div&gt;&lt;div&gt;&lt;br /&gt;&lt;/div&gt;&lt;div&gt;&lt;div&gt;&lt;span&gt;&amp;nbsp;&amp;nbsp; &amp;nbsp;&lt;/span&gt;The history and physical exam are crucial in the initial diagnosis of osteosarcoma. A common presenting complaint is new-onset pain over several months duration; it may occur at rest, disrupt sleep, or only occur after physical activity. This complaint must be investigated with due diligence by the clinician. Pain associated with weightbearing or activity is typically due to an inflammatory process. However, pain at rest or at night is biologic pain possibly secondary to the growth of a lesion in bone. A patient with an osteosarcoma may also present with a palpable mass in the extremity that is painful. Symptoms such as fevers, chills, weight loss, and fatigue are late signs of disease.&lt;/div&gt;&lt;/div&gt;&lt;div&gt;&lt;br /&gt;&lt;/div&gt;&lt;div&gt;&lt;div&gt;&lt;span&gt;&amp;nbsp;&amp;nbsp; &amp;nbsp;&lt;/span&gt;After a physical exam, plain radiographs of the region in question must be obtained. An osteosarcoma classically arises from the metaphyseal region of a bone and appears as a &lt;b&gt;mixed sclerotic and lytic lesion&lt;/b&gt; that can infiltrate bone and surrounding cortex, causing a periosteal reaction and soft tissue mass. A&lt;b&gt; Codman&amp;nbsp;&lt;/b&gt;&lt;b&gt;triangle&lt;/b&gt; may be seen on radiographs ( Figure 38–2 ). Plain radiographs concerning for a malignant bone lesion require advanced imaging, such as an MRI, to evaluate for surrounding soft tissue, neurovascular, and bone marrow involvement, as well as skip lesions, which represent discontinuous metastases.&lt;/div&gt;&lt;/div&gt;&lt;div&gt;&lt;br /&gt;&lt;/div&gt;&lt;div class=&quot;separator&quot; style=&quot;clear: both; text-align: center;&quot;&gt;&lt;a href=&quot;https://blogger.googleusercontent.com/img/b/R29vZ2xl/AVvXsEiTGigskYV9XTyYHEG_Vy61pSfq2H3dUerOvGZ7UNZ9ei6pQJCKGgEp1tB7or9DQoqegVmJ67N_AmfygU9R6FAukpT5RuSLpri7So6Ziu83QYaTSR8tia-8Vj97_lb7_MN2VCA48s4lnSMtq6NU1s69mGj7dQFcrTb6Ug9gqNuXYaSsiFc0zHQ6TgBjQA/s624/Osteosarcoma-with-Codman-triangle.jpg&quot; style=&quot;margin-left: 1em; margin-right: 1em;&quot;&gt;&lt;img alt=&quot;Osteosarcoma with Codman triangle&quot; border=&quot;0&quot; data-original-height=&quot;624&quot; data-original-width=&quot;296&quot; height=&quot;640&quot; src=&quot;https://blogger.googleusercontent.com/img/b/R29vZ2xl/AVvXsEiTGigskYV9XTyYHEG_Vy61pSfq2H3dUerOvGZ7UNZ9ei6pQJCKGgEp1tB7or9DQoqegVmJ67N_AmfygU9R6FAukpT5RuSLpri7So6Ziu83QYaTSR8tia-8Vj97_lb7_MN2VCA48s4lnSMtq6NU1s69mGj7dQFcrTb6Ug9gqNuXYaSsiFc0zHQ6TgBjQA/w304-h640/Osteosarcoma-with-Codman-triangle.jpg&quot; title=&quot;Osteosarcoma with Codman triangle&quot; width=&quot;304&quot; /&gt;&lt;/a&gt;&lt;/div&gt;&lt;div&gt;&lt;div&gt;&lt;b&gt;&lt;span style=&quot;color: #017087;&quot;&gt;Figure 38–2.&lt;/span&gt;&lt;/b&gt; Osteosarcoma with Codman triangle, a new area of subperiosteal bone formed when a&lt;/div&gt;&lt;div&gt;tumor raises the periosteum away from the bone. (Reproduced, with permission, from Doherty GM.&lt;/div&gt;&lt;div&gt;&lt;i&gt;Current Diagnosis &amp;amp; Treatment: Surgery&lt;/i&gt;. 13th ed. New York, NY: McGraw-Hill; 2010:Fig. 40-38.)&lt;/div&gt;&lt;/div&gt;&lt;div&gt;&lt;br /&gt;&lt;/div&gt;&lt;div&gt;&lt;div&gt;&lt;span&gt;&amp;nbsp;&amp;nbsp; &amp;nbsp;&lt;/span&gt;A biopsy of the lesion is required when the imaging, examination, and history are consistent with a pathologic process; it is the only definitive means of determining whether a tumor is benign or malignant. An orthopaedic oncologist must perform the biopsy, which, if improperly obtained, can compromise the ability to perform limb salvage procedures. Definitive osteosarcoma resection must include the biopsy tract, as it too can be contaminated with tumor cells. In the setting of osteosarcoma, histologic analysis often shows &lt;b&gt;diffuse cellular atypia with nuclear pleomorphism&amp;nbsp;&lt;/b&gt;&lt;b&gt;and “lacey” osteoid matrix production.&lt;/b&gt;&lt;/div&gt;&lt;/div&gt;&lt;div&gt;&lt;br /&gt;&lt;/div&gt;&lt;div&gt;&lt;div&gt;&lt;span&gt;&amp;nbsp;&amp;nbsp; &amp;nbsp;&lt;/span&gt;Laboratory testing is not diagnostic for osteosarcoma, but does play a role in its workup and management. Before initiation of chemotherapy, laboratory values (ie, complete blood count with differential, basic metabolic profile, renal and liver function values) must be obtained to provide a baseline assessment of organ function. Additionally, elevated alkaline phosphatase (up to 2-3 times the normal value) and lactate dehydrogenase levels are considered poor prognostic factors.&lt;/div&gt;&lt;/div&gt;&lt;div&gt;&lt;br /&gt;&lt;/div&gt;&lt;div&gt;&lt;div&gt;&lt;span&gt;&amp;nbsp;&amp;nbsp; &amp;nbsp;&lt;/span&gt;Chest CT and bone scan evaluate for metastatic disease, which is the most important prognostic and staging factor in osteosarcoma. In approximately 10% of patients, pulmonary or lymph node metastases are detected at presentation. It is not uncommon for osteosarcoma to have early hematogenous metastases. Micrometastases, which are undetectable with conventional imaging, are likely present in the lungs in most patients on initial presentation and without appropriate treatment will become evident on imaging within 1 to 2 years. A bone scan will be “hot” in osteosarcoma, specifically in areas of tumor. Those areas of the body showing increased metabolic activity must be further evaluated with imaging.&lt;/div&gt;&lt;/div&gt;&lt;div&gt;&lt;br /&gt;&lt;/div&gt;&lt;div&gt;&lt;div&gt;&lt;b&gt;&lt;span&gt;&amp;nbsp;&amp;nbsp; &amp;nbsp;&lt;/span&gt;The majority of patients with osteosarcoma present as having Enneking stage&amp;nbsp;&lt;/b&gt;&lt;b&gt;IIB disease at diagnosis.&lt;/b&gt; For most types, treatment is multimodal. Two to three cycles of neoadjuvant chemotherapy are commonly given before wide-margin surgical resection is performed, which involves achieving the largest possible margin without unnecessary functional sacrifice. &lt;b&gt;Fortunately, approximately 90% of patients with&amp;nbsp;&lt;/b&gt;&lt;b&gt;osteosarcoma are able to undergo limb salvage procedures and do not require&amp;nbsp;&lt;/b&gt;&lt;b&gt;amputation.&lt;/b&gt; The excised segment of bone may be replaced with a prosthesis and/or allograft. Postoperative chemotherapy is also typically used. Radiation is not used in the treatment of osteosarcoma.&lt;/div&gt;&lt;/div&gt;&lt;div&gt;&lt;br /&gt;&lt;/div&gt;&lt;div&gt;&lt;div&gt;&lt;span&gt;&amp;nbsp;&amp;nbsp; &amp;nbsp;&lt;/span&gt;The overall 5-year survival rates of patients with osteosarcoma are 60% to 80%. With the initiation of neoadjuvant chemotherapy protocols, 5-year survival rates are almost 80% in patients with nonmetastatic disease and 10% to 20% in patients with metastatic disease.&lt;b&gt; In addition to elevated alkaline phosphatase and lactate&amp;nbsp;&lt;/b&gt;&lt;b&gt;dehydrogenase, poor prognostic factors include metastatic disease at diagnosis, a&amp;nbsp;&lt;/b&gt;&lt;b&gt;poor response to chemotherapy (defined as &amp;lt; 90% necrosis of resected tumor),&amp;nbsp;&lt;/b&gt;&lt;b&gt;and pathologic fracture.&lt;/b&gt;&lt;/div&gt;&lt;/div&gt;&lt;div&gt;&lt;br /&gt;&lt;/div&gt;&lt;div&gt;&lt;br /&gt;&lt;/div&gt;&lt;div&gt;&lt;b&gt;&lt;span style=&quot;color: #017087; font-size: medium;&quot;&gt;&lt;u&gt;COMPREHENSION QUESTIONS&lt;/u&gt;&lt;/span&gt;&lt;/b&gt;&lt;/div&gt;&lt;div&gt;&lt;br /&gt;&lt;/div&gt;&lt;div&gt;&lt;div&gt;38.1 Which of the following most accurately lists the most common location of osteosarcoma in increasing frequency?&lt;/div&gt;&lt;/div&gt;&lt;blockquote style=&quot;border: none; margin: 0px 0px 0px 40px; padding: 0px; text-align: left;&quot;&gt;&lt;div&gt;&lt;div&gt;A. Proximal tibia &amp;lt; axial skeleton &amp;lt; proximal humerus &amp;lt; proximal femur&lt;/div&gt;&lt;/div&gt;&lt;div&gt;&lt;div&gt;B. Axial skeleton &amp;lt; proximal humerus &amp;lt; proximal femur &amp;lt; proximal tibia&lt;/div&gt;&lt;/div&gt;&lt;div&gt;&lt;div&gt;C. Axial skeleton &amp;lt; proximal humerus &amp;lt; proximal tibia &amp;lt; proximal femur&lt;/div&gt;&lt;/div&gt;&lt;div&gt;&lt;div&gt;D. Proximal humerus &amp;lt; axial skeleton &amp;lt; proximal tibia &amp;lt; proximal femur&lt;/div&gt;&lt;/div&gt;&lt;/blockquote&gt;&lt;div&gt;&lt;div&gt;&lt;br /&gt;&lt;/div&gt;&lt;div&gt;38.2 In addition to metastases, which of the following is also a poor prognostic indicator in the setting of osteosarcoma?&lt;/div&gt;&lt;/div&gt;&lt;blockquote style=&quot;border: none; margin: 0px 0px 0px 40px; padding: 0px; text-align: left;&quot;&gt;&lt;div&gt;&lt;div&gt;A. High alkaline phosphatase&lt;/div&gt;&lt;/div&gt;&lt;div&gt;&lt;div&gt;B. Pain&lt;/div&gt;&lt;/div&gt;&lt;div&gt;&lt;div&gt;C. Low lactate dehydrogenase&lt;/div&gt;&lt;/div&gt;&lt;div&gt;&lt;div&gt;D. A and B&lt;/div&gt;&lt;/div&gt;&lt;/blockquote&gt;&lt;div&gt;&lt;br /&gt;&lt;/div&gt;&lt;div&gt;&lt;div&gt;38.3 A 14-year-old male presents with an 8-month history of increasing right knee pain and swelling. He is otherwise healthy, but is a known carrier of the retinoblastoma gene. A mass is found on the posterior aspect of his knee, and a biopsy is taken. If found to be an osteosarcoma, what would histologic analysis of the specimen likely show?&lt;/div&gt;&lt;/div&gt;&lt;blockquote style=&quot;border: none; margin: 0px 0px 0px 40px; padding: 0px; text-align: left;&quot;&gt;&lt;div&gt;&lt;div&gt;A. Densely packed uniform small blue-stained cells in sheets&lt;/div&gt;&lt;/div&gt;&lt;div&gt;&lt;div&gt;B. Numerous multinucleated giant cells&lt;/div&gt;&lt;/div&gt;&lt;div&gt;&lt;div&gt;C. Diffuse cellular atypia with nuclear pleomorphism and “lacey” osteoid matrix production&lt;/div&gt;&lt;/div&gt;&lt;div&gt;&lt;div&gt;D. Cystic spaces filled with blood&lt;/div&gt;&lt;/div&gt;&lt;/blockquote&gt;&lt;div&gt;&lt;br /&gt;&lt;/div&gt;&lt;div&gt;&lt;br /&gt;&lt;/div&gt;&lt;div&gt;&lt;b&gt;&lt;span style=&quot;color: #017087; font-size: medium;&quot;&gt;&lt;u&gt;ANSWERS&lt;/u&gt;&lt;/span&gt;&lt;/b&gt;&lt;/div&gt;&lt;div&gt;&lt;br /&gt;&lt;/div&gt;&lt;div&gt;&lt;div&gt;38.1 &lt;b&gt;C.&lt;/b&gt; The distal femur is the most common location of osteosarcoma. This is followed by the proximal tibia, proximal humerus, and then the axial skeleton (ie, spine and sacrum).&lt;/div&gt;&lt;div&gt;&lt;br /&gt;&lt;/div&gt;&lt;div&gt;38.2 &lt;b&gt;A.&lt;/b&gt; Poor prognostic factors include elevated alkaline phosphatase and lactate dehydrogenase, metastatic disease at diagnosis, a poor response to chemotherapy (defined as &amp;lt; 90% necrosis of resected tumor), and pathologic fracture.&lt;/div&gt;&lt;div&gt;&lt;br /&gt;&lt;/div&gt;&lt;div&gt;38.3 &lt;b&gt;C.&lt;/b&gt; Histologic analysis of osteosarcoma often reveals diffuse cellular atypia with nuclear pleomorphism and “lacey” osteoid matrix production. Other malignant characteristics include stromal cells with atypia, a high nuclear to cytoplasmic ratio, and abnormal mitotic figures. A describes a Ewing sarcoma, B a giant cell tumor, and D an aneurysmal bone cyst.&lt;/div&gt;&lt;/div&gt;&lt;div&gt;&lt;br /&gt;&lt;/div&gt;&lt;div&gt;&lt;b style=&quot;background-color: #4093a5;&quot;&gt;&amp;nbsp; &amp;nbsp;&lt;span style=&quot;font-size: medium;&quot;&gt;&amp;nbsp;&lt;/span&gt;&lt;span style=&quot;color: white;&quot;&gt;&lt;span style=&quot;font-size: medium;&quot;&gt;CLINICAL PEARLS&lt;/span&gt;&lt;span&gt;&amp;nbsp; &amp;nbsp;&amp;nbsp;&lt;/span&gt;&lt;/span&gt;&lt;/b&gt;&lt;/div&gt;&lt;div&gt;&lt;table border=&quot;1&quot; cellpadding=&quot;0&quot; cellspacing=&quot;0&quot; class=&quot;MsoTableGrid&quot; style=&quot;border-collapse: collapse; border: none; mso-border-alt: solid windowtext .5pt; mso-padding-alt: 0in 5.4pt 0in 5.4pt; mso-yfti-tbllook: 1184;&quot;&gt;
  34. &lt;tbody&gt;&lt;tr style=&quot;height: 19.15pt; mso-yfti-firstrow: yes; mso-yfti-irow: 0; mso-yfti-lastrow: yes;&quot;&gt;
  35.  &lt;td style=&quot;background: rgb(221, 237, 239); border-left: solid #017087 1.0pt; border: 1pt solid rgb(64, 147, 165); height: 19.15pt; padding: 0in 5.4pt; width: 433.7pt;&quot; valign=&quot;top&quot; width=&quot;578&quot;&gt;&lt;div style=&quot;line-height: normal; margin-bottom: 0in;&quot;&gt;&lt;span style=&quot;color: #017087;&quot;&gt;&lt;br /&gt;&lt;/span&gt;&lt;/div&gt;&lt;div style=&quot;line-height: normal; margin-bottom: 0in;&quot;&gt;&lt;span style=&quot;color: #017087;&quot;&gt;►&amp;nbsp;&lt;/span&gt;The history and physical exam are crucial in the initial diagnosis of osteosarcoma.&lt;/div&gt;&lt;div style=&quot;line-height: normal; margin-bottom: 0in;&quot;&gt;&lt;span style=&quot;color: #017087;&quot;&gt;&lt;br /&gt;&lt;/span&gt;&lt;/div&gt;&lt;div style=&quot;line-height: normal; margin-bottom: 0in;&quot;&gt;&lt;span style=&quot;color: #017087;&quot;&gt;►&lt;/span&gt;&amp;nbsp;MRI is effective at evaluating an osteosarcoma because it can clearly identify surrounding soft tissue, neurovascular and bone marrow involvement, and skip lesions, which represent discontinuous metastases.&lt;/div&gt;&lt;div style=&quot;line-height: normal; margin-bottom: 0in;&quot;&gt;&lt;span style=&quot;color: #017087;&quot;&gt;&lt;br /&gt;&lt;/span&gt;&lt;/div&gt;&lt;div style=&quot;line-height: normal; margin-bottom: 0in;&quot;&gt;&lt;span style=&quot;color: #017087;&quot;&gt;►&lt;/span&gt;&amp;nbsp;Chest CT and bone scan are used to evaluate for metastatic disease.&lt;/div&gt;&lt;div style=&quot;line-height: normal; margin-bottom: 0in;&quot;&gt;&lt;span style=&quot;color: #017087;&quot;&gt;&lt;br /&gt;&lt;/span&gt;&lt;/div&gt;&lt;div style=&quot;line-height: normal; margin-bottom: 0in;&quot;&gt;&lt;span style=&quot;color: #017087;&quot;&gt;►&lt;/span&gt;&amp;nbsp;The majority of patients with osteosarcoma present with &lt;b&gt;Enneking&lt;/b&gt; stage IIB disease at diagnosis.&lt;/div&gt;&lt;div style=&quot;line-height: normal; margin-bottom: 0in;&quot;&gt;&lt;span style=&quot;color: #017087;&quot;&gt;&lt;br /&gt;&lt;/span&gt;&lt;/div&gt;&lt;div style=&quot;line-height: normal; margin-bottom: 0in;&quot;&gt;&lt;span style=&quot;color: #017087;&quot;&gt;►&lt;/span&gt;&amp;nbsp;Treatment commonly involves neoadjuvant chemotherapy, limb-salvage surgery, and postoperative chemotherapy.&lt;/div&gt;&lt;div style=&quot;line-height: normal; margin-bottom: 0in;&quot;&gt;&lt;span style=&quot;color: #017087;&quot;&gt;&lt;br /&gt;&lt;/span&gt;&lt;/div&gt;&lt;div style=&quot;line-height: normal; margin-bottom: 0in;&quot;&gt;&lt;span style=&quot;color: #017087;&quot;&gt;►&lt;/span&gt;&amp;nbsp;The overall 5-year survival rates of patients with osteosarcoma are 60% to 80%.&lt;/div&gt;&lt;div style=&quot;line-height: normal; margin-bottom: 0in;&quot;&gt;&lt;br /&gt;&lt;/div&gt;
  36.  &lt;/td&gt;
  37. &lt;/tr&gt;
  38. &lt;/tbody&gt;&lt;/table&gt;&lt;br /&gt;&lt;/div&gt;
  39.  
  40. &lt;details close=&quot;&quot;&gt;
  41.  &lt;summary&gt;&lt;b&gt;&lt;span style=&quot;color: #00788b;&quot;&gt;REFERENCES&lt;/span&gt;&lt;/b&gt;&lt;/summary&gt;
  42.  &lt;p&gt;Leet, Arabella I. Chapter 64: Pediatric tumors and hematologic diseases. In: Aiona MD. OKU 10:
  43. Orthopaedic Knowledge Update . Rosemont, IL: American Academy of Orthopaedic Surgeons;
  44. 2011:827.&amp;nbsp;&lt;/p&gt;&lt;p&gt;Messerschmitt PJ, Garcia RM, Abdul-Karim FW, Greenfield EM, Getty PJ. Osteosarcoma. J Am Acad
  45. Orthop Surg. 2009;17:515-527.&lt;/p&gt;
  46. &lt;/details&gt;&lt;/div&gt;</content><link rel='replies' type='application/atom+xml' href='https://medical-phd.blogspot.com/feeds/2159250416480394026/comments/default' title='Post Comments'/><link rel='replies' type='text/html' href='https://medical-phd.blogspot.com/2022/03/osteosarcoma-case-file.html#comment-form' title='0 Comments'/><link rel='edit' type='application/atom+xml' href='https://www.blogger.com/feeds/3302710210518459416/posts/default/2159250416480394026'/><link rel='self' type='application/atom+xml' href='https://www.blogger.com/feeds/3302710210518459416/posts/default/2159250416480394026'/><link rel='alternate' type='text/html' href='https://medical-phd.blogspot.com/2022/03/osteosarcoma-case-file.html' title='Osteosarcoma Case File'/><author><name>Medical Group</name><uri>http://www.blogger.com/profile/01780656800578397393</uri><email>noreply@blogger.com</email><gd:image rel='http://schemas.google.com/g/2005#thumbnail' width='32' height='32' src='//blogger.googleusercontent.com/img/b/R29vZ2xl/AVvXsEiMuROkBDv6yw4SmxGqRA5BPQhOXSwFld-Gufmy4kUqfpoAGQRl7cv-3naew6wjAnCjRFSe_FKX8z7CdE6cFKxxBpnqBuwUTSr-15xd3mFGcr_wJOX2y73G_zJCGND98Q/s220/Final-logo.jpg'/></author><media:thumbnail xmlns:media="http://search.yahoo.com/mrss/" url="https://blogger.googleusercontent.com/img/b/R29vZ2xl/AVvXsEjfeOgM38nd88M3Qj0xx5nVYN74GgsYp6scv-TSV-6bYqZlSceowy-TYT6sjcg_cKLo7IXGWzL4jF4wuW0xVFLIrWDXZmyXDHnz3wp3DtsRMFkPrMntFN7blpMuBtk6c87gwcPu91i_OcghGSzZigxjdcy4SCVlikFIfCLNe0CZjF_p8CHSHX1nb650KQ/s72-w468-h640-c/AP-view-of-the-distal-femur.jpg" height="72" width="72"/><thr:total>0</thr:total><georss:featurename>United States</georss:featurename><georss:point>37.09024 -95.712891</georss:point><georss:box>8.780006163821156 -130.869141 65.400473836178847 -60.556641</georss:box></entry><entry><id>tag:blogger.com,1999:blog-3302710210518459416.post-6239165778581713238</id><published>2022-03-31T20:54:00.000-07:00</published><updated>2022-03-31T20:54:04.339-07:00</updated><category scheme="http://www.blogger.com/atom/ns#" term="Case Files"/><category scheme="http://www.blogger.com/atom/ns#" term="Orthopaedic Surgery Case File"/><title type='text'>Calcaneus Fracture Case File</title><content type='html'>&lt;div class=&quot;scrollbox&quot;&gt;
  47.  
  48. &lt;div style=&quot;text-align: left;&quot;&gt;&lt;b&gt;&lt;span style=&quot;font-size: large;&quot;&gt;Calcaneus Fracture Case File&lt;/span&gt;&lt;/b&gt;&lt;/div&gt;&lt;div&gt;Eugene C. Toy, MD, Andrew J. Rosenbaum, MD, Timothy T. Roberts, MD, Joshua S. Dines, MD&lt;/div&gt;&lt;div&gt;&lt;br /&gt;&lt;/div&gt;&lt;div&gt;&lt;b&gt;&lt;span style=&quot;color: #017087; font-size: medium;&quot;&gt;CASE 37&lt;/span&gt;&lt;/b&gt;&lt;/div&gt;&lt;div&gt;&lt;div&gt;A 51-year-old man presents to the emergency department (ED) after falling 6 feet from a ladder onto his left foot while moving heavy boxes. The patient has no past medical history, works as a self-employed engineer, and is a nonsmoker. On initial presentation, he complains of isolated left foot and heel pain. He states he cannot bear any weight on his affected limb. Physical exam of the foot reveals no open wounds, marked soft tissue swelling, ecchymosis, and tenderness to palpation. Sensation to light touch is grossly preserved, but his ankle and foot range of motion is significantly limited by pain. Pedal pulses are palpable and symmetric to the contralateral side. Initial radiographic evaluation with anteroposterior (AP), lateral, and Harris views of the foot are shown in Figures 37–1 , 37–2 , and 37–3 .&lt;/div&gt;&lt;/div&gt;&lt;div&gt;&lt;br /&gt;&lt;/div&gt;&lt;div class=&quot;separator&quot; style=&quot;clear: both; text-align: center;&quot;&gt;&lt;a href=&quot;https://blogger.googleusercontent.com/img/b/R29vZ2xl/AVvXsEhWehiYZutupAtL7gScBdAneO_gw5WJ-mPsd47hEZ_ETKGWd8r_yAAZwwDfFZXLdoZ4Of_yGkrImomoJgVd4sNV4_LuUf4xot5GHWjzSkC62XY2W_6gVh1oKE0bzXsSAWH_SWhoscjyJgcpJuL580XUdg3NsR_5fWGoC_tUiCYhZ1D18PwOx-b2Jcl-ag/s500/AP-view-of-the-foot.jpg&quot; style=&quot;margin-left: 1em; margin-right: 1em;&quot;&gt;&lt;img alt=&quot;AP view of the foot&quot; border=&quot;0&quot; data-original-height=&quot;500&quot; data-original-width=&quot;292&quot; height=&quot;400&quot; src=&quot;https://blogger.googleusercontent.com/img/b/R29vZ2xl/AVvXsEhWehiYZutupAtL7gScBdAneO_gw5WJ-mPsd47hEZ_ETKGWd8r_yAAZwwDfFZXLdoZ4Of_yGkrImomoJgVd4sNV4_LuUf4xot5GHWjzSkC62XY2W_6gVh1oKE0bzXsSAWH_SWhoscjyJgcpJuL580XUdg3NsR_5fWGoC_tUiCYhZ1D18PwOx-b2Jcl-ag/w234-h400/AP-view-of-the-foot.jpg&quot; title=&quot;AP view of the foot&quot; width=&quot;234&quot; /&gt;&lt;/a&gt;&lt;/div&gt;&lt;div&gt;&lt;b&gt;&lt;span style=&quot;color: #017087;&quot;&gt;Figure 37–1.&lt;/span&gt;&lt;/b&gt; AP view of the foot.&lt;/div&gt;&lt;div&gt;&lt;br /&gt;&lt;/div&gt;&lt;div&gt;&lt;div&gt;&lt;span style=&quot;color: #017087;&quot;&gt;►&lt;/span&gt;&amp;nbsp;What is the most likely diagnosis?&lt;/div&gt;&lt;div&gt;&lt;span style=&quot;color: #017087;&quot;&gt;►&lt;/span&gt;&amp;nbsp;What additional studies should be obtained?&lt;/div&gt;&lt;div&gt;&lt;span style=&quot;color: #017087;&quot;&gt;►&lt;/span&gt;&amp;nbsp;What are the treatment options?&lt;/div&gt;&lt;div&gt;&lt;span style=&quot;color: #017087;&quot;&gt;►&lt;/span&gt;&amp;nbsp;What are the potential complications?&lt;/div&gt;&lt;/div&gt;&lt;div&gt;&lt;br /&gt;&lt;/div&gt;&lt;div class=&quot;separator&quot; style=&quot;clear: both; text-align: center;&quot;&gt;&lt;a href=&quot;https://blogger.googleusercontent.com/img/b/R29vZ2xl/AVvXsEj5--d2uTv1ZWJKtsiQARkvk-dCbE0vh5PTkQR8QC7nE-5SjvwIbx8mnrjzZSYez8QVcm_X_AUqAfqSloGgFwAMVrKfsSYRkQNLUFlg4oDc7OZIQQQHFtLpf0yAEz1rq6M0LgRaa1UqPTzej2Vkaq25K0xA-XhAc_G81PzEkexVBhAAEV-b692ktr7i1Q/s500/Lateral-view-of-the-foot.jpg&quot; style=&quot;margin-left: 1em; margin-right: 1em;&quot;&gt;&lt;img alt=&quot;Lateral view of the foot&quot; border=&quot;0&quot; data-original-height=&quot;301&quot; data-original-width=&quot;500&quot; height=&quot;241&quot; src=&quot;https://blogger.googleusercontent.com/img/b/R29vZ2xl/AVvXsEj5--d2uTv1ZWJKtsiQARkvk-dCbE0vh5PTkQR8QC7nE-5SjvwIbx8mnrjzZSYez8QVcm_X_AUqAfqSloGgFwAMVrKfsSYRkQNLUFlg4oDc7OZIQQQHFtLpf0yAEz1rq6M0LgRaa1UqPTzej2Vkaq25K0xA-XhAc_G81PzEkexVBhAAEV-b692ktr7i1Q/w400-h241/Lateral-view-of-the-foot.jpg&quot; title=&quot;Lateral view of the foot&quot; width=&quot;400&quot; /&gt;&lt;/a&gt;&lt;/div&gt;&lt;div&gt;&lt;b&gt;&lt;span style=&quot;color: #017087;&quot;&gt;Figure 37–2.&lt;/span&gt;&lt;/b&gt; Lateral view of the foot.&lt;/div&gt;&lt;div&gt;&lt;br /&gt;&lt;/div&gt;&lt;div class=&quot;separator&quot; style=&quot;clear: both; text-align: center;&quot;&gt;&lt;a href=&quot;https://blogger.googleusercontent.com/img/b/R29vZ2xl/AVvXsEgdqpAArQgyxy9YxpSNx0kpr8e2tZt76GytlUZ8s_ofxIeUVDs4KsNBJmPhQMRVEd4SUKkQ73h6mIHAh1sVVuE65WQsm670T9-XYj8zlNMDoelFwd6ieYQqGqf_aPtHoTFjsmOORvLOuIOV54WCIUntd5t_GeglI1ilyFXV7Syjo8C3o3Rw1dodiFv5xA/s500/Harris-view-of-the-foot.jpg&quot; style=&quot;margin-left: 1em; margin-right: 1em;&quot;&gt;&lt;img alt=&quot;Harris view of the foot&quot; border=&quot;0&quot; data-original-height=&quot;500&quot; data-original-width=&quot;380&quot; height=&quot;400&quot; src=&quot;https://blogger.googleusercontent.com/img/b/R29vZ2xl/AVvXsEgdqpAArQgyxy9YxpSNx0kpr8e2tZt76GytlUZ8s_ofxIeUVDs4KsNBJmPhQMRVEd4SUKkQ73h6mIHAh1sVVuE65WQsm670T9-XYj8zlNMDoelFwd6ieYQqGqf_aPtHoTFjsmOORvLOuIOV54WCIUntd5t_GeglI1ilyFXV7Syjo8C3o3Rw1dodiFv5xA/w304-h400/Harris-view-of-the-foot.jpg&quot; title=&quot;Harris view of the foot&quot; width=&quot;304&quot; /&gt;&lt;/a&gt;&lt;/div&gt;&lt;div&gt;&lt;b&gt;&lt;span style=&quot;color: #017087;&quot;&gt;Figure 37–3.&lt;/span&gt;&lt;/b&gt; Harris view of the foot.&lt;/div&gt;&lt;div&gt;&lt;br /&gt;&lt;/div&gt;&lt;div&gt;&lt;br /&gt;&lt;/div&gt;&lt;div&gt;&lt;div&gt;&lt;b&gt;&lt;span style=&quot;font-size: medium;&quot;&gt;ANSWER TO CASE 37:&lt;/span&gt;&lt;/b&gt;&lt;/div&gt;&lt;div&gt;&lt;span style=&quot;background-color: #daebf0; font-size: medium;&quot;&gt;&lt;span&gt;&lt;span style=&quot;background-color: transparent;&quot;&gt;&lt;span style=&quot;color: #017087;&quot;&gt;&lt;b&gt;Calcaneus Fracture&lt;/b&gt;&lt;/span&gt;&lt;/span&gt;&lt;b&gt;&amp;nbsp;&amp;nbsp;&lt;/b&gt;&lt;/span&gt;&lt;span style=&quot;font-weight: bold;&quot;&gt;&amp;nbsp;&amp;nbsp; &amp;nbsp;&lt;/span&gt;&lt;span style=&quot;font-weight: bold;&quot;&gt;&amp;nbsp;&amp;nbsp; &amp;nbsp;&lt;/span&gt;&lt;span style=&quot;font-weight: bold;&quot;&gt;&amp;nbsp;&amp;nbsp; &amp;nbsp;&lt;/span&gt;&lt;span style=&quot;font-weight: bold;&quot;&gt;&amp;nbsp;&amp;nbsp; &amp;nbsp;&lt;/span&gt;&lt;span style=&quot;font-weight: bold;&quot;&gt;&amp;nbsp; &amp;nbsp;&lt;/span&gt;&lt;/span&gt;&lt;/div&gt;&lt;/div&gt;&lt;div&gt;&lt;br /&gt;&lt;/div&gt;&lt;div&gt;&lt;div&gt;&lt;i&gt;Summary&lt;/i&gt;: A 51-year-old man presents with left foot pain, swelling, and ecchymosis after a 6-foot fall. He is unable to bear weight on his left foot. Radiographs are consistent with a left displaced intraarticular joint depression calcaneus fracture.&lt;/div&gt;&lt;div&gt;&lt;ul style=&quot;text-align: left;&quot;&gt;&lt;li&gt;&lt;b&gt;Most likely diagnosis:&lt;/b&gt; Displaced intraarticular joint depression calcaneus fracture.&lt;/li&gt;&lt;li&gt;&lt;b&gt;Additional studies:&lt;/b&gt; To further define the calcaneus fracture pattern and plan for definitive management, an axial computed tomography (CT) scan of the left calcaneus should be obtained. Coronal and sagittal plane reconstructions are frequently useful as well. Physical exam of the spine with further imaging is indicated, as there is a 10% incidence of associated spine injuries with calcaneus fractures. Although this patient complains of isolated foot pain, other regions such as the ipsilateral hip and knee should also be examined for pain, and if pain is present, appropriate radiographs of the affected area should be obtained.&lt;/li&gt;&lt;li&gt;&lt;b&gt;Treatment options:&lt;/b&gt; Nonoperative with immobilization in a nonweightbearing short leg splint or cast; open-reduction internal fixation.&lt;/li&gt;&lt;li&gt;&lt;b&gt;Potential complications: &lt;/b&gt;Wound complications/infection, calcaneal malunion, peroneal tendon irritation, anterior ankle impingement, subtalar arthritis, calcaneocuboid arthritis.&lt;/li&gt;&lt;/ul&gt;&lt;/div&gt;&lt;/div&gt;&lt;div&gt;&lt;br /&gt;&lt;/div&gt;&lt;div&gt;&lt;br /&gt;&lt;/div&gt;&lt;div&gt;&lt;div&gt;&lt;b&gt;&lt;span style=&quot;color: #017087; font-size: medium;&quot;&gt;ANALYSIS&lt;/span&gt;&lt;/b&gt;&lt;/div&gt;&lt;div&gt;&lt;b&gt;&lt;span style=&quot;font-size: medium;&quot;&gt;&lt;i&gt;Objectives&lt;/i&gt;&lt;/span&gt;&lt;/b&gt;&lt;/div&gt;&lt;/div&gt;&lt;div&gt;&lt;div&gt;&lt;ol style=&quot;text-align: left;&quot;&gt;&lt;li&gt;Identify the calcaneus fracture pattern using AP, lateral, and Harris radiographs.&lt;/li&gt;&lt;li&gt;Define and measure the Bohler angle and the critical angle of Gissane.&lt;/li&gt;&lt;li&gt;Recognize the fracture pattern on CT scan and apply the Sanders classification.&lt;/li&gt;&lt;li&gt;Understand the indications for open-reduction internal fixation.&lt;/li&gt;&lt;li&gt;Identify common complications of either nonoperative or operative treatment.&lt;/li&gt;&lt;/ol&gt;&lt;/div&gt;&lt;/div&gt;&lt;div&gt;&lt;br /&gt;&lt;/div&gt;&lt;div&gt;&lt;br /&gt;&lt;/div&gt;&lt;div&gt;&lt;div&gt;&lt;b&gt;&lt;i&gt;&lt;span style=&quot;font-size: medium;&quot;&gt;Considerations&lt;/span&gt;&lt;/i&gt;&lt;/b&gt;&lt;/div&gt;&lt;div&gt;&lt;br /&gt;&lt;/div&gt;&lt;div&gt;On initial presentation of a fractured calcaneus, the patient’s chief complaint will be pain about the hindfoot and an inability to bear weight on the affected limb. Observation of the skin and soft tissues may reveal soft tissue swelling, ecchymosis, and fracture blisters. The limb must be fully uncovered and circumferentially examined. A detailed motor and sensory examination should be performed. Because of the violent nature of the axial heel loading, there often exist fine disturbances to the plantar sensory nerves, which should be fully documented. A pulse exam must similarly be documented. Any lacerations, typically medially, reflect an open calcaneal fracture unless proven otherwise. Open fractures, especially those associated with contamination, should undergo emergent debridement and irrigation in the operative theatre.&lt;/div&gt;&lt;/div&gt;&lt;div&gt;&lt;br /&gt;&lt;/div&gt;&lt;div&gt;&lt;div&gt;&lt;span&gt;&amp;nbsp;&amp;nbsp; &amp;nbsp;&lt;/span&gt;Extensive soft tissue swelling may also predispose the patient to compartment syndrome of the foot. &lt;b&gt;There are 9 compartments within the foot, which are individually defined by an inflexible investing fascia. &lt;/b&gt;As muscle swelling increases, the closed spaces within the foot do not permit an increase in volume, and the individual compartment becomes pressurized. This pressure head exceeds the venous outflow pressure, which results in ischemia to the nervous and muscular components in a time-dependent manner. The most common signs are increasing pain requirements and pain on passive stretch, specifically great toe abduction and adduction. Diagnosis requires a coherent patient, not under the influence of excessive sedating medications. If the patient is unable to interact appropriately, pressure measurements may be objectively obtained through needle manometry. If compartment syndrome is present, the compartments must be emergently released via surgical fasciotomies.&lt;/div&gt;&lt;/div&gt;&lt;div&gt;&lt;br /&gt;&lt;/div&gt;&lt;div&gt;&lt;div&gt;&lt;span&gt;&amp;nbsp;&amp;nbsp; &amp;nbsp;&lt;/span&gt;In addition to the calcaneus fracture, the patient may present with additional injuries. A thorough history and physical exam should be performed, with additional imaging obtained as indicated. &lt;b&gt;Approximately 10% of patients with calcaneus&amp;nbsp;&lt;/b&gt;&lt;b&gt;fractures have associated spine injuries, and upward of 26% of patients have&amp;nbsp;&lt;/b&gt;&lt;b&gt;other extremity injuries.&lt;/b&gt; Missed injuries can be avoided by a careful clinical and radiographic evaluation of the coherent patient at the time of injury.&lt;/div&gt;&lt;/div&gt;&lt;div&gt;&lt;br /&gt;&lt;/div&gt;&lt;div&gt;&lt;br /&gt;&lt;/div&gt;&lt;div&gt;&lt;div&gt;&lt;div&gt;&lt;b&gt;&lt;span style=&quot;font-size: medium;&quot;&gt;APPROACH TO:&lt;/span&gt;&lt;/b&gt;&lt;/div&gt;&lt;div&gt;&lt;span style=&quot;background-color: #daebf0; color: #017087; font-size: medium;&quot;&gt;&lt;span&gt;&lt;span style=&quot;background-color: transparent;&quot;&gt;&lt;b&gt;Calcaneus Fracture&lt;/b&gt;&lt;/span&gt;&lt;b&gt;&amp;nbsp; &amp;nbsp;&amp;nbsp;&lt;/b&gt;&lt;/span&gt;&lt;span style=&quot;font-weight: bold;&quot;&gt;&amp;nbsp;&amp;nbsp; &amp;nbsp;&lt;/span&gt;&lt;span style=&quot;font-weight: bold;&quot;&gt;&amp;nbsp;&amp;nbsp; &amp;nbsp;&lt;/span&gt;&lt;span style=&quot;font-weight: bold;&quot;&gt;&amp;nbsp;&amp;nbsp; &amp;nbsp;&lt;/span&gt;&lt;span style=&quot;font-weight: bold;&quot;&gt;&amp;nbsp;&amp;nbsp; &amp;nbsp;&lt;/span&gt;&lt;span style=&quot;font-weight: bold;&quot;&gt;&amp;nbsp;&amp;nbsp; &amp;nbsp;&lt;/span&gt;&lt;span style=&quot;font-weight: bold;&quot;&gt;&amp;nbsp;&amp;nbsp; &amp;nbsp;&lt;/span&gt;&lt;span style=&quot;font-weight: bold;&quot;&gt;&amp;nbsp;&amp;nbsp;&amp;nbsp;&lt;/span&gt;&lt;span style=&quot;font-weight: bold;&quot;&gt;&amp;nbsp;&lt;/span&gt;&lt;/span&gt;&lt;/div&gt;&lt;/div&gt;&lt;div&gt;&lt;br /&gt;&lt;/div&gt;&lt;div&gt;&lt;b&gt;&lt;span style=&quot;color: #017087; font-size: medium;&quot;&gt;DEFINITIONS&lt;/span&gt;&lt;/b&gt;&lt;/div&gt;&lt;/div&gt;&lt;div&gt;&lt;br /&gt;&lt;/div&gt;&lt;div&gt;&lt;div&gt;&lt;b&gt;INTRAARTICULAR: &lt;/b&gt;With respect to the calcaneus, a fracture that extends into the subtalar joint. More specifically, a fracture that exists within the posterior facet of the talocalcaneal articulation.&lt;/div&gt;&lt;div&gt;&lt;b&gt;&lt;br /&gt;&lt;/b&gt;&lt;/div&gt;&lt;div&gt;&lt;b&gt;EXTRAARTICULAR:&lt;/b&gt; The fracture does not extend into the subtalar joint, particularly the posterior facet of the talocalcaneal articulation.&lt;/div&gt;&lt;div&gt;&lt;b&gt;&lt;br /&gt;&lt;/b&gt;&lt;/div&gt;&lt;div&gt;&lt;b&gt;BOHLER ANGLE:&lt;/b&gt; An angle created from the intersection of 2 radiographic lines observed on a lateral calcaneus x-ray. The first line connects the most cephalad point of the calcaneal anterior process with the most cephalad point of the calcaneal posterior facet. The second line connects the most cephalad point of the calcaneal posterior facet with the most cephalad point of the calcaneal tuberosity. An uninjured calcaneus typically demonstrates an angle between 20 and 40 degrees.&lt;/div&gt;&lt;div&gt;&lt;b&gt;&lt;br /&gt;&lt;/b&gt;&lt;/div&gt;&lt;div&gt;&lt;b&gt;CRITICAL ANGLE OF GISSANE:&lt;/b&gt; Another angle created from the intersection of 2 radiographic lines observed on the lateral calcaneus x-ray. The first line runs tangential along the superior aspect of the anterior process. The second line runs tangential to the superior border of the posterior facet. A typical value for the uninjured calcaneus measures between 95 and 105 degrees.&lt;/div&gt;&lt;/div&gt;&lt;div&gt;&lt;br /&gt;&lt;/div&gt;&lt;div&gt;&lt;br /&gt;&lt;/div&gt;&lt;div&gt;&lt;b&gt;&lt;span style=&quot;color: #017087; font-size: medium;&quot;&gt;CLINICAL APPROACH&lt;/span&gt;&lt;/b&gt;&lt;/div&gt;&lt;div&gt;&lt;br /&gt;&lt;/div&gt;&lt;div&gt;&lt;div&gt;&lt;b&gt;&lt;i&gt;&lt;span style=&quot;font-size: medium;&quot;&gt;Etiology&lt;/span&gt;&lt;/i&gt;&lt;/b&gt;&lt;/div&gt;&lt;div&gt;&lt;br /&gt;&lt;/div&gt;&lt;div&gt;Fractures of the calcaneus typically result from high-energy trauma such as falls from a height or motor vehicle collision and represent approximately 2% of all fractures.&lt;/div&gt;&lt;/div&gt;&lt;div&gt;&lt;br /&gt;&lt;/div&gt;&lt;div&gt;&lt;br /&gt;&lt;/div&gt;&lt;div&gt;&lt;span style=&quot;color: #017087; font-size: medium;&quot;&gt;&lt;b&gt;DIAGNOSIS&lt;/b&gt;&lt;/span&gt;&lt;/div&gt;&lt;div&gt;&lt;br /&gt;&lt;/div&gt;&lt;div&gt;&lt;div&gt;Initial radiographic evaluation of a suspected calcaneus fracture should include an AP of the foot, lateral of the foot, and a Harris axial projection, along with AP, lateral, and mortise views of the ankle. These radiographs will help classify the fracture pattern, whether joint depression or tongue type. The diagnosis is most evident on the lateral projection, which will demonstrate the degree of calcaneal compression quantified by the &lt;b&gt;Bohler angle ( Figure 37–4). The degree of height loss has prognostic&amp;nbsp;&lt;/b&gt;&lt;b&gt;implications, with a smaller Bohler angle correlating with poorer functional&amp;nbsp;&lt;/b&gt;&lt;b&gt;outcomes.&lt;/b&gt; CT will help identify each fracture line and fracture fragment. Reformats in the coronal and sagittal planes parallel the findings in the Harris axial and lateral views, respectively. Additional information can be obtained from the CT, including an evaluation of lateral wall expulsion and peroneal tendon impingement or dislocation.&lt;/div&gt;&lt;/div&gt;&lt;div&gt;&lt;br /&gt;&lt;/div&gt;&lt;div&gt;&lt;div&gt;&lt;span&gt;&amp;nbsp;&amp;nbsp; &amp;nbsp;&lt;/span&gt;From the CT scan, the fracture can be classified according to &lt;b&gt;Sanders.&lt;/b&gt; The posterior facet is divided into 3 fragments: lateral, central, and medial. With the addition of the sustentaculum, there are 4 defined fragments. The primary fracture line is identified and the number of fragments created by the fracture line is noted. A letter is assigned to where the fracture line crosses the posterior facet. The number and location of posterior facet fracture lines have been demonstrated to correlate with outcomes after surgical fixation, with poorer outcome measures associated with more comminuted patterns. The collective information obtained by careful evaluation of the radiographs and CT scan aid in definitive management and preoperative planning.&lt;/div&gt;&lt;/div&gt;&lt;div&gt;&lt;br /&gt;&lt;/div&gt;&lt;div class=&quot;separator&quot; style=&quot;clear: both; text-align: center;&quot;&gt;&lt;a href=&quot;https://blogger.googleusercontent.com/img/b/R29vZ2xl/AVvXsEh-FQXhC6zS7O53yyyoiFqSGlShWThm7q0wJv68If9Ewckp58DW9lPbcaw0PS6QCGFOgxL1YJGqcaT_djxrhD1EU4fQs9Tte89tnq-tLgXumaW0HiWdHLkpvVCiidBfxMITSYAGO02IpLGRN7o2nx6paMJcfZbzXloxN2mYb3JwbAX5qLmECiPU30mc6Q/s526/Calcaneus-fracture.jpg&quot; style=&quot;margin-left: 1em; margin-right: 1em;&quot;&gt;&lt;img alt=&quot;Calcaneus fracture&quot; border=&quot;0&quot; data-original-height=&quot;353&quot; data-original-width=&quot;526&quot; height=&quot;269&quot; src=&quot;https://blogger.googleusercontent.com/img/b/R29vZ2xl/AVvXsEh-FQXhC6zS7O53yyyoiFqSGlShWThm7q0wJv68If9Ewckp58DW9lPbcaw0PS6QCGFOgxL1YJGqcaT_djxrhD1EU4fQs9Tte89tnq-tLgXumaW0HiWdHLkpvVCiidBfxMITSYAGO02IpLGRN7o2nx6paMJcfZbzXloxN2mYb3JwbAX5qLmECiPU30mc6Q/w400-h269/Calcaneus-fracture.jpg&quot; title=&quot;Calcaneus fracture&quot; width=&quot;400&quot; /&gt;&lt;/a&gt;&lt;/div&gt;&lt;div&gt;&lt;div&gt;&lt;b&gt;&lt;span style=&quot;color: #017087;&quot;&gt;Figure 37–4.&lt;/span&gt;&lt;/b&gt; Calcaneus fracture: Bohler angle. The Bohler angle is formed by the intersection of lines&lt;/div&gt;&lt;div&gt;drawn tangentially to the (&lt;b&gt;A&lt;/b&gt;) anterior and (&lt;b&gt;B&lt;/b&gt;) posterior elements of the (&lt;b&gt;C&lt;/b&gt;) superior surface of the&lt;/div&gt;&lt;div&gt;calcaneus. A normal angle is approximately 20 to 40 degrees. (Reproduced, with permission, from&lt;/div&gt;&lt;div&gt;Knoop KJ, Stack LB, Storrow AB, et al. &lt;i&gt;Atlas of Emergency Medicine&lt;/i&gt;. 3rd ed. New York, NY: McGraw-Hill; 2009:Fig. 11-83.)&lt;/div&gt;&lt;/div&gt;&lt;div&gt;&lt;br /&gt;&lt;/div&gt;&lt;div&gt;&lt;br /&gt;&lt;/div&gt;&lt;div&gt;&lt;b style=&quot;color: #017087;&quot;&gt;&lt;span style=&quot;font-size: medium;&quot;&gt;TREATMENT&lt;/span&gt;&lt;/b&gt;&lt;/div&gt;&lt;div&gt;&lt;br /&gt;&lt;/div&gt;&lt;div&gt;&lt;div&gt;&lt;b&gt;&lt;i&gt;&lt;span style=&quot;font-size: medium;&quot;&gt;Nonoperative&lt;/span&gt;&lt;/i&gt;&lt;/b&gt;&lt;/div&gt;&lt;div&gt;&lt;br /&gt;&lt;/div&gt;&lt;div&gt;Nonoperative treatment consists of early ankle and subtalar range-of-motion exercises and &lt;b&gt;nonweightbearing for approximately 3 months.&lt;/b&gt; Initial management consists of splinting the foot and ankle in neutral dorsiflexion to prevent an equinus contracture, ice, elevation, and pain control. Once the swelling subsides, the splint is removed while ankle and subtalar range-of-motion exercises are performed. Clinical and radiographic follow-up is carried out at routine intervals to document healing. Specific indications for nonoperative treatment include a nondisplaced fracture, a patient too physiologically compromised to tolerate surgery, or a nonambulatory patient. In addition, patients unable or unwilling to comply with postoperative weightbearing restrictions are best managed nonoperatively. Factors that affect local wound healing, such as a history of smoking, severe peripheral vascular disease, or insulin-dependent diabetes, are relative contraindications to surgery.&lt;/div&gt;&lt;/div&gt;&lt;div&gt;&lt;br /&gt;&lt;/div&gt;&lt;div&gt;&lt;br /&gt;&lt;/div&gt;&lt;div&gt;&lt;div&gt;&lt;b&gt;&lt;i&gt;&lt;span style=&quot;font-size: medium;&quot;&gt;Open-Reduction Internal Fixation&lt;/span&gt;&lt;/i&gt;&lt;/b&gt;&lt;/div&gt;&lt;div&gt;&lt;br /&gt;&lt;/div&gt;&lt;div&gt;Historically, fractures of the calcaneus were treated nonoperatively by means of re-impacting the fracture by striking the lateral heel with a hammer. Surgeons would rather perform this procedure over open surgery because of fears over infection, malunion, nonunion, and need for amputation. With the development of aseptic techniques and the modern principles of reduction and fixation, calcaneal surgery could be performed with an improved complication profile. A prospective, randomized, controlled multicenter trial by &lt;b&gt;Buckley et al&lt;/b&gt; warrants mention. In this study, 309 patients were prospectively randomized to either nonoperative or operative treatment and followed for a minimum of 2 years. Operative treatment demonstrated improved outcomes in women, patients not receiving worker’s compensation, young men (&amp;lt; 30 years of age), patients with a higher Bohler angle, patients with a lighter workload, and patients with a simple displaced intraarticular fracture pattern. Anatomic reductions also improved outcomes. Patients treated nonoperatively were more likely to require late arthrodesis. As our understanding of the outcomes of nonoperative and operative treatment improves, we can identify those patients best managed by open-reduction internal fixation.&lt;/div&gt;&lt;/div&gt;&lt;div&gt;&lt;br /&gt;&lt;/div&gt;&lt;div&gt;&lt;div&gt;&lt;span&gt;&amp;nbsp;&amp;nbsp; &amp;nbsp;&lt;/span&gt;Once the decision has been made to proceed with open-reduction internal fixation, timing and surgical treatment goals must be discussed. Timing of surgery is an important consideration. Calcaneal fractures are the result of high-energy trauma, and the associated soft tissue injury is often extensive. Incisions across a compromised soft tissue envelope risk postoperative wound complications and deep sepsis. Serous and hemorrhagic fracture blisters should be unroofed and treated with dry bandages until epithelialization occurs. Despite signs of healing, the blister beds should be avoided if possible. Before surgical intervention, the foot must achieve soft tissue quiescence. This readiness of the soft tissues is indicated by the appearance of fine skin wrinkles with passive foot dorsiflexion, known as a positive &lt;b&gt;“wrinkle&amp;nbsp;&lt;/b&gt;&lt;b&gt;test.”&lt;/b&gt; A waiting period of 10 to 14 days is common before proceeding to surgery. Although waiting for an appropriate surgical window is critical, treatment within the first 3 weeks is recommended to avoid a malunion. This is a situation in which the fracture heals in an undesirable position. The malunited calcaneus is frequently shortened and in varus.&lt;/div&gt;&lt;/div&gt;&lt;div&gt;&lt;br /&gt;&lt;/div&gt;&lt;div&gt;&lt;div&gt;&lt;span&gt;&amp;nbsp;&amp;nbsp; &amp;nbsp;&lt;/span&gt;The goal of surgical intervention is to anatomically restore the calcaneus. Critical components in this restoration include anatomic reduction of the posterior facet and reconstitution of calcaneal height, width, and length. Medial and lateral approaches to fixation have been described. An &lt;b&gt;extensile lateral approach&lt;/b&gt; is most often used. The vertical limb of the incision is made anterior to the lateral border of the Achilles tendon and continued in a right angle fashion horizontally along the border of the foot where the glabrous and plantar skin meet. This approach allows for direct reduction of the anterior process, the posterior facet, and the tuberosity. Kirschner wires can be used to temporarily hold the reduction, which should be checked fluoroscopically and under direct visualization. Calcaneal height is checked on the lateral view. Width and varus heel positioning is imaged on the axial view. The reduction of the posterior facet is directly visualized and can be imaged through the use of the Broden view. Once the reduction is satisfactory, the Kirschner wires are replaced with screws or a combination of screws and plates for the final construct. Careful handling of the flap is crucial during wound closure to minimize the risk of skin necrosis and wound complications. A Hemovac drain is typically left in place and a splint applied in neutral dorsiflexion to prevent an equinus contracture. Postoperative radiographs and CT are performed to check the accuracy of the reduction. The patient is made nonweightbearing for approximately 3 months, at the discretion of the treating surgeon. During the healing period, range-of-motion exercises for the ankle and subtalar joint can be initiated as soon as the incision appears stable and healing. Postoperative follow-up is at routine intervals to document fracture healing. Good to excellent results can be achieved with an anatomic articular reduction.&lt;/div&gt;&lt;/div&gt;&lt;div&gt;&lt;br /&gt;&lt;/div&gt;&lt;div&gt;&lt;br /&gt;&lt;/div&gt;&lt;div&gt;&lt;div&gt;&lt;b&gt;&lt;i&gt;&lt;span style=&quot;font-size: medium;&quot;&gt;Complications&lt;/span&gt;&lt;/i&gt;&lt;/b&gt;&lt;/div&gt;&lt;div&gt;&lt;br /&gt;&lt;/div&gt;&lt;div&gt;&lt;b&gt;&lt;span style=&quot;font-size: medium;&quot;&gt;Subtalar Arthritis:&lt;/span&gt; Posttraumatic arthritis&lt;/b&gt; of the subtalar joint can develop after both nonoperative and operative treatment. A nonanatomic articular reduction and cartilage damage from the injury itself are the primary contributing factors. Patients will primarily complain of pain. The pain is dependent on activity and worsens as the day progresses. As a result of subtalar stiffness, patients will complain that it is&amp;nbsp;&lt;b&gt;difficult to navigate over uneven walking surfaces.&lt;/b&gt; Radiographic evaluation with standard x-rays and CT may reveal joint space narrowing, osteophyte formation, and subchondral sclerosis. Injecting a local anesthetic into the subtalar joint and documenting relief of symptoms can confirm the diagnosis. Nonoperative measures include nonsteroidal anti-inflammatory medications, shoe wear modifications, or custom orthoses. These measures should be tried before operative intervention is considered. If nonoperative measures fail, operative treatment consists of a subtalar fusion.&lt;/div&gt;&lt;/div&gt;&lt;div&gt;&lt;br /&gt;&lt;/div&gt;&lt;div&gt;&lt;div&gt;&lt;b&gt;&lt;span style=&quot;font-size: medium;&quot;&gt;Wound Complications/Infection:&lt;/span&gt;&lt;/b&gt; The most common complication after openreduction internal fixation is related to the surgical wound. Wound complications including dehiscence and infection have been noted to occur in up to 20% of patients with closed fractures. The reported rates of deep infections and calcaneal osteomyelitis range from 0% to 20% in closed fractures and from 19% to 31% in open fractures. Cellulitis and superficial wound sloughs can be treated with dressing changes and the administration of antibiotics. Persistent drainage or purulence necessitates open debridement coupled with administration of antibiotics. In the case of superficial infection, the hardware can be retained. If a deep infection is noted or there is evidence of diffuse osteomyelitis at the time of open debridement, the hardware must be removed. Patient selection, meticulous surgical technique, and compliance with postoperative protocols can minimize the risks of wound complications.&lt;/div&gt;&lt;/div&gt;&lt;div&gt;&lt;br /&gt;&lt;/div&gt;&lt;div&gt;&lt;div&gt;&lt;b&gt;&lt;span style=&quot;font-size: medium;&quot;&gt;Calcaneocuboid Arthritis:&lt;/span&gt;&lt;/b&gt; Arthritis can develop after both nonoperative and operative treatment secondary to an imperfect reduction of the anterolateral fragment. Similar to the diagnosis of subtalar arthritis detailed previously, physical exam, radiographs, and a trial injection can be performed to elucidate whether the calcaneocuboid joint is the source of the patient’s pain. First-line treatment is nonoperative with nonsteroidal anti- inflammatory medications, shoe wear modifications, or custom orthoses. If nonoperative treatment fails, calcaneocuboid fusion may be considered and is typically performed in conjunction with a talonavicular and talocalcaneal fusion.&lt;/div&gt;&lt;/div&gt;&lt;div&gt;&lt;br /&gt;&lt;/div&gt;&lt;div&gt;&lt;div&gt;&lt;b&gt;&lt;span style=&quot;font-size: medium;&quot;&gt;Peroneal Irritation:&lt;/span&gt; This occurs more commonly in patients managed&amp;nbsp;&lt;/b&gt;&lt;b&gt;nonoperatively.&lt;/b&gt; During the axial loading event, the lateral wall often becomes a separate fragment, which translates laterally. Both the peroneus longus and brevis tendons traverse the lateral calcaneus in this region and are subjected to compression and irritation. Dislocations or subluxations are common. Operative treatment addresses both the lateral wall displacement and the peroneal tendon instability. In operative cases, postoperative scarring or prominent hardware is more frequently the culprit. The initial treatment is nonoperative with nonsteroidal antiinflammatory medications and shoe wear modifications or orthotics. Elective surgery includes removal of hardware, lateral wall reconstruction, or exostectomy, along with a peroneal tendon reconstruction. These procedures should be considered if the patient is experiencing pain refractory to nonoperative measures.&lt;/div&gt;&lt;/div&gt;&lt;div&gt;&lt;br /&gt;&lt;/div&gt;&lt;div&gt;&lt;div&gt;&lt;b&gt;&lt;span style=&quot;font-size: medium;&quot;&gt;Anterior Ankle Impingement:&lt;/span&gt;&lt;/b&gt; Anterior ankle impingement can occur after nonoperative or operative treatment. The impingement is the result of loss of calcaneal height and a decreased lateral talocalcaneal angle. With nonoperative management or incomplete restoration of calcaneal height, the talus adopts a pathologic horizontal position. The result is direct contact with the dorsal talar neck&lt;/div&gt;&lt;div&gt;and the anterior tibial plafond. On clinical evaluation, the patients present with pain and &lt;b&gt;restriction in ankle dorsiflexion.&lt;/b&gt; Often the examiner can feel a firm block when the ankle is passively dorsiflexed. Surgical management can include exostectomy of the talar osteophytes, but &lt;b&gt;distraction subtalar bone block fusion is the preferred&amp;nbsp;&lt;/b&gt;&lt;b&gt;procedure.&lt;/b&gt; The goal of this procedure is to restore the calcaneal height and thus create an improved lateral talocalcaneal angle.&lt;/div&gt;&lt;/div&gt;&lt;div&gt;&lt;br /&gt;&lt;/div&gt;&lt;div&gt;&lt;div&gt;&lt;b&gt;&lt;span style=&quot;font-size: medium;&quot;&gt;Calcaneal Malunion:&lt;/span&gt;&lt;/b&gt; Malunion can occur in both nonoperative and operative management of calcaneal fractures. &lt;b&gt;Varus angulation of the hindfoot is the&amp;nbsp;&lt;/b&gt;&lt;b&gt;most common residual deformity. &lt;/b&gt;This usually occurs from a varus malunion of the tuberosity. Treatment with subtalar fusion and a calcaneal osteotomy has been described, with variations, by several authors. To plan the malunion repair, Stephens and Sanders developed a CT classification for malunions. Type I involves a large lateral exostosis with or without arthrosis of the subtalar joint and is treated with peroneal tenolysis and a lateral exostectomy. Type II involves a lateral exostosis with arthrosis across the subtalar joint and is treated with peroneal tenolysis, a lateral exostectomy, and in situ subtalar fusion. Type III involves a lateral exostosis, severe subtalar arthrosis, and varus or valgus angulation of the calcaneal body and is treated with peroneal tenolysis, a lateral exostectomy, subtalar fusion, and a calcaneal osteotomy.&lt;/div&gt;&lt;/div&gt;&lt;div&gt;&lt;br /&gt;&lt;/div&gt;&lt;div&gt;&lt;br /&gt;&lt;/div&gt;&lt;div&gt;&lt;b&gt;&lt;span style=&quot;color: #017087; font-size: medium;&quot;&gt;&lt;u&gt;COMPREHENSION QUESTIONS&lt;/u&gt;&lt;/span&gt;&lt;/b&gt;&lt;/div&gt;&lt;div&gt;&lt;br /&gt;&lt;/div&gt;&lt;div&gt;&lt;div&gt;37.1 A 29-year-old man presents with significant right hindfoot swelling and pain after a 25-foot jump from a cliff. Imaging confirms a comminuted intraarticular calcaneus fracture involving the posterior facet. As the treating physician, you request that radiographs of the spine be obtained. What percentage of patients with fractures of the calcaneus have associated spine injuries?&lt;/div&gt;&lt;/div&gt;&lt;blockquote style=&quot;border: none; margin: 0px 0px 0px 40px; padding: 0px; text-align: left;&quot;&gt;&lt;div&gt;&lt;div&gt;A. 10%&lt;/div&gt;&lt;/div&gt;&lt;div&gt;&lt;div&gt;B. 30%&lt;/div&gt;&lt;/div&gt;&lt;div&gt;&lt;div&gt;C. 50%&lt;/div&gt;&lt;/div&gt;&lt;div&gt;&lt;div&gt;D. 80%&lt;/div&gt;&lt;/div&gt;&lt;/blockquote&gt;&lt;div&gt;&lt;div&gt;&lt;br /&gt;&lt;/div&gt;&lt;div&gt;37.2 Which of the following factors is associated with worse outcomes after operative treatment of fractures of the calcaneus?&lt;/div&gt;&lt;/div&gt;&lt;blockquote style=&quot;border: none; margin: 0px 0px 0px 40px; padding: 0px; text-align: left;&quot;&gt;&lt;div&gt;&lt;div&gt;A. Female sex&lt;/div&gt;&lt;/div&gt;&lt;div&gt;&lt;div&gt;B. Age &amp;lt; 30 years&lt;/div&gt;&lt;/div&gt;&lt;div&gt;&lt;div&gt;C. Worker’s compensation&lt;/div&gt;&lt;/div&gt;&lt;div&gt;&lt;div&gt;D. Bohler angle &amp;gt; 0 degrees&lt;/div&gt;&lt;/div&gt;&lt;div&gt;&lt;div&gt;E. Simple articular fracture pattern&lt;/div&gt;&lt;/div&gt;&lt;/blockquote&gt;&lt;div&gt;&lt;div&gt;&lt;br /&gt;&lt;/div&gt;&lt;div&gt;37.3 A 42-year-old man presents with a displaced comminuted intraarticular calcaneus fracture. On physical exam, there is extensive soft tissue swelling and hemorrhagic blisters. The patient is splinted and discharged home, as the soft tissues are not ready for open-reduction internal fixation. He returns to clinic 10 days later with clawing of his toes. What is the most likely explanation for this finding?&lt;/div&gt;&lt;/div&gt;&lt;blockquote style=&quot;border: none; margin: 0px 0px 0px 40px; padding: 0px; text-align: left;&quot;&gt;&lt;div&gt;&lt;div&gt;A. Tendon laceration from displaced fracture fragment&lt;/div&gt;&lt;/div&gt;&lt;div&gt;&lt;div&gt;B. Nerve injury from displaced fracture fragment&lt;/div&gt;&lt;/div&gt;&lt;div&gt;&lt;div&gt;C. Compartment syndrome&lt;/div&gt;&lt;/div&gt;&lt;div&gt;&lt;div&gt;D. Weakness secondary to pain and prolonged immobilization&lt;/div&gt;&lt;/div&gt;&lt;/blockquote&gt;&lt;div&gt;&lt;br /&gt;&lt;/div&gt;&lt;div&gt;&lt;br /&gt;&lt;/div&gt;&lt;div&gt;&lt;b&gt;&lt;span style=&quot;color: #017087; font-size: medium;&quot;&gt;&lt;u&gt;ANSWERS&lt;/u&gt;&lt;/span&gt;&lt;/b&gt;&lt;/div&gt;&lt;div&gt;&lt;br /&gt;&lt;/div&gt;&lt;div&gt;&lt;div&gt;37.1 &lt;b&gt;A.&lt;/b&gt; Approximately 10% of patients who present with calcaneus fractures have associated spine injuries. A thorough history and physical exam, with additional imaging as indicated, is important to avoid missed injuries.&lt;/div&gt;&lt;div&gt;&lt;br /&gt;&lt;/div&gt;&lt;div&gt;37.2 &lt;b&gt;C.&lt;/b&gt; A prospective, randomized, controlled multicenter trial by Buckley et al followed 309 patients for a period of 2 to 8 years. Overall the outcomes for nonoperative and operative management were not found to be different. After closer examination of the patient population, operative treatment demonstrated improved outcomes in women, patients not receiving worker’s compensation, young men (&amp;lt; 30 years), patients with a higher Bohler angle, patients with a lighter workload, and patients with a simple displaced intraarticular fracture pattern. Anatomic reductions also improved outcomes. Patients treated nonoperatively were more likely to require late arthrodesis.&lt;/div&gt;&lt;div&gt;&lt;br /&gt;&lt;/div&gt;&lt;div&gt;37.3 &lt;b&gt;C.&lt;/b&gt; Foot compartment syndrome is a known complication of calcaneus fractures. There are 9 compartments within the foot, which are individually defined by an inflexible investing fascia. As muscle swelling increases, the closed spaces within the foot do not permit an increase in volume, and the individual compartment becomes pressurized. This pressure head exceeds the venous outflow pressure, which results in ischemia to the nervous and muscular components in a time-dependent manner. If this is unrecognized, nerve and muscle death occur. This can result in contractures of the muscles in the affected compartments, which is evident on clinical evaluation.&lt;/div&gt;&lt;/div&gt;&lt;div&gt;&lt;br /&gt;&lt;/div&gt;&lt;div&gt;&lt;b style=&quot;background-color: #4093a5;&quot;&gt;&amp;nbsp; &amp;nbsp;&lt;span style=&quot;font-size: medium;&quot;&gt;&amp;nbsp;&lt;/span&gt;&lt;span style=&quot;color: white;&quot;&gt;&lt;span style=&quot;font-size: medium;&quot;&gt;CLINICAL PEARLS&lt;/span&gt;&lt;span&gt;&amp;nbsp; &amp;nbsp;&amp;nbsp;&lt;/span&gt;&lt;/span&gt;&lt;/b&gt;&lt;/div&gt;&lt;div&gt;&lt;table border=&quot;1&quot; cellpadding=&quot;0&quot; cellspacing=&quot;0&quot; class=&quot;MsoTableGrid&quot; style=&quot;border-collapse: collapse; border: none; mso-border-alt: solid windowtext .5pt; mso-padding-alt: 0in 5.4pt 0in 5.4pt; mso-yfti-tbllook: 1184;&quot;&gt;
  49. &lt;tbody&gt;&lt;tr style=&quot;height: 19.15pt; mso-yfti-firstrow: yes; mso-yfti-irow: 0; mso-yfti-lastrow: yes;&quot;&gt;
  50.  &lt;td style=&quot;background: rgb(221, 237, 239); border-left: solid #017087 1.0pt; border: 1pt solid rgb(64, 147, 165); height: 19.15pt; padding: 0in 5.4pt; width: 433.7pt;&quot; valign=&quot;top&quot; width=&quot;578&quot;&gt;
  51.  &lt;div style=&quot;line-height: normal; margin-bottom: 0in;&quot;&gt;&lt;span style=&quot;color: #017087;&quot;&gt;&lt;br /&gt;&lt;/span&gt;&lt;/div&gt;&lt;div style=&quot;line-height: normal; margin-bottom: 0in;&quot;&gt;&lt;span style=&quot;color: #017087;&quot;&gt;►&amp;nbsp;&lt;/span&gt;Fractures of the calcaneus represent high-energy trauma with a significant incidence of associated spine and extremity injuries. A thorough history and physical exam are crucial at the time of presentation.&lt;/div&gt;&lt;div style=&quot;line-height: normal; margin-bottom: 0in;&quot;&gt;&lt;span style=&quot;color: #017087;&quot;&gt;&lt;br /&gt;&lt;/span&gt;&lt;/div&gt;&lt;div style=&quot;line-height: normal; margin-bottom: 0in;&quot;&gt;&lt;span style=&quot;color: #017087;&quot;&gt;►&lt;/span&gt;&amp;nbsp;AP, lateral, and Harris radiographs should be ordered along with a CT scan in the axial plane with sagittal and coronal reconstructions to evaluate the fracture pattern and plan for treatment.&lt;/div&gt;&lt;div style=&quot;line-height: normal; margin-bottom: 0in;&quot;&gt;&lt;span style=&quot;color: #017087;&quot;&gt;&lt;br /&gt;&lt;/span&gt;&lt;/div&gt;&lt;div style=&quot;line-height: normal; margin-bottom: 0in;&quot;&gt;&lt;span style=&quot;color: #017087;&quot;&gt;►&lt;/span&gt;&amp;nbsp;Open-reduction internal fixation should be delayed until soft tissue swelling is reduced and blisters have epithelialized.&lt;/div&gt;&lt;div style=&quot;line-height: normal; margin-bottom: 0in;&quot;&gt;&lt;span style=&quot;color: #017087;&quot;&gt;&lt;br /&gt;&lt;/span&gt;&lt;/div&gt;&lt;div style=&quot;line-height: normal; margin-bottom: 0in;&quot;&gt;&lt;span style=&quot;color: #017087;&quot;&gt;►&lt;/span&gt;&amp;nbsp;Wound complications including dehiscence and infection are the most common problems associated with operative management.&lt;/div&gt;&lt;div style=&quot;line-height: normal; margin-bottom: 0in;&quot;&gt;&lt;span style=&quot;color: #017087;&quot;&gt;&lt;br /&gt;&lt;/span&gt;&lt;/div&gt;&lt;div style=&quot;line-height: normal; margin-bottom: 0in;&quot;&gt;&lt;span style=&quot;color: #017087;&quot;&gt;►&lt;/span&gt;&amp;nbsp;Outcomes after operative treatment are best in women, non–worker’s compensation cases, young (age &amp;lt; 30 years) men, higher Bohler angle (&amp;gt; 0 degrees), patients with a lighter workload, and those with simple articular fracture patterns.&lt;/div&gt;&lt;div style=&quot;line-height: normal; margin-bottom: 0in;&quot;&gt;&lt;br /&gt;&lt;/div&gt;
  52.  &lt;/td&gt;
  53. &lt;/tr&gt;
  54. &lt;/tbody&gt;&lt;/table&gt;&lt;br /&gt;&lt;/div&gt;
  55.  
  56. &lt;details close=&quot;&quot;&gt;
  57.  &lt;summary&gt;&lt;b&gt;&lt;span style=&quot;color: #00788b;&quot;&gt;REFERENCES&lt;/span&gt;&lt;/b&gt;&lt;/summary&gt;
  58.  &lt;p&gt;Benirschke SK, Kramer PA. Wound healing complications in closed and open calcaneal fractures.
  59. J Orthop Trauma. 2004;18:1-6.&amp;nbsp;&lt;/p&gt;&lt;p&gt;Buckley R, Tough S, McCormack R, Pate G, Leighton R, Petrie D, Galpin R. Operative compared with
  60. nonoperative treatment of displaced intra-articular calcaneal fractures: a prospective, randomized,
  61. controlled multicenter trial. J Bone Joint Surg Am . 2002;84-A:1733-1744.&amp;nbsp;&lt;/p&gt;&lt;p&gt;Howard JL, Buckley R, McCormack R, Pate G, Leighton R, Petrie D, Galpin R. Complications following
  62. management of displaced intra-articular calcaneal fractures: a prospective randomized trial comparing
  63. open reduction internal fixation with nonoperative management. J Orthop Trauma. 2003;17:241-249.&amp;nbsp;&lt;/p&gt;&lt;p&gt;Sanders R. Current concepts review: displaced intra-articular fractures of the calcaneus. J Bone Joint Surg
  64. Am. 2000;82-A:225-250.&amp;nbsp;&lt;/p&gt;&lt;p&gt;Sanders R, Fortin P, DiPasquale T, Walling A. Operative treatment in 120 displaced intraarticular calcaneal
  65. fractures. Results using a prognostic computed tomography scan classification. Clin Orthop.
  66. 1993;290:87-95.&amp;nbsp;&lt;/p&gt;&lt;p&gt;Sanders R, Gregory P. Operative treatment of intra-articular fractures of the calcaneus. Orthop Clin North
  67. Am. 1995;26:203-214.&lt;/p&gt;
  68. &lt;/details&gt;&lt;/div&gt;</content><link rel='replies' type='application/atom+xml' href='https://medical-phd.blogspot.com/feeds/6239165778581713238/comments/default' title='Post Comments'/><link rel='replies' type='text/html' href='https://medical-phd.blogspot.com/2022/03/calcaneus-fracture-case-file.html#comment-form' title='0 Comments'/><link rel='edit' type='application/atom+xml' href='https://www.blogger.com/feeds/3302710210518459416/posts/default/6239165778581713238'/><link rel='self' type='application/atom+xml' href='https://www.blogger.com/feeds/3302710210518459416/posts/default/6239165778581713238'/><link rel='alternate' type='text/html' href='https://medical-phd.blogspot.com/2022/03/calcaneus-fracture-case-file.html' title='Calcaneus Fracture Case File'/><author><name>Medical Group</name><uri>http://www.blogger.com/profile/01780656800578397393</uri><email>noreply@blogger.com</email><gd:image rel='http://schemas.google.com/g/2005#thumbnail' width='32' height='32' src='//blogger.googleusercontent.com/img/b/R29vZ2xl/AVvXsEiMuROkBDv6yw4SmxGqRA5BPQhOXSwFld-Gufmy4kUqfpoAGQRl7cv-3naew6wjAnCjRFSe_FKX8z7CdE6cFKxxBpnqBuwUTSr-15xd3mFGcr_wJOX2y73G_zJCGND98Q/s220/Final-logo.jpg'/></author><media:thumbnail xmlns:media="http://search.yahoo.com/mrss/" url="https://blogger.googleusercontent.com/img/b/R29vZ2xl/AVvXsEhWehiYZutupAtL7gScBdAneO_gw5WJ-mPsd47hEZ_ETKGWd8r_yAAZwwDfFZXLdoZ4Of_yGkrImomoJgVd4sNV4_LuUf4xot5GHWjzSkC62XY2W_6gVh1oKE0bzXsSAWH_SWhoscjyJgcpJuL580XUdg3NsR_5fWGoC_tUiCYhZ1D18PwOx-b2Jcl-ag/s72-w234-h400-c/AP-view-of-the-foot.jpg" height="72" width="72"/><thr:total>0</thr:total><georss:featurename>United States</georss:featurename><georss:point>37.09024 -95.712891</georss:point><georss:box>8.780006163821156 -130.869141 65.400473836178847 -60.556641</georss:box></entry><entry><id>tag:blogger.com,1999:blog-3302710210518459416.post-6466141448318133777</id><published>2022-03-31T19:46:00.003-07:00</published><updated>2022-03-31T19:46:19.018-07:00</updated><category scheme="http://www.blogger.com/atom/ns#" term="Case Files"/><category scheme="http://www.blogger.com/atom/ns#" term="Orthopaedic Surgery Case File"/><title type='text'>Lisfranc Injury Case File</title><content type='html'>&lt;div class=&quot;scrollbox&quot;&gt;
  69.  
  70. &lt;div style=&quot;text-align: left;&quot;&gt;&lt;b&gt;&lt;span style=&quot;font-size: large;&quot;&gt;Lisfranc Injury Case File&lt;/span&gt;&lt;/b&gt;&lt;/div&gt;&lt;div&gt;Eugene C. Toy, MD, Andrew J. Rosenbaum, MD, Timothy T. Roberts, MD, Joshua S. Dines, MD&lt;/div&gt;&lt;div&gt;&lt;br /&gt;&lt;/div&gt;&lt;div&gt;&lt;b&gt;&lt;span style=&quot;color: #017087; font-size: medium;&quot;&gt;CASE 36&lt;/span&gt;&lt;/b&gt;&lt;/div&gt;&lt;div&gt;&lt;div&gt;A 37-year-old man arrives in the emergency department after a motorcycle accident in which he hit a tree traveling at approximately 50 mph. He was helmeted and denies any loss of consciousness. His blood pressure is 130/80 mmHg, his heart rate is 90 beats/minute, and his temperature is 99°F. He complains of significant pain localized to his right foot. On physical exam of the right foot, the skin is found to be intact. However, there is significant swelling over the dorsum of the foot as well as plantar ecchymosis. His foot is tender, predominantly over the midfoot and forefoot. The patient will not move his toes secondary to pain. His sensation is grossly intact. The posterior tibial pulse is palpable, whereas the dorsalis pedis pulse is only appreciated with a Doppler probe. Capillary refill of the toes is brisk and less than 2 seconds. His pain does not worsen with passive dorsiflexion of the toes.&lt;/div&gt;&lt;div&gt;&lt;br /&gt;&lt;/div&gt;&lt;div&gt;&lt;span style=&quot;color: #017087;&quot;&gt;►&lt;/span&gt;&amp;nbsp;What initial radiographic studies are most appropriate?&lt;/div&gt;&lt;div&gt;&lt;span style=&quot;color: #017087;&quot;&gt;►&lt;/span&gt;&amp;nbsp;What is the most likely diagnosis?&lt;/div&gt;&lt;div&gt;&lt;span style=&quot;color: #017087;&quot;&gt;►&lt;/span&gt;&amp;nbsp;Given the mechanism of injury, what other considerations must be taken before initiating appropriate treatment for the patient’s foot?&lt;/div&gt;&lt;div&gt;&lt;span style=&quot;color: #017087;&quot;&gt;►&lt;/span&gt;&amp;nbsp;How is this injury classified?&lt;/div&gt;&lt;div&gt;&lt;span style=&quot;color: #017087;&quot;&gt;►&lt;/span&gt;&amp;nbsp;What is the most appropriate treatment for this patient?&lt;/div&gt;&lt;/div&gt;&lt;div&gt;&lt;br /&gt;&lt;/div&gt;&lt;div&gt;&lt;br /&gt;&lt;/div&gt;&lt;div&gt;&lt;div&gt;&lt;b&gt;&lt;span style=&quot;font-size: medium;&quot;&gt;ANSWER TO CASE 36:&lt;/span&gt;&lt;/b&gt;&lt;/div&gt;&lt;div&gt;&lt;span style=&quot;background-color: #daebf0; font-size: medium;&quot;&gt;&lt;span&gt;&lt;span style=&quot;background-color: transparent;&quot;&gt;&lt;span style=&quot;color: #017087;&quot;&gt;&lt;b&gt;Lisfranc Injury&lt;/b&gt;&lt;/span&gt;&lt;/span&gt;&lt;b&gt;&amp;nbsp;&amp;nbsp;&lt;/b&gt;&lt;/span&gt;&lt;span style=&quot;font-weight: bold;&quot;&gt;&amp;nbsp;&amp;nbsp; &amp;nbsp;&lt;/span&gt;&lt;span style=&quot;font-weight: bold;&quot;&gt;&amp;nbsp;&amp;nbsp; &amp;nbsp;&lt;/span&gt;&lt;span style=&quot;font-weight: bold;&quot;&gt;&amp;nbsp;&amp;nbsp; &amp;nbsp;&lt;/span&gt;&lt;span style=&quot;font-weight: bold;&quot;&gt;&amp;nbsp;&amp;nbsp; &amp;nbsp;&lt;/span&gt;&lt;span style=&quot;font-weight: bold;&quot;&gt;&amp;nbsp; &amp;nbsp;&lt;/span&gt;&lt;/span&gt;&lt;/div&gt;&lt;/div&gt;&lt;div&gt;&lt;div&gt;&lt;i&gt;&lt;br /&gt;&lt;/i&gt;&lt;/div&gt;&lt;div&gt;&lt;i&gt;Summary&lt;/i&gt;: A 37-year-old man complains of severe right foot pain after a motorcycle accident in which he hit a tree at approximately 50 mph. On physical exam of his foot, diffuse tenderness and dorsal swelling are noted, as is medial plantar ecchymosis.&lt;/div&gt;&lt;div&gt;&lt;ul style=&quot;text-align: left;&quot;&gt;&lt;li&gt;&lt;b&gt;Radiographs to order:&lt;/b&gt; Anteroposterior (AP), lateral, and 30-degree oblique views of the right foot.&lt;/li&gt;&lt;li&gt;&lt;b&gt;Most likely diagnosis: &lt;/b&gt;Tarsometatarsal joint complex injury, commonly referred to as a Lisfranc injury.&lt;/li&gt;&lt;li&gt;&lt;b&gt;Other considerations:&lt;/b&gt; Given the high-energy mechanism of injury, this patient must be managed with strict adherence to advanced trauma life support protocol. A complete trauma workup and evaluation should be performed before orthopaedic intervention for the left foot.&lt;/li&gt;&lt;li&gt;&lt;b&gt;Classification of Lisfranc injury:&lt;/b&gt; Although multiple classifications exist, none have been proven to be useful for determining treatment or prognosis. One commonly used descriptive classification describes homolateral, isolated, and divergent patterns.&lt;/li&gt;&lt;li&gt;&lt;b&gt;Treatment:&lt;/b&gt; Open-reduction internal fixation when soft tissue swelling subsides.&lt;/li&gt;&lt;/ul&gt;&lt;/div&gt;&lt;/div&gt;&lt;div&gt;&lt;br /&gt;&lt;/div&gt;&lt;div&gt;&lt;br /&gt;&lt;/div&gt;&lt;div&gt;&lt;div&gt;&lt;b&gt;&lt;span style=&quot;color: #017087; font-size: medium;&quot;&gt;ANALYSIS&lt;/span&gt;&lt;/b&gt;&lt;/div&gt;&lt;div&gt;&lt;b&gt;&lt;span style=&quot;font-size: medium;&quot;&gt;&lt;i&gt;Objectives&lt;/i&gt;&lt;/span&gt;&lt;/b&gt;&lt;/div&gt;&lt;/div&gt;&lt;div&gt;&lt;div&gt;&lt;ol style=&quot;text-align: left;&quot;&gt;&lt;li&gt;Be familiar with the general presentation of a Lisfranc injury.&lt;/li&gt;&lt;li&gt;Understand the workup and imaging studies required for diagnosis of a Lisfranc injury.&lt;/li&gt;&lt;li&gt;Know the treatment options for and complications associated with Lisfranc injuries.&lt;/li&gt;&lt;/ol&gt;&lt;/div&gt;&lt;/div&gt;&lt;div&gt;&lt;br /&gt;&lt;/div&gt;&lt;div&gt;&lt;br /&gt;&lt;/div&gt;&lt;div&gt;&lt;b&gt;&lt;i&gt;&lt;span style=&quot;font-size: medium;&quot;&gt;Considerations&lt;/span&gt;&lt;/i&gt;&lt;/b&gt;&lt;/div&gt;&lt;div&gt;&lt;div&gt;&lt;br /&gt;&lt;/div&gt;&lt;div&gt;This patient’s history and presentation are common for an injury to the tarsometatarsal (TMT) joint complex. The diffuse pain and swelling, specifically over the midfoot and forefoot, are consistent with a high-energy injury mechanism, such as a motorcycle accident, which is often associated with Lisfranc injuries. &lt;b&gt;Additionally,&amp;nbsp;&lt;/b&gt;&lt;b&gt;the medial plantar ecchymosis is often thought to be pathognomonic for this injury.&amp;nbsp;&lt;/b&gt;The absence of a palpable dorsalis pedis pulse is likely due to the soft tissue swelling over the dorsum of the foot, another common finding. Although the patient’s clinical picture is consistent with a Lisfranc injury, &lt;b&gt;up to 20% of these initially go&amp;nbsp;&lt;/b&gt;&lt;b&gt;undiagnosed.&lt;/b&gt; The swelling is concerning, as compartment syndrome of the foot can be seen in association with injuries to the TMT joint complex. The practitioner must carefully and serially examine the patient in this setting and even consider measuring compartment pressures if they believe an acute compartment syndrome is present. The orthopaedic workup of this patient should also include imaging of the right foot, including AP, lateral, and oblique views. A secondary survey should also be performed to rule out additional musculoskeletal injuries requiring care. In the emergency department, this patient’s foot should be splinted, iced, and elevated to control swelling, as the timing of potential operative intervention is dependent on this. Soft tissue swelling can delay operative treatment of Lisfranc injuries by up to several weeks. &lt;b&gt;In addition to an orthopaedic surgery consultation, this patient&amp;nbsp;&lt;/b&gt;&lt;b&gt;requires an evaluation by the trauma team; intracranial, thoracic, abdominal, and&amp;nbsp;&lt;/b&gt;&lt;b&gt;spine injuries can be present concurrently.&lt;/b&gt;&lt;/div&gt;&lt;/div&gt;&lt;div&gt;&lt;br /&gt;&lt;/div&gt;&lt;div&gt;&lt;br /&gt;&lt;/div&gt;&lt;div&gt;&lt;div&gt;&lt;div&gt;&lt;b&gt;&lt;span style=&quot;font-size: medium;&quot;&gt;APPROACH TO:&lt;/span&gt;&lt;/b&gt;&lt;/div&gt;&lt;div&gt;&lt;span style=&quot;background-color: #daebf0; color: #017087; font-size: medium;&quot;&gt;&lt;span&gt;&lt;span style=&quot;background-color: transparent;&quot;&gt;&lt;b&gt;Lisfranc Injuries&lt;/b&gt;&lt;/span&gt;&lt;b&gt;&amp;nbsp; &amp;nbsp;&amp;nbsp;&lt;/b&gt;&lt;/span&gt;&lt;span style=&quot;font-weight: bold;&quot;&gt;&amp;nbsp;&amp;nbsp; &amp;nbsp;&lt;/span&gt;&lt;span style=&quot;font-weight: bold;&quot;&gt;&amp;nbsp;&amp;nbsp; &amp;nbsp;&lt;/span&gt;&lt;span style=&quot;font-weight: bold;&quot;&gt;&amp;nbsp;&amp;nbsp; &amp;nbsp;&lt;/span&gt;&lt;span style=&quot;font-weight: bold;&quot;&gt;&amp;nbsp;&amp;nbsp; &amp;nbsp;&lt;/span&gt;&lt;span style=&quot;font-weight: bold;&quot;&gt;&amp;nbsp;&amp;nbsp; &amp;nbsp;&lt;/span&gt;&lt;span style=&quot;font-weight: bold;&quot;&gt;&amp;nbsp;&amp;nbsp; &amp;nbsp;&lt;/span&gt;&lt;span style=&quot;font-weight: bold;&quot;&gt;&amp;nbsp;&amp;nbsp;&amp;nbsp;&lt;/span&gt;&lt;span style=&quot;font-weight: bold;&quot;&gt;&amp;nbsp;&lt;/span&gt;&lt;/span&gt;&lt;/div&gt;&lt;/div&gt;&lt;div&gt;&lt;br /&gt;&lt;/div&gt;&lt;div&gt;&lt;b&gt;&lt;span style=&quot;color: #017087; font-size: medium;&quot;&gt;DEFINITIONS&lt;/span&gt;&lt;/b&gt;&lt;/div&gt;&lt;/div&gt;&lt;div&gt;&lt;br /&gt;&lt;/div&gt;&lt;div&gt;&lt;div&gt;&lt;b&gt;JACQUES LISFRANC:&lt;/b&gt; A field surgeon in Napoleon’s army. He described an amputation involving the TMT joint due to a severe gangrene that developed when a soldier fell from a horse with his foot caught in a stirrup.&lt;/div&gt;&lt;div&gt;&lt;b&gt;&lt;br /&gt;&lt;/b&gt;&lt;/div&gt;&lt;div&gt;&lt;b&gt;LISFRANC JOINT:&lt;/b&gt; This refers to the TMT joint complex, an osseous and capsule- ligamentous network that includes the 5 metatarsals (MTs), their articulations with the cuneiforms and cuboid, and the Lisfranc ligament, a strong interosseous attachment between the medial cuneiform and second MT.&lt;/div&gt;&lt;div&gt;&lt;b&gt;&lt;br /&gt;&lt;/b&gt;&lt;/div&gt;&lt;div&gt;&lt;b&gt;LISFRANC INJURY:&lt;/b&gt; Refers to a spectrum of processes involving the TMT joint complex, including both fractures and dislocation. It does not delineate a specific injury.&lt;/div&gt;&lt;div&gt;&lt;b&gt;&lt;br /&gt;&lt;/b&gt;&lt;/div&gt;&lt;div&gt;&lt;b&gt;HOMOLATERAL LISFRANC INJURY:&lt;/b&gt; All 5 metatarsals are displaced in the same direction.&lt;/div&gt;&lt;div&gt;&lt;b&gt;&lt;br /&gt;&lt;/b&gt;&lt;/div&gt;&lt;div&gt;&lt;b&gt;ISOLATED LISFRANC INJURY:&lt;/b&gt; 1 or 2 metatarsals are displaced from the others.&lt;/div&gt;&lt;div&gt;&lt;b&gt;&lt;br /&gt;&lt;/b&gt;&lt;/div&gt;&lt;div&gt;&lt;b&gt;DIVERGENT LISFRANC INJURY:&lt;/b&gt; The metatarsals are displaced in both the coronal and sagittal planes.&lt;/div&gt;&lt;div&gt;&lt;b&gt;&lt;br /&gt;&lt;/b&gt;&lt;/div&gt;&lt;div&gt;&lt;b&gt;COMPARTMENT SYNDROME:&lt;/b&gt; A surgical emergency that occurs as a result of increased pressure within a muscle compartment, at times compromising blood flow and damaging nerves, muscle, and surrounding soft tissue structures. It is often encountered in the setting of trauma and extremity fractures. Although a clinical diagnosis, intracompartmental pressure measurements of greater than 30 mmHg or a less than 30-mmHg difference between the patient’s intracompartmental pressure and diastolic blood pressure are consistent with the diagnosis of compartment syndrome (see Case 13).&lt;/div&gt;&lt;/div&gt;&lt;div&gt;&lt;br /&gt;&lt;/div&gt;&lt;div&gt;&lt;br /&gt;&lt;/div&gt;&lt;div&gt;&lt;b&gt;&lt;span style=&quot;color: #017087; font-size: medium;&quot;&gt;CLINICAL APPROACH&lt;/span&gt;&lt;/b&gt;&lt;/div&gt;&lt;div&gt;&lt;br /&gt;&lt;/div&gt;&lt;div&gt;&lt;div&gt;&lt;b&gt;&lt;i&gt;&lt;span style=&quot;font-size: medium;&quot;&gt;Anatomy&lt;/span&gt;&lt;/i&gt;&lt;/b&gt;&lt;/div&gt;&lt;div&gt;&lt;br /&gt;&lt;/div&gt;&lt;div&gt;The Lisfranc joint divides the midfoot and the forefoot, forming an oblique line running from the lateral aspect of the proximal forefoot to the medial aspect of the distal forefoot. Bony elements provide the primary structural support to this articulation, with ligamentous contributions adding additional stability. The 3 medial MTs articulate with the 3 cuneiforms; the fourth and fifth MTs articulate with the cuboid. The middle cuneiform–second MT articulation forms the keystone of the arch, preventing mediolateral MT motion at the Lisfranc joint. This transverse arch also prevents plantar displacement of the 3 medial MTs. The most significant and strongest ligamentous structure is the &lt;b&gt;oblique interosseous ligament,&lt;/b&gt; referred to as Lisfranc ligament. Originating on the lateral surface of the medial cuneiform, it passes in front of the intercuneiform ligament and ultimately inserts on the medial aspect of the second MT base near the plantar surface.&lt;/div&gt;&lt;div&gt;&lt;br /&gt;&lt;/div&gt;&lt;div&gt;&lt;b&gt;&lt;i&gt;&lt;span style=&quot;font-size: medium;&quot;&gt;Mechanism&lt;/span&gt;&lt;/i&gt;&lt;/b&gt;&lt;/div&gt;&lt;div&gt;&lt;br /&gt;&lt;/div&gt;&lt;div&gt;Direct and indirect injury patterns can damage the Lisfranc joint. Direct mechanisms,&amp;nbsp;&lt;b&gt;most commonly crush injuries,&lt;/b&gt; are due to high-energy blunt trauma to the dorsum of the foot. These often result in worse clinical outcome as compared with indirect types, secondary to the associated soft tissue trauma. The indirect mechanism usually involves axial loading of a plantarflexed foot, external rotation on a pronated forefoot, or an abduction stress to the midfoot. Indirect mechanisms are stratified into high-energy and low-energy subtypes. Motor vehicle accidents are the most common cause of high-energy Lisfranc injury. &lt;b&gt;Low-energy injuries include&amp;nbsp;&lt;/b&gt;&lt;b&gt;those incurred during athletic competition.&lt;/b&gt; The fracture pattern and direction of dislocation in direct injuries are dependent on the force vector applied. Indirect injuries are more predictable and most commonly involve failure of the weaker dorsal TMT ligaments in tension with subsequent dorsal or dorsolateral MT dislocation.&lt;/div&gt;&lt;/div&gt;&lt;div&gt;&lt;br /&gt;&lt;/div&gt;&lt;div&gt;&lt;br /&gt;&lt;/div&gt;&lt;div&gt;&lt;div&gt;&lt;b&gt;&lt;i&gt;&lt;span style=&quot;font-size: medium;&quot;&gt;Workup and Diagnosis&lt;/span&gt;&lt;/i&gt;&lt;/b&gt;&lt;/div&gt;&lt;div&gt;&lt;br /&gt;&lt;/div&gt;&lt;div&gt;The diagnosis of high-energy Lisfranc injuries is straightforward; physical exam will reveal swelling and many times obvious deformity, including widening or flattening of the forefoot. Additionally, it might be an open injury with disruption of the skin and subcutaneous tissue. TMT joint injury and intercuneiform disruption is suggested in the presence of a gap between the first and second toes, known as a &lt;b&gt;positive gap&amp;nbsp;&lt;/b&gt;&lt;b&gt;sign.&lt;/b&gt; Although associated vascular injury is rare, it may be difficult to palpate a dorsalis pedis pulse secondary to swelling. The clinician must also consider a concurrent&amp;nbsp;&lt;b&gt;compartment syndrome&lt;/b&gt; with severe swelling and pain with passive dorsiflexion of the toes. If unsure, measurement of pressures is warranted.&lt;/div&gt;&lt;/div&gt;&lt;div&gt;&lt;br /&gt;&lt;/div&gt;&lt;div&gt;&lt;div&gt;&lt;span&gt;&amp;nbsp;&amp;nbsp; &amp;nbsp;&lt;/span&gt;In the setting of a low-energy injury, physical exam may only reveal a patient with an inability to bear weight and possibly midfoot and forefoot swelling. Paying attention to the reported mechanism and the clinical appearance of the foot is pivotal to making the correct diagnosis. An additional finding may be &lt;b&gt;plantar arch&amp;nbsp;&lt;/b&gt;&lt;b&gt;ecchymosis,&lt;/b&gt; which is considered pathognomonic for Lisfranc injury. To increase the accuracy of diagnosis, various tests and stress maneuvers have been described to aid in the diagnosis of subtle injury. Of note, pain on passive abduction and pronation of the forefoot is suggestive of injury to the TMT complex.&lt;/div&gt;&lt;/div&gt;&lt;div&gt;&lt;br /&gt;&lt;/div&gt;&lt;div&gt;&lt;div&gt;&lt;span&gt;&amp;nbsp;&amp;nbsp; &amp;nbsp;&lt;/span&gt;Initial imaging to obtain include nonweightbearing anteroposterior (AP), lateral, and 30-degree oblique views of the foot ( Figures 36–1A , 36–1B , and 36–1C ).&lt;/div&gt;&lt;/div&gt;&lt;div&gt;&lt;br /&gt;&lt;/div&gt;&lt;div class=&quot;separator&quot; style=&quot;clear: both; text-align: center;&quot;&gt;&lt;a href=&quot;https://blogger.googleusercontent.com/img/b/R29vZ2xl/AVvXsEilszp7yJNTc-_Bk9qfkgQfON2jz1mQcJItlWHfYDhd7YmzJJ09ggfLKSshfqTyk09-vyIhGIEIK8ah2ikbQttV7a7N3w9WmcvU_PiYHIny3I7oQ1UtXgf3-j8wGTANZNWUsnpssFVxPzrwVwG92mMHZWrMIBLLGErRjkpZGLK7X-iEuUxQrZhcVVeKDw/s424/AP-Lisfranc-Injury.jpg&quot; style=&quot;margin-left: 1em; margin-right: 1em;&quot;&gt;&lt;img alt=&quot;AP Lisfranc Injury&quot; border=&quot;0&quot; data-original-height=&quot;424&quot; data-original-width=&quot;325&quot; height=&quot;400&quot; src=&quot;https://blogger.googleusercontent.com/img/b/R29vZ2xl/AVvXsEilszp7yJNTc-_Bk9qfkgQfON2jz1mQcJItlWHfYDhd7YmzJJ09ggfLKSshfqTyk09-vyIhGIEIK8ah2ikbQttV7a7N3w9WmcvU_PiYHIny3I7oQ1UtXgf3-j8wGTANZNWUsnpssFVxPzrwVwG92mMHZWrMIBLLGErRjkpZGLK7X-iEuUxQrZhcVVeKDw/w306-h400/AP-Lisfranc-Injury.jpg&quot; title=&quot;AP Lisfranc Injury&quot; width=&quot;306&quot; /&gt;&lt;/a&gt;&lt;/div&gt;&lt;br /&gt;&lt;div&gt;&lt;div class=&quot;separator&quot; style=&quot;clear: both; text-align: center;&quot;&gt;&lt;a href=&quot;https://blogger.googleusercontent.com/img/b/R29vZ2xl/AVvXsEh56GfKCRq8ImvYa6LEqCLAdD85QLhph4D9vTBGc2la5DvkaL_RX8WxAMephGDZh0UpeFv5_KEGRlgM6qdmJRPXlb09RRAHtB5_K6CowBJ6m0v2ltmNABkRjhJjZ2bSR827-uIcxTiGVLYQVGGyMiyV11c8GigofzAon8rlvuexZFexN4Q7a0bLLe4KAg/s426/lateral-Lisfranc-Injury.jpg&quot; style=&quot;margin-left: 1em; margin-right: 1em;&quot;&gt;&lt;img alt=&quot;lateral Lisfranc Injury&quot; border=&quot;0&quot; data-original-height=&quot;426&quot; data-original-width=&quot;289&quot; height=&quot;400&quot; src=&quot;https://blogger.googleusercontent.com/img/b/R29vZ2xl/AVvXsEh56GfKCRq8ImvYa6LEqCLAdD85QLhph4D9vTBGc2la5DvkaL_RX8WxAMephGDZh0UpeFv5_KEGRlgM6qdmJRPXlb09RRAHtB5_K6CowBJ6m0v2ltmNABkRjhJjZ2bSR827-uIcxTiGVLYQVGGyMiyV11c8GigofzAon8rlvuexZFexN4Q7a0bLLe4KAg/w271-h400/lateral-Lisfranc-Injury.jpg&quot; title=&quot;lateral Lisfranc Injury&quot; width=&quot;271&quot; /&gt;&lt;/a&gt;&lt;/div&gt;&lt;br /&gt;&lt;/div&gt;&lt;div class=&quot;separator&quot; style=&quot;clear: both; text-align: center;&quot;&gt;&lt;a href=&quot;https://blogger.googleusercontent.com/img/b/R29vZ2xl/AVvXsEgQ80LkGnlTdCMeELsxMyYpuzcY9jG-IWp30iiXjU23ZK98WAXiIUCL-p7o7pDHYawJTjzqlVSGR81LijRWgdhqOUZlA25r_NOmcyeBba1_xv7LMkt4sl2hTy0xER3WSZ3UT6jscIogqByNBbwigqYrqAvv5B77Thl8D01foN2hYG10xnf-xPkMTepwuQ/s427/oblique-radiographs-Lisfranc-Injury.jpg&quot; style=&quot;margin-left: 1em; margin-right: 1em;&quot;&gt;&lt;img alt=&quot;oblique radiographs Lisfranc Injury&quot; border=&quot;0&quot; data-original-height=&quot;427&quot; data-original-width=&quot;289&quot; height=&quot;400&quot; src=&quot;https://blogger.googleusercontent.com/img/b/R29vZ2xl/AVvXsEgQ80LkGnlTdCMeELsxMyYpuzcY9jG-IWp30iiXjU23ZK98WAXiIUCL-p7o7pDHYawJTjzqlVSGR81LijRWgdhqOUZlA25r_NOmcyeBba1_xv7LMkt4sl2hTy0xER3WSZ3UT6jscIogqByNBbwigqYrqAvv5B77Thl8D01foN2hYG10xnf-xPkMTepwuQ/w271-h400/oblique-radiographs-Lisfranc-Injury.jpg&quot; title=&quot;oblique radiographs Lisfranc Injury&quot; width=&quot;271&quot; /&gt;&lt;/a&gt;&lt;/div&gt;&lt;div&gt;&lt;br /&gt;&lt;/div&gt;&lt;div&gt;&lt;div&gt;However, 50% of subtle Lisfranc injuries will have normal nonweightbearing imaging. Thus to diagnose these injuries, a &lt;b&gt;weightbearing film&lt;/b&gt; with both feet on a single cassette or an &lt;b&gt;abduction-pronation stress view&lt;/b&gt; should be obtained. Admittedly, these are rarely obtained secondary to pain, necessitating a bone scan or magnetic resonance imaging. Computed tomography may be beneficial as a diagnostic adjunct, as it is effective at delineating fractures.&lt;/div&gt;&lt;/div&gt;&lt;div&gt;&lt;br /&gt;&lt;/div&gt;&lt;div&gt;&lt;div&gt;&lt;b&gt;&lt;span&gt;&amp;nbsp;&amp;nbsp; &amp;nbsp;&lt;/span&gt;On the AP radiograph, findings suggestive of a Lisfranc injury include incongruity at the first and second MT joints, misalignment of the medial border of the second&amp;nbsp;&lt;/b&gt;&lt;b&gt;MT and medial border of the middle cuneiform, and a diastasis of ê 2 mm between&amp;nbsp;&lt;/b&gt;&lt;b&gt;the first and second MTs as compared with the contralateral foot ( Figure 36–1A ).&amp;nbsp;&lt;/b&gt;The oblique radiograph should show alignment of the medial border of the fourth MT and the medial border of the cuboid in a normal foot ( Figure 36–1C ). Misalignment may suggest a TMT joint complex injury. Potentially seen on this view as well as the AP is the &lt;b&gt;“fleck” sign,&lt;/b&gt; as described by Myerson et al. It refers to the presence of a small bony fragment between the base of the second MT and the medial cuneiform and represents an avulsion of either the proximal or distal attachment of the Lisfranc ligament. The lateral radiograph will show flattening of the longitudinal arch and/or dorsal displacement at the second TMT joint ( Figure 36–1B ). The fifth MT is normally plantar in relation to the medial cuneiform. Flattening of the midfoot arch positions the medial cuneiform plantar to the fifth MT.&lt;/div&gt;&lt;/div&gt;&lt;div&gt;&lt;br /&gt;&lt;/div&gt;&lt;div&gt;&lt;br /&gt;&lt;/div&gt;&lt;div&gt;&lt;b style=&quot;color: #017087;&quot;&gt;&lt;span style=&quot;font-size: medium;&quot;&gt;TREATMENT&lt;/span&gt;&lt;/b&gt;&lt;/div&gt;&lt;div&gt;&lt;br /&gt;&lt;/div&gt;&lt;div&gt;&lt;div&gt;Indications for both operative and nonoperative treatment exist, all with the goal of reestablishing a painless and stable foot. Any disruption of normal anatomy warrants surgical correction. Nonoperative treatment is limited to stable TMT joint complex injuries and include those that are nondisplaced, without fracture, and stable under radiographic stress examination. Treatment involves protected weightbearing in a Cam Walker® boot, with frequent follow-up radiographs to ensure no change in alignment. It takes approximately 4 months to recover from a nonsurgical Lisfranc injury.&lt;/div&gt;&lt;div&gt;&lt;br /&gt;&lt;/div&gt;&lt;div&gt;&lt;span&gt;&amp;nbsp;&amp;nbsp; &amp;nbsp;&lt;/span&gt;In unstable Lisfranc injuries, open-reduction internal fixation is performed to restore anatomic alignment. Commonly, screw fixation is used to fix the first, second, and third TMT joints, whereas the fourth and fifth may be pinned with K-wires. Although this is one common approach to operative management, many forms of fixation are available and depend on both the nature of the injury and the surgeon’s preference. Although controversial, TMT joint arthrodesis is recommended by some for purely ligamentous Lisfranc injuries. Traditionally, this procedure was considered a salvage operation for failed ORIF and in cases of posttraumatic arthritis. Postoperatively, protected weightbearing for 3 to 5 months and therapy emphasizing passive midfoot range of motion are allowed. Regardless of the modality, anatomic alignment is the standard to decrease the risk of posttraumatic arthritis, chronic instability, and pain.&lt;/div&gt;&lt;/div&gt;&lt;div&gt;&lt;br /&gt;&lt;/div&gt;&lt;div&gt;&lt;br /&gt;&lt;/div&gt;&lt;div&gt;&lt;b&gt;&lt;span style=&quot;color: #017087; font-size: medium;&quot;&gt;&lt;u&gt;COMPREHENSION QUESTIONS&lt;/u&gt;&lt;/span&gt;&lt;/b&gt;&lt;/div&gt;&lt;div&gt;&lt;br /&gt;&lt;/div&gt;&lt;div&gt;&lt;div&gt;36.1 A 40-year-old equestrian complains of right midfoot pain associated with difficulty ambulating for the last 2 days, when her foot got caught in a stirrup while riding. Radiographs show a 4-mm diastasis between the first and second metatarsals. What is the most appropriate treatment?&lt;/div&gt;&lt;/div&gt;&lt;blockquote style=&quot;border: none; margin: 0px 0px 0px 40px; padding: 0px; text-align: left;&quot;&gt;&lt;div&gt;&lt;div&gt;A. Open-reduction internal fixation&lt;/div&gt;&lt;/div&gt;&lt;div&gt;&lt;div&gt;B. Nonweightbearing in a short leg cast&lt;/div&gt;&lt;/div&gt;&lt;div&gt;&lt;div&gt;C. Weightbearing as tolerated in an aircast&lt;/div&gt;&lt;/div&gt;&lt;div&gt;&lt;div&gt;D. Chevron osteotomy&lt;/div&gt;&lt;/div&gt;&lt;/blockquote&gt;&lt;div&gt;&lt;div&gt;&lt;br /&gt;&lt;/div&gt;&lt;div&gt;36.2 A football player who is lying on the ground after being tackled attempts to stand up. While he is still prone on the ground, another player falls directly on his left heel. He immediately experiences midfoot pain and is unable to place any weight on his left foot. In this setting, what should the team physician be most concerned about?&lt;/div&gt;&lt;/div&gt;&lt;blockquote style=&quot;border: none; margin: 0px 0px 0px 40px; padding: 0px; text-align: left;&quot;&gt;&lt;div&gt;&lt;div&gt;A. High ankle sprain&lt;/div&gt;&lt;/div&gt;&lt;div&gt;&lt;div&gt;B. Achilles tendon rupture&lt;/div&gt;&lt;/div&gt;&lt;div&gt;&lt;div&gt;C. Lisfranc injury&lt;/div&gt;&lt;/div&gt;&lt;div&gt;&lt;div&gt;D. Ankle fracture&lt;/div&gt;&lt;/div&gt;&lt;/blockquote&gt;&lt;div&gt;&lt;div&gt;&lt;br /&gt;&lt;/div&gt;&lt;div&gt;36.3 A 54-year-old woman sustains a twisting injury to her foot. An AP radiograph of the foot reveals a 4-mm diastasis between the first and second metatarsals. Which structure connects the medial cuneiform to the base of the second metatarsal?&lt;/div&gt;&lt;/div&gt;&lt;blockquote style=&quot;border: none; margin: 0px 0px 0px 40px; padding: 0px; text-align: left;&quot;&gt;&lt;div&gt;&lt;div&gt;A. Chopart ligament&lt;/div&gt;&lt;/div&gt;&lt;div&gt;&lt;div&gt;B. Deltoid ligament&lt;/div&gt;&lt;/div&gt;&lt;div&gt;&lt;div&gt;C. Lisfranc ligament&lt;/div&gt;&lt;/div&gt;&lt;div&gt;&lt;div&gt;D. Spring ligament&lt;/div&gt;&lt;/div&gt;&lt;/blockquote&gt;&lt;div&gt;&lt;br /&gt;&lt;/div&gt;&lt;div&gt;&lt;br /&gt;&lt;/div&gt;&lt;div&gt;&lt;b&gt;&lt;span style=&quot;color: #017087; font-size: medium;&quot;&gt;&lt;u&gt;ANSWERS&lt;/u&gt;&lt;/span&gt;&lt;/b&gt;&lt;/div&gt;&lt;div&gt;&lt;br /&gt;&lt;/div&gt;&lt;div&gt;&lt;div&gt;36.1 &lt;b&gt;A.&lt;/b&gt; The patient has sustained an injury to her TMT joint complex. The Lisfranc ligament, which connects the base of the second metatarsal to the medial cuneiform, has been disrupted. Anatomic reduction of the Lisfranc joint is necessary and in this case requires open reduction and internal fixation. Postoperatively, protected weightbearing for 3 to 5 months and therapy emphasizing passive midfoot range of motion are allowed. Posttraumatic arthritis, midfoot instability, and pain are the long-term outcomes of a nonreduced joint.&lt;/div&gt;&lt;/div&gt;&lt;div&gt;&lt;br /&gt;&lt;/div&gt;&lt;div&gt;&lt;div&gt;36.2 &lt;b&gt;C.&lt;/b&gt; Lisfranc injuries are common in football players, with up to approximately 4% of American football players sustaining them each season. External rotation on a pronated forefoot is responsible for this and can lead to an unstable Lisfranc ligamentous injury. Other sports in which TMT joint complex injuries are encountered include equestrian events and windsurfing, in which the use of a stirrup can lead to one’s forefoot being abducted around a fixed hindfoot. When this occurs, dislocation of the second metatarsal and lateral displacement of the other metatarsals occurs.&lt;/div&gt;&lt;div&gt;&lt;br /&gt;&lt;/div&gt;&lt;div&gt;36.3 &lt;b&gt;C.&lt;/b&gt; This woman sustained a TMT joint complex injury, as evidenced by the large diastasis between the first and second metatarsals. The most significant and strongest ligamentous structure of the TMT joint complex is the oblique interosseous ligament, more commonly referred to as the Lisfranc ligament. Originating on the lateral surface of the medial cuneiform, it passes in front of the intercuneiform ligament and ultimately inserts on the medial aspect of the second MT base near the plantar surface. A, B, and D are incorrect—the Chopart, or bifurcate, ligament provides stability to the calcaneocuboid joint; the Deltoid ligament stabilizes the medial side of the ankle; and the Spring ligament stabilizes the talonavicular joint.&lt;/div&gt;&lt;/div&gt;&lt;div&gt;&lt;br /&gt;&lt;/div&gt;&lt;div&gt;&lt;b style=&quot;background-color: #4093a5;&quot;&gt;&amp;nbsp; &amp;nbsp;&lt;span style=&quot;font-size: medium;&quot;&gt;&amp;nbsp;&lt;/span&gt;&lt;span style=&quot;color: white;&quot;&gt;&lt;span style=&quot;font-size: medium;&quot;&gt;CLINICAL PEARLS&lt;/span&gt;&lt;span&gt;&amp;nbsp; &amp;nbsp;&amp;nbsp;&lt;/span&gt;&lt;/span&gt;&lt;/b&gt;&lt;/div&gt;&lt;div&gt;&lt;table border=&quot;1&quot; cellpadding=&quot;0&quot; cellspacing=&quot;0&quot; class=&quot;MsoTableGrid&quot; style=&quot;border-collapse: collapse; border: none; mso-border-alt: solid windowtext .5pt; mso-padding-alt: 0in 5.4pt 0in 5.4pt; mso-yfti-tbllook: 1184;&quot;&gt;
  71. &lt;tbody&gt;&lt;tr style=&quot;height: 19.15pt; mso-yfti-firstrow: yes; mso-yfti-irow: 0; mso-yfti-lastrow: yes;&quot;&gt;
  72.  &lt;td style=&quot;background: rgb(221, 237, 239); border-left: solid #017087 1.0pt; border: 1pt solid rgb(64, 147, 165); height: 19.15pt; padding: 0in 5.4pt; width: 433.7pt;&quot; valign=&quot;top&quot; width=&quot;578&quot;&gt;
  73.  &lt;div style=&quot;line-height: normal; margin-bottom: 0in;&quot;&gt;&lt;div style=&quot;line-height: normal; margin-bottom: 0in;&quot;&gt;&lt;br /&gt;&lt;/div&gt;&lt;div style=&quot;line-height: normal; margin-bottom: 0in;&quot;&gt;&lt;span style=&quot;color: #017087;&quot;&gt;►&lt;/span&gt;&amp;nbsp;The Lisfranc joint refers to the TMT joint complex, an osseous and capsuloligamentous network that includes the 5 metatarsals, their articulations with the cuneiforms and cuboid, and the Lisfranc ligament.&lt;/div&gt;&lt;div style=&quot;line-height: normal; margin-bottom: 0in;&quot;&gt;&lt;span style=&quot;color: #017087;&quot;&gt;&lt;br /&gt;&lt;/span&gt;&lt;/div&gt;&lt;div style=&quot;line-height: normal; margin-bottom: 0in;&quot;&gt;&lt;span style=&quot;color: #017087;&quot;&gt;►&lt;/span&gt;&amp;nbsp;Radiographic findings indicative of a Lisfranc injury include diastasis of ≥ 2 mm between the first and second MTs as compared with the contralateral foot, the fleck sign, flattening of the longitudinal arch of the foot, and dorsal displacement at the second TMT joint.&lt;/div&gt;&lt;div style=&quot;line-height: normal; margin-bottom: 0in;&quot;&gt;&lt;span style=&quot;color: #017087;&quot;&gt;&lt;br /&gt;&lt;/span&gt;&lt;/div&gt;&lt;div style=&quot;line-height: normal; margin-bottom: 0in;&quot;&gt;&lt;span style=&quot;color: #017087;&quot;&gt;►&lt;/span&gt;&amp;nbsp;The goal of both operative and nonoperative management of Lisfranc injuries is restoration of the normal anatomy of the TMT joint complex.&lt;/div&gt;&lt;div style=&quot;line-height: normal; margin-bottom: 0in;&quot;&gt;&lt;span style=&quot;color: #017087;&quot;&gt;&lt;br /&gt;&lt;/span&gt;&lt;/div&gt;&lt;div style=&quot;line-height: normal; margin-bottom: 0in;&quot;&gt;&lt;span style=&quot;color: #017087;&quot;&gt;►&lt;/span&gt;&amp;nbsp;Open-reduction internal fixation is the predominant surgical intervention for unstable Lisfranc injuries.&lt;/div&gt;&lt;div style=&quot;line-height: normal; margin-bottom: 0in;&quot;&gt;&lt;span style=&quot;color: #017087;&quot;&gt;&lt;br /&gt;&lt;/span&gt;&lt;/div&gt;&lt;div style=&quot;line-height: normal; margin-bottom: 0in;&quot;&gt;&lt;span style=&quot;color: #017087;&quot;&gt;►&lt;/span&gt;&amp;nbsp;Posttraumatic arthritis, chronic instability, and pain are the long-term complications associated with inadequate and nonanatomic reduction of the Lisfranc joint and its structures.&lt;/div&gt;&lt;div style=&quot;line-height: normal; margin-bottom: 0in;&quot;&gt;&lt;br /&gt;&lt;/div&gt;&lt;/div&gt;
  74.  &lt;/td&gt;
  75. &lt;/tr&gt;
  76. &lt;/tbody&gt;&lt;/table&gt;&lt;br /&gt;&lt;/div&gt;
  77.  
  78. &lt;details close=&quot;&quot;&gt;
  79.  &lt;summary&gt;&lt;b&gt;&lt;span style=&quot;color: #00788b;&quot;&gt;REFERENCES&lt;/span&gt;&lt;/b&gt;&lt;/summary&gt;
  80.  &lt;p&gt;Davies MS, Saxby TS. Intercuneiform instability and the “gap” sign. Foot Ankle Int. 1999;20:606-609.&amp;nbsp;&lt;/p&gt;&lt;p&gt;Myerson MS, Fisher RT, Burgess AR, Kenzora JE. Fracture dislocations of the tarsometatarsal joints: end
  81. results correlated with pathology and treatment. Foot Ankle. 1986;5:225-242.&amp;nbsp;&lt;/p&gt;&lt;p&gt;Nunley JA, Vertullo CJ. Classification, investigation, and management of midfoot sprains: Lisfranc injuries
  82. in the athlete. Am J Sports Med. 2002;30:871-878.&amp;nbsp;&lt;/p&gt;&lt;p&gt;Ross G, Cronin R, Hauzenblas J, Juliano P. Plantar ecchymosis sign: a clinical aid to diagnose occult
  83. Lisfranc tarsometatarsal injuries. J Orthop Trauma. 1996;10:119-122.&amp;nbsp;&lt;/p&gt;&lt;p&gt;Thompson MC, Mormino MA. Injury to the tarsometatarsal joint complex. J Am Acad Orthop Surg.
  84. 2003;11:260-267.&amp;nbsp;&lt;/p&gt;&lt;p&gt;Watson TS, Shurnas PS, Denker J. Treatment of Lisfranc joint injury: current concepts. J Am Acad
  85. Orthop Surg. 2010;18:718-728.&lt;/p&gt;
  86. &lt;/details&gt;&lt;/div&gt;</content><link rel='replies' type='application/atom+xml' href='https://medical-phd.blogspot.com/feeds/6466141448318133777/comments/default' title='Post Comments'/><link rel='replies' type='text/html' href='https://medical-phd.blogspot.com/2022/03/lisfranc-injury-case-file.html#comment-form' title='0 Comments'/><link rel='edit' type='application/atom+xml' href='https://www.blogger.com/feeds/3302710210518459416/posts/default/6466141448318133777'/><link rel='self' type='application/atom+xml' href='https://www.blogger.com/feeds/3302710210518459416/posts/default/6466141448318133777'/><link rel='alternate' type='text/html' href='https://medical-phd.blogspot.com/2022/03/lisfranc-injury-case-file.html' title='Lisfranc Injury Case File'/><author><name>Medical Group</name><uri>http://www.blogger.com/profile/01780656800578397393</uri><email>noreply@blogger.com</email><gd:image rel='http://schemas.google.com/g/2005#thumbnail' width='32' height='32' src='//blogger.googleusercontent.com/img/b/R29vZ2xl/AVvXsEiMuROkBDv6yw4SmxGqRA5BPQhOXSwFld-Gufmy4kUqfpoAGQRl7cv-3naew6wjAnCjRFSe_FKX8z7CdE6cFKxxBpnqBuwUTSr-15xd3mFGcr_wJOX2y73G_zJCGND98Q/s220/Final-logo.jpg'/></author><media:thumbnail xmlns:media="http://search.yahoo.com/mrss/" url="https://blogger.googleusercontent.com/img/b/R29vZ2xl/AVvXsEilszp7yJNTc-_Bk9qfkgQfON2jz1mQcJItlWHfYDhd7YmzJJ09ggfLKSshfqTyk09-vyIhGIEIK8ah2ikbQttV7a7N3w9WmcvU_PiYHIny3I7oQ1UtXgf3-j8wGTANZNWUsnpssFVxPzrwVwG92mMHZWrMIBLLGErRjkpZGLK7X-iEuUxQrZhcVVeKDw/s72-w306-h400-c/AP-Lisfranc-Injury.jpg" height="72" width="72"/><thr:total>0</thr:total><georss:featurename>United States</georss:featurename><georss:point>37.09024 -95.712891</georss:point><georss:box>8.780006163821156 -130.869141 65.400473836178847 -60.556641</georss:box></entry><entry><id>tag:blogger.com,1999:blog-3302710210518459416.post-6377178870811475493</id><published>2022-03-29T21:00:00.007-07:00</published><updated>2022-03-29T21:00:58.566-07:00</updated><category scheme="http://www.blogger.com/atom/ns#" term="Case Files"/><category scheme="http://www.blogger.com/atom/ns#" term="Orthopaedic Surgery Case File"/><title type='text'>Adult Acquired Flatfoot Deformity Case File</title><content type='html'>&lt;div class=&quot;scrollbox&quot;&gt;
  87.  
  88. &lt;div style=&quot;text-align: left;&quot;&gt;&lt;b&gt;&lt;span style=&quot;font-size: large;&quot;&gt;Adult Acquired Flatfoot Deformity Case File&lt;/span&gt;&lt;/b&gt;&lt;/div&gt;&lt;div&gt;Eugene C. Toy, MD, Andrew J. Rosenbaum, MD, Timothy T. Roberts, MD, Joshua S. Dines, MD&lt;/div&gt;&lt;div&gt;&lt;br /&gt;&lt;/div&gt;&lt;div&gt;&lt;b&gt;&lt;span style=&quot;color: #017087; font-size: medium;&quot;&gt;CASE 35&lt;/span&gt;&lt;/b&gt;&lt;/div&gt;&lt;div&gt;&lt;div&gt;A 47-year-old woman complains of pain at the medial side of her ankle. She works constantly on her feet as real estate agent. She says that she has had a relatively flatfoot her whole life, but it has worsened over the last 3.5 years and believes that the arch is collapsing. She says when she is in the shower it feels like the foot cups the floor. She also has a bunion that seems to have progressed over time. It is dramatically affecting her ability to walk and work. She has tried conservative care in the form of an orthotic and an ankle brace, but the condition continues to worsen her quality of life. On her exam, she has good ankle and subtalar range of motion. She has obvious increased heel valgus on the left side compared with the right. There is a sense of fullness and pain to palpation over the medial aspect of the ankle. On sitting, she can invert, evert, plantar flex, and dorsiflex the ankle with normal strength. However, on standing, she has increased foot abduction, and the arch appears to be collapsed. She cannot perform a single heel raise on the left side but can do so normally on the right side. She also has gastrocnemius tightness. The first ray is hypermobile.&lt;/div&gt;&lt;div&gt;&lt;br /&gt;&lt;/div&gt;&lt;div&gt;&lt;span style=&quot;color: #017087;&quot;&gt;►&lt;/span&gt;&amp;nbsp;What is the most likely diagnosis?&lt;/div&gt;&lt;div&gt;&lt;span style=&quot;color: #017087;&quot;&gt;►&lt;/span&gt;&amp;nbsp;What imaging will help confirm the diagnosis?&lt;/div&gt;&lt;div&gt;&lt;span style=&quot;color: #017087;&quot;&gt;►&lt;/span&gt;&amp;nbsp;What is the surgical treatment of this condition?&lt;/div&gt;&lt;/div&gt;&lt;div&gt;&lt;br /&gt;&lt;/div&gt;&lt;div&gt;&lt;br /&gt;&lt;/div&gt;&lt;div&gt;&lt;div&gt;&lt;b&gt;&lt;span style=&quot;font-size: medium;&quot;&gt;ANSWER TO CASE 35:&lt;/span&gt;&lt;/b&gt;&lt;/div&gt;&lt;div&gt;&lt;span style=&quot;background-color: #daebf0; font-size: medium;&quot;&gt;&lt;span&gt;&lt;span style=&quot;background-color: transparent;&quot;&gt;&lt;span style=&quot;color: #017087;&quot;&gt;&lt;b&gt;Adult Acquired Flatfoot Deformity&lt;/b&gt;&lt;/span&gt;&lt;/span&gt;&lt;b&gt;&amp;nbsp;&amp;nbsp;&lt;/b&gt;&lt;/span&gt;&lt;span style=&quot;font-weight: bold;&quot;&gt;&amp;nbsp;&amp;nbsp; &amp;nbsp;&lt;/span&gt;&lt;span style=&quot;font-weight: bold;&quot;&gt;&amp;nbsp;&amp;nbsp; &amp;nbsp;&lt;/span&gt;&lt;span style=&quot;font-weight: bold;&quot;&gt;&amp;nbsp;&amp;nbsp; &amp;nbsp;&lt;/span&gt;&lt;span style=&quot;font-weight: bold;&quot;&gt;&amp;nbsp;&amp;nbsp; &amp;nbsp;&lt;/span&gt;&lt;span style=&quot;font-weight: bold;&quot;&gt;&amp;nbsp; &amp;nbsp;&lt;/span&gt;&lt;/span&gt;&lt;/div&gt;&lt;/div&gt;&lt;div&gt;&lt;br /&gt;&lt;/div&gt;&lt;div&gt;&lt;div&gt;&lt;i&gt;Summary&lt;/i&gt;: A 47-year-old woman presents with medial ankle pain and worsening flat foot over the last 3.5 years. The patient has tried orthotics and an ankle brace. On exam, she has increased heel valgus, decreased arch, and inability to single heel raise, with pain over the medial aspect of the ankle.&lt;/div&gt;&lt;div&gt;&lt;ul style=&quot;text-align: left;&quot;&gt;&lt;li&gt;&lt;b&gt;Most likely diagnosis:&lt;/b&gt; Adult acquired flatfoot deformity (AAFD; originally known as posterior tibial tendon insufficiency).&lt;/li&gt;&lt;li&gt;&lt;b&gt;Imaging:&lt;/b&gt; Initial plain radiographs of the foot should be obtained in a weightbearing stance, including an anteroposterior (AP) view to evaluate the talonavicular coverage angle and a lateral view to evaluate the first tarsometatarsal angle (Meary angle), the calcaneal pitch, and the alignment of the hindfoot. Additionally, radiographs of the ankle should be obtained to evaluate for the presence of a valgus tilt.&lt;/li&gt;&lt;li&gt;&lt;b&gt;Surgical treatment:&lt;/b&gt; Medializing calcaneal osteotomy with a flexor digitorum longus transfer to the navicular. Other accessory procedures depend on the nature of the deformity.&lt;/li&gt;&lt;/ul&gt;&lt;/div&gt;&lt;/div&gt;&lt;div&gt;&lt;br /&gt;&lt;/div&gt;&lt;div&gt;&lt;br /&gt;&lt;/div&gt;&lt;div&gt;&lt;div&gt;&lt;b&gt;&lt;span style=&quot;color: #017087; font-size: medium;&quot;&gt;ANALYSIS&lt;/span&gt;&lt;/b&gt;&lt;/div&gt;&lt;div&gt;&lt;b&gt;&lt;span style=&quot;font-size: medium;&quot;&gt;&lt;i&gt;Objectives&lt;/i&gt;&lt;/span&gt;&lt;/b&gt;&lt;/div&gt;&lt;/div&gt;&lt;div&gt;&lt;div&gt;&lt;ol style=&quot;text-align: left;&quot;&gt;&lt;li&gt;Understand the pathophysiology behind the development of AAFD, and appreciate the bony, tendinous, and ligamentous anatomy implicated in its development.&lt;/li&gt;&lt;li&gt;Understand the nonoperative approach to patients with AAFD.&lt;/li&gt;&lt;li&gt;Appreciate the indications and purpose of each step in the algorithm for surgically treating AAFD.&lt;/li&gt;&lt;/ol&gt;&lt;/div&gt;&lt;/div&gt;&lt;div&gt;&lt;br /&gt;&lt;/div&gt;&lt;div&gt;&lt;br /&gt;&lt;/div&gt;&lt;div&gt;&lt;div&gt;&lt;b&gt;&lt;i&gt;&lt;span style=&quot;font-size: medium;&quot;&gt;Considerations&lt;/span&gt;&lt;/i&gt;&lt;/b&gt;&lt;/div&gt;&lt;div&gt;&lt;br /&gt;&lt;/div&gt;&lt;div&gt;This 47-year-old woman has a history of a flatfoot that has worsened over time. The medial arch pain is most likely caused by inflammation and tenderness of the posterior tibial tendon. She has already failed a number of conservative treatments designed to maintain the foot in a more anatomic position. This includes the orthotic, which gives arch support and also tilts the heel out of the valgus position. Figures 35–1 , 35–2 , 35–3 , and 35–4 comprise a standard radiographic workup for AAFD. These radiographs demonstrate collapse at the medial arch (lateral view), a forefoot abduction deformity (AP view), and increased heel valgus (ankle AP and mortise views). An MRI is also very helpful and will show thickening of the posterior tibial tendon and failure of the ligaments of the medial arch. Surgical treatment can be performed after failure of conservative treatment. This consists of a variety of corrective osteotomies and tendon transfers in patients with flexible deformity as present in this patient. Patients with rigid deformity are better served with selective fusions of the hindfoot.&lt;/div&gt;&lt;/div&gt;&lt;div&gt;&lt;br /&gt;&lt;/div&gt;&lt;div class=&quot;separator&quot; style=&quot;clear: both; text-align: center;&quot;&gt;&lt;a href=&quot;https://blogger.googleusercontent.com/img/b/R29vZ2xl/AVvXsEgRM74CBRPGH0uOo8Dc0bDrrB12cqk52zxS7DcbNaYw00xeeMTsqHVmUy6KnwVJR8fAOjMapJivROOq5AS8uXODQSzvxjDMgOl90uxZS2nTmsBKcp8ayoBCEYRiR_L_hNsNACzLfzc51D8dkxe0HQ-UGiNsVJleCb-UjiKLghSdOqM_X5LhnWwVS9hclA/s500/radiograph-of-the-left-ankle.jpg&quot; style=&quot;margin-left: 1em; margin-right: 1em;&quot;&gt;&lt;img alt=&quot;radiograph of the left ankle&quot; border=&quot;0&quot; data-original-height=&quot;500&quot; data-original-width=&quot;412&quot; height=&quot;640&quot; src=&quot;https://blogger.googleusercontent.com/img/b/R29vZ2xl/AVvXsEgRM74CBRPGH0uOo8Dc0bDrrB12cqk52zxS7DcbNaYw00xeeMTsqHVmUy6KnwVJR8fAOjMapJivROOq5AS8uXODQSzvxjDMgOl90uxZS2nTmsBKcp8ayoBCEYRiR_L_hNsNACzLfzc51D8dkxe0HQ-UGiNsVJleCb-UjiKLghSdOqM_X5LhnWwVS9hclA/w528-h640/radiograph-of-the-left-ankle.jpg&quot; title=&quot;radiograph of the left ankle&quot; width=&quot;528&quot; /&gt;&lt;/a&gt;&lt;/div&gt;&lt;div&gt;&lt;b&gt;&lt;span style=&quot;color: #017087;&quot;&gt;Figure 35–1.&lt;/span&gt;&lt;/b&gt; Mortise view plain radiograph of the left ankle. (Courtesy of Scott Ellis, MD)&lt;/div&gt;&lt;div&gt;&lt;br /&gt;&lt;/div&gt;&lt;div class=&quot;separator&quot; style=&quot;clear: both; text-align: center;&quot;&gt;&lt;a href=&quot;https://blogger.googleusercontent.com/img/b/R29vZ2xl/AVvXsEgjGBWTGmyogok_vXE0YPQkXumtchWUnn1lGt4R8IzFRDtmLFY41FuLx42WfL-A0P35EhAIRNZQZeB56JmwR1XJ042mekanJ06ZK9TuW6ODsJR5d0WSHwtdh-DeJ7m2mf5cEbEJh1TIqllhoICM8eJH9peiS7wMNxT3l8W5V7txpHANVBBttYBO_WnDmw/s600/Lateral-foot-plain-radiograph.jpg&quot; style=&quot;margin-left: 1em; margin-right: 1em;&quot;&gt;&lt;img alt=&quot;Lateral foot plain radiograph&quot; border=&quot;0&quot; data-original-height=&quot;400&quot; data-original-width=&quot;600&quot; height=&quot;426&quot; src=&quot;https://blogger.googleusercontent.com/img/b/R29vZ2xl/AVvXsEgjGBWTGmyogok_vXE0YPQkXumtchWUnn1lGt4R8IzFRDtmLFY41FuLx42WfL-A0P35EhAIRNZQZeB56JmwR1XJ042mekanJ06ZK9TuW6ODsJR5d0WSHwtdh-DeJ7m2mf5cEbEJh1TIqllhoICM8eJH9peiS7wMNxT3l8W5V7txpHANVBBttYBO_WnDmw/w640-h426/Lateral-foot-plain-radiograph.jpg&quot; title=&quot;Lateral foot plain radiograph&quot; width=&quot;640&quot; /&gt;&lt;/a&gt;&lt;/div&gt;&lt;div&gt;&lt;div&gt;&lt;b&gt;&lt;span style=&quot;color: #017087;&quot;&gt;Figure 35–2.&lt;/span&gt;&lt;/b&gt; Lateral foot plain radiograph showing a decreased pitch at the calcaneus and a&lt;/div&gt;&lt;div&gt;decreased first tarsometatarsal angle. (Courtesy of Scott Ellis, MD)&lt;/div&gt;&lt;/div&gt;&lt;div&gt;&lt;br /&gt;&lt;/div&gt;&lt;div class=&quot;separator&quot; style=&quot;clear: both; text-align: center;&quot;&gt;&lt;a href=&quot;https://blogger.googleusercontent.com/img/b/R29vZ2xl/AVvXsEhkuHV6QHxEvcBta5pDkTMxGn3DQRmj-u8AvslnHT9YwOAsJw9Rb2oYTApWMusa99o2H2LhzHMP9nWul-HA82Sf1jU1hqw9mrRw_7F-QlggfLLrBJYIqzOIcgebd7tAagOxklx88F8aqbVxMZ0j2OPCy3ROTTUspWuuPKUmcLPLxGx3Tmb3-fH3VEF9cA/s507/Anteroposterior-plain-radiograph.jpg&quot; style=&quot;margin-left: 1em; margin-right: 1em;&quot;&gt;&lt;img alt=&quot;Anteroposterior plain radiograph&quot; border=&quot;0&quot; data-original-height=&quot;507&quot; data-original-width=&quot;400&quot; height=&quot;640&quot; src=&quot;https://blogger.googleusercontent.com/img/b/R29vZ2xl/AVvXsEhkuHV6QHxEvcBta5pDkTMxGn3DQRmj-u8AvslnHT9YwOAsJw9Rb2oYTApWMusa99o2H2LhzHMP9nWul-HA82Sf1jU1hqw9mrRw_7F-QlggfLLrBJYIqzOIcgebd7tAagOxklx88F8aqbVxMZ0j2OPCy3ROTTUspWuuPKUmcLPLxGx3Tmb3-fH3VEF9cA/w504-h640/Anteroposterior-plain-radiograph.jpg&quot; title=&quot;Anteroposterior plain radiograph&quot; width=&quot;504&quot; /&gt;&lt;/a&gt;&lt;/div&gt;&lt;div&gt;&lt;div&gt;&lt;b&gt;&lt;span style=&quot;color: #017087;&quot;&gt;Figure 35–3.&lt;/span&gt;&lt;/b&gt; Anteroposterior plain radiograph of the left foot demonstrating increased forefoot&lt;/div&gt;&lt;div&gt;abduction through the talonavicular joint and the presence of a severe bunion (hallux valgus deformity).&lt;/div&gt;&lt;div&gt;(Courtesy of Scott Ellis, MD)&lt;/div&gt;&lt;/div&gt;&lt;div&gt;&lt;br /&gt;&lt;/div&gt;&lt;div class=&quot;separator&quot; style=&quot;clear: both; text-align: center;&quot;&gt;&lt;a href=&quot;https://blogger.googleusercontent.com/img/b/R29vZ2xl/AVvXsEgKVWOGYleoVGSY18Dqcsxnavbs11MT-yOXlCt9N5a1cv9_lB3z6eNbcUUBhVnIF2mR-K7Q-QOH7y75OHeRkkdD_fEza--Rak6d9xBZGSZZ1rHIrVWruvH4sZmcZ_mGZfnN1xKhLiE78lMFb-mRf2e7QtXjAindF9WFTZ1V0oVlMPTOEebfEO_o6aHhSw/s545/hindfoot-valgus.jpg&quot; style=&quot;margin-left: 1em; margin-right: 1em;&quot;&gt;&lt;img alt=&quot;hindfoot valgus&quot; border=&quot;0&quot; data-original-height=&quot;450&quot; data-original-width=&quot;545&quot; height=&quot;528&quot; src=&quot;https://blogger.googleusercontent.com/img/b/R29vZ2xl/AVvXsEgKVWOGYleoVGSY18Dqcsxnavbs11MT-yOXlCt9N5a1cv9_lB3z6eNbcUUBhVnIF2mR-K7Q-QOH7y75OHeRkkdD_fEza--Rak6d9xBZGSZZ1rHIrVWruvH4sZmcZ_mGZfnN1xKhLiE78lMFb-mRf2e7QtXjAindF9WFTZ1V0oVlMPTOEebfEO_o6aHhSw/w640-h528/hindfoot-valgus.jpg&quot; title=&quot;hindfoot valgus&quot; width=&quot;640&quot; /&gt;&lt;/a&gt;&lt;/div&gt;&lt;div&gt;&lt;b&gt;&lt;span style=&quot;color: #017087;&quot;&gt;Figure 35–4.&lt;/span&gt;&lt;/b&gt; Hindfoot alignment view shows increased hindfoot valgus. (Courtesy of Scott Ellis, MD)&lt;/div&gt;&lt;div&gt;&lt;br /&gt;&lt;/div&gt;&lt;div&gt;&lt;br /&gt;&lt;/div&gt;&lt;div&gt;&lt;div&gt;&lt;div&gt;&lt;div&gt;&lt;b&gt;&lt;span style=&quot;font-size: medium;&quot;&gt;APPROACH TO:&lt;/span&gt;&lt;/b&gt;&lt;/div&gt;&lt;div&gt;&lt;span style=&quot;background-color: #daebf0; color: #017087; font-size: medium;&quot;&gt;&lt;span&gt;&lt;span style=&quot;background-color: transparent;&quot;&gt;&lt;b&gt;Adult Acquired Flatfoot Deformity&lt;/b&gt;&lt;/span&gt;&lt;b&gt;&amp;nbsp; &amp;nbsp;&amp;nbsp;&lt;/b&gt;&lt;/span&gt;&lt;span style=&quot;font-weight: bold;&quot;&gt;&amp;nbsp;&amp;nbsp; &amp;nbsp;&lt;/span&gt;&lt;span style=&quot;font-weight: bold;&quot;&gt;&amp;nbsp;&amp;nbsp; &amp;nbsp;&lt;/span&gt;&lt;span style=&quot;font-weight: bold;&quot;&gt;&amp;nbsp;&amp;nbsp; &amp;nbsp;&lt;/span&gt;&lt;span style=&quot;font-weight: bold;&quot;&gt;&amp;nbsp;&amp;nbsp; &amp;nbsp;&lt;/span&gt;&lt;span style=&quot;font-weight: bold;&quot;&gt;&amp;nbsp;&amp;nbsp; &amp;nbsp;&lt;/span&gt;&lt;span style=&quot;font-weight: bold;&quot;&gt;&amp;nbsp;&amp;nbsp; &amp;nbsp;&lt;/span&gt;&lt;span style=&quot;font-weight: bold;&quot;&gt;&amp;nbsp;&amp;nbsp;&amp;nbsp;&lt;/span&gt;&lt;span style=&quot;font-weight: bold;&quot;&gt;&amp;nbsp;&lt;/span&gt;&lt;/span&gt;&lt;/div&gt;&lt;/div&gt;&lt;div&gt;&lt;br /&gt;&lt;/div&gt;&lt;div&gt;&lt;b&gt;&lt;span style=&quot;color: #017087; font-size: medium;&quot;&gt;DEFINITIONS&lt;/span&gt;&lt;/b&gt;&lt;/div&gt;&lt;/div&gt;&lt;div&gt;&lt;br /&gt;&lt;/div&gt;&lt;/div&gt;&lt;div&gt;&lt;div&gt;&lt;b&gt;POSTERIOR TIBIAL TENDON:&lt;/b&gt; The major invertor of the hindfoot, which has its origin along the posterior aspect of the leg and its insertion on the navicular.&lt;/div&gt;&lt;div&gt;&lt;b&gt;&lt;br /&gt;&lt;/b&gt;&lt;/div&gt;&lt;div&gt;&lt;b&gt;MEDIALIZING CALCANEAL OSTEOTOMY:&lt;/b&gt; Surgically cutting the calcaneal bone from the lateral side, which is used to translate the posterior portion of the bone attached to the Achilles in a medial fashion. The osteotomy is fixed by 2 screws passed axially up the long axis of the heel bone or calcaneus.&lt;/div&gt;&lt;div&gt;&lt;b&gt;&lt;br /&gt;&lt;/b&gt;&lt;/div&gt;&lt;div&gt;&lt;b&gt;FLEXOR DIGITORUM LONGUS TRANSFER:&lt;/b&gt; A surgical rerouting of the tendon, which normally attaches to the plantar surface of the base of the distal phalanges of the 4 lesser toes, to the navicular to replace or enhance the action of the posterior tibial tendon.&lt;/div&gt;&lt;div&gt;&lt;b&gt;&lt;br /&gt;&lt;/b&gt;&lt;/div&gt;&lt;div&gt;&lt;b&gt;GASTROCNEMIUS RECESSION:&lt;/b&gt; This is a selective lengthening of the fascia just distal to the muscle body of the gastrocnemius that does not compromise the soleus muscle and therefore lengthens the Achilles complex in a modest fashion.&lt;/div&gt;&lt;div&gt;&lt;b&gt;&lt;br /&gt;&lt;/b&gt;&lt;/div&gt;&lt;div&gt;&lt;b&gt;LATERAL COLUMN LENGTHENING:&lt;/b&gt; A surgical cut of the anterior/lateral aspect of the calcaneus that is used to place a graft and move the forefoot out of relative abduction.&lt;/div&gt;&lt;div&gt;&lt;b&gt;&lt;br /&gt;&lt;/b&gt;&lt;/div&gt;&lt;div&gt;&lt;b&gt;SPRING LIGAMENT:&lt;/b&gt; The calcaneonavicular ligament, which is a sleeve of tissue that extends from the calcaneus to the navicular and helps support the talar head.&lt;/div&gt;&lt;div&gt;&lt;b&gt;&lt;br /&gt;&lt;/b&gt;&lt;/div&gt;&lt;div&gt;&lt;b&gt;DELTOID LIGAMENT:&lt;/b&gt; The ligament connecting the medial malleolus to the talus and calcaneus that helps to prevent the ankle joint from tilting into valgus.&lt;/div&gt;&lt;div&gt;&lt;b&gt;&lt;br /&gt;&lt;/b&gt;&lt;/div&gt;&lt;div&gt;&lt;b&gt;ARIZONA BRACE:&lt;/b&gt; A brace that incorporates a custom-made orthotic with a lace-up ankle brace used to help control adult acquired flatfoot deformity.&lt;/div&gt;&lt;div&gt;&lt;b&gt;&lt;br /&gt;&lt;/b&gt;&lt;/div&gt;&lt;div&gt;&lt;b&gt;HINDFOOT VALGUS:&lt;/b&gt; A malalignment of the heel with its base displaced laterally when viewed from behind.&lt;/div&gt;&lt;div&gt;&lt;b&gt;&lt;br /&gt;&lt;/b&gt;&lt;/div&gt;&lt;div&gt;&lt;b&gt;FOREFOOT ABDUCTION:&lt;/b&gt; A relative position of the toes further away from the midline of the body than normal.&lt;/div&gt;&lt;/div&gt;&lt;div&gt;&lt;br /&gt;&lt;/div&gt;&lt;div&gt;&lt;br /&gt;&lt;/div&gt;&lt;div&gt;&lt;b&gt;&lt;span style=&quot;color: #017087; font-size: medium;&quot;&gt;CLINICAL APPROACH&lt;/span&gt;&lt;/b&gt;&lt;/div&gt;&lt;div&gt;&lt;br /&gt;&lt;/div&gt;&lt;div&gt;&lt;div&gt;&lt;i&gt;&lt;b&gt;&lt;span style=&quot;font-size: medium;&quot;&gt;Etiologies&lt;/span&gt;&lt;/b&gt;&lt;/i&gt;&lt;/div&gt;&lt;div&gt;&lt;br /&gt;&lt;/div&gt;&lt;div&gt;The true cause of the AAFD is not known. Generally, however, it is thought to occur more commonly in women, particularly those who are ligamentously lax and in those who are overweight, which places more strain on the posterior tibial tendon. This tendon goes on to stretch and fail along with the ligaments that support the arch, including the spring ligament. Patients generally complain of having a flatfoot their whole life, which worsens over time. It is thought that the &lt;b&gt;hypovascular&amp;nbsp;&lt;/b&gt;nature of the tendon in the area of failure is a leading cause. This generally occurs in women beginning in their fifties.&lt;/div&gt;&lt;/div&gt;&lt;div&gt;&lt;br /&gt;&lt;/div&gt;&lt;div&gt;&lt;br /&gt;&lt;/div&gt;&lt;div&gt;&lt;div&gt;&lt;b&gt;&lt;i&gt;&lt;span style=&quot;font-size: medium;&quot;&gt;Clinical Presentation&lt;/span&gt;&lt;/i&gt;&lt;/b&gt;&lt;/div&gt;&lt;div&gt;&lt;br /&gt;&lt;/div&gt;&lt;div&gt;Most patients present first with pain in the medial arch area. This corresponds with the area of the posterior tibial tendon. They also note a decrease in the arch and the position of their heel in more valgus. The foot generally begins to feel weaker. As the disease progresses, pain may shift laterally. This is commonly because the heel begins to impinge underneath the fibula.&lt;/div&gt;&lt;div&gt;&lt;br /&gt;&lt;/div&gt;&lt;div&gt;&lt;span&gt;&amp;nbsp;&amp;nbsp; &amp;nbsp;&lt;/span&gt;The physical exam shows the patient to have a &lt;b&gt;decreased arch&lt;/b&gt; with the &lt;b&gt;heel&amp;nbsp;&lt;/b&gt;&lt;b&gt;positioned in valgus.&lt;/b&gt; Patients commonly have an &lt;b&gt;abducted forefoot.&lt;/b&gt; A helpful diagnostic test is the single heel raise. &lt;b&gt;Patients with posterior tibial tendon insufficiency&amp;nbsp;&lt;/b&gt;&lt;b&gt;are unable to raise their heel from the ground or, if they can do so, are&amp;nbsp;&lt;/b&gt;&lt;b&gt;not able to fully invert the heel.&lt;/b&gt; Patients also commonly present with a bunion and a tight gastrocnemius muscle, which occurs as the heel moves in the more relative position of valgus and shortens the Achilles.&lt;/div&gt;&lt;/div&gt;&lt;div&gt;&lt;br /&gt;&lt;/div&gt;&lt;div&gt;&lt;br /&gt;&lt;/div&gt;&lt;div&gt;&lt;div&gt;&lt;b&gt;&lt;i&gt;&lt;span style=&quot;font-size: medium;&quot;&gt;Diagnosis&lt;/span&gt;&lt;/i&gt;&lt;/b&gt;&lt;/div&gt;&lt;div&gt;&lt;br /&gt;&lt;/div&gt;&lt;div&gt;The diagnosis is commonly made based on the history. The inability to perform a single heel raise is very suggestive as well. Radiographs indicate the location and amount of deformity ( Figures 35–1 , 35–2 , 35–3 , and 35–4 ). On a lateral x-ray, patients have a decreased first tarsometatarsal angle showing collapse on the inside arch. The hindfoot alignment view generally shows that the heel is in relative position of valgus. AP x-rays show that the navicular bone moves lateral relative to the talus, which suggests that there is forefoot abduction. An MRI can be helpful as well, which shows not only enlargement and degeneration of the posterior tibial tendon, but also failure of the spring ligament complex.&lt;/div&gt;&lt;div&gt;&lt;br /&gt;&lt;/div&gt;&lt;div&gt;&lt;span&gt;&amp;nbsp;&amp;nbsp; &amp;nbsp;&lt;/span&gt;Once the diagnosis of AAFD is made, it can be classified. Classification into 4 stages of severity helps guide treatment and prognosis. In stage I, the foot and ankle are without deformity, however there maybe pain from posterior tibial tendinosis. In stage II, the flatfoot deformity is apparent, but it is flexible throughout, meaning, the examiner can pull it into appropriate alignment. In stage III, the flatfoot deformity becomes rigid and is uncorrectable by manipulation. Finally, by stage IV, there is an additional rigid valgus tilt to the ankle due to compromise of the deltoid ligament.&lt;/div&gt;&lt;/div&gt;&lt;div&gt;&lt;br /&gt;&lt;/div&gt;&lt;div&gt;&lt;br /&gt;&lt;/div&gt;&lt;div&gt;&lt;b style=&quot;color: #017087;&quot;&gt;&lt;span style=&quot;font-size: medium;&quot;&gt;TREATMENT&lt;/span&gt;&lt;/b&gt;&lt;/div&gt;&lt;div&gt;&lt;br /&gt;&lt;/div&gt;&lt;div&gt;&lt;div&gt;Treatment depends on the stage of the deformity. In general, those with &lt;b&gt;stage I&lt;/b&gt; or no deformity can be treated conservatively with an orthotic and relative rest. Patients with &lt;b&gt;stage II&lt;/b&gt; deformity generally are first approached with an orthotic, which gives arch support, and a medial post, which is used to raise the inside aspect of the heel and tilt the heel out of valgus. An ankle brace that is usually a lace-up or velcro brace can be used to help stabilize the hindfoot. Commonly, patients must go on to try an Arizona brace, which is a custom-made orthotic built in with an ankle brace. Patients with &lt;b&gt;stage III&lt;/b&gt; AAFD respond less favorably to orthotics and bracing given that the deformity is rigid and not correctible. &lt;b&gt;Stage IV&lt;/b&gt; generally also requires more aggressive treatment, given the collapse that is occurring through the ankle unless the patient is relatively asymptomatic.&lt;/div&gt;&lt;/div&gt;&lt;div&gt;&lt;br /&gt;&lt;/div&gt;&lt;div&gt;&lt;div&gt;&lt;span&gt;&amp;nbsp;&amp;nbsp; &amp;nbsp;&lt;/span&gt;Surgical treatment also depends on the stage. Surgery is rarely indicated for patients without deformity or stage I. In stage II, a medializing heel slide is commonly performed, along with a transfer of the flexor digitorum longus tendon to the navicular. If the posterior tibial tendon is very diseased, it can be resected. A gastrocnemius resection, lateral column lengthening, and medial column procedures such as first tarsometatarsal fusion or osteotomy to the medial cuneiform (Cotton osteotomy) can be also used. For stage III deformity, a fusion of the triple joint complex, which includes the subtalar, talonavicular, and calcaneal cuboid joint, must be performed. In stage IV, the ankle must be addressed concomitantly with the foot either through trying to reconstruct the deltoid, performing an ankle fusion, or performing ankle replacement.&lt;/div&gt;&lt;/div&gt;&lt;div&gt;&lt;br /&gt;&lt;/div&gt;&lt;div&gt;&lt;br /&gt;&lt;/div&gt;&lt;div&gt;&lt;b&gt;&lt;span style=&quot;color: #017087; font-size: medium;&quot;&gt;&lt;u&gt;COMPREHENSION QUESTIONS&lt;/u&gt;&lt;/span&gt;&lt;/b&gt;&lt;/div&gt;&lt;div&gt;&lt;br /&gt;&lt;/div&gt;&lt;div&gt;&lt;div&gt;35.1 Which of the following best defines the “spring ligament”?&lt;/div&gt;&lt;/div&gt;&lt;blockquote style=&quot;border: none; margin: 0px 0px 0px 40px; padding: 0px; text-align: left;&quot;&gt;&lt;div&gt;&lt;div&gt;A. Talocalcaneal ligament&lt;/div&gt;&lt;/div&gt;&lt;div&gt;&lt;div&gt;B. Calcaneonavicular ligament&lt;/div&gt;&lt;/div&gt;&lt;div&gt;&lt;div&gt;C. Talonavicular ligament&lt;/div&gt;&lt;/div&gt;&lt;div&gt;&lt;div&gt;D. Calcaneofibular ligament&lt;/div&gt;&lt;/div&gt;&lt;/blockquote&gt;&lt;div&gt;&lt;div&gt;&lt;br /&gt;&lt;/div&gt;&lt;div&gt;35.2 Which of the following best describes the action of the posterior tibial tendon?&lt;/div&gt;&lt;/div&gt;&lt;blockquote style=&quot;border: none; margin: 0px 0px 0px 40px; padding: 0px; text-align: left;&quot;&gt;&lt;div&gt;&lt;div&gt;A. Evert the hindfoot&lt;/div&gt;&lt;/div&gt;&lt;div&gt;&lt;div&gt;B. Dorsiflex the ankle&lt;/div&gt;&lt;/div&gt;&lt;div&gt;&lt;div&gt;C. Plantar flex the ankle&lt;/div&gt;&lt;/div&gt;&lt;div&gt;&lt;div&gt;D. Invert the hindfoot&lt;/div&gt;&lt;/div&gt;&lt;/blockquote&gt;&lt;div&gt;&lt;div&gt;&lt;br /&gt;&lt;/div&gt;&lt;div&gt;35.3 A 55-year-old woman presents with a severe flatfoot deformity with arthritis in the subtalar joint. On exam, she has increased hindfoot valgus and forefoot abduction, which is not correctable on physical exam. Which stage is this flatfoot deformity?&lt;/div&gt;&lt;/div&gt;&lt;blockquote style=&quot;border: none; margin: 0px 0px 0px 40px; padding: 0px; text-align: left;&quot;&gt;&lt;div&gt;&lt;div&gt;A. Stage I&lt;/div&gt;&lt;/div&gt;&lt;div&gt;&lt;div&gt;B. Stage II&lt;/div&gt;&lt;/div&gt;&lt;div&gt;&lt;div&gt;C. Stage III&lt;/div&gt;&lt;/div&gt;&lt;div&gt;&lt;div&gt;D. Stage IV&lt;/div&gt;&lt;/div&gt;&lt;/blockquote&gt;&lt;div&gt;&lt;br /&gt;&lt;/div&gt;&lt;div&gt;&lt;div&gt;35.4 A 60-year-old man presents with a 2-year history of medial ankle pain and a severe flatfoot deformity, which is correctable on physical exam. Which of the following best describes the orthotic that may be used to help treat this deformity?&lt;/div&gt;&lt;/div&gt;&lt;blockquote style=&quot;border: none; margin: 0px 0px 0px 40px; padding: 0px; text-align: left;&quot;&gt;&lt;div&gt;&lt;div&gt;A. Lateral posting and a first metatarsal well&lt;/div&gt;&lt;/div&gt;&lt;div&gt;&lt;div&gt;B. Lateral posting with a metatarsal pad&lt;/div&gt;&lt;/div&gt;&lt;div&gt;&lt;div&gt;C. Medial heel posting with arch support&lt;/div&gt;&lt;/div&gt;&lt;div&gt;&lt;div&gt;D. A carbon-graphite insert with an extension to protect the great toe&lt;/div&gt;&lt;/div&gt;&lt;/blockquote&gt;&lt;div&gt;&lt;div&gt;&lt;br /&gt;&lt;/div&gt;&lt;div&gt;35.5 A 52-year-old woman who is moderately obese and who has had flatfoot deformity her whole life is now presenting with worsening hindfoot pain, particularly around the medial aspect of her ankle. She has increased heel valgus on exam with forefoot abduction and a decreased arch. She cannot perform a single heel raise. MRI confirms a decreased arch and posterior tibial tendinosis. Which of the following best describes the surgical treatment?&lt;/div&gt;&lt;/div&gt;&lt;blockquote style=&quot;border: none; margin: 0px 0px 0px 40px; padding: 0px; text-align: left;&quot;&gt;&lt;div&gt;&lt;div&gt;A. Lateralizing heel slide and a flexor digitorum longus transfer to the navicular&lt;/div&gt;&lt;/div&gt;&lt;div&gt;&lt;div&gt;B. Lateralizing heel slide and a deltoid ligament repair&lt;/div&gt;&lt;/div&gt;&lt;div&gt;&lt;div&gt;C. Medializing heel slide with lateral column shortening through the anterior calcaneus&lt;/div&gt;&lt;/div&gt;&lt;div&gt;&lt;div&gt;D. Medializing heel slide with a flexor digitorum longus transfer to the navicular&lt;/div&gt;&lt;/div&gt;&lt;/blockquote&gt;&lt;div&gt;&lt;br /&gt;&lt;/div&gt;&lt;div&gt;&lt;br /&gt;&lt;/div&gt;&lt;div&gt;&lt;b&gt;&lt;span style=&quot;color: #017087; font-size: medium;&quot;&gt;&lt;u&gt;ANSWERS&lt;/u&gt;&lt;/span&gt;&lt;/b&gt;&lt;/div&gt;&lt;div&gt;&lt;br /&gt;&lt;/div&gt;&lt;div&gt;&lt;div&gt;35.1 &lt;b&gt;B.&lt;/b&gt; The spring ligament connects the calcaneus and navicular, holding up the talar head and helping support the medial arch.&lt;/div&gt;&lt;div&gt;&lt;br /&gt;&lt;/div&gt;&lt;div&gt;35.2 &lt;b&gt;D.&lt;/b&gt; The posterior tibia tendon inserts on the navicular. Its primary action is to invert the hindfoot and can be assessed with the single heel raise test.&lt;/div&gt;&lt;div&gt;&lt;br /&gt;&lt;/div&gt;&lt;div&gt;35.3 &lt;b&gt;C.&lt;/b&gt; A flatfoot deformity that does not correct on exam is considered rigid and therefore classified as stage III. Arthritis is commonly present.&lt;/div&gt;&lt;div&gt;&lt;br /&gt;&lt;/div&gt;&lt;div&gt;35.4 &lt;b&gt;C.&lt;/b&gt; A medial heel post helps tilt the heel out of valgus. The arch support helps to maintain the medial arch. Metatarsal posts are generally used for pain under the metatarsals, and the carbon-graphite insert above is generally used for patients with arthritis in the great toe joint.&lt;/div&gt;&lt;div&gt;&lt;br /&gt;&lt;/div&gt;&lt;div&gt;35.5 &lt;b&gt;D.&lt;/b&gt; The heel must be cut and translated medially to bring the calcaneus out of valgus. The flexor digitorum transfer to the navicular helps restores inversion power to the foot.&lt;/div&gt;&lt;/div&gt;&lt;div&gt;&lt;br /&gt;&lt;/div&gt;&lt;div&gt;&lt;b style=&quot;background-color: #4093a5;&quot;&gt;&amp;nbsp; &amp;nbsp;&lt;span style=&quot;font-size: medium;&quot;&gt;&amp;nbsp;&lt;/span&gt;&lt;span style=&quot;color: white;&quot;&gt;&lt;span style=&quot;font-size: medium;&quot;&gt;CLINICAL PEARLS&lt;/span&gt;&lt;span&gt;&amp;nbsp; &amp;nbsp;&amp;nbsp;&lt;/span&gt;&lt;/span&gt;&lt;/b&gt;&lt;/div&gt;&lt;div&gt;&lt;table border=&quot;1&quot; cellpadding=&quot;0&quot; cellspacing=&quot;0&quot; class=&quot;MsoTableGrid&quot; style=&quot;border-collapse: collapse; border: none; mso-border-alt: solid windowtext .5pt; mso-padding-alt: 0in 5.4pt 0in 5.4pt; mso-yfti-tbllook: 1184;&quot;&gt;
  89. &lt;tbody&gt;&lt;tr style=&quot;height: 19.15pt; mso-yfti-firstrow: yes; mso-yfti-irow: 0; mso-yfti-lastrow: yes;&quot;&gt;
  90.  &lt;td style=&quot;background: rgb(221, 237, 239); border-left: solid #017087 1.0pt; border: 1pt solid rgb(64, 147, 165); height: 19.15pt; padding: 0in 5.4pt; width: 433.7pt;&quot; valign=&quot;top&quot; width=&quot;578&quot;&gt;&lt;div style=&quot;line-height: normal; margin-bottom: 0in;&quot;&gt;&lt;span style=&quot;color: #017087;&quot;&gt;&lt;br /&gt;&lt;/span&gt;&lt;/div&gt;&lt;div style=&quot;line-height: normal; margin-bottom: 0in;&quot;&gt;&lt;span style=&quot;color: #017087;&quot;&gt;►&lt;/span&gt;AAFD usually is caused by failure of the ligaments supporting the medial arch along with the posterior tibial tendon.&lt;/div&gt;&lt;div style=&quot;line-height: normal; margin-bottom: 0in;&quot;&gt;&lt;span style=&quot;color: #017087;&quot;&gt;&lt;br /&gt;&lt;/span&gt;&lt;/div&gt;&lt;div style=&quot;line-height: normal; margin-bottom: 0in;&quot;&gt;&lt;span style=&quot;color: #017087;&quot;&gt;►&lt;/span&gt;&amp;nbsp;Patients with AAFD present with hindfoot valgus and an inability to do a single heel raise.&lt;/div&gt;&lt;div style=&quot;line-height: normal; margin-bottom: 0in;&quot;&gt;&lt;span style=&quot;color: #017087;&quot;&gt;&lt;br /&gt;&lt;/span&gt;&lt;/div&gt;&lt;div style=&quot;line-height: normal; margin-bottom: 0in;&quot;&gt;&lt;span style=&quot;color: #017087;&quot;&gt;►&lt;/span&gt;&amp;nbsp;Standard weightbearing radiographs often diagnose flatfoot deformity due to decreased first tarsometatarsal angle on a lateral view, increased forefoot abduction through the talonavicular joint on an anteroposterior view, and a hindfoot alignment view showing increased heel valgus.&lt;/div&gt;&lt;div style=&quot;line-height: normal; margin-bottom: 0in;&quot;&gt;&lt;span style=&quot;color: #017087;&quot;&gt;&lt;br /&gt;&lt;/span&gt;&lt;/div&gt;&lt;div style=&quot;line-height: normal; margin-bottom: 0in;&quot;&gt;&lt;span style=&quot;color: #017087;&quot;&gt;►&lt;/span&gt;&amp;nbsp;MRI can also show the failure of the posterior tendon and medial arch ligaments.&lt;/div&gt;&lt;div style=&quot;line-height: normal; margin-bottom: 0in;&quot;&gt;&lt;span style=&quot;color: #017087;&quot;&gt;&lt;br /&gt;&lt;/span&gt;&lt;/div&gt;&lt;div style=&quot;line-height: normal; margin-bottom: 0in;&quot;&gt;&lt;span style=&quot;color: #017087;&quot;&gt;►&lt;/span&gt;&amp;nbsp;Conservative treatment generally consists of orthotics and bracing.&lt;/div&gt;&lt;div style=&quot;line-height: normal; margin-bottom: 0in;&quot;&gt;&lt;span style=&quot;color: #017087;&quot;&gt;&lt;br /&gt;&lt;/span&gt;&lt;/div&gt;&lt;div style=&quot;line-height: normal; margin-bottom: 0in;&quot;&gt;&lt;span style=&quot;color: #017087;&quot;&gt;►&lt;/span&gt;&amp;nbsp;Surgical reconstruction usually consists of an osteotomy of the calcaneus and a tendon transfer in the foot in patients with flexible deformity.&lt;/div&gt;&lt;div style=&quot;line-height: normal; margin-bottom: 0in;&quot;&gt;&lt;br /&gt;&lt;/div&gt;
  91.  &lt;/td&gt;
  92. &lt;/tr&gt;
  93. &lt;/tbody&gt;&lt;/table&gt;&lt;br /&gt;&lt;/div&gt;
  94.  
  95. &lt;details close=&quot;&quot;&gt;
  96.  &lt;summary&gt;&lt;b&gt;&lt;span style=&quot;color: #00788b;&quot;&gt;REFERENCES&lt;/span&gt;&lt;/b&gt;&lt;/summary&gt;
  97.  &lt;p&gt;Deland JT. Adult-acquired flatfoot deformity. J Am Acad Orthop Surg. 2008;16:399-406.&amp;nbsp;&lt;/p&gt;&lt;p&gt;Deland JT, Page A, Sung IH, O’Malley MJ, Inda D, Choung S. Posterior tibial tendon insufficiency
  98. results at different stages. HSS J. 2006;2:157-160.&amp;nbsp;&lt;/p&gt;&lt;p&gt;Haddad SL, Myerson MS, Younger A, Anderson RB, Davis WH, Manoli A 2nd. Symposium: adult
  99. acquired flatfoot deformity. Foot Ankle Int. 2011;32:95-111.&amp;nbsp;&lt;/p&gt;&lt;p&gt;Johnson KA, Strom DE. Tibialis posterior tendon dysfunction. Clin Orthop Relat Res. 1989:196-206.&amp;nbsp;&lt;/p&gt;&lt;p&gt;Myerson MS. Adult acquired flatfoot deformity: treatment of dysfunction of the posterior tibial tendon.
  100. Instr Course Lect. 1997;46:393-405.&amp;nbsp;&lt;/p&gt;&lt;p&gt;Younger AS, Sawatzky B, Dryden P. Radiographic assessment of adult flatfoot. Foot Ankle Int. 2005;26:
  101. 820-825.&lt;/p&gt;
  102. &lt;/details&gt;&lt;/div&gt;</content><link rel='replies' type='application/atom+xml' href='https://medical-phd.blogspot.com/feeds/6377178870811475493/comments/default' title='Post Comments'/><link rel='replies' type='text/html' href='https://medical-phd.blogspot.com/2022/03/adult-acquired-flatfoot-deformity-case.html#comment-form' title='0 Comments'/><link rel='edit' type='application/atom+xml' href='https://www.blogger.com/feeds/3302710210518459416/posts/default/6377178870811475493'/><link rel='self' type='application/atom+xml' href='https://www.blogger.com/feeds/3302710210518459416/posts/default/6377178870811475493'/><link rel='alternate' type='text/html' href='https://medical-phd.blogspot.com/2022/03/adult-acquired-flatfoot-deformity-case.html' title='Adult Acquired Flatfoot Deformity Case File'/><author><name>Medical Group</name><uri>http://www.blogger.com/profile/01780656800578397393</uri><email>noreply@blogger.com</email><gd:image rel='http://schemas.google.com/g/2005#thumbnail' width='32' height='32' src='//blogger.googleusercontent.com/img/b/R29vZ2xl/AVvXsEiMuROkBDv6yw4SmxGqRA5BPQhOXSwFld-Gufmy4kUqfpoAGQRl7cv-3naew6wjAnCjRFSe_FKX8z7CdE6cFKxxBpnqBuwUTSr-15xd3mFGcr_wJOX2y73G_zJCGND98Q/s220/Final-logo.jpg'/></author><media:thumbnail xmlns:media="http://search.yahoo.com/mrss/" url="https://blogger.googleusercontent.com/img/b/R29vZ2xl/AVvXsEgRM74CBRPGH0uOo8Dc0bDrrB12cqk52zxS7DcbNaYw00xeeMTsqHVmUy6KnwVJR8fAOjMapJivROOq5AS8uXODQSzvxjDMgOl90uxZS2nTmsBKcp8ayoBCEYRiR_L_hNsNACzLfzc51D8dkxe0HQ-UGiNsVJleCb-UjiKLghSdOqM_X5LhnWwVS9hclA/s72-w528-h640-c/radiograph-of-the-left-ankle.jpg" height="72" width="72"/><thr:total>0</thr:total><georss:featurename>United States</georss:featurename><georss:point>37.09024 -95.712891</georss:point><georss:box>8.780006163821156 -130.869141 65.400473836178847 -60.556641</georss:box></entry><entry><id>tag:blogger.com,1999:blog-3302710210518459416.post-4055021515462911602</id><published>2022-03-29T20:05:00.001-07:00</published><updated>2022-03-29T20:06:19.990-07:00</updated><category scheme="http://www.blogger.com/atom/ns#" term="Case Files"/><category scheme="http://www.blogger.com/atom/ns#" term="Orthopaedic Surgery Case File"/><title type='text'>Achilles Tendon Rupture Case File</title><content type='html'>&lt;div class=&quot;scrollbox&quot;&gt;
  103.  
  104. &lt;div style=&quot;text-align: left;&quot;&gt;&lt;b&gt;&lt;span style=&quot;font-size: large;&quot;&gt;Achilles Tendon Rupture Case File&lt;/span&gt;&lt;/b&gt;&lt;/div&gt;&lt;div&gt;Eugene C. Toy, MD, Andrew J. Rosenbaum, MD, Timothy T. Roberts, MD, Joshua S. Dines, MD&lt;/div&gt;&lt;div&gt;&lt;br /&gt;&lt;/div&gt;&lt;div&gt;&lt;b&gt;&lt;span style=&quot;color: #017087; font-size: medium;&quot;&gt;CASE 34&lt;/span&gt;&lt;/b&gt;&lt;/div&gt;&lt;div&gt;&lt;div&gt;A 35-year-old man is brought to the emergency department (ED) by his basketball teammates for evaluation of his right leg. The patient was playing a pick-up game at the local gym earlier in the day, at which time he planted his right foot for a jump shot and suddenly felt as though someone kicked him in the back of his leg, although when he looked back no one was close by. The patient reports feeling a “snap” in his calf, experiencing intense pain, and falling to the ground. In the ED, his vital signs are within normal limits. Examination of his left lower extremity reveals a palpable, tender “bulging” mass over his calf and a soft depression on the posterior aspect of his heel. There is no bony tenderness of the left lower extremity. On passive range of motion, there is increased dorsiflexion in the left foot compared with the right foot. The patient cannot actively plantarflex his left foot.&lt;/div&gt;&lt;div&gt;&lt;br /&gt;&lt;/div&gt;&lt;div&gt;&lt;span style=&quot;color: #017087;&quot;&gt;►&lt;/span&gt;&amp;nbsp;What is the most likely diagnosis?&lt;/div&gt;&lt;div&gt;&lt;span style=&quot;color: #017087;&quot;&gt;►&lt;/span&gt;&amp;nbsp;What is the next step in workup?&lt;/div&gt;&lt;div&gt;&lt;span style=&quot;color: #017087;&quot;&gt;►&lt;/span&gt;&amp;nbsp;What is the best treatment?&lt;/div&gt;&lt;/div&gt;&lt;div&gt;&lt;br /&gt;&lt;/div&gt;&lt;div&gt;&lt;br /&gt;&lt;/div&gt;&lt;div&gt;&lt;div&gt;&lt;b&gt;&lt;span style=&quot;font-size: medium;&quot;&gt;ANSWER TO CASE 34:&lt;/span&gt;&lt;/b&gt;&lt;/div&gt;&lt;div&gt;&lt;span style=&quot;background-color: #daebf0; font-size: medium;&quot;&gt;&lt;span&gt;&lt;span style=&quot;background-color: transparent;&quot;&gt;&lt;span style=&quot;color: #017087;&quot;&gt;&lt;b&gt;Achilles Tendon Rupture&lt;/b&gt;&lt;/span&gt;&lt;/span&gt;&lt;b&gt;&amp;nbsp;&amp;nbsp;&lt;/b&gt;&lt;/span&gt;&lt;span style=&quot;font-weight: bold;&quot;&gt;&amp;nbsp;&amp;nbsp; &amp;nbsp;&lt;/span&gt;&lt;span style=&quot;font-weight: bold;&quot;&gt;&amp;nbsp;&amp;nbsp; &amp;nbsp;&lt;/span&gt;&lt;span style=&quot;font-weight: bold;&quot;&gt;&amp;nbsp;&amp;nbsp; &amp;nbsp;&lt;/span&gt;&lt;span style=&quot;font-weight: bold;&quot;&gt;&amp;nbsp;&amp;nbsp; &amp;nbsp;&lt;/span&gt;&lt;span style=&quot;font-weight: bold;&quot;&gt;&amp;nbsp;&amp;nbsp; &amp;nbsp;&lt;/span&gt;&lt;span style=&quot;font-weight: bold;&quot;&gt;&amp;nbsp;&amp;nbsp; &amp;nbsp;&lt;/span&gt;&lt;span style=&quot;font-weight: bold;&quot;&gt;&amp;nbsp; &amp;nbsp;&amp;nbsp;&lt;/span&gt;&lt;/span&gt;&lt;/div&gt;&lt;/div&gt;&lt;div&gt;&lt;br /&gt;&lt;/div&gt;&lt;div&gt;&lt;div&gt;&lt;i&gt;Summary&lt;/i&gt;: A 35-year-old man presents with left lower extremity pain after hearing a “snap” in the back of his leg while planting his foot to take a shot during a basketball game. He states that it felt like someone kicked him. Immediately afterward he noticed significant posterior heel swelling. He is unable to actively plantarflex his foot. On exam, you note increased passive range of dorsiflexion on his left side when compared with his right as well as a palpable defect in his Achilles tendon.&lt;/div&gt;&lt;div&gt;&lt;ul style=&quot;text-align: left;&quot;&gt;&lt;li&gt;&lt;b&gt;Most likely diagnosis:&lt;/b&gt; Achilles tendon rupture.&lt;/li&gt;&lt;li&gt;&lt;b&gt;Next step in workup:&lt;/b&gt; Pain control, plain radiographs and magnetic resonance imaging (MRI) of the left distal tibia and fibula and ankle if the diagnosis is unclear. Ultrasound is an effective alternative modality for diagnosing an Achilles tendon rupture if MRI is unavailable or contraindicated.&lt;/li&gt;&lt;li&gt;&lt;b&gt;Best treatment:&lt;/b&gt; Primary surgical repair.&lt;/li&gt;&lt;/ul&gt;&lt;/div&gt;&lt;/div&gt;&lt;div&gt;&lt;br /&gt;&lt;/div&gt;&lt;div&gt;&lt;br /&gt;&lt;/div&gt;&lt;div&gt;&lt;div&gt;&lt;b&gt;&lt;span style=&quot;color: #017087; font-size: medium;&quot;&gt;ANALYSIS&lt;/span&gt;&lt;/b&gt;&lt;/div&gt;&lt;div&gt;&lt;b&gt;&lt;span style=&quot;font-size: medium;&quot;&gt;&lt;i&gt;Objectives&lt;/i&gt;&lt;/span&gt;&lt;/b&gt;&lt;/div&gt;&lt;/div&gt;&lt;div&gt;&lt;div&gt;&lt;ol style=&quot;text-align: left;&quot;&gt;&lt;li&gt;Understand how to clinically diagnose an acute Achilles tendon rupture using history and specific physical exam maneuvers.&lt;/li&gt;&lt;li&gt;Be familiar with treatment options, both nonoperative and operative, for managing Achilles tendon ruptures.&lt;/li&gt;&lt;li&gt;Recognize the indications for various treatment options and understand their respective advantages and disadvantages.&lt;/li&gt;&lt;/ol&gt;&lt;/div&gt;&lt;/div&gt;&lt;div&gt;&lt;br /&gt;&lt;/div&gt;&lt;div&gt;&lt;br /&gt;&lt;/div&gt;&lt;div&gt;&lt;div&gt;&lt;b&gt;&lt;i&gt;&lt;span style=&quot;font-size: medium;&quot;&gt;Considerations&lt;/span&gt;&lt;/i&gt;&lt;/b&gt;&lt;/div&gt;&lt;div&gt;&lt;br /&gt;&lt;/div&gt;&lt;div&gt;Initial treatment for this 35-year-old man with a presumed Achilles tendon rupture should focus on pain relief and placing the affected leg in a splint that ensures immobilization in a plantarflexed position. The goal of this initial stabilization is to restore the natural length of the musculotendinous unit disrupted by the injury. The primary goal of treatment is to alleviate pain and restore function. There is debate in the orthopaedic community regarding the best treatment modality for acute Achilles tendon ruptures. Nonoperative treatment includes a period of 6 to 8 weeks of splint or cast immobilization. Operative surgical treatments include open tendon repair and minimally invasive suturing techniques to restore the length and integrity of the torn tendon.&lt;/div&gt;&lt;/div&gt;&lt;div&gt;&lt;br /&gt;&lt;/div&gt;&lt;div&gt;&lt;br /&gt;&lt;/div&gt;&lt;div&gt;&lt;div&gt;&lt;div&gt;&lt;b&gt;&lt;span style=&quot;font-size: medium;&quot;&gt;APPROACH TO:&lt;/span&gt;&lt;/b&gt;&lt;/div&gt;&lt;div&gt;&lt;span style=&quot;background-color: #daebf0; color: #017087; font-size: medium;&quot;&gt;&lt;span&gt;&lt;span style=&quot;background-color: transparent;&quot;&gt;&lt;b&gt;Achilles Tendon Rupture&lt;/b&gt;&lt;/span&gt;&lt;b&gt;&amp;nbsp; &amp;nbsp;&amp;nbsp;&lt;/b&gt;&lt;/span&gt;&lt;span style=&quot;font-weight: bold;&quot;&gt;&amp;nbsp;&amp;nbsp; &amp;nbsp;&lt;/span&gt;&lt;span style=&quot;font-weight: bold;&quot;&gt;&amp;nbsp;&amp;nbsp; &amp;nbsp;&lt;/span&gt;&lt;span style=&quot;font-weight: bold;&quot;&gt;&amp;nbsp;&amp;nbsp; &amp;nbsp;&lt;/span&gt;&lt;span style=&quot;font-weight: bold;&quot;&gt;&amp;nbsp;&amp;nbsp; &amp;nbsp;&lt;/span&gt;&lt;span style=&quot;font-weight: bold;&quot;&gt;&amp;nbsp;&amp;nbsp; &amp;nbsp;&lt;/span&gt;&lt;span style=&quot;font-weight: bold;&quot;&gt;&amp;nbsp;&amp;nbsp; &amp;nbsp;&lt;/span&gt;&lt;span style=&quot;font-weight: bold;&quot;&gt;&amp;nbsp;&amp;nbsp;&amp;nbsp;&lt;/span&gt;&lt;span style=&quot;font-weight: bold;&quot;&gt;&amp;nbsp;&lt;/span&gt;&lt;/span&gt;&lt;/div&gt;&lt;/div&gt;&lt;div&gt;&lt;br /&gt;&lt;/div&gt;&lt;div&gt;&lt;b&gt;&lt;span style=&quot;color: #017087; font-size: medium;&quot;&gt;DEFINITIONS&lt;/span&gt;&lt;/b&gt;&lt;/div&gt;&lt;/div&gt;&lt;div&gt;&lt;br /&gt;&lt;/div&gt;&lt;div&gt;&lt;div&gt;&lt;b&gt;THOMPSON TEST: &lt;/b&gt;With the patient in the prone position, the examiner, squeezes the posterior calf musculature ( Figure 34–1 ). A positive Thompson test occurs when there is an absence of plantarflexion in the affected extremity. The disrupted musculotendinous unit is no longer able to plantarflex the foot on the affected side. This is compared with the unaffected side.&lt;/div&gt;&lt;div&gt;&lt;b&gt;&lt;br /&gt;&lt;/b&gt;&lt;/div&gt;&lt;div&gt;&lt;b&gt;ECCENTRIC CONTRACTION:&lt;/b&gt; A muscular contraction during which muscle fibers actually elongate while firing due to overwhelming antagonistic forces. This occurs because the forces that the muscle is trying to overcome are greater than the force generated by the muscle body. Conversely, &lt;b&gt;concentric contractions&lt;/b&gt; occur when the muscle body shortens with contraction. In concentric contractions, the force generated by the muscle is greater than the antagonistic forces applied to it.&lt;/div&gt;&lt;/div&gt;&lt;div&gt;&lt;br /&gt;&lt;/div&gt;&lt;div class=&quot;separator&quot; style=&quot;clear: both; text-align: center;&quot;&gt;&lt;a href=&quot;https://blogger.googleusercontent.com/img/b/R29vZ2xl/AVvXsEi5sPwS254sz5Lf25NUyNwYIInGc5vI8Uyspj3C6OD9FbsbwjjiTpMcz9C57h_jefWbNvZMGvFw57QMDdbDtwVxYZ9zfnbEnFxsvGWbSTbXmf0-1wKljiqsi_YO-uBF6SlSoy3PGvTahIg1UsXDbx3t94IdB9rm2t9AGbwuKiLhbFDNrVZzC9I-sAZlBA/s542/Thompson-test.jpg&quot; style=&quot;margin-left: 1em; margin-right: 1em;&quot;&gt;&lt;img alt=&quot;Achilles Tendon Rupture&quot; border=&quot;0&quot; data-original-height=&quot;542&quot; data-original-width=&quot;464&quot; height=&quot;640&quot; src=&quot;https://blogger.googleusercontent.com/img/b/R29vZ2xl/AVvXsEi5sPwS254sz5Lf25NUyNwYIInGc5vI8Uyspj3C6OD9FbsbwjjiTpMcz9C57h_jefWbNvZMGvFw57QMDdbDtwVxYZ9zfnbEnFxsvGWbSTbXmf0-1wKljiqsi_YO-uBF6SlSoy3PGvTahIg1UsXDbx3t94IdB9rm2t9AGbwuKiLhbFDNrVZzC9I-sAZlBA/w548-h640/Thompson-test.jpg&quot; title=&quot;Achilles Tendon Rupture&quot; width=&quot;548&quot; /&gt;&lt;/a&gt;&lt;/div&gt;&lt;div&gt;&lt;div&gt;&lt;b&gt;&lt;span style=&quot;color: #017087;&quot;&gt;Figure 34–1.&lt;/span&gt;&lt;/b&gt; Thompson test, in which compression of the gastrocnemius-soleus complex normally&lt;/div&gt;&lt;div&gt;produces plantarflexion of the foot (&lt;b&gt;1&lt;/b&gt;). If the tendon is completely ruptured, this will not occur (&lt;b&gt;2&lt;/b&gt;).&lt;/div&gt;&lt;div&gt;(Reproduced, with permission, from Knoop KJ, Stack LB, Storrow AB, et al. &lt;i&gt;Atlas of Emergency Medicine&lt;/i&gt;. 3rd ed. New York, NY: McGraw-Hill; 2009:Fig. 11-77.)&lt;/div&gt;&lt;/div&gt;&lt;div&gt;&lt;br /&gt;&lt;/div&gt;&lt;div&gt;&lt;br /&gt;&lt;/div&gt;&lt;div&gt;&lt;b&gt;&lt;span style=&quot;color: #017087; font-size: medium;&quot;&gt;CLINICAL APPROACH&lt;/span&gt;&lt;/b&gt;&lt;/div&gt;&lt;div&gt;&lt;br /&gt;&lt;/div&gt;&lt;div&gt;&lt;div&gt;&lt;b&gt;&lt;i&gt;&lt;span style=&quot;font-size: medium;&quot;&gt;Etiologies&lt;/span&gt;&lt;/i&gt;&lt;/b&gt;&lt;/div&gt;&lt;div&gt;&lt;br /&gt;&lt;/div&gt;&lt;div&gt;Diagnosis of Achilles tendon ruptures are increasing in incidence as more “weekend warrior” athletes are pushing the limits of their typically sedentary and deconditioned bodies. Additionally, the medical community is becoming more aware of and skilled at diagnosing the injury. This injury is commonly observed in men in their fourth through sixth decades of life.&lt;/div&gt;&lt;/div&gt;&lt;div&gt;&lt;br /&gt;&lt;/div&gt;&lt;div&gt;&lt;br /&gt;&lt;/div&gt;&lt;div&gt;&lt;div&gt;&lt;b&gt;&lt;i&gt;&lt;span style=&quot;font-size: medium;&quot;&gt;Pathoanatomy&lt;/span&gt;&lt;/i&gt;&lt;/b&gt;&lt;/div&gt;&lt;div&gt;&lt;br /&gt;&lt;/div&gt;&lt;div&gt;The gastrocnemius and soleus muscle bodies are contained within the superficial posterior compartment of the leg. These muscle bodies converge to form a common Achilles tendon, which inserts on the posterior tuberosity of the calcaneus. This musculotendinous unit acts to plantarflex the foot. The Achilles tendon most commonly ruptures when the gastrocnemius-soleus complex undergoes eccentric loading with the ankle dorsiflexed and the knee extended. Patients are usually able to describe the incident in which they were pushing off or landing on a plantarflexed foot. Most Achilles tendon injuries occur in a hypovascular watershed area located approximately 2 to 6 cm proximal to its insertion. Repetitive microtrauma to this vulnerable hypovascular region is thought to predispose it to rupture.&lt;/div&gt;&lt;/div&gt;&lt;div&gt;&lt;br /&gt;&lt;/div&gt;&lt;div&gt;&lt;br /&gt;&lt;/div&gt;&lt;div&gt;&lt;div&gt;&lt;b&gt;&lt;i&gt;&lt;span style=&quot;font-size: medium;&quot;&gt;Physical Exam&lt;/span&gt;&lt;/i&gt;&lt;/b&gt;&lt;/div&gt;&lt;div&gt;&lt;br /&gt;&lt;/div&gt;&lt;div&gt;Along with the clinical history, the physical exam is key to diagnosing an Achilles tendon rupture. Despite increasing awareness in the medical community, approximately 25% of Achilles tendon ruptures are initially misdiagnosed as ankle sprains. Visual inspection of the affected extremities will likely reveal significant soft tissue swelling in the heel region. A palpable gap, erythema, and/or a bulging muscle mass may be noted after an acute rupture. The Thompson test ( Figure 34–1 ) will aid the clinician in establishing the diagnosis of an Achilles tendon rupture. Although plantarflexion is frequently compromised, be aware that the patient may still exhibit active plantarflexion due to the functions of the unaffected toe flexor and posterior tibialis muscles.&lt;/div&gt;&lt;/div&gt;&lt;div&gt;&lt;br /&gt;&lt;/div&gt;&lt;div&gt;&lt;br /&gt;&lt;/div&gt;&lt;div&gt;&lt;div&gt;&lt;b&gt;&lt;span style=&quot;font-size: medium;&quot;&gt;&lt;i&gt;Imaging&lt;/i&gt;&lt;/span&gt;&lt;/b&gt;&lt;/div&gt;&lt;div&gt;&lt;br /&gt;&lt;/div&gt;&lt;div&gt;The diagnosis of an Achilles tendon rupture is primarily a &lt;b&gt;clinical diagnosis.&lt;/b&gt; Imaging studies are generally reserved for situations in which the physical exam is inconclusive. Plain radiographs of the lower leg have limited use in diagnosing soft tissues injuries but can adequately rule out any bony injuries and sometimes demonstrate soft tissue swelling. Ultrasound is another imaging modality that can be useful. Although inexpensive, its utility is greatly operator dependent. MRI is the gold standard imaging modality but is not necessary in a clinically obvious diagnosis of Achilles tendon rupture ( Figure 34–2 ). It may, however, be useful in discerning subtleties of partial or chronic Achilles tendon injuries. Of note, the American Academy of Orthopaedic Surgeons Clinical Practice Guideline Summary for acute Achilles tendon rupture states that “We are unable to recommend for or against the routine use of MRI, ultrasonography, and radiography to confirm the diagnosis of acute Achilles tendon rupture.” In other words, it remains a clinical diagnosis.&lt;/div&gt;&lt;/div&gt;&lt;div&gt;&lt;br /&gt;&lt;/div&gt;&lt;div class=&quot;separator&quot; style=&quot;clear: both; text-align: center;&quot;&gt;&lt;a href=&quot;https://blogger.googleusercontent.com/img/b/R29vZ2xl/AVvXsEhIkop3z3CL0MPv1zhp7S6dCPUgwQYRcAQxVHkUg-FjmKyBNQd91BEPNiwKG28wftliB3s9_KEUaLJOqHrKQzLHMmwD5OW45yDymZuidLJqmx95SZOSYtQFMUsoMCGRIEAZK_iddfyVFSmk9GH3aWXbrOvZ79NOcVN1bl9IQPvEeBG5BODQ1WmXDWgonA/s500/Sagittal-T2-weighted-MRI-image.jpg&quot; style=&quot;margin-left: 1em; margin-right: 1em;&quot;&gt;&lt;img alt=&quot;Sagittal T2-weighted MRI image&quot; border=&quot;0&quot; data-original-height=&quot;500&quot; data-original-width=&quot;340&quot; height=&quot;640&quot; src=&quot;https://blogger.googleusercontent.com/img/b/R29vZ2xl/AVvXsEhIkop3z3CL0MPv1zhp7S6dCPUgwQYRcAQxVHkUg-FjmKyBNQd91BEPNiwKG28wftliB3s9_KEUaLJOqHrKQzLHMmwD5OW45yDymZuidLJqmx95SZOSYtQFMUsoMCGRIEAZK_iddfyVFSmk9GH3aWXbrOvZ79NOcVN1bl9IQPvEeBG5BODQ1WmXDWgonA/w436-h640/Sagittal-T2-weighted-MRI-image.jpg&quot; title=&quot;Sagittal T2-weighted MRI image&quot; width=&quot;436&quot; /&gt;&lt;/a&gt;&lt;/div&gt;&lt;div&gt;&lt;div&gt;&lt;b&gt;&lt;span style=&quot;color: #017087;&quot;&gt;Figure 34–2.&lt;/span&gt;&lt;/b&gt; Sagittal T2-weighted MRI image of an Achilles tendon rupture. Note the hyperintensity&lt;/div&gt;&lt;div&gt;surrounding its calcaneal insertion that is indicative of edema and an absence of tendinous attachment.&lt;/div&gt;&lt;div&gt;(Reproduced, with permission, from Skinner HB. &lt;i&gt;Current Diagnosis &amp;amp; Treatment in Orthopedics.&lt;/i&gt;&lt;/div&gt;&lt;div&gt;4th ed. New York, NY: McGraw-Hill; 2006:Fig. 9-58.)&lt;/div&gt;&lt;/div&gt;&lt;div&gt;&lt;br /&gt;&lt;/div&gt;&lt;div&gt;&lt;br /&gt;&lt;/div&gt;&lt;div&gt;&lt;b style=&quot;color: #017087;&quot;&gt;&lt;span style=&quot;font-size: medium;&quot;&gt;TREATMENT&lt;/span&gt;&lt;/b&gt;&lt;/div&gt;&lt;div&gt;&lt;br /&gt;&lt;/div&gt;&lt;div&gt;&lt;div&gt;The treatment for an Achilles tendon rupture is controversial and should be individualized to each patient. The primary goal is to reestablish the normal anatomy of the muscle and tendon, in turn restoring function to the musculotendinous unit. If the tendon is left to heal in either extreme of dorsiflexion or plantarflexion, the mechanical functioning of the gastrocnemius-soleus complex with the tendon could be compromised.&lt;/div&gt;&lt;div&gt;&lt;br /&gt;&lt;/div&gt;&lt;div&gt;&lt;span&gt;&amp;nbsp;&amp;nbsp; &amp;nbsp;&lt;/span&gt;Nonoperative treatment is reserved for older, sedentary individuals with multiple medical comorbidities that may compromise wound healing. It focuses on immobilizing the affected leg in a gravity equinus position that maximizes apposition of the 2 ends of the ruptured tendon. Posterior splints, removable boots and short leg casts for 6 to 8 weeks can all stabilize the foot in this manner. Gradual range of motion and resistance exercises begin at 8 to 10 weeks. Such management lacks the risks of surgery, which include infection, wound breakdown, and nerve injury. However, the limitations of conservative management include suboptimal tendon length after healing and diminished muscle function. Patients must be warned that they will likely have residual weakness and that it can take up to 1 year before maximal plantarflexion is achieved. Furthermore, some studies have shown an increased re-rupture rate with nonoperative management.&lt;/div&gt;&lt;/div&gt;&lt;div&gt;&lt;br /&gt;&lt;/div&gt;&lt;div&gt;&lt;div&gt;&lt;span&gt;&amp;nbsp;&amp;nbsp; &amp;nbsp;&lt;/span&gt;Common indications for operative treatment include acute ruptures in young, healthy, active individuals, and following re-rupture of previously immobilizationtreated Achilles ruptures. It can be accomplished via open and percutaneous techniques. The surgical approach is on the medial side of the Achilles tendon sheath. The frayed edges are debrided, and the foot is positioned in equinus. Two heavy nonabsorbable sutures are then woven through 3 to 4 cm of each tendon edge, using a Bunnell or Kessler stitch technique. The repair can be augmented with lighter, absorbable sutures or with the plantaris tendon if it is present. Although surgical repair decreases re-rupture rate and increases maximal plantarflexion strength, it does have risks and complications, including infection, skin sloughing and necrosis, and sural nerve damage.&lt;/div&gt;&lt;/div&gt;&lt;div&gt;&lt;br /&gt;&lt;/div&gt;&lt;div&gt;&lt;br /&gt;&lt;/div&gt;&lt;div&gt;&lt;b&gt;&lt;span style=&quot;color: #017087; font-size: medium;&quot;&gt;&lt;u&gt;COMPREHENSION QUESTIONS&lt;/u&gt;&lt;/span&gt;&lt;/b&gt;&lt;/div&gt;&lt;div&gt;&lt;br /&gt;&lt;/div&gt;&lt;div&gt;&lt;div&gt;34.1 A 28-year-old man with an acute Achilles tendon rupture is deciding between operative and nonoperative treatment for his condition. Which of the following statements is true regarding operative treatment?&lt;/div&gt;&lt;/div&gt;&lt;blockquote style=&quot;border: none; margin: 0px 0px 0px 40px; padding: 0px; text-align: left;&quot;&gt;&lt;div&gt;&lt;div&gt;A. Lower infection rate&lt;/div&gt;&lt;/div&gt;&lt;div&gt;&lt;div&gt;B. Lower incidence of tendon re-rupture&lt;/div&gt;&lt;/div&gt;&lt;div&gt;&lt;div&gt;C. Decreased muscle strength&lt;/div&gt;&lt;/div&gt;&lt;div&gt;&lt;div&gt;D. Increased sensation over the skin of the posterior calf&lt;/div&gt;&lt;/div&gt;&lt;/blockquote&gt;&lt;div&gt;&lt;div&gt;&lt;br /&gt;&lt;/div&gt;&lt;div&gt;34.2 A 45-year-old man presents with severe ankle pain and an inability to bear weight after tripping into a sand trap while golfing. Which initial diagnostic test is most appropriate?&lt;/div&gt;&lt;/div&gt;&lt;blockquote style=&quot;border: none; margin: 0px 0px 0px 40px; padding: 0px; text-align: left;&quot;&gt;&lt;div&gt;&lt;div&gt;A. MRI&lt;/div&gt;&lt;/div&gt;&lt;div&gt;&lt;div&gt;B. X-ray&lt;/div&gt;&lt;/div&gt;&lt;div&gt;&lt;div&gt;C. Bone scan&lt;/div&gt;&lt;/div&gt;&lt;div&gt;&lt;div&gt;D. Ultrasound&lt;/div&gt;&lt;/div&gt;&lt;/blockquote&gt;&lt;div&gt;&lt;div&gt;&lt;br /&gt;&lt;/div&gt;&lt;div&gt;34.3 Achilles tendon rupture typically occurs during which type of muscle contraction?&lt;/div&gt;&lt;/div&gt;&lt;blockquote style=&quot;border: none; margin: 0px 0px 0px 40px; padding: 0px; text-align: left;&quot;&gt;&lt;div&gt;&lt;div&gt;A. Eccentric muscle contraction&lt;/div&gt;&lt;/div&gt;&lt;div&gt;&lt;div&gt;B. Isometric muscle contraction&lt;/div&gt;&lt;/div&gt;&lt;div&gt;&lt;div&gt;C. Concentric muscle contraction&lt;/div&gt;&lt;/div&gt;&lt;div&gt;&lt;div&gt;D. Rhythmic muscle contraction&lt;/div&gt;&lt;/div&gt;&lt;/blockquote&gt;&lt;div&gt;&lt;br /&gt;&lt;/div&gt;&lt;div&gt;&lt;br /&gt;&lt;/div&gt;&lt;div&gt;&lt;b&gt;&lt;span style=&quot;color: #017087; font-size: medium;&quot;&gt;&lt;u&gt;ANSWERS&lt;/u&gt;&lt;/span&gt;&lt;/b&gt;&lt;/div&gt;&lt;div&gt;&lt;br /&gt;&lt;/div&gt;&lt;div&gt;&lt;div&gt;34.1 &lt;b&gt;B. &lt;/b&gt;Surgical repair is associated with a significantly reduced risk of re-rupture when compared with nonsurgical treatment.&lt;/div&gt;&lt;/div&gt;&lt;div&gt;&lt;br /&gt;&lt;/div&gt;&lt;div&gt;&lt;div&gt;34.2 &lt;b&gt;B.&lt;/b&gt; In a patient with ankle pain and an inability to bear weight, potential fracture must first be ruled out. Although MRI is the gold standard for diagnosing Achilles tendon ruptures, MRI, and similarly ultrasound, should be considered only after other potential injuries are excluded. Bone scans are effective at diagnosing occult bony abnormalities but rarely play a role in diagnosing acute injuries about the ankle.&lt;/div&gt;&lt;div&gt;&lt;br /&gt;&lt;/div&gt;&lt;div&gt;34.3 &lt;b&gt;A.&lt;/b&gt; The Achilles tendon most commonly ruptures when the gastrocnemiussoleus complex is undergoing eccentric loading with the ankle dorsiflexed and the knee extended.&lt;/div&gt;&lt;/div&gt;&lt;div&gt;&lt;br /&gt;&lt;/div&gt;&lt;div&gt;&lt;b style=&quot;background-color: #4093a5;&quot;&gt;&amp;nbsp; &amp;nbsp;&lt;span style=&quot;font-size: medium;&quot;&gt;&amp;nbsp;&lt;/span&gt;&lt;span style=&quot;color: white;&quot;&gt;&lt;span style=&quot;font-size: medium;&quot;&gt;CLINICAL PEARLS&lt;/span&gt;&lt;span&gt;&amp;nbsp; &amp;nbsp;&amp;nbsp;&lt;/span&gt;&lt;/span&gt;&lt;/b&gt;&lt;/div&gt;&lt;div&gt;&lt;table border=&quot;1&quot; cellpadding=&quot;0&quot; cellspacing=&quot;0&quot; class=&quot;MsoTableGrid&quot; style=&quot;border-collapse: collapse; border: none; mso-border-alt: solid windowtext .5pt; mso-padding-alt: 0in 5.4pt 0in 5.4pt; mso-yfti-tbllook: 1184;&quot;&gt;
  105. &lt;tbody&gt;&lt;tr style=&quot;height: 19.15pt; mso-yfti-firstrow: yes; mso-yfti-irow: 0; mso-yfti-lastrow: yes;&quot;&gt;
  106.  &lt;td style=&quot;background: rgb(221, 237, 239); border-left: solid #017087 1.0pt; border: 1pt solid rgb(64, 147, 165); height: 19.15pt; padding: 0in 5.4pt; width: 433.7pt;&quot; valign=&quot;top&quot; width=&quot;578&quot;&gt;
  107.  &lt;div style=&quot;line-height: normal; margin-bottom: 0in;&quot;&gt;&lt;span style=&quot;color: #017087;&quot;&gt;&lt;br /&gt;&lt;/span&gt;&lt;/div&gt;&lt;div style=&quot;line-height: normal; margin-bottom: 0in;&quot;&gt;&lt;span style=&quot;color: #017087;&quot;&gt;►&lt;/span&gt;&amp;nbsp;Approximately 25% of Achilles tendon ruptures are initially misdiagnosed as ankle sprains.&lt;/div&gt;&lt;div style=&quot;line-height: normal; margin-bottom: 0in;&quot;&gt;&lt;span style=&quot;color: #017087;&quot;&gt;&lt;br /&gt;&lt;/span&gt;&lt;/div&gt;&lt;div style=&quot;line-height: normal; margin-bottom: 0in;&quot;&gt;&lt;span style=&quot;color: #017087;&quot;&gt;►&lt;/span&gt;&amp;nbsp;Signs and symptoms of an acute rupture include pain, swelling, and ecchymosis around the heel, and a palpable gap in the tendon. Patients classically recall a “popping” sensation during injury or describe the false sensation of being struck on the heel with a blunt object.&lt;/div&gt;&lt;div style=&quot;line-height: normal; margin-bottom: 0in;&quot;&gt;&lt;span style=&quot;color: #017087;&quot;&gt;&lt;br /&gt;&lt;/span&gt;&lt;/div&gt;&lt;div style=&quot;line-height: normal; margin-bottom: 0in;&quot;&gt;&lt;span style=&quot;color: #017087;&quot;&gt;►&lt;/span&gt;&amp;nbsp;The Thompson test is an effective physical exam maneuver used to diagnose Achilles ruptures.&lt;/div&gt;&lt;div style=&quot;line-height: normal; margin-bottom: 0in;&quot;&gt;&lt;span style=&quot;color: #017087;&quot;&gt;&lt;br /&gt;&lt;/span&gt;&lt;/div&gt;&lt;div style=&quot;line-height: normal; margin-bottom: 0in;&quot;&gt;&lt;span style=&quot;color: #017087;&quot;&gt;►&lt;/span&gt;&amp;nbsp;While Achilles tendon rupture is a predominantly clinical diagnosis, imaging may be necessary in the setting of chronic symptomatology, potential concomitant injuries, or unclear exam findings.&lt;/div&gt;&lt;div style=&quot;line-height: normal; margin-bottom: 0in;&quot;&gt;&lt;br /&gt;&lt;/div&gt;
  108.  &lt;/td&gt;
  109. &lt;/tr&gt;
  110. &lt;/tbody&gt;&lt;/table&gt;&lt;br /&gt;&lt;/div&gt;
  111.  
  112. &lt;details close=&quot;&quot;&gt;
  113.  &lt;summary&gt;&lt;b&gt;&lt;span style=&quot;color: #00788b;&quot;&gt;REFERENCES&lt;/span&gt;&lt;/b&gt;&lt;/summary&gt;
  114.  &lt;p&gt;Azar FM. Traumatic disorders. In: Canale ST, Beaty JH, eds. Campbell’s Operative Orthopaedics. 11th ed.
  115. Philadelphia: Mosby Elsevier; 2008:2737-2788.&amp;nbsp;&lt;/p&gt;&lt;p&gt;Casillas MM, Mann RA. Tendon disorders of the foot and ankle. In: Chapman MW, Szabo RM, Marder
  116. R, et al, eds. Chapman’s Orthopaedic Surgery. 3rd ed. Philadelphia: Lippincott Williams &amp;amp; Wilkins;
  117. 2001:3123-3136.&amp;nbsp;&lt;/p&gt;&lt;p&gt;Chiodo CP, et al. AAOS Clinical practice guideline summary: diagnosis and treatment of acute Achilles
  118. tendon rupture. J Am Acad Orthop Surg. 2010;18:503-510.&amp;nbsp;&lt;/p&gt;&lt;p&gt;Chiodo CP, Wilson MG. Current concepts review: acute ruptures of the Achilles tendon. Foot Ankle Int.
  119. 2006;27:305-313.&amp;nbsp;&lt;/p&gt;&lt;p&gt;Nilsson-Helander K, et al. Acute Achilles tendon rupture: a randomized, controlled study comparing
  120. surgical and nonsurgical treatments using validated outcome measures. Am J Sports Med. 2010;38:
  121. 2186-2193.&lt;/p&gt;
  122. &lt;/details&gt;&lt;/div&gt;</content><link rel='replies' type='application/atom+xml' href='https://medical-phd.blogspot.com/feeds/4055021515462911602/comments/default' title='Post Comments'/><link rel='replies' type='text/html' href='https://medical-phd.blogspot.com/2022/03/achilles-tendon-rupture-case-file.html#comment-form' title='0 Comments'/><link rel='edit' type='application/atom+xml' href='https://www.blogger.com/feeds/3302710210518459416/posts/default/4055021515462911602'/><link rel='self' type='application/atom+xml' href='https://www.blogger.com/feeds/3302710210518459416/posts/default/4055021515462911602'/><link rel='alternate' type='text/html' href='https://medical-phd.blogspot.com/2022/03/achilles-tendon-rupture-case-file.html' title='Achilles Tendon Rupture Case File'/><author><name>Medical Group</name><uri>http://www.blogger.com/profile/01780656800578397393</uri><email>noreply@blogger.com</email><gd:image rel='http://schemas.google.com/g/2005#thumbnail' width='32' height='32' src='//blogger.googleusercontent.com/img/b/R29vZ2xl/AVvXsEiMuROkBDv6yw4SmxGqRA5BPQhOXSwFld-Gufmy4kUqfpoAGQRl7cv-3naew6wjAnCjRFSe_FKX8z7CdE6cFKxxBpnqBuwUTSr-15xd3mFGcr_wJOX2y73G_zJCGND98Q/s220/Final-logo.jpg'/></author><media:thumbnail xmlns:media="http://search.yahoo.com/mrss/" url="https://blogger.googleusercontent.com/img/b/R29vZ2xl/AVvXsEi5sPwS254sz5Lf25NUyNwYIInGc5vI8Uyspj3C6OD9FbsbwjjiTpMcz9C57h_jefWbNvZMGvFw57QMDdbDtwVxYZ9zfnbEnFxsvGWbSTbXmf0-1wKljiqsi_YO-uBF6SlSoy3PGvTahIg1UsXDbx3t94IdB9rm2t9AGbwuKiLhbFDNrVZzC9I-sAZlBA/s72-w548-h640-c/Thompson-test.jpg" height="72" width="72"/><thr:total>0</thr:total><georss:featurename>United States</georss:featurename><georss:point>37.09024 -95.712891</georss:point><georss:box>8.780006163821156 -130.869141 65.400473836178847 -60.556641</georss:box></entry><entry><id>tag:blogger.com,1999:blog-3302710210518459416.post-7696503260018442618</id><published>2022-03-29T14:06:00.000-07:00</published><updated>2022-03-29T14:06:09.673-07:00</updated><category scheme="http://www.blogger.com/atom/ns#" term="Case Files"/><category scheme="http://www.blogger.com/atom/ns#" term="Orthopaedic Surgery Case File"/><title type='text'>Lateral Epicondylitis Case File</title><content type='html'>&lt;div class=&quot;scrollbox&quot;&gt;
  123.  
  124. &lt;div style=&quot;text-align: left;&quot;&gt;&lt;b&gt;&lt;span style=&quot;font-size: large;&quot;&gt;Lateral Epicondylitis Case File&lt;/span&gt;&lt;/b&gt;&lt;/div&gt;&lt;div&gt;Eugene C. Toy, MD, Andrew J. Rosenbaum, MD, Timothy T. Roberts, MD, Joshua S. Dines, MD&lt;/div&gt;&lt;div&gt;&lt;br /&gt;&lt;/div&gt;&lt;div&gt;&lt;b&gt;&lt;span style=&quot;color: #017087; font-size: medium;&quot;&gt;CASE 33&lt;/span&gt;&lt;/b&gt;&lt;/div&gt;&lt;div&gt;&lt;div&gt;A 43-year-old dentist presents to the clinic with debilitating left elbow pain and stiffness. He states that his left elbow has been bothering him on and off for several weeks but has recently been getting worse. He states that he has been having trouble grasping and gripping objects and feels as if his hand is not working properly. The patient is concerned because the pain and stiffness are beginning to limit him in what he can do in his practice. On physical exam, he is found to have pain on palpation of his lateral elbow and pain with resisted wrist extension and radial deviation. There is minimal swelling present. All motor and sensory findings are normal. His past medical history and review of systems are otherwise unremarkable.&lt;/div&gt;&lt;div&gt;&lt;br /&gt;&lt;/div&gt;&lt;div&gt;&lt;span style=&quot;color: #017087;&quot;&gt;►&lt;/span&gt;&amp;nbsp;What is the most likely diagnosis?&lt;/div&gt;&lt;div&gt;&lt;span style=&quot;color: #017087;&quot;&gt;►&lt;/span&gt;&amp;nbsp;What is the initial treatment?&lt;/div&gt;&lt;div&gt;&lt;span style=&quot;color: #017087;&quot;&gt;►&lt;/span&gt;&amp;nbsp;Should the patient receive a steroid injection?&lt;/div&gt;&lt;/div&gt;&lt;div&gt;&lt;br /&gt;&lt;/div&gt;&lt;div&gt;&lt;br /&gt;&lt;/div&gt;&lt;div&gt;&lt;div&gt;&lt;b&gt;&lt;span style=&quot;font-size: medium;&quot;&gt;ANSWER TO CASE 33:&lt;/span&gt;&lt;/b&gt;&lt;/div&gt;&lt;div&gt;&lt;span style=&quot;background-color: #daebf0; font-size: medium;&quot;&gt;&lt;span&gt;&lt;span style=&quot;background-color: transparent;&quot;&gt;&lt;span style=&quot;color: #017087;&quot;&gt;&lt;b&gt;Lateral Epicondylitis&lt;/b&gt;&lt;/span&gt;&lt;/span&gt;&lt;b&gt;&amp;nbsp;&amp;nbsp;&lt;/b&gt;&lt;/span&gt;&lt;span style=&quot;font-weight: bold;&quot;&gt;&amp;nbsp;&amp;nbsp; &amp;nbsp;&lt;/span&gt;&lt;span style=&quot;font-weight: bold;&quot;&gt;&amp;nbsp;&amp;nbsp; &amp;nbsp;&lt;/span&gt;&lt;span style=&quot;font-weight: bold;&quot;&gt;&amp;nbsp;&amp;nbsp; &amp;nbsp;&lt;/span&gt;&lt;span style=&quot;font-weight: bold;&quot;&gt;&amp;nbsp;&amp;nbsp; &amp;nbsp;&lt;/span&gt;&lt;span style=&quot;font-weight: bold;&quot;&gt;&amp;nbsp;&amp;nbsp; &amp;nbsp;&lt;/span&gt;&lt;span style=&quot;font-weight: bold;&quot;&gt;&amp;nbsp;&amp;nbsp; &amp;nbsp;&lt;/span&gt;&lt;span style=&quot;font-weight: bold;&quot;&gt;&amp;nbsp; &amp;nbsp;&amp;nbsp;&lt;/span&gt;&lt;/span&gt;&lt;/div&gt;&lt;/div&gt;&lt;div&gt;&lt;br /&gt;&lt;/div&gt;&lt;div&gt;&lt;div&gt;&lt;i&gt;Summary&lt;/i&gt;: A 43-year-old otherwise healthy dentist presents with left elbow pain localized to its lateral aspect. The pain has progressively worsened over the last several weeks, and he now reports stiffness and difficulty with grasping objects. The patient is found to have tenderness with palpation of the lateral epicondyle, specifically at the origin of the forearm extensors, as well as pain with resisted wrist extension and pain with resisted radial deviation. All motor and sensory findings are normal. Radiographs of the elbow are negative for any pathology.&lt;/div&gt;&lt;div&gt;&lt;ul style=&quot;text-align: left;&quot;&gt;&lt;li&gt;&lt;b&gt;Most likely diagnosis:&lt;/b&gt; Lateral epicondylitis (tennis elbow).&lt;/li&gt;&lt;li&gt;&lt;b&gt;Initial treatment:&lt;/b&gt; Rest and activity modifications designed to avoid repetitive movements that stress the wrist and forearm extensor tendons. Nonsteroidal anti-inflammatory medications (NSAIDs) can be prescribed, and icing may provide symptomatic relief. A forearm band or “counterforce brace” that is worn distal to the origin of the extensor group could be worn. Once pain has subsided, stretching and strengthening of the forearm extensors should be performed to prevent recurrence of symptoms.&lt;/li&gt;&lt;li&gt;&lt;b&gt;Utility of a steroid injection:&lt;/b&gt; Corticosteroids, usually administered concurrently with a local anesthetic, into the area of maximal tenderness, have been shown to be beneficial in achieving transient pain relief. However, multiple corticosteroid injections may be associated with rupture of the extensor tendon origin and/or lateral collateral ligament, both devastating complications.&lt;/li&gt;&lt;/ul&gt;&lt;/div&gt;&lt;/div&gt;&lt;div&gt;&lt;br /&gt;&lt;/div&gt;&lt;div&gt;&lt;br /&gt;&lt;/div&gt;&lt;div&gt;&lt;div&gt;&lt;b&gt;&lt;span style=&quot;color: #017087; font-size: medium;&quot;&gt;ANALYSIS&lt;/span&gt;&lt;/b&gt;&lt;/div&gt;&lt;div&gt;&lt;b&gt;&lt;span style=&quot;font-size: medium;&quot;&gt;&lt;i&gt;Objectives&lt;/i&gt;&lt;/span&gt;&lt;/b&gt;&lt;/div&gt;&lt;/div&gt;&lt;div&gt;&lt;div&gt;&lt;ol style=&quot;text-align: left;&quot;&gt;&lt;li&gt;Know the causes of lateral epicondylitis and the populations most affected.&lt;/li&gt;&lt;li&gt;Be familiar with the differential diagnosis for lateral epicondylitis.&lt;/li&gt;&lt;li&gt;Know the treatment options for lateral epicondylitis.&lt;/li&gt;&lt;/ol&gt;&lt;/div&gt;&lt;/div&gt;&lt;div&gt;&lt;br /&gt;&lt;/div&gt;&lt;div&gt;&lt;br /&gt;&lt;/div&gt;&lt;div&gt;&lt;div&gt;&lt;b&gt;&lt;i&gt;&lt;span style=&quot;font-size: medium;&quot;&gt;Considerations&lt;/span&gt;&lt;/i&gt;&lt;/b&gt;&lt;/div&gt;&lt;div&gt;&lt;br /&gt;&lt;/div&gt;&lt;div&gt;This 43-year-old dentist presents with left elbow pain localized to the extensor origin at the lateral elbow. His history and physical exam are concerning for lateral epicondylitis, more commonly known as tennis elbow. However, this diagnosis is one of exclusion, and it is essential to rule out other possible injuries, such as a radial and posterior interosseous nerve (PIN) entrapment syndromes, occult fractures, radial head arthritis, C6 and C7 nerve root compression, posterolateral plica, and osteochondral loose bodies. Plain radiographs and physical exam are effective at evaluating for such pathology. In this patient, exam findings that make the diagnosis of lateral epicondylitis more likely include localized elbow tenderness at the origin of the forearm extensors, pain with resisted wrist extension, and pain with resisted radial deviation.&lt;/div&gt;&lt;/div&gt;&lt;div&gt;&lt;br /&gt;&lt;/div&gt;&lt;div&gt;&lt;br /&gt;&lt;/div&gt;&lt;div&gt;&lt;div&gt;&lt;div&gt;&lt;b&gt;&lt;span style=&quot;font-size: medium;&quot;&gt;APPROACH TO:&lt;/span&gt;&lt;/b&gt;&lt;/div&gt;&lt;div&gt;&lt;span style=&quot;background-color: #daebf0; color: #017087; font-size: medium;&quot;&gt;&lt;span&gt;&lt;span style=&quot;background-color: transparent;&quot;&gt;&lt;b&gt;Lateral Epicondylitis&lt;/b&gt;&lt;/span&gt;&lt;b&gt;&amp;nbsp; &amp;nbsp;&amp;nbsp;&lt;/b&gt;&lt;/span&gt;&lt;span style=&quot;font-weight: bold;&quot;&gt;&amp;nbsp;&amp;nbsp; &amp;nbsp;&lt;/span&gt;&lt;span style=&quot;font-weight: bold;&quot;&gt;&amp;nbsp;&amp;nbsp; &amp;nbsp;&lt;/span&gt;&lt;span style=&quot;font-weight: bold;&quot;&gt;&amp;nbsp;&amp;nbsp; &amp;nbsp;&lt;/span&gt;&lt;span style=&quot;font-weight: bold;&quot;&gt;&amp;nbsp;&amp;nbsp; &amp;nbsp;&lt;/span&gt;&lt;span style=&quot;font-weight: bold;&quot;&gt;&amp;nbsp;&amp;nbsp; &amp;nbsp;&lt;/span&gt;&lt;span style=&quot;font-weight: bold;&quot;&gt;&amp;nbsp;&amp;nbsp; &amp;nbsp;&lt;/span&gt;&lt;span style=&quot;font-weight: bold;&quot;&gt;&amp;nbsp;&amp;nbsp;&amp;nbsp;&lt;/span&gt;&lt;span style=&quot;font-weight: bold;&quot;&gt;&amp;nbsp;&lt;/span&gt;&lt;/span&gt;&lt;/div&gt;&lt;/div&gt;&lt;div&gt;&lt;br /&gt;&lt;/div&gt;&lt;div&gt;&lt;b&gt;&lt;span style=&quot;color: #017087; font-size: medium;&quot;&gt;DEFINITIONS&lt;/span&gt;&lt;/b&gt;&lt;/div&gt;&lt;/div&gt;&lt;div&gt;&lt;br /&gt;&lt;/div&gt;&lt;div&gt;&lt;div&gt;&lt;b&gt;LATERAL EPICONDYLITIS:&lt;/b&gt; An overuse syndrome or tendinosis affecting the lateral humeral epicondyle. It most commonly involves the extensor carpi radialis brevis (ECRB) but may also involve the extensor digitorum communis (EDC). Lateral epicondylitis is more commonly referred to as tennis elbow.&lt;/div&gt;&lt;div&gt;&lt;b&gt;&lt;br /&gt;&lt;/b&gt;&lt;/div&gt;&lt;div&gt;&lt;b&gt;MAUDSLEY TEST:&lt;/b&gt; A test used to evaluate for tennis elbow. It is positive when the patient experiences pain in the region of the lateral epicondyle during resisted extension of the middle finger.&lt;/div&gt;&lt;/div&gt;&lt;div&gt;&lt;br /&gt;&lt;/div&gt;&lt;div&gt;&lt;br /&gt;&lt;/div&gt;&lt;div&gt;&lt;b&gt;&lt;span style=&quot;color: #017087; font-size: medium;&quot;&gt;CLINICAL APPROACH&lt;/span&gt;&lt;/b&gt;&lt;/div&gt;&lt;div&gt;&lt;br /&gt;&lt;/div&gt;&lt;div&gt;&lt;div&gt;&lt;b&gt;&lt;i&gt;&lt;span style=&quot;font-size: medium;&quot;&gt;Etiology&lt;/span&gt;&lt;/i&gt;&lt;/b&gt;&lt;/div&gt;&lt;div&gt;&lt;br /&gt;&lt;/div&gt;&lt;div&gt;Lateral epicondylitis is a condition that typically affects middle-aged men and women, resulting in mild to severe discomfort and functional impairment. Although it is also known as tennis elbow because of the vulnerability of tennis players to this injury, javelin throwers, bowlers, swimmers, golfers, and pitchers are also susceptible because of the stress that is placed on the forearm and wrist extensor tendons in those sports. Individuals whose vocational demands include repetitious upper extremity movements, such as carpenters, plumbers, shoemakers, surgeons, and musicians (ie, violinists), are also at risk of developing lateral epicondylitis. The ECRB is most often implicated, but EDC involvement also occurs.&lt;/div&gt;&lt;/div&gt;&lt;div&gt;&lt;br /&gt;&lt;/div&gt;&lt;div&gt;&lt;div&gt;&lt;span&gt;&amp;nbsp;&amp;nbsp; &amp;nbsp;&lt;/span&gt;Although epicondylitis is commonly thought of as an inflammatory process, this is a misconception; histology reveals neither acute nor chronic inflammatory changes, but instead hyaline degeneration and vascular proliferation, typically at the ECRB origin ( Figure 33–1 ). These histologic findings have been termed &lt;b&gt;&lt;i&gt;angiofibroblastic&amp;nbsp;&lt;/i&gt;&lt;/b&gt;&lt;b&gt;&lt;i&gt;dysplasia.&lt;/i&gt;&lt;/b&gt; One proposed mechanism by which this occurs involves repetitive microtrauma to the ECRB with an incomplete healing and regenerative response.&lt;/div&gt;&lt;/div&gt;&lt;div&gt;&lt;br /&gt;&lt;/div&gt;&lt;div&gt;&lt;br /&gt;&lt;/div&gt;&lt;div&gt;&lt;div&gt;&lt;b&gt;&lt;i&gt;&lt;span style=&quot;font-size: medium;&quot;&gt;Clinical Presentation and Diagnosis&lt;/span&gt;&lt;/i&gt;&lt;/b&gt;&lt;/div&gt;&lt;div&gt;&lt;br /&gt;&lt;/div&gt;&lt;div&gt;On presentation, patients complain of pain and sometimes stiffness around the lateral aspect of the elbow. They may also describe subjective feelings of tightness in the forearm. It is usually the dominant arm that is affected, and it is rarely seen bilaterally. Physical exam should include evaluation of range of motion at the wrist and elbow, motor strength of the forearm extensor muscles, and palpation of the radial head. Tenderness localized to the lateral epicondyle, where the extensor muscles originate, is present on palpation, especially when the elbow is held in extension, the forearm held in pronation, and the wrist held in flexion. Additionally, both resisted wrist extension and resisted radial deviation may cause pain in the region of the lateral epicondyle. The &lt;b&gt;Maudsley test&lt;/b&gt; should also be performed and is positive when the patient complains of pain in the region of the lateral epicondyle during resisted extension of the middle finger. A lidocaine injection test can be used to differentiate between tennis elbow and posterior interosseous nerve (PIN) syndrome, an entrapment neuropathy that is commonly misdiagnosed as lateral epicondylitis as a result of a similar constellation of symptoms. In the case of radial tunnel syndrome, an injection given 4 fingerbreadths distal to the lateral epicondyle will result in a temporary PIN palsy and in the setting of radial tunnel syndrome, a temporary relief in pain. If the injection fails to provide relief, the diagnosis of lateral epicondylitis is more likely and can be confirmed with transient pain relief from a lidocaine injection at the origin of the ECRB tendon.&lt;/div&gt;&lt;/div&gt;&lt;div&gt;&lt;br /&gt;&lt;/div&gt;&lt;div class=&quot;separator&quot; style=&quot;clear: both; text-align: center;&quot;&gt;&lt;a href=&quot;https://blogger.googleusercontent.com/img/b/R29vZ2xl/AVvXsEgyrzUDy8wbGSm81QWRmQ6JQnBEZODxZ_nuLEDrJ4FGqOoRJe0fIVj9Y9UXNXTE-6DTf4L5aaLimZPRqlJ8Zh2GQysuz0Jwt2YANJoOcu1SAayhCklgKiJziPFXBpUc6ef4_LeBqOyGZ2LTkGyy2c6m7xpF2fbAzDDod6OZtx51kpBK4ZTo5eE68zaSBA/s783/Lateral-Epicondylitis.jpg&quot; style=&quot;margin-left: 1em; margin-right: 1em;&quot;&gt;&lt;img alt=&quot;Lateral Epicondylitis&quot; border=&quot;0&quot; data-original-height=&quot;625&quot; data-original-width=&quot;783&quot; height=&quot;510&quot; src=&quot;https://blogger.googleusercontent.com/img/b/R29vZ2xl/AVvXsEgyrzUDy8wbGSm81QWRmQ6JQnBEZODxZ_nuLEDrJ4FGqOoRJe0fIVj9Y9UXNXTE-6DTf4L5aaLimZPRqlJ8Zh2GQysuz0Jwt2YANJoOcu1SAayhCklgKiJziPFXBpUc6ef4_LeBqOyGZ2LTkGyy2c6m7xpF2fbAzDDod6OZtx51kpBK4ZTo5eE68zaSBA/w640-h510/Lateral-Epicondylitis.jpg&quot; title=&quot;Lateral Epicondylitis&quot; width=&quot;640&quot; /&gt;&lt;/a&gt;&lt;/div&gt;&lt;div&gt;&lt;div&gt;&lt;b&gt;&lt;span style=&quot;color: #017087;&quot;&gt;Figure 33–1.&lt;/span&gt;&lt;/b&gt; (&lt;b&gt;A&lt;/b&gt;) Lateral view of the forearm. (&lt;b&gt;B&lt;/b&gt;) Superficial and (&lt;b&gt;C&lt;/b&gt;) deep muscles of the posterior&lt;/div&gt;&lt;div&gt;forearm. (Reproduced, with permission, from Morton DA, Foreman KB, Albertine KH. &lt;i&gt;The Big Picture&lt;/i&gt;: Gross Anatomy. New York, NY: McGraw-Hill; 2011:Fig. 32-2.)&lt;/div&gt;&lt;/div&gt;&lt;div&gt;&lt;br /&gt;&lt;/div&gt;&lt;div&gt;&lt;div&gt;Imaging has limited utility in the diagnosis of lateral epicondylitis and is most beneficial for ruling out other processes, such as arthritis or fractures. Plain radiographs only rarely show soft-tissue calcification near the lateral epicondyle, which if present would be suggestive of tennis elbow. Ultrasound and magnetic resonance imaging can be used to visualize the extensor tendons and elbow joint, but are only indicated if the patient’s symptoms fail to improve after 3 months of conservative treatment.&lt;/div&gt;&lt;/div&gt;&lt;div&gt;&lt;br /&gt;&lt;/div&gt;&lt;div&gt;&lt;br /&gt;&lt;/div&gt;&lt;div&gt;&lt;b style=&quot;color: #017087;&quot;&gt;&lt;span style=&quot;font-size: medium;&quot;&gt;TREATMENT&lt;/span&gt;&lt;/b&gt;&lt;/div&gt;&lt;div&gt;&lt;br /&gt;&lt;/div&gt;&lt;div&gt;&lt;div&gt;Conservative, nonoperative treatment modalities are always attempted first, with operative interventions reserved for refractory cases. Pain reduction is the first treatment goal, and NASIDs, rest, and splinting are often trialled. Patients should be advised to attempt to reduce performing strenuous activities that exacerbate their symptoms for at least 6 weeks. Wrist splints are particularly useful if the elbow tenderness is exacerbated by resisted wrist extension. A counterforce brace can also be used and functions as an inelastic cuff around the proximal forearm against the extensor compartment that reduces the forces generated by the muscles. Other conservative measures that may relieve pain include corticosteroid injections adjacent to the ECRB tendon and extracorporeal shock wave therapy. More recently, injection of platelet-rich plasma has also shown some promise in providing symptomatic relief. Once pain relief is achieved, extensor compartment stretching and strengthening exercises should commence and will help to prevent symptom recurrence.&lt;/div&gt;&lt;/div&gt;&lt;div&gt;&lt;br /&gt;&lt;/div&gt;&lt;div&gt;&lt;div&gt;&lt;span&gt;&amp;nbsp;&amp;nbsp; &amp;nbsp;&lt;/span&gt;Cases in which symptoms fail to resolve with 6 to 12 months of conservative treatment can be treated with operative intervention, in which any degenerative, angiofibrotic tissue is debrided, and the ERCB, common extensor tendon, and its aponeurosis are repaired if torn. Both open and arthroscopic techniques have been described. Surgery is followed by a period of rest followed by progressive rehabilitation and strengthening. The major complication following this procedure is posterolateral elbow instability, which results from excessive debridement of collateral ligament and extensor muscle origins on the lateral epicondyle.&lt;/div&gt;&lt;/div&gt;&lt;div&gt;&lt;br /&gt;&lt;/div&gt;&lt;div&gt;&lt;br /&gt;&lt;/div&gt;&lt;div&gt;&lt;b&gt;&lt;span style=&quot;color: #017087; font-size: medium;&quot;&gt;&lt;u&gt;COMPREHENSION QUESTIONS&lt;/u&gt;&lt;/span&gt;&lt;/b&gt;&lt;/div&gt;&lt;div&gt;&lt;br /&gt;&lt;/div&gt;&lt;div&gt;&lt;div&gt;33.1 A 45-year-old carpenter has chronic pain and stiffness over the lateral aspect of the elbow, especially when using a hammer. On exam, Maudsley test is positive. Which muscle attachment is likely to be involved?&lt;/div&gt;&lt;/div&gt;&lt;blockquote style=&quot;border: none; margin: 0px 0px 0px 40px; padding: 0px; text-align: left;&quot;&gt;&lt;div&gt;&lt;div&gt;A. Extensor carpi radialis longus&lt;/div&gt;&lt;/div&gt;&lt;div&gt;&lt;div&gt;B. Brachioradialis&lt;/div&gt;&lt;/div&gt;&lt;div&gt;&lt;div&gt;C. Extensor carpi radialis brevis&lt;/div&gt;&lt;/div&gt;&lt;div&gt;&lt;div&gt;D. Supinator&lt;/div&gt;&lt;/div&gt;&lt;/blockquote&gt;&lt;div&gt;&lt;div&gt;&lt;br /&gt;&lt;/div&gt;&lt;div&gt;33.2 A 65-year-old tennis player presents to your office complaining of pain localized over the insertion of the extensor carpi radialis brevis. On exam, you also note pain with resisted wrist extension and subsequently diagnosis the patient with lateral epicondylitis. What is the histologic term used to describe tennis elbow?&lt;/div&gt;&lt;/div&gt;&lt;blockquote style=&quot;border: none; margin: 0px 0px 0px 40px; padding: 0px; text-align: left;&quot;&gt;&lt;div&gt;&lt;div&gt;A. Reactive hyperemia&lt;/div&gt;&lt;/div&gt;&lt;div&gt;&lt;div&gt;B. Lateral epicondylitis&lt;/div&gt;&lt;/div&gt;&lt;div&gt;&lt;div&gt;C. Angiofibroblastic dysplasia&lt;/div&gt;&lt;/div&gt;&lt;div&gt;&lt;div&gt;D. Apoptosis&lt;/div&gt;&lt;/div&gt;&lt;/blockquote&gt;&lt;div&gt;&lt;br /&gt;&lt;/div&gt;&lt;div&gt;&lt;div&gt;33.3 A 40-year-old woman who you have recently diagnosed with lateral epicondylitis wants to know her treatment options. Which of the following are considered first-line interventions?&lt;/div&gt;&lt;/div&gt;&lt;blockquote style=&quot;border: none; margin: 0px 0px 0px 40px; padding: 0px; text-align: left;&quot;&gt;&lt;div&gt;&lt;div&gt;A. NSAIDs&lt;/div&gt;&lt;/div&gt;&lt;div&gt;&lt;div&gt;B. Wrist splinting&lt;/div&gt;&lt;/div&gt;&lt;div&gt;&lt;div&gt;C. ECRB debridement&lt;/div&gt;&lt;/div&gt;&lt;div&gt;&lt;div&gt;D. Counterforce brace&lt;/div&gt;&lt;/div&gt;&lt;div&gt;&lt;div&gt;E. A, B, D&lt;/div&gt;&lt;/div&gt;&lt;/blockquote&gt;&lt;div&gt;&lt;br /&gt;&lt;/div&gt;&lt;div&gt;&lt;br /&gt;&lt;/div&gt;&lt;div&gt;&lt;b&gt;&lt;span style=&quot;color: #017087; font-size: medium;&quot;&gt;&lt;u&gt;ANSWERS&lt;/u&gt;&lt;/span&gt;&lt;/b&gt;&lt;/div&gt;&lt;div&gt;&lt;br /&gt;&lt;/div&gt;&lt;div&gt;&lt;div&gt;33.1 &lt;b&gt;C.&lt;/b&gt; The extensor carpi radialis brevis is the most common forearm extensor associated with lateral epicondylitis. The extensor digitorum communis has also been implicated. The extensor carpi radialis longus and extensor carpi ulnaris can also be involved, but it is very rare.&lt;/div&gt;&lt;div&gt;&lt;br /&gt;&lt;/div&gt;&lt;div&gt;33.2 &lt;b&gt;C.&lt;/b&gt; The histologic term used to describe tennis elbow is angiofibroblastic dysplasia. This is because tennis elbow results from hyaline degeneration and vascular proliferation at the ECRB origin. Microscopic evaluation shows fibroblast hypertrophy, disorganized collagen, and vascular hyperplasia. Although tennis elbow is named lateral epicondylitis, it is not due to inflammatory processes.&lt;/div&gt;&lt;div&gt;&lt;br /&gt;&lt;/div&gt;&lt;div&gt;33.3 &lt;b&gt;E. &lt;/b&gt;Conservative measures used in the treatment of lateral epicondylitis include rest, ice, NSAIDs, physical therapy, bracing, steroid injections, extracorporeal shock wave therapy, and platelet-rich plasma. ECRB debridement with possible tendon repair is only performed after 6 to 12 months of failed conservative treatment.&lt;/div&gt;&lt;/div&gt;&lt;div&gt;&lt;br /&gt;&lt;/div&gt;&lt;div&gt;&lt;b style=&quot;background-color: #4093a5;&quot;&gt;&amp;nbsp; &amp;nbsp;&lt;span style=&quot;font-size: medium;&quot;&gt;&amp;nbsp;&lt;/span&gt;&lt;span style=&quot;color: white;&quot;&gt;&lt;span style=&quot;font-size: medium;&quot;&gt;CLINICAL PEARLS&lt;/span&gt;&lt;span&gt;&amp;nbsp; &amp;nbsp;&amp;nbsp;&lt;/span&gt;&lt;/span&gt;&lt;/b&gt;&lt;/div&gt;&lt;div&gt;&lt;table border=&quot;1&quot; cellpadding=&quot;0&quot; cellspacing=&quot;0&quot; class=&quot;MsoTableGrid&quot; style=&quot;border-collapse: collapse; border: none; mso-border-alt: solid windowtext .5pt; mso-padding-alt: 0in 5.4pt 0in 5.4pt; mso-yfti-tbllook: 1184;&quot;&gt;
  125. &lt;tbody&gt;&lt;tr style=&quot;height: 19.15pt; mso-yfti-firstrow: yes; mso-yfti-irow: 0; mso-yfti-lastrow: yes;&quot;&gt;
  126.  &lt;td style=&quot;background: rgb(221, 237, 239); border-left: solid #017087 1.0pt; border: 1pt solid rgb(64, 147, 165); height: 19.15pt; padding: 0in 5.4pt; width: 433.7pt;&quot; valign=&quot;top&quot; width=&quot;578&quot;&gt;
  127.  &lt;div style=&quot;line-height: normal; margin-bottom: 0in;&quot;&gt;&lt;span style=&quot;color: #017087;&quot;&gt;&lt;br /&gt;&lt;/span&gt;&lt;/div&gt;&lt;div style=&quot;line-height: normal; margin-bottom: 0in;&quot;&gt;&lt;span style=&quot;color: #017087;&quot;&gt;►&amp;nbsp;&lt;/span&gt;Lateral epicondylitis, or tennis elbow, is a diagnosis of exclusion.&lt;/div&gt;&lt;div style=&quot;line-height: normal; margin-bottom: 0in;&quot;&gt;&lt;span style=&quot;color: #017087;&quot;&gt;&lt;br /&gt;&lt;/span&gt;&lt;/div&gt;&lt;div style=&quot;line-height: normal; margin-bottom: 0in;&quot;&gt;&lt;span style=&quot;color: #017087;&quot;&gt;►&lt;/span&gt;&amp;nbsp;The ECRB is most often implicated, but EDC involvement also occurs in tennis elbow.&lt;/div&gt;&lt;div style=&quot;line-height: normal; margin-bottom: 0in;&quot;&gt;&lt;span style=&quot;color: #017087;&quot;&gt;&lt;br /&gt;&lt;/span&gt;&lt;/div&gt;&lt;div style=&quot;line-height: normal; margin-bottom: 0in;&quot;&gt;&lt;span style=&quot;color: #017087;&quot;&gt;►&lt;/span&gt;&amp;nbsp;Exam findings consistent with lateral epicondylitis include point tenderness at the forearm extensor origin on the lateral epicondyle, pain with resisted wrist extension, pain with resisted radial deviation, and a positive Maudsley test.&lt;/div&gt;&lt;div style=&quot;line-height: normal; margin-bottom: 0in;&quot;&gt;&lt;span style=&quot;color: #017087;&quot;&gt;&lt;br /&gt;&lt;/span&gt;&lt;/div&gt;&lt;div style=&quot;line-height: normal; margin-bottom: 0in;&quot;&gt;&lt;span style=&quot;color: #017087;&quot;&gt;►&lt;/span&gt;&amp;nbsp;Imaging rarely has a role in the diagnosis of tennis elbow.&lt;/div&gt;&lt;div style=&quot;line-height: normal; margin-bottom: 0in;&quot;&gt;&lt;span style=&quot;color: #017087;&quot;&gt;&lt;br /&gt;&lt;/span&gt;&lt;/div&gt;&lt;div style=&quot;line-height: normal; margin-bottom: 0in;&quot;&gt;&lt;span style=&quot;color: #017087;&quot;&gt;►&lt;/span&gt;&amp;nbsp;Conservative treatment modalities should always be tried first, with operative intervention reserved for refractory cases.&lt;/div&gt;&lt;div style=&quot;line-height: normal; margin-bottom: 0in;&quot;&gt;&lt;span style=&quot;color: #017087;&quot;&gt;&lt;br /&gt;&lt;/span&gt;&lt;/div&gt;&lt;div style=&quot;line-height: normal; margin-bottom: 0in;&quot;&gt;&lt;span style=&quot;color: #017087;&quot;&gt;►&lt;/span&gt;&amp;nbsp;Despite its name, lateral epicondylitis is not technically the result of an inflammatory response, but rather a pathological process of hyaline degeneration and vascular proliferation termed &lt;i&gt;angiofibroblastic&amp;nbsp;&lt;/i&gt;&lt;i&gt;dysplasia.&lt;/i&gt;&lt;/div&gt;&lt;div style=&quot;line-height: normal; margin-bottom: 0in;&quot;&gt;&lt;br /&gt;&lt;/div&gt;
  128.  &lt;/td&gt;
  129. &lt;/tr&gt;
  130. &lt;/tbody&gt;&lt;/table&gt;&lt;br /&gt;&lt;/div&gt;
  131.  
  132. &lt;details close=&quot;&quot;&gt;
  133.  &lt;summary&gt;&lt;b&gt;&lt;span style=&quot;color: #00788b;&quot;&gt;REFERENCES&lt;/span&gt;&lt;/b&gt;&lt;/summary&gt;
  134.  &lt;p&gt;De Smedt T, et al. Lateral epicondylitis in tennis: update on aetiology, biomechanics and treatment.
  135. Br J Sports Med . 2007;41:816-819.&amp;nbsp;&lt;/p&gt;&lt;p&gt;Faro F, Wolf JM. Lateral epicondylitis: review and current concepts. J Hand Surg Am . 2007;32:
  136. 1271-1279.&lt;/p&gt;
  137. &lt;/details&gt;&lt;/div&gt;</content><link rel='replies' type='application/atom+xml' href='https://medical-phd.blogspot.com/feeds/7696503260018442618/comments/default' title='Post Comments'/><link rel='replies' type='text/html' href='https://medical-phd.blogspot.com/2022/03/lateral-epicondylitis-case-file.html#comment-form' title='0 Comments'/><link rel='edit' type='application/atom+xml' href='https://www.blogger.com/feeds/3302710210518459416/posts/default/7696503260018442618'/><link rel='self' type='application/atom+xml' href='https://www.blogger.com/feeds/3302710210518459416/posts/default/7696503260018442618'/><link rel='alternate' type='text/html' href='https://medical-phd.blogspot.com/2022/03/lateral-epicondylitis-case-file.html' title='Lateral Epicondylitis Case File'/><author><name>Medical Group</name><uri>http://www.blogger.com/profile/01780656800578397393</uri><email>noreply@blogger.com</email><gd:image rel='http://schemas.google.com/g/2005#thumbnail' width='32' height='32' src='//blogger.googleusercontent.com/img/b/R29vZ2xl/AVvXsEiMuROkBDv6yw4SmxGqRA5BPQhOXSwFld-Gufmy4kUqfpoAGQRl7cv-3naew6wjAnCjRFSe_FKX8z7CdE6cFKxxBpnqBuwUTSr-15xd3mFGcr_wJOX2y73G_zJCGND98Q/s220/Final-logo.jpg'/></author><media:thumbnail xmlns:media="http://search.yahoo.com/mrss/" url="https://blogger.googleusercontent.com/img/b/R29vZ2xl/AVvXsEgyrzUDy8wbGSm81QWRmQ6JQnBEZODxZ_nuLEDrJ4FGqOoRJe0fIVj9Y9UXNXTE-6DTf4L5aaLimZPRqlJ8Zh2GQysuz0Jwt2YANJoOcu1SAayhCklgKiJziPFXBpUc6ef4_LeBqOyGZ2LTkGyy2c6m7xpF2fbAzDDod6OZtx51kpBK4ZTo5eE68zaSBA/s72-w640-h510-c/Lateral-Epicondylitis.jpg" height="72" width="72"/><thr:total>0</thr:total><georss:featurename>United States</georss:featurename><georss:point>37.09024 -95.712891</georss:point><georss:box>8.780006163821156 -130.869141 65.400473836178847 -60.556641</georss:box></entry><entry><id>tag:blogger.com,1999:blog-3302710210518459416.post-3558360047720998195</id><published>2022-03-26T14:53:00.000-07:00</published><updated>2022-03-26T14:53:07.079-07:00</updated><category scheme="http://www.blogger.com/atom/ns#" term="Case Files"/><category scheme="http://www.blogger.com/atom/ns#" term="Orthopaedic Surgery Case File"/><title type='text'>Trigger Finger Case File</title><content type='html'>&lt;div class=&quot;scrollbox&quot;&gt;
  138.  
  139. &lt;div style=&quot;text-align: left;&quot;&gt;&lt;b&gt;&lt;span style=&quot;font-size: large;&quot;&gt;Trigger Finger Case File&lt;/span&gt;&lt;/b&gt;&lt;/div&gt;&lt;div&gt;Eugene C. Toy, MD, Andrew J. Rosenbaum, MD, Timothy T. Roberts, MD, Joshua S. Dines, MD&lt;/div&gt;&lt;div&gt;&lt;br /&gt;&lt;/div&gt;&lt;div&gt;&lt;b&gt;&lt;span style=&quot;color: #017087; font-size: medium;&quot;&gt;CASE 32&lt;/span&gt;&lt;/b&gt;&lt;/div&gt;&lt;div&gt;&lt;div&gt;A 60-year-old right-hand dominant woman presents complaining of 12 weeks of right ring finger pain. She is an avid gardener but denies any injury. She notes a clicking sensation when she moves her finger. She occasionally experiences the finger “getting stuck” when she bends it and she has to use her other hand to force the finger back into extension. She reports minimal pain at rest. The pain does not awaken her at night, but she does note ring finger stiffness in the morning. She denies swelling or redness. She has tried resting and limiting her gardening, and she has been taking ibuprofen. Her medical history includes diabetes. On examination, the skin is intact and appears normal. There is no atrophy, swelling, ecchymosis, or erythema. She has a full range of passive and active motion of her fingers and wrist. There is tenderness over the palmar aspect of the ring finger metacarpophalangeal (MCP) joint and palpable popping with active finger flexion and extension. Strength is 5/5 in finger flexion, extension, and abduction/adduction. Sensation is intact to light touch. She has 2+ pulses and capillary refill is less than 2 seconds. Compression testing at the carpal tunnel is negative, as is Phalen and Tinel testing. Hand radiographs show intact joint spaces and normal alignment.&lt;/div&gt;&lt;div&gt;&lt;br /&gt;&lt;/div&gt;&lt;div&gt;&lt;span style=&quot;color: #017087;&quot;&gt;►&lt;/span&gt;&amp;nbsp;What is the most likely diagnosis?&lt;/div&gt;&lt;div&gt;&lt;span style=&quot;color: #017087;&quot;&gt;►&lt;/span&gt;&amp;nbsp;What is the next diagnostic step?&lt;/div&gt;&lt;div&gt;&lt;span style=&quot;color: #017087;&quot;&gt;►&lt;/span&gt;&amp;nbsp;What is the next step in therapy?&lt;/div&gt;&lt;/div&gt;&lt;div&gt;&lt;br /&gt;&lt;/div&gt;&lt;div&gt;&lt;br /&gt;&lt;/div&gt;&lt;div&gt;&lt;div&gt;&lt;b&gt;&lt;span style=&quot;font-size: medium;&quot;&gt;ANSWER TO CASE 32:&lt;/span&gt;&lt;/b&gt;&lt;/div&gt;&lt;div&gt;&lt;span style=&quot;background-color: #daebf0; font-size: medium;&quot;&gt;&lt;span&gt;&lt;span style=&quot;background-color: transparent;&quot;&gt;&lt;span style=&quot;color: #017087;&quot;&gt;&lt;b&gt;Trigger Finger&lt;/b&gt;&lt;/span&gt;&lt;/span&gt;&lt;b&gt;&amp;nbsp;&amp;nbsp;&lt;/b&gt;&lt;/span&gt;&lt;span style=&quot;font-weight: bold;&quot;&gt;&amp;nbsp;&amp;nbsp; &amp;nbsp;&lt;/span&gt;&lt;span style=&quot;font-weight: bold;&quot;&gt;&amp;nbsp;&amp;nbsp; &amp;nbsp;&lt;/span&gt;&lt;span style=&quot;font-weight: bold;&quot;&gt;&amp;nbsp;&amp;nbsp; &amp;nbsp;&lt;/span&gt;&lt;span style=&quot;font-weight: bold;&quot;&gt;&amp;nbsp;&amp;nbsp; &amp;nbsp;&lt;/span&gt;&lt;span style=&quot;font-weight: bold;&quot;&gt;&amp;nbsp;&amp;nbsp; &amp;nbsp;&lt;/span&gt;&lt;span style=&quot;font-weight: bold;&quot;&gt;&amp;nbsp;&amp;nbsp; &amp;nbsp;&lt;/span&gt;&lt;span style=&quot;font-weight: bold;&quot;&gt;&amp;nbsp; &amp;nbsp;&amp;nbsp;&lt;/span&gt;&lt;/span&gt;&lt;/div&gt;&lt;/div&gt;&lt;div&gt;&lt;br /&gt;&lt;/div&gt;&lt;div&gt;&lt;div&gt;&lt;i&gt;Summary&lt;/i&gt;: A 60-year-old diabetic woman presents to the office with 6 weeks of right ring finger pain, popping, and locking with no history of injury. There is tenderness to palpation over the flexor tendons overlying the MCP joint and palpable triggering. There is full range of motion and good strength.&lt;/div&gt;&lt;div&gt;&lt;ul style=&quot;text-align: left;&quot;&gt;&lt;li&gt;&lt;b&gt;Most likely diagnosis:&lt;/b&gt; Trigger finger.&lt;/li&gt;&lt;li&gt;&lt;b&gt;Next diagnostic step:&lt;/b&gt; Physical exam is usually sufficient to diagnose this condition. Further confirmation may be achieved with an injection of local anesthetic into the flexor tendon sheath. This results in a temporary resolution of pain so that triggering, if present, can be readily elicited.&lt;/li&gt;&lt;li&gt;&lt;b&gt;Next step in therapy:&lt;/b&gt; This patient has a 12-week history of symptoms and has not seen improvement after rest, activity modification, and NSAIDs. The next step is steroid injection into the flexor tendon sheath.&lt;/li&gt;&lt;/ul&gt;&lt;/div&gt;&lt;/div&gt;&lt;div&gt;&lt;br /&gt;&lt;/div&gt;&lt;div&gt;&lt;br /&gt;&lt;/div&gt;&lt;div&gt;&lt;div&gt;&lt;b&gt;&lt;span style=&quot;color: #017087; font-size: medium;&quot;&gt;ANALYSIS&lt;/span&gt;&lt;/b&gt;&lt;/div&gt;&lt;div&gt;&lt;b&gt;&lt;span style=&quot;font-size: medium;&quot;&gt;&lt;i&gt;Objectives&lt;/i&gt;&lt;/span&gt;&lt;/b&gt;&lt;/div&gt;&lt;/div&gt;&lt;div&gt;&lt;div&gt;&lt;ol style=&quot;text-align: left;&quot;&gt;&lt;li&gt;Understand the pathoanatomy of trigger finger.&lt;/li&gt;&lt;li&gt;Appreciate the functions of the flexor tendons and the flexor pulley system and how they relate to trigger finger.&lt;/li&gt;&lt;li&gt;Be familiar with the treatment options for trigger finger.&lt;/li&gt;&lt;/ol&gt;&lt;/div&gt;&lt;/div&gt;&lt;div&gt;&lt;br /&gt;&lt;/div&gt;&lt;div&gt;&lt;br /&gt;&lt;/div&gt;&lt;div&gt;&lt;div&gt;&lt;i&gt;&lt;b&gt;&lt;span style=&quot;font-size: medium;&quot;&gt;Considerations&lt;/span&gt;&lt;/b&gt;&lt;/i&gt;&lt;/div&gt;&lt;div&gt;&lt;br /&gt;&lt;/div&gt;&lt;div&gt;This 60-year-old woman with finger pain and popping has tendon entrapment, also known as trigger finger. The diagnosis can generally be made with a thorough history and physical examination. It is important to rule out more serious causes for finger pain, including infection or fracture. Infection would have associated swelling, erythema, and possibly constitutional symptoms such as fever and chills. Fractures result from an injury and initially will have swelling and ecchymosis. These conditions can easily be ruled out in this case by the history and exam. If doubt exists, plain radiographs can be used to rule out fracture, arthritis, or underlying bony deformity. Other common conditions should also be ruled out, including arthritis and carpal tunnel syndrome. Arthritis is a common cause of joint pain and stiffness in the hand. Examination will reveal tenderness over the joint. In this patient’s case, the tenderness is over the palmar aspect of the MCP joint. Anatomically, this region of the joint is covered by the flexor tendons and pulley system. The radial and ulnar sides of the MCP joint are more accessible to palpation and are nontender in this patient.&lt;/div&gt;&lt;div&gt;&lt;br /&gt;&lt;/div&gt;&lt;div&gt;&lt;span&gt;&amp;nbsp;&amp;nbsp; &amp;nbsp;&lt;/span&gt;The severity of the mechanical symptoms in a patient with trigger finger should be determined. Early on, some patients may complain of pain alone, whereas in more advanced cases, patients may have significant popping and locking to the point that the finger becomes fixed in a flexed position. This can lead to permanent loss of motion in the finger if untreated.&lt;/div&gt;&lt;/div&gt;&lt;div&gt;&lt;br /&gt;&lt;/div&gt;&lt;div&gt;&lt;br /&gt;&lt;/div&gt;&lt;div&gt;&lt;div&gt;&lt;div&gt;&lt;b&gt;&lt;span style=&quot;font-size: medium;&quot;&gt;APPROACH TO:&lt;/span&gt;&lt;/b&gt;&lt;/div&gt;&lt;div&gt;&lt;span style=&quot;background-color: #daebf0; color: #017087; font-size: medium;&quot;&gt;&lt;span&gt;&lt;span style=&quot;background-color: transparent;&quot;&gt;&lt;b&gt;Trigger Finger&lt;/b&gt;&lt;/span&gt;&lt;b&gt;&amp;nbsp; &amp;nbsp;&amp;nbsp;&lt;/b&gt;&lt;/span&gt;&lt;span style=&quot;font-weight: bold;&quot;&gt;&amp;nbsp;&amp;nbsp; &amp;nbsp;&lt;/span&gt;&lt;span style=&quot;font-weight: bold;&quot;&gt;&amp;nbsp;&amp;nbsp; &amp;nbsp;&lt;/span&gt;&lt;span style=&quot;font-weight: bold;&quot;&gt;&amp;nbsp;&amp;nbsp; &amp;nbsp;&lt;/span&gt;&lt;span style=&quot;font-weight: bold;&quot;&gt;&amp;nbsp;&amp;nbsp; &amp;nbsp;&lt;/span&gt;&lt;span style=&quot;font-weight: bold;&quot;&gt;&amp;nbsp;&amp;nbsp; &amp;nbsp;&lt;/span&gt;&lt;span style=&quot;font-weight: bold;&quot;&gt;&amp;nbsp;&amp;nbsp; &amp;nbsp;&lt;/span&gt;&lt;span style=&quot;font-weight: bold;&quot;&gt;&amp;nbsp;&amp;nbsp;&amp;nbsp;&lt;/span&gt;&lt;span style=&quot;font-weight: bold;&quot;&gt;&amp;nbsp;&lt;/span&gt;&lt;/span&gt;&lt;/div&gt;&lt;/div&gt;&lt;div&gt;&lt;br /&gt;&lt;/div&gt;&lt;div&gt;&lt;b&gt;&lt;span style=&quot;color: #017087; font-size: medium;&quot;&gt;DEFINITIONS&lt;/span&gt;&lt;/b&gt;&lt;/div&gt;&lt;/div&gt;&lt;div&gt;&lt;br /&gt;&lt;/div&gt;&lt;div&gt;&lt;div&gt;&lt;b&gt;TENOSYNOVIUM:&lt;/b&gt; Thin synovial lining surrounding tendons that provides lubricating fluid and nutrition to the tendons.&lt;/div&gt;&lt;div&gt;&lt;br /&gt;&lt;/div&gt;&lt;div&gt;&lt;b&gt;PULLEY:&lt;/b&gt; Fibrous sheath through which the flexor tendon glides. The flexor tendon sheath functions to improve mechanics and maintain the tendon against the metacarpal and phalanges. In each finger, there are 4 to 5 annular (circular) pulleys and 3 cruciate (crossing) pulleys that make up the tendon sheath ( Figure 32–1 ).&lt;/div&gt;&lt;/div&gt;&lt;div&gt;&lt;br /&gt;&lt;/div&gt;&lt;div class=&quot;separator&quot; style=&quot;clear: both; text-align: center;&quot;&gt;&lt;a href=&quot;https://blogger.googleusercontent.com/img/b/R29vZ2xl/AVvXsEh8hs1pWaVK9HUwOZONN249NuTQJiCvZJLXroBzCuMN-wVlESWrBVva7bVJW3NDYL0ONFHjKSkaVu3LM0lXgVjlnhKsEtTQw3c_NsUKFarGc8sKu9aEa9V9yxoblE0NrnLlYjGklHjlskUQUh-38eYL0dDuuDUb_oyHh9SJ-RL_qEccHBGV8Vb5CQyMLQ/s603/Finger-flexor-tendon-pulley-system.jpg&quot; style=&quot;margin-left: 1em; margin-right: 1em;&quot;&gt;&lt;img alt=&quot;Finger flexor tendon pulley system&quot; border=&quot;0&quot; data-original-height=&quot;603&quot; data-original-width=&quot;326&quot; height=&quot;640&quot; src=&quot;https://blogger.googleusercontent.com/img/b/R29vZ2xl/AVvXsEh8hs1pWaVK9HUwOZONN249NuTQJiCvZJLXroBzCuMN-wVlESWrBVva7bVJW3NDYL0ONFHjKSkaVu3LM0lXgVjlnhKsEtTQw3c_NsUKFarGc8sKu9aEa9V9yxoblE0NrnLlYjGklHjlskUQUh-38eYL0dDuuDUb_oyHh9SJ-RL_qEccHBGV8Vb5CQyMLQ/w346-h640/Finger-flexor-tendon-pulley-system.jpg&quot; title=&quot;Finger flexor tendon pulley system&quot; width=&quot;346&quot; /&gt;&lt;/a&gt;&lt;/div&gt;&lt;div&gt;&lt;div&gt;&lt;b&gt;&lt;span style=&quot;color: #017087;&quot;&gt;Figure 32–1.&lt;/span&gt;&lt;/b&gt; Finger flexor tendon pulley system. The pulleys are attached to the palmar surface of&lt;/div&gt;&lt;div&gt;each finger metacarpal (MC), proximal phalanx (PP), middle pharynx (MP), and distal pharynx (DP).&lt;/div&gt;&lt;div&gt;There are 5 annular pulleys, A1 through A5, and 3 cruciate pulleys (C1-3). &lt;b&gt;Triggering occurs when&lt;/b&gt;&lt;/div&gt;&lt;div&gt;&lt;b&gt;there is narrowing at the A1 pulley.&lt;/b&gt; (Reproduced, with permission, from Brunicardi FC, Andersen DK, Billiar TR, et al. &lt;i&gt;Schwartz’s Principles of Surgery&lt;/i&gt;. 9th ed. New York, NY: McGraw-Hill; 2010:Fig. 44-4.)&lt;/div&gt;&lt;/div&gt;&lt;div&gt;&lt;br /&gt;&lt;/div&gt;&lt;div&gt;&lt;div&gt;&lt;b&gt;TRIGGER FINGER&lt;/b&gt; &lt;b&gt;(ie, tendon entrapment):&lt;/b&gt; Inflammation of the tenosynovium and narrowing of the first pulley, resulting in pain and popping with finger movement.&lt;/div&gt;&lt;div&gt;&lt;b&gt;&lt;br /&gt;&lt;/b&gt;&lt;/div&gt;&lt;div&gt;&lt;b&gt;FINGER FLEXOR TENDONS: &lt;/b&gt;Each finger has 1 tendon from the flexor digitorum profundus, which attaches to the distal phalanx and flexes the distal interphalangeal joint (DIP), and 2 slips (derived from a single tendon) from the flexor digitorum superficialis (FDS), which attach to the middle phalanx and flex the proximal interphalangeal joint (PIP). The thumb pulley system contains the flexor pollicis longus and attaches to the distal phalanx, flexing the thumb interphalangeal joint.&lt;/div&gt;&lt;/div&gt;&lt;div&gt;&lt;br /&gt;&lt;/div&gt;&lt;div&gt;&lt;br /&gt;&lt;/div&gt;&lt;div&gt;&lt;b&gt;&lt;span style=&quot;color: #017087; font-size: medium;&quot;&gt;CLINICAL APPROACH&lt;/span&gt;&lt;/b&gt;&lt;/div&gt;&lt;div&gt;&lt;br /&gt;&lt;/div&gt;&lt;div&gt;&lt;div&gt;&lt;b&gt;&lt;i&gt;&lt;span style=&quot;font-size: medium;&quot;&gt;Etiologies&lt;/span&gt;&lt;/i&gt;&lt;/b&gt;&lt;/div&gt;&lt;div&gt;&lt;br /&gt;&lt;/div&gt;&lt;div&gt;Tendon entrapment can occur in any finger or the thumb. It is also common to see patients with multiple fingers affected. Trigger finger is most common in patients 55 to 60 years of age and is at least twice as common in women as in men. It is generally thought to be a degenerative condition resulting from long-term repetitive use of the hand, especially with repeated gripping or pinching. It is more commonly seen in the dominant hand. Systemic factors that can cause or exacerbate tendon entrapment include diabetes, gout, and rheumatoid arthritis.&lt;/div&gt;&lt;div&gt;&lt;br /&gt;&lt;/div&gt;&lt;div&gt;&lt;span&gt;&amp;nbsp;&amp;nbsp; &amp;nbsp;&lt;/span&gt;The flexor tendons glide through a series of fibrous pulleys ( Figure 32–1 ) that make up the tendon sheath. The pulleys act to constrain the tendons against the bone. This increases the force transmission and allows tendon gliding with finger motion. There are 4 to 5 annular pulleys and 3 cruciate pulleys. &lt;b&gt;The A2 and A4 pulleys are most critical&amp;nbsp;&lt;/b&gt;&lt;b&gt;for function.&lt;/b&gt; The first pulley (A1) may become narrowed over time, leading to tendon entrapment. The tendons are lined with a thin tenosynovium that provides lubrication and nutrition. With repetitive use of the hand, the tenosynovium may become inflamed, resulting in pain over the tendon aggravated by use (termed tenosynovitis ). As the condition worsens, degenerative changes in the fibrous pulley occur, resulting in thickening and inflammatory cell infiltration. As the A1 pulley becomes more hypertrophied, stenosis or narrowing occurs. This leads to a popping or catching sensation with finger flexion and extension as the tendon bunches up and gets stuck in the pulley.&lt;/div&gt;&lt;/div&gt;&lt;div&gt;&lt;br /&gt;&lt;/div&gt;&lt;div&gt;&lt;br /&gt;&lt;/div&gt;&lt;div&gt;&lt;div&gt;&lt;b&gt;&lt;i&gt;&lt;span style=&quot;font-size: medium;&quot;&gt;Clinical Presentation&lt;/span&gt;&lt;/i&gt;&lt;/b&gt;&lt;/div&gt;&lt;div&gt;&lt;br /&gt;&lt;/div&gt;&lt;div&gt;In the early stages of the condition, pain is the primary complaint. Pain is reported with activities that involve use of the hand, especially gripping or holding objects. Most patients report minimal pain at rest but may experience sharp pain with aggravating activities. As the condition worsens, popping begins to occur with active finger motion. The finger may begin to “catch” or temporarily get stuck and, in more advanced cases, may lock in a flexed position, requiring assistance to force the finger back in extension. Patients often complain of stiffness upon waking in the morning and may note that the finger is locked in flexion when they wake up. There is generally no swelling, numbness, or tingling. However, some patients can feel a bump or a nodule in the flexor tendons that is tender to the touch.&lt;/div&gt;&lt;div&gt;&lt;br /&gt;&lt;/div&gt;&lt;div&gt;&lt;span&gt;&amp;nbsp;&amp;nbsp; &amp;nbsp;&lt;/span&gt;Physical examination usually confirms the diagnosis. The A1 pulley, which overlies the metacarpal head and can be felt as a bump at the base of each finger in the distal aspect of the palm, is palpated and is very tender to touch in the setting of trigger finger. There is generally no tenderness elsewhere. Next, range of motion should be examined and is usually normal. In severe cases with chronic locking, some patients may develop a flexion contracture where they cannot fully extend the finger. The examiner should then palpate over the A1 pulley while the patient actively flexes and extends the finger to check for popping. One may also note a nodularity in the tendon as it glides with finger motion. A complete examination of the hand and wrist should be performed to rule out other conditions.&lt;/div&gt;&lt;/div&gt;&lt;div&gt;&lt;br /&gt;&lt;/div&gt;&lt;div&gt;&lt;div&gt;&lt;span&gt;&amp;nbsp;&amp;nbsp; &amp;nbsp;&lt;/span&gt;Radiographs are routinely obtained to rule out other conditions including arthritis and fracture. There are no radiographic abnormalities associated with trigger finger.&amp;nbsp;&lt;b&gt;The diagnosis can be confirmed with an injection of local anesthetic into the&amp;nbsp;&lt;/b&gt;&lt;b&gt;flexor tendon sheath. This will result in complete pain resolution.&lt;/b&gt;&lt;/div&gt;&lt;/div&gt;&lt;div&gt;&lt;br /&gt;&lt;/div&gt;&lt;div&gt;&lt;br /&gt;&lt;/div&gt;&lt;div&gt;&lt;b style=&quot;color: #017087;&quot;&gt;&lt;span style=&quot;font-size: medium;&quot;&gt;TREATMENT&lt;/span&gt;&lt;/b&gt;&lt;/div&gt;&lt;div&gt;&lt;br /&gt;&lt;/div&gt;&lt;div&gt;&lt;div&gt;Conservative treatment is effective in most patients with a trigger finger. In early cases with mild symptoms, simple activity modification, rest, and oral anti-inflammatories may be effective. With more significant pain and popping and in more chronic cases, the next step is usually a steroid injection into the flexor tendon sheath. &lt;b&gt;Injections have been reported to result in complete symptom resolution in 57% of cases, but some patients may require more than one injection.&lt;/b&gt; Patients with systemic causes for trigger finger such as diabetes have worse results with conservative treatment, with success seen in less than 50%.&lt;/div&gt;&lt;/div&gt;&lt;div&gt;&lt;br /&gt;&lt;/div&gt;&lt;div&gt;&lt;div&gt;&lt;span&gt;&amp;nbsp;&amp;nbsp; &amp;nbsp;&lt;/span&gt;In cases that do not respond to conservative treatment, surgery is an option. This involves surgically dividing the A1 pulley to relieve the tendon entrapment. The A2 and other distal pulleys remain intact, and no function is compromised.&amp;nbsp;&lt;b&gt;The success rate of surgery is very high (&amp;gt; 90% by most reports).&lt;/b&gt;&lt;/div&gt;&lt;/div&gt;&lt;div&gt;&lt;br /&gt;&lt;/div&gt;&lt;div&gt;&lt;br /&gt;&lt;/div&gt;&lt;div&gt;&lt;b&gt;&lt;span style=&quot;color: #017087; font-size: medium;&quot;&gt;&lt;u&gt;COMPREHENSION QUESTIONS&lt;/u&gt;&lt;/span&gt;&lt;/b&gt;&lt;/div&gt;&lt;div&gt;&lt;br /&gt;&lt;/div&gt;&lt;div&gt;&lt;div&gt;32.1 A 55-year-old woman presents with sudden catching and locking of her ring finger when trying to extend it. She experiences severe pain and notes tenderness in her distal palm. You suspect that this is a trigger finger. What is the best test for confirming this diagnosis?&lt;/div&gt;&lt;/div&gt;&lt;blockquote style=&quot;border: none; margin: 0px 0px 0px 40px; padding: 0px; text-align: left;&quot;&gt;&lt;div&gt;&lt;div&gt;A. X-rays&lt;/div&gt;&lt;/div&gt;&lt;div&gt;&lt;div&gt;B. Computed tomography scan&lt;/div&gt;&lt;/div&gt;&lt;div&gt;&lt;div&gt;C. Magnetic resonance imaging&lt;/div&gt;&lt;/div&gt;&lt;div&gt;&lt;div&gt;D. Lidocaine injection&lt;/div&gt;&lt;/div&gt;&lt;div&gt;&lt;div&gt;E. Ultrasound of the hand&lt;/div&gt;&lt;/div&gt;&lt;/blockquote&gt;&lt;div&gt;&lt;div&gt;&lt;br /&gt;&lt;/div&gt;&lt;div&gt;32.2 A 45-year-old woman whom you have recently diagnosed with trigger finger asks you to explain in greater detail why her ring finger locks with flexion. Which of the following statements is most accurate regarding the mechanism for triggering?&lt;/div&gt;&lt;/div&gt;&lt;blockquote style=&quot;border: none; margin: 0px 0px 0px 40px; padding: 0px; text-align: left;&quot;&gt;&lt;div&gt;&lt;div&gt;A. Trigger finger is caused by entrapment of the flexor tendons at the level of the A1 pulley.&lt;/div&gt;&lt;/div&gt;&lt;div&gt;&lt;div&gt;B. The mechanism that causes the finger to trigger is unknown.&lt;/div&gt;&lt;/div&gt;&lt;div&gt;&lt;div&gt;C. Trigger finger is due to a vascular anomaly.&lt;/div&gt;&lt;/div&gt;&lt;div&gt;&lt;div&gt;D. Trigger finger is caused by a traumatic rupture of the A1 pulley.&lt;/div&gt;&lt;/div&gt;&lt;/blockquote&gt;&lt;div&gt;&lt;br /&gt;&lt;/div&gt;&lt;div&gt;&lt;div&gt;32.3 A diabetic patient with a chronic trigger finger has locking of the finger with flexion and has failed 2 prior steroid injections. What is the most effective treatment option for this patient?&lt;/div&gt;&lt;/div&gt;&lt;blockquote style=&quot;border: none; margin: 0px 0px 0px 40px; padding: 0px; text-align: left;&quot;&gt;&lt;div&gt;&lt;div&gt;A. Surgery&lt;/div&gt;&lt;/div&gt;&lt;div&gt;&lt;div&gt;B. Repeat steroid injection&lt;/div&gt;&lt;/div&gt;&lt;div&gt;&lt;div&gt;C. Physical therapy&lt;/div&gt;&lt;/div&gt;&lt;div&gt;&lt;div&gt;D. Finger splinting&lt;/div&gt;&lt;/div&gt;&lt;div&gt;&lt;div&gt;E. Oral anti-inflammatories&lt;/div&gt;&lt;/div&gt;&lt;/blockquote&gt;&lt;div&gt;&lt;br /&gt;&lt;/div&gt;&lt;div&gt;&lt;br /&gt;&lt;/div&gt;&lt;div&gt;&lt;b&gt;&lt;span style=&quot;color: #017087; font-size: medium;&quot;&gt;&lt;u&gt;ANSWERS&lt;/u&gt;&lt;/span&gt;&lt;/b&gt;&lt;/div&gt;&lt;div&gt;&lt;br /&gt;&lt;/div&gt;&lt;div&gt;&lt;div&gt;32.1 &lt;b&gt;D.&lt;/b&gt; A lidocaine injection into the flexor tendon sheath resulting in complete pain relief confirms the diagnosis. However, history and physical exam are usually adequate for the diagnosis, and further confirmation is not required in most cases.&lt;/div&gt;&lt;div&gt;&lt;br /&gt;&lt;/div&gt;&lt;div&gt;32.2 &lt;b&gt;A.&lt;/b&gt; The flexor tendons glide through a series of fibrous pulleys that make up the tendon sheath. The first pulley (A1) may become narrowed over time, and as it becomes more hypertrophied, stenosis or narrowing occurs. This leads to a popping or catching sensation with finger flexion and extension as the tendon bunches up and gets stuck in the pulley.&lt;/div&gt;&lt;div&gt;&lt;br /&gt;&lt;/div&gt;&lt;div&gt;32.3 &lt;b&gt;A.&lt;/b&gt; This patient has failed 2 injections, and other conservative treatments are unlikely to be effective because of the associated diabetes. Surgical release of the A1 pulley has a success rate of greater than 90%.&lt;/div&gt;&lt;/div&gt;&lt;div&gt;&lt;br /&gt;&lt;/div&gt;&lt;div&gt;&lt;b style=&quot;background-color: #4093a5;&quot;&gt;&amp;nbsp; &amp;nbsp;&lt;span style=&quot;font-size: medium;&quot;&gt;&amp;nbsp;&lt;/span&gt;&lt;span style=&quot;color: white;&quot;&gt;&lt;span style=&quot;font-size: medium;&quot;&gt;CLINICAL PEARLS&lt;/span&gt;&lt;span&gt;&amp;nbsp; &amp;nbsp;&amp;nbsp;&lt;/span&gt;&lt;/span&gt;&lt;/b&gt;&lt;/div&gt;&lt;div&gt;&lt;table border=&quot;1&quot; cellpadding=&quot;0&quot; cellspacing=&quot;0&quot; class=&quot;MsoTableGrid&quot; style=&quot;border-collapse: collapse; border: none; mso-border-alt: solid windowtext .5pt; mso-padding-alt: 0in 5.4pt 0in 5.4pt; mso-yfti-tbllook: 1184;&quot;&gt;
  140. &lt;tbody&gt;&lt;tr style=&quot;height: 19.15pt; mso-yfti-firstrow: yes; mso-yfti-irow: 0; mso-yfti-lastrow: yes;&quot;&gt;
  141.  &lt;td style=&quot;background: rgb(221, 237, 239); border-left: solid #017087 1.0pt; border: 1pt solid rgb(64, 147, 165); height: 19.15pt; padding: 0in 5.4pt; width: 433.7pt;&quot; valign=&quot;top&quot; width=&quot;578&quot;&gt;
  142.  &lt;div style=&quot;line-height: normal; margin-bottom: 0in;&quot;&gt;&lt;span style=&quot;color: #017087;&quot;&gt;&lt;br /&gt;&lt;/span&gt;&lt;/div&gt;&lt;div style=&quot;line-height: normal; margin-bottom: 0in;&quot;&gt;&lt;span style=&quot;color: #017087;&quot;&gt;►&lt;/span&gt;&amp;nbsp;Trigger finger is caused by flexor tendon entrapment in a narrowed A1 pulley.&lt;/div&gt;&lt;div style=&quot;line-height: normal; margin-bottom: 0in;&quot;&gt;&lt;span style=&quot;color: #017087;&quot;&gt;&lt;br /&gt;&lt;/span&gt;&lt;/div&gt;&lt;div style=&quot;line-height: normal; margin-bottom: 0in;&quot;&gt;&lt;span style=&quot;color: #017087;&quot;&gt;►&lt;/span&gt;&amp;nbsp;Trigger finger usually responds to conservative treatment.&lt;/div&gt;&lt;div style=&quot;line-height: normal; margin-bottom: 0in;&quot;&gt;&lt;span style=&quot;color: #017087;&quot;&gt;&lt;br /&gt;&lt;/span&gt;&lt;/div&gt;&lt;div style=&quot;line-height: normal; margin-bottom: 0in;&quot;&gt;&lt;span style=&quot;color: #017087;&quot;&gt;►&lt;/span&gt;&amp;nbsp;A steroid injection into the tendon sheath is an effective treatment in most cases and usually should be attempted before considering surgery.&lt;/div&gt;&lt;div style=&quot;line-height: normal; margin-bottom: 0in;&quot;&gt;&lt;span style=&quot;color: #017087;&quot;&gt;&lt;br /&gt;&lt;/span&gt;&lt;/div&gt;&lt;div style=&quot;line-height: normal; margin-bottom: 0in;&quot;&gt;&lt;span style=&quot;color: #017087;&quot;&gt;►&lt;/span&gt;&amp;nbsp;Surgical release of the A1 pulley is very effective in most cases that do not respond to conservative treatment.&lt;/div&gt;&lt;div style=&quot;line-height: normal; margin-bottom: 0in;&quot;&gt;&lt;br /&gt;&lt;/div&gt;
  143.  &lt;/td&gt;
  144. &lt;/tr&gt;
  145. &lt;/tbody&gt;&lt;/table&gt;&lt;br /&gt;&lt;/div&gt;
  146.  
  147. &lt;details close=&quot;&quot;&gt;
  148.  &lt;summary&gt;&lt;b&gt;&lt;span style=&quot;color: #00788b;&quot;&gt;REFERENCES&lt;/span&gt;&lt;/b&gt;&lt;/summary&gt;
  149.  &lt;p&gt;Fleisch SB, Spindler KP, Lee DH. Corticosteroid injections in the treatment of trigger finger: a level I
  150. and II systematic review. J Am Acad Orthop Surg. 2007;15:166-171.&amp;nbsp;&lt;/p&gt;&lt;p&gt;Saldana MJ. Trigger digits: diagnosis and treatment. J Am Acad Orthop Surg. 2001;9:246-252.&amp;nbsp;&lt;/p&gt;&lt;p&gt;Wolfe SW, et al. Green’s Operative Hand Surgery . 6th ed. Philadelphia: Elsevier; 2011:2071-2079.&lt;/p&gt;
  151. &lt;/details&gt;&lt;/div&gt;</content><link rel='replies' type='application/atom+xml' href='https://medical-phd.blogspot.com/feeds/3558360047720998195/comments/default' title='Post Comments'/><link rel='replies' type='text/html' href='https://medical-phd.blogspot.com/2022/03/trigger-finger-case-file.html#comment-form' title='0 Comments'/><link rel='edit' type='application/atom+xml' href='https://www.blogger.com/feeds/3302710210518459416/posts/default/3558360047720998195'/><link rel='self' type='application/atom+xml' href='https://www.blogger.com/feeds/3302710210518459416/posts/default/3558360047720998195'/><link rel='alternate' type='text/html' href='https://medical-phd.blogspot.com/2022/03/trigger-finger-case-file.html' title='Trigger Finger Case File'/><author><name>Medical Group</name><uri>http://www.blogger.com/profile/01780656800578397393</uri><email>noreply@blogger.com</email><gd:image rel='http://schemas.google.com/g/2005#thumbnail' width='32' height='32' src='//blogger.googleusercontent.com/img/b/R29vZ2xl/AVvXsEiMuROkBDv6yw4SmxGqRA5BPQhOXSwFld-Gufmy4kUqfpoAGQRl7cv-3naew6wjAnCjRFSe_FKX8z7CdE6cFKxxBpnqBuwUTSr-15xd3mFGcr_wJOX2y73G_zJCGND98Q/s220/Final-logo.jpg'/></author><media:thumbnail xmlns:media="http://search.yahoo.com/mrss/" url="https://blogger.googleusercontent.com/img/b/R29vZ2xl/AVvXsEh8hs1pWaVK9HUwOZONN249NuTQJiCvZJLXroBzCuMN-wVlESWrBVva7bVJW3NDYL0ONFHjKSkaVu3LM0lXgVjlnhKsEtTQw3c_NsUKFarGc8sKu9aEa9V9yxoblE0NrnLlYjGklHjlskUQUh-38eYL0dDuuDUb_oyHh9SJ-RL_qEccHBGV8Vb5CQyMLQ/s72-w346-h640-c/Finger-flexor-tendon-pulley-system.jpg" height="72" width="72"/><thr:total>0</thr:total><georss:featurename>United States</georss:featurename><georss:point>37.09024 -95.712891</georss:point><georss:box>8.780006163821156 -130.869141 65.400473836178847 -60.556641</georss:box></entry><entry><id>tag:blogger.com,1999:blog-3302710210518459416.post-1479417503835675289</id><published>2022-03-26T14:21:00.006-07:00</published><updated>2022-03-26T14:21:43.596-07:00</updated><category scheme="http://www.blogger.com/atom/ns#" term="Case Files"/><category scheme="http://www.blogger.com/atom/ns#" term="Orthopaedic Surgery Case File"/><title type='text'>Dupuytren Disease Case File</title><content type='html'>&lt;div class=&quot;scrollbox&quot;&gt;
  152.  
  153. &lt;div style=&quot;text-align: left;&quot;&gt;&lt;div&gt;&lt;b&gt;&lt;span style=&quot;font-size: large;&quot;&gt;Dupuytren Disease Case File&lt;/span&gt;&lt;/b&gt;&lt;/div&gt;&lt;div&gt;Eugene C. Toy, MD, Andrew J. Rosenbaum, MD, Timothy T. Roberts, MD, Joshua S. Dines, MD&lt;/div&gt;&lt;div&gt;&lt;br /&gt;&lt;/div&gt;&lt;div&gt;&lt;b&gt;&lt;span style=&quot;color: #017087; font-size: medium;&quot;&gt;CASE 31&lt;/span&gt;&lt;/b&gt;&lt;/div&gt;&lt;div&gt;&lt;div&gt;A 64-year-old, right-hand dominant male is referred to your office with complaints of progressive hand stiffness and deformity over the past 2 years. The small finger of the right hand is most affected. The left hand is also affected, but not as severe. He denies any history of trauma or previous hand problems, and he has no complaints of pain, nor finger locking or clicking. His father had similar problems with his hands.&lt;/div&gt;&lt;div&gt;&lt;br /&gt;&lt;/div&gt;&lt;div&gt;&lt;span&gt;&amp;nbsp;&amp;nbsp; &amp;nbsp;&lt;/span&gt;The patient’s past medical history includes high cholesterol and blood pressure. More than 5 years ago, he underwent surgery for early-stage colon cancer, and since then has been cancer-free. He currently does not drink alcohol, although he drank heavily during his younger years. He is of English-Irish decent.&lt;/div&gt;&lt;div&gt;&lt;br /&gt;&lt;/div&gt;&lt;div&gt;&lt;span&gt;&amp;nbsp;&amp;nbsp; &amp;nbsp;&lt;/span&gt;Examination of both hands reveals nodules and skin thickening in the palm along the small finger ray. The right small finger metacarpophalangeal (MCP) joint is fixed at 45 degrees of flexion, and the proximal interphalangeal (PIP) joint is fixed at 20 degrees. There is no redness, tenderness, edema, or pain with passive flexion. The left hand has a nodule and cord with a resting flexion contracture of the small finger of 20 degrees that is improved with passive extension. Additionally, examination of his left foot reveals nontender nodules along the first ray proximal to the metatarsal head.&lt;/div&gt;&lt;div&gt;&lt;br /&gt;&lt;/div&gt;&lt;div&gt;&lt;span style=&quot;color: #017087;&quot;&gt;►&lt;/span&gt;&amp;nbsp;What is the most likely diagnosis?&lt;/div&gt;&lt;div&gt;&lt;span style=&quot;color: #017087;&quot;&gt;►&lt;/span&gt;&amp;nbsp;What is the most appropriate treatment for this patient?&lt;/div&gt;&lt;/div&gt;&lt;div&gt;&lt;br /&gt;&lt;/div&gt;&lt;div&gt;&lt;br /&gt;&lt;/div&gt;&lt;div&gt;&lt;div&gt;&lt;b&gt;&lt;span style=&quot;font-size: medium;&quot;&gt;ANSWER TO CASE 31:&lt;/span&gt;&lt;/b&gt;&lt;/div&gt;&lt;div&gt;&lt;span style=&quot;background-color: #daebf0; font-size: medium;&quot;&gt;&lt;span&gt;&lt;span style=&quot;background-color: transparent;&quot;&gt;&lt;span style=&quot;color: #017087;&quot;&gt;&lt;b&gt;Dupuytren Disease&lt;/b&gt;&lt;/span&gt;&lt;/span&gt;&lt;b&gt;&amp;nbsp;&lt;/b&gt;&lt;/span&gt;&lt;span style=&quot;font-weight: bold;&quot;&gt;&amp;nbsp;&amp;nbsp; &amp;nbsp;&lt;/span&gt;&lt;span style=&quot;font-weight: bold;&quot;&gt;&amp;nbsp;&amp;nbsp; &amp;nbsp;&lt;/span&gt;&lt;span style=&quot;font-weight: bold;&quot;&gt;&amp;nbsp;&amp;nbsp; &amp;nbsp;&lt;/span&gt;&lt;span style=&quot;font-weight: bold;&quot;&gt;&amp;nbsp;&amp;nbsp; &amp;nbsp;&lt;/span&gt;&lt;span style=&quot;font-weight: bold;&quot;&gt;&amp;nbsp;&amp;nbsp; &amp;nbsp;&lt;/span&gt;&lt;span style=&quot;font-weight: bold;&quot;&gt;&amp;nbsp;&amp;nbsp; &amp;nbsp;&lt;/span&gt;&lt;span style=&quot;font-weight: bold;&quot;&gt;&amp;nbsp; &amp;nbsp;&amp;nbsp;&lt;/span&gt;&lt;/span&gt;&lt;/div&gt;&lt;/div&gt;&lt;div&gt;&lt;br /&gt;&lt;/div&gt;&lt;div&gt;&lt;div&gt;&lt;i&gt;Summary&lt;/i&gt;: This right-hand dominant, 64-year-old male patient has a progressive soft tissue condition leading to digital flexion deformity or contracture. The stiffness is associated with decreased functionality. The contracture in the right hand is not passively correctible.&lt;/div&gt;&lt;div&gt;&lt;ul style=&quot;text-align: left;&quot;&gt;&lt;li&gt;&lt;b&gt;Most likely diagnosis:&lt;/b&gt; Dupuytren disease.&lt;/li&gt;&lt;li&gt;&lt;b&gt;Most appropriate treatment:&lt;/b&gt; Collagenase injection(s), needle aponeurotomy, or&amp;nbsp; surgical fasciotomies, depending on the extent of disease and surgeon preference.&lt;/li&gt;&lt;/ul&gt;&lt;/div&gt;&lt;/div&gt;&lt;div&gt;&lt;br /&gt;&lt;/div&gt;&lt;div&gt;&lt;br /&gt;&lt;/div&gt;&lt;div&gt;&lt;div&gt;&lt;b&gt;&lt;span style=&quot;color: #017087; font-size: medium;&quot;&gt;ANALYSIS&lt;/span&gt;&lt;/b&gt;&lt;/div&gt;&lt;div&gt;&lt;b&gt;&lt;span style=&quot;font-size: medium;&quot;&gt;&lt;i&gt;Objectives&lt;/i&gt;&lt;/span&gt;&lt;/b&gt;&lt;/div&gt;&lt;/div&gt;&lt;div&gt;&lt;div&gt;&lt;ol style=&quot;text-align: left;&quot;&gt;&lt;li&gt;Understand the clinical presentation, etiology, risk factors, anatomic features, and pathoanatomy of Dupuytren disease.&lt;/li&gt;&lt;li&gt;Be familiar with basic treatment options and associated complications.&lt;/li&gt;&lt;li&gt;Identify criteria to guide choice of treatment.&lt;/li&gt;&lt;li&gt;Recognize prognostic factors for recurrence and poor outcome.&lt;/li&gt;&lt;/ol&gt;&lt;/div&gt;&lt;/div&gt;&lt;div&gt;&lt;br /&gt;&lt;/div&gt;&lt;div&gt;&lt;br /&gt;&lt;/div&gt;&lt;div&gt;&lt;div&gt;&lt;b&gt;&lt;i&gt;&lt;span style=&quot;font-size: medium;&quot;&gt;Considerations&lt;/span&gt;&lt;/i&gt;&lt;/b&gt;&lt;/div&gt;&lt;div&gt;&lt;br /&gt;&lt;/div&gt;&lt;div&gt;This patient presents with progressive right small finger stiffness and deformity. His MCP joint is fixed in 45 degrees of flexion and his PIP joint is fixed in 20 degrees of flexion. His deformity (see Figure 31–1 ) is most obvious as he tries to extend all his fingers. Furthermore, the patient denies any locking symptoms, and there is no redness, tenderness, edema, or pain with passive flexion. The differential diagnosis for digital flexion contractures includes Dupuytren disease, trigger finger(s), stenosing tenosynovitis, ganglion cysts, soft-tissue masses, hyperkeratosis, intrinsic joint disease, and traumatic scarring. Despite this broad differential, there are several components of this patient’s presentation that favor a diagnosis of Dupuytren disease. These include a family history of similar pathology, a history of heavy alcohol use, being of Northern European decent, and presence of similar, nontender nodules in his left foot. After the &lt;b&gt;clinical diagnosis&lt;/b&gt; of Dupuytren disease is made, treatment options can be considered, including nonoperative and operative modalities.&lt;/div&gt;&lt;/div&gt;&lt;div&gt;&lt;br /&gt;&lt;/div&gt;&lt;div class=&quot;separator&quot; style=&quot;clear: both; text-align: center;&quot;&gt;&lt;a href=&quot;https://blogger.googleusercontent.com/img/b/R29vZ2xl/AVvXsEg-jh2dJ52EDQZuQpgx1f630p6EfNQMVc9Ap0-pH8j3oaNkskKRVHelhmlMzp71urdooEJVbKK9GUX-VsfllB3ayXVoNpJysu9eWsY3WiTobggTRp8Xr783bTFs6CR0WBCX0-eGLDIZ61ZusN8Ty9N0Rx30LWEID1j38pwpy-axuS-cLHFVxVJaC_FzHQ/s486/Dupuytren-disease-of-the-small-finger.jpg&quot; style=&quot;margin-left: 1em; margin-right: 1em;&quot;&gt;&lt;img alt=&quot;Dupuytren disease of the small finger&quot; border=&quot;0&quot; data-original-height=&quot;384&quot; data-original-width=&quot;486&quot; height=&quot;316&quot; src=&quot;https://blogger.googleusercontent.com/img/b/R29vZ2xl/AVvXsEg-jh2dJ52EDQZuQpgx1f630p6EfNQMVc9Ap0-pH8j3oaNkskKRVHelhmlMzp71urdooEJVbKK9GUX-VsfllB3ayXVoNpJysu9eWsY3WiTobggTRp8Xr783bTFs6CR0WBCX0-eGLDIZ61ZusN8Ty9N0Rx30LWEID1j38pwpy-axuS-cLHFVxVJaC_FzHQ/w400-h316/Dupuytren-disease-of-the-small-finger.jpg&quot; title=&quot;Dupuytren disease of the small finger&quot; width=&quot;400&quot; /&gt;&lt;/a&gt;&lt;/div&gt;&lt;div&gt;&lt;div&gt;&lt;b&gt;&lt;span style=&quot;color: #017087;&quot;&gt;Figure 31–1.&lt;/span&gt;&lt;/b&gt; Dupuytren disease of the small finger. Note the MCP and PIP flexion contracture as the&lt;/div&gt;&lt;div&gt;patient attempts to extend his fingers.&lt;/div&gt;&lt;/div&gt;&lt;div&gt;&lt;br /&gt;&lt;/div&gt;&lt;div&gt;&lt;br /&gt;&lt;/div&gt;&lt;div&gt;&lt;div&gt;&lt;div&gt;&lt;b&gt;&lt;span style=&quot;font-size: medium;&quot;&gt;APPROACH TO:&lt;/span&gt;&lt;/b&gt;&lt;/div&gt;&lt;div&gt;&lt;span style=&quot;background-color: #daebf0; color: #017087; font-size: medium;&quot;&gt;&lt;span&gt;&lt;span style=&quot;background-color: transparent;&quot;&gt;&lt;b&gt;Dupuytren Disease&lt;/b&gt;&lt;/span&gt;&lt;b&gt;&amp;nbsp;&lt;/b&gt;&lt;/span&gt;&lt;span style=&quot;font-weight: bold;&quot;&gt;&amp;nbsp;&amp;nbsp; &amp;nbsp;&lt;/span&gt;&lt;span style=&quot;font-weight: bold;&quot;&gt;&amp;nbsp;&amp;nbsp; &amp;nbsp;&lt;/span&gt;&lt;span style=&quot;font-weight: bold;&quot;&gt;&amp;nbsp;&amp;nbsp; &amp;nbsp;&lt;/span&gt;&lt;span style=&quot;font-weight: bold;&quot;&gt;&amp;nbsp;&amp;nbsp; &amp;nbsp;&lt;/span&gt;&lt;span style=&quot;font-weight: bold;&quot;&gt;&amp;nbsp;&amp;nbsp; &amp;nbsp;&lt;/span&gt;&lt;span style=&quot;font-weight: bold;&quot;&gt;&amp;nbsp;&amp;nbsp; &amp;nbsp;&lt;/span&gt;&lt;span style=&quot;font-weight: bold;&quot;&gt;&amp;nbsp;&amp;nbsp; &amp;nbsp;&lt;/span&gt;&lt;span style=&quot;font-weight: bold;&quot;&gt;&amp;nbsp;&amp;nbsp;&amp;nbsp;&lt;/span&gt;&lt;span style=&quot;font-weight: bold;&quot;&gt;&amp;nbsp;&lt;/span&gt;&lt;/span&gt;&lt;/div&gt;&lt;/div&gt;&lt;div&gt;&lt;br /&gt;&lt;/div&gt;&lt;div&gt;&lt;b&gt;&lt;span style=&quot;color: #017087; font-size: medium;&quot;&gt;DEFINITIONS&lt;/span&gt;&lt;/b&gt;&lt;/div&gt;&lt;/div&gt;&lt;div&gt;&lt;br /&gt;&lt;/div&gt;&lt;div&gt;&lt;div&gt;&lt;b&gt;DUPUYTREN DISEASE:&lt;/b&gt; A benign fibroproliferative disorder involving the palmar and digital fascia.&lt;/div&gt;&lt;div&gt;&lt;b&gt;&lt;br /&gt;&lt;/b&gt;&lt;/div&gt;&lt;div&gt;&lt;b&gt;SPIRAL CORD:&lt;/b&gt; An arrangement of abnormal fibrous tissue associated with PIP joint flexion contractures. Spiral cords can displace digital neurovascular bundles centrally and volarly (palmar superficially), making them especially prone to injury during surgical release.&lt;/div&gt;&lt;div&gt;&lt;b&gt;&lt;br /&gt;&lt;/b&gt;&lt;/div&gt;&lt;div&gt;&lt;b&gt;DUPUYTREN DIATHESIS:&lt;/b&gt; Refers to patients with an aggressive form of the disease, manifesting with early onset, rapid progression, frequent bilaterality, and/or involvement of the radial digits.&lt;/div&gt;&lt;/div&gt;&lt;div&gt;&lt;br /&gt;&lt;/div&gt;&lt;div&gt;&lt;br /&gt;&lt;/div&gt;&lt;div&gt;&lt;b&gt;&lt;span style=&quot;color: #017087; font-size: medium;&quot;&gt;CLINICAL APPROACH&lt;/span&gt;&lt;/b&gt;&lt;/div&gt;&lt;div&gt;&lt;br /&gt;&lt;/div&gt;&lt;div&gt;&lt;div&gt;&lt;i&gt;&lt;span style=&quot;font-size: medium;&quot;&gt;&lt;b&gt;Etiology and Anatomic Features&lt;/b&gt;&lt;/span&gt;&lt;/i&gt;&lt;/div&gt;&lt;div&gt;&lt;br /&gt;&lt;/div&gt;&lt;div&gt;Dupuytren disease is a benign fibroproliferative disorder involving the fascia of the hand and fingers,&lt;b&gt; most commonly involving the ring and small fingers.&lt;/b&gt; It can present locally as a nodule in the palm or a dimple in the skin crease. Dupuytrens may then progress distally to involve the joints of the fingers.&lt;/div&gt;&lt;div&gt;&lt;br /&gt;&lt;/div&gt;&lt;div&gt;&lt;span&gt;&amp;nbsp;&amp;nbsp; &amp;nbsp;&lt;/span&gt;The exact etiology remains unclear, although a strong genetic predisposition exists. It occurs more commonly in males of northern European descent. Dupuytren disease has been associated with tobacco and alcohol use, diabetes, epilepsy, chronic pulmonary disease, prior myocardial infarction, human immunodeficiency virus (HIV) infection, trauma, and tuberculosis. However, idiopathic presentation is most common.&lt;/div&gt;&lt;/div&gt;&lt;div&gt;&lt;br /&gt;&lt;/div&gt;&lt;div&gt;&lt;div&gt;&lt;span&gt;&amp;nbsp;&amp;nbsp; &amp;nbsp;&lt;/span&gt;The pathophysiology of Dupuytren disease is not completely understood, but has been found to involve &lt;b&gt;abnormal proliferations of myofibroblasts,&lt;/b&gt; cells normally responsible for producing contractile scar tissue when the body attempts to close a wound. In Dupuytrens, hyperactive myofibroblasts deposit disproportionate quantities of extracellular collagen that form collections of fibrous bundles. Additionally, the deposited fibrous matrix is itself abnormal, consisting of greater concentrations of relatively thick and immature collagen type III, versus the mature collagen type I that is found in normal fascia. Dupuytren disease develops in 3 distinct stages: &lt;b&gt;(1) the proliferative stage,&lt;/b&gt; in which abnormal numbers of oversized myofibroblasts are produced, &lt;b&gt;(2) the involutional stage,&lt;/b&gt; in which abnormal concentrations and quantities of collagen type III are deposited, and&amp;nbsp;&lt;b&gt;(3) the residual stage,&lt;/b&gt; in which the myofibroblasts disappear, leaving behind their dense fibrous deposits. Simply put, Dupuytrens results from a pathological triggering of tissue healing in the absence of inciting trauma. Thick scar-like deposits of abnormal collagen coalesce into the characteristic Dupuytren nodules and cords.&lt;/div&gt;&lt;/div&gt;&lt;div&gt;&lt;br /&gt;&lt;/div&gt;&lt;div&gt;&lt;div&gt;&lt;span&gt;&amp;nbsp;&amp;nbsp; &amp;nbsp;&lt;/span&gt;Structural anomalies result from contraction of the collagenous palmar fascia and overlying skin and fingers. Normal structures involved in the disease process include the pretendinous band, natatory band, spiral band, Grayson ligament, retrovascular band, and lateral digital sheet. These normal structures, or bands, when diseased, become cords and are thereafter referred to as the pretendinous cord, central cord, lateral cord, spiral cord, abductor digiti minimi cord, and intercommissural cord of the first web space. Spiral cords lead to PIP contractures and receive special attention for putting the neurovascular bundle at risk by displacing it more centrally and volarly as the PIP joint contracture increases.&lt;/div&gt;&lt;/div&gt;&lt;div&gt;&lt;br /&gt;&lt;/div&gt;&lt;div&gt;&lt;div&gt;&lt;span&gt;&amp;nbsp;&amp;nbsp; &amp;nbsp;&lt;/span&gt;The natural history and risk for progression for a single individual remain indeterminable. Patients with Dupuytren diathesis have a more aggressive form of the disease that is associated with onset before age 40 years, bilateral hand involvement, thumb and index finger contractures, and fibrotic disease of the feet and penis. Patients need to be informed that recurrence is as high as 40% to 50%. When the fibroproliferative disease results in permanent curvature of the penis, it is known as&amp;nbsp;&lt;b&gt;Peyronie disease;&lt;/b&gt; similarly, if it involves the plantar foot, it is known as &lt;b&gt;Ledderhose&amp;nbsp;&lt;/b&gt;&lt;b&gt;disease.&lt;/b&gt;&lt;/div&gt;&lt;/div&gt;&lt;div&gt;&lt;br /&gt;&lt;/div&gt;&lt;div&gt;&lt;br /&gt;&lt;/div&gt;&lt;div&gt;&lt;div&gt;&lt;i&gt;&lt;span style=&quot;font-size: medium;&quot;&gt;&lt;b&gt;Clinical Presentation and Evaluation&lt;/b&gt;&lt;/span&gt;&lt;/i&gt;&lt;/div&gt;&lt;div&gt;&lt;br /&gt;&lt;/div&gt;&lt;div&gt;Patients may present early or with mild forms that manifest as a tender palmar nodule (arising over a pretendinous band), skin dimpling, or a palpable cord. &lt;b&gt;Classic&amp;nbsp;&lt;/b&gt;&lt;b&gt;features include fixed finger flexion deformities that impair function of daily&amp;nbsp;&lt;/b&gt;&lt;b&gt;living or work and indicate more advanced disease.&lt;/b&gt; There can be fixed flexion of either or both the MCP and PIP joints. (The distal interphalangeal joints are rarely involved and, when they are, are believed to result from formation of the retrovascular cord.) Disease involvement occurs most often in the ring and little fingers, with decreasing order of frequency in the long, thumb, and index digits. It is bilateral in 45% of patients.&lt;/div&gt;&lt;/div&gt;&lt;div&gt;&lt;br /&gt;&lt;/div&gt;&lt;div&gt;&lt;div&gt;&lt;span&gt;&amp;nbsp;&amp;nbsp; &amp;nbsp;&lt;/span&gt;Dupuytren disease is diagnosed clinically. Flexion contractures are rarely subtle. MCP flexion may result from a pretendinous cord or a spiral cord. Limitation in finger abduction is frequently present and results from involvement of the web space and the presence of a natatory cord. Fixed flexion of the PIP joint may arise from contracture of the central cord that continues from a pretendinous cord in the palm that extends into the finger. Involvement of the thumb may manifest as contracture of the web space in addition to a fixed flexion contracture.&lt;/div&gt;&lt;/div&gt;&lt;div&gt;&lt;br /&gt;&lt;/div&gt;&lt;div&gt;&lt;br /&gt;&lt;/div&gt;&lt;div&gt;&lt;b style=&quot;color: #017087;&quot;&gt;&lt;span style=&quot;font-size: medium;&quot;&gt;TREATMENT&lt;/span&gt;&lt;/b&gt;&lt;/div&gt;&lt;div&gt;&lt;br /&gt;&lt;/div&gt;&lt;div&gt;&lt;div&gt;The indications for surgery include functional impairment such as inability to wear a glove, reach into pockets, or securely hold or grasp objects. Physical exam criteria include a positive tabletop test (the inability to place the hand flat on a tabletop), &lt;b&gt;MCP flexion contracture of greater than 30 degrees, and any PIP&amp;nbsp;&lt;/b&gt;&lt;b&gt;flexion contracture.&lt;/b&gt;&lt;/div&gt;&lt;/div&gt;&lt;div&gt;&lt;br /&gt;&lt;/div&gt;&lt;div&gt;&lt;div&gt;&lt;span&gt;&amp;nbsp;&amp;nbsp; &amp;nbsp;&lt;/span&gt;Different surgical techniques have been described. These include percutaneous needle aponeurotomy, limited fasciectomy, and total palmar fasciectomy. Total palmar fasciectomy has recently fallen from favor because recurrence rates after surgery were no better than those of less extensive procedures and it was associated with higher rates of wound complications. The open-palm technique of McCash leaves the wounds open after surgery, resulting in diminished edema and hematoma formation and allowing for early postoperative motion. This historic technique was once preferred for older patients at increased risk for stiffness; but because of wound complications and poor acceptance on the part of the patient, this technique has since been abandoned. Skin coverage deficits are not uncommon after surgical treatment and may be addressed with skin grafting, Z-plasty, or healing by secondary intention.&lt;/div&gt;&lt;/div&gt;&lt;div&gt;&lt;br /&gt;&lt;/div&gt;&lt;div&gt;&lt;div&gt;&lt;span&gt;&amp;nbsp;&amp;nbsp; &amp;nbsp;&lt;/span&gt;Nonoperative treatment of Dupuytren disease is evolving. In early stages, corticosteroid injections may provide some relief by softening and flattening prominent or painful nodules. For more advanced disease, an injectable collagenase (clostridial collagenase histolyticum or Xiaflex, which was approved by the US Food and Drug Administration in 2010) is receiving increasing popularity. This drug has been shown to chemically dissolve the Dupuytren’s cord, allowing for the subsequent manipulation necessary to mechanically disrupt the contracted tissue and correct the flexion deformity.&lt;/div&gt;&lt;/div&gt;&lt;div&gt;&lt;br /&gt;&lt;/div&gt;&lt;div&gt;&lt;br /&gt;&lt;/div&gt;&lt;div&gt;&lt;div&gt;&lt;b&gt;&lt;i&gt;&lt;span style=&quot;font-size: medium;&quot;&gt;Outcomes and Complications&lt;/span&gt;&lt;/i&gt;&lt;/b&gt;&lt;/div&gt;&lt;div&gt;&lt;br /&gt;&lt;/div&gt;&lt;div&gt;The most common complications after collagenase injection include edema, bruising, and redness localized to the site of injection. More serious but less frequent complications include flexor tendon rupture or neurovascular injury.&lt;/div&gt;&lt;div&gt;&lt;br /&gt;&lt;/div&gt;&lt;div&gt;&lt;span&gt;&amp;nbsp;&amp;nbsp; &amp;nbsp;&lt;/span&gt;The most common complication after operative treatment is recurrence, with long-term rates as high as 40% to 50%. Attention to postoperative therapy with active range of motion and splinting is a major determinant of improved outcomes. Potential complications include infection, digital neurovascular injury, complex regional pain syndrome, hematoma, skin loss, and amputation.&lt;/div&gt;&lt;/div&gt;&lt;div&gt;&lt;br /&gt;&lt;/div&gt;&lt;div&gt;&lt;br /&gt;&lt;/div&gt;&lt;div&gt;&lt;b&gt;&lt;span style=&quot;color: #017087; font-size: medium;&quot;&gt;&lt;u&gt;COMPREHENSION QUESTIONS&lt;/u&gt;&lt;/span&gt;&lt;/b&gt;&lt;/div&gt;&lt;div&gt;&lt;br /&gt;&lt;/div&gt;&lt;div&gt;&lt;div&gt;31.1 A right-hand dominant 48-year-old woman presents with worsening hand stiffness and deformity. On exam, you note nodules and skin thickening in the palm along the small finger ray. You ultimately diagnose the patient with Dupuytren disease and begin discussing the specifics of this condition with her. Which of the following statements regarding Dupuytren disease is true?&lt;/div&gt;&lt;/div&gt;&lt;/div&gt;&lt;blockquote style=&quot;border: none; margin: 0px 0px 0px 40px; padding: 0px; text-align: left;&quot;&gt;&lt;div style=&quot;text-align: left;&quot;&gt;&lt;div&gt;&lt;div&gt;A. Dupuytren disease is rarely bilateral.&lt;/div&gt;&lt;/div&gt;&lt;/div&gt;&lt;div style=&quot;text-align: left;&quot;&gt;&lt;div&gt;&lt;div&gt;B. Dupuytren disease is associated with pain.&lt;/div&gt;&lt;/div&gt;&lt;/div&gt;&lt;div style=&quot;text-align: left;&quot;&gt;&lt;div&gt;&lt;div&gt;C. Females are at greater risk than males.&lt;/div&gt;&lt;/div&gt;&lt;/div&gt;&lt;div style=&quot;text-align: left;&quot;&gt;&lt;div&gt;&lt;div&gt;D. Recurrence after surgery is as high as 50%.&lt;/div&gt;&lt;/div&gt;&lt;/div&gt;&lt;div style=&quot;text-align: left;&quot;&gt;&lt;div&gt;&lt;div&gt;E. Rapidly progressive disease suggests malignancy.&lt;/div&gt;&lt;/div&gt;&lt;/div&gt;&lt;/blockquote&gt;&lt;div style=&quot;text-align: left;&quot;&gt;&lt;div&gt;&lt;div&gt;&lt;br /&gt;&lt;/div&gt;&lt;div&gt;31.2 Which of the following patients with Dupuytren contracture would benefit the most from total palmar fasciectomy?&lt;/div&gt;&lt;/div&gt;&lt;/div&gt;&lt;blockquote style=&quot;border: none; margin: 0px 0px 0px 40px; padding: 0px; text-align: left;&quot;&gt;&lt;div style=&quot;text-align: left;&quot;&gt;&lt;div&gt;&lt;div&gt;A. 50-year-old police officer with ring and small finger involvement of only the MCP joints&lt;/div&gt;&lt;/div&gt;&lt;/div&gt;&lt;div style=&quot;text-align: left;&quot;&gt;&lt;div&gt;&lt;div&gt;B. 70-year-old sedentary male with small finger involvement including the MCP and PIP joints&lt;/div&gt;&lt;/div&gt;&lt;/div&gt;&lt;div style=&quot;text-align: left;&quot;&gt;&lt;div&gt;&lt;div&gt;C. 45-year-old female tennis player with ring and small finger involvement including MCP and PIP joints&lt;/div&gt;&lt;/div&gt;&lt;/div&gt;&lt;div style=&quot;text-align: left;&quot;&gt;&lt;div&gt;&lt;div&gt;D. None of the above, as total palmar fasciectomy has fallen out of popularity due to its associated recurrence rates and complications, as opposed to regional, or limited fasciectomy&lt;/div&gt;&lt;/div&gt;&lt;/div&gt;&lt;/blockquote&gt;&lt;div style=&quot;text-align: left;&quot;&gt;&lt;div&gt;&lt;div&gt;&lt;br /&gt;&lt;/div&gt;&lt;div&gt;31.3 A 40-year-old man presents to your hand clinic with a diagnosis of Dupuytren disease involving his left small finger. The patient was given this diagnosis last week while at a routine follow-up appointment with your partner for his right total hip arthroplasty. Your partner felt that you, a hand surgeon, would be best equipped to manage the Dupuytren. After receiving this diagnosis, the patient did an internet search on this condition and comes in today inquiring about treatment with Xiaflex. Which of the following statements regarding Xiaflex is most accurate?&lt;/div&gt;&lt;/div&gt;&lt;/div&gt;&lt;blockquote style=&quot;border: none; margin: 0px 0px 0px 40px; padding: 0px; text-align: left;&quot;&gt;&lt;div style=&quot;text-align: left;&quot;&gt;&lt;div&gt;&lt;div&gt;A. This drug has been shown to chemically dissolve the Dupuytren cord.&lt;/div&gt;&lt;/div&gt;&lt;/div&gt;&lt;div style=&quot;text-align: left;&quot;&gt;&lt;div&gt;&lt;div&gt;B. Xiaflex is a time-tested and effective means of treating Dupuytren disease.&lt;/div&gt;&lt;/div&gt;&lt;/div&gt;&lt;div style=&quot;text-align: left;&quot;&gt;&lt;div&gt;&lt;div&gt;C. Xiaflex is great because it has no known side effects.&lt;/div&gt;&lt;/div&gt;&lt;/div&gt;&lt;div style=&quot;text-align: left;&quot;&gt;&lt;div&gt;&lt;div&gt;D. Xiaflex can be purchased as an over-the-counter medication.&lt;/div&gt;&lt;/div&gt;&lt;/div&gt;&lt;/blockquote&gt;&lt;div style=&quot;text-align: left;&quot;&gt;&lt;div&gt;&lt;br /&gt;&lt;/div&gt;&lt;div&gt;&lt;br /&gt;&lt;/div&gt;&lt;div&gt;&lt;b&gt;&lt;span style=&quot;color: #017087; font-size: medium;&quot;&gt;&lt;u&gt;ANSWERS&lt;/u&gt;&lt;/span&gt;&lt;/b&gt;&lt;/div&gt;&lt;div&gt;&lt;br /&gt;&lt;/div&gt;&lt;div&gt;&lt;div&gt;31.1 &lt;b&gt;D.&lt;/b&gt; Patients with Dupuytren disease should be counseled about the high risk for recurrence after surgery. Disease is frequently bilateral, most common in middle-aged men of northern European descent, and not histologically malignant, although patients with Dupuytren diathesis or with a genetic predisposition may have a more clinically aggressive form with rapid progression and high recurrence.&lt;/div&gt;&lt;div&gt;&lt;br /&gt;&lt;/div&gt;&lt;div&gt;31.2 &lt;b&gt;D.&lt;/b&gt; As stated in D, total palmar fasciectomy has fallen out of popularity as a result of its associated recurrence rates and complications, as opposed to regional, or limited fasciectomy.&lt;/div&gt;&lt;div&gt;&lt;br /&gt;&lt;/div&gt;&lt;div&gt;31.3 &lt;b&gt;A.&lt;/b&gt; Xiaflex has been shown to chemically dissolve the Dupuytren cord, allowing for the subsequent manipulation necessary to mechanically disrupt the contracted tissue and correct the flexion deformity. Although data are promising, Xiaflex was only approved by the US Food and Drug Administration in 2010. Serious side effects include tendon rupture, ligament damage, nerve injury, and allergic reactions. Common side effects include swelling, bleeding, bruising, pain, and/or tenderness at the injection site or hand.&lt;/div&gt;&lt;/div&gt;&lt;div&gt;&lt;br /&gt;&lt;/div&gt;&lt;div&gt;&lt;b style=&quot;background-color: #4093a5;&quot;&gt;&amp;nbsp; &amp;nbsp;&lt;span style=&quot;font-size: medium;&quot;&gt;&amp;nbsp;&lt;/span&gt;&lt;span style=&quot;color: white;&quot;&gt;&lt;span style=&quot;font-size: medium;&quot;&gt;CLINICAL PEARLS&lt;/span&gt;&lt;span&gt;&amp;nbsp; &amp;nbsp;&amp;nbsp;&lt;/span&gt;&lt;/span&gt;&lt;/b&gt;&lt;/div&gt;&lt;div&gt;&lt;table border=&quot;1&quot; cellpadding=&quot;0&quot; cellspacing=&quot;0&quot; class=&quot;MsoTableGrid&quot; style=&quot;border-collapse: collapse; border: none; mso-border-alt: solid windowtext .5pt; mso-padding-alt: 0in 5.4pt 0in 5.4pt; mso-yfti-tbllook: 1184;&quot;&gt;
  154. &lt;tbody&gt;&lt;tr style=&quot;height: 19.15pt; mso-yfti-firstrow: yes; mso-yfti-irow: 0; mso-yfti-lastrow: yes;&quot;&gt;
  155.  &lt;td style=&quot;background: rgb(221, 237, 239); border-left: solid #017087 1.0pt; border: 1pt solid rgb(64, 147, 165); height: 19.15pt; padding: 0in 5.4pt; width: 433.7pt;&quot; valign=&quot;top&quot; width=&quot;578&quot;&gt;
  156.  &lt;div style=&quot;line-height: normal; margin-bottom: 0in;&quot;&gt;&lt;span style=&quot;color: #017087;&quot;&gt;&lt;br /&gt;&lt;/span&gt;&lt;/div&gt;&lt;div style=&quot;line-height: normal; margin-bottom: 0in;&quot;&gt;&lt;span style=&quot;color: #017087;&quot;&gt;►&lt;/span&gt;&amp;nbsp;Dupuytren disease is a clinical diagnosis, and imaging or laboratory tests are needed only to assess or rule out other incidental conditions.&lt;/div&gt;&lt;div style=&quot;line-height: normal; margin-bottom: 0in;&quot;&gt;&lt;span style=&quot;color: #017087;&quot;&gt;&lt;br /&gt;&lt;/span&gt;&lt;/div&gt;&lt;div style=&quot;line-height: normal; margin-bottom: 0in;&quot;&gt;&lt;span style=&quot;color: #017087;&quot;&gt;►&lt;/span&gt;&amp;nbsp;Treatment is indicated when the fixed contractures interfere with hand function; standard criteria to provide treatment include the contractures of the MCP of 30 degrees and any degree of fixed PIP flexion, but decision to treat should be tailored to the patient.&lt;/div&gt;&lt;div style=&quot;line-height: normal; margin-bottom: 0in;&quot;&gt;&lt;span style=&quot;color: #017087;&quot;&gt;&lt;br /&gt;&lt;/span&gt;&lt;/div&gt;&lt;div style=&quot;line-height: normal; margin-bottom: 0in;&quot;&gt;&lt;span style=&quot;color: #017087;&quot;&gt;►&lt;/span&gt;&amp;nbsp;Patients with aggressive disease should be warned that there is high recurrence after treatment.&lt;/div&gt;&lt;div style=&quot;line-height: normal; margin-bottom: 0in;&quot;&gt;&lt;span style=&quot;color: #017087;&quot;&gt;&lt;br /&gt;&lt;/span&gt;&lt;/div&gt;&lt;div style=&quot;line-height: normal; margin-bottom: 0in;&quot;&gt;&lt;span style=&quot;color: #017087;&quot;&gt;►&lt;/span&gt;&amp;nbsp;Patients should be informed of irreversible complications from treatment including flexor tendon rupture, wound complications (eg, hematoma, infection), and injury to neurovascular structures.&lt;/div&gt;&lt;div style=&quot;line-height: normal; margin-bottom: 0in;&quot;&gt;&lt;br /&gt;&lt;/div&gt;
  157.  &lt;/td&gt;
  158. &lt;/tr&gt;
  159. &lt;/tbody&gt;&lt;/table&gt;&lt;br /&gt;&lt;/div&gt;&lt;/div&gt;
  160.  
  161. &lt;details close=&quot;&quot;&gt;
  162.  &lt;summary&gt;&lt;b&gt;&lt;span style=&quot;color: #00788b;&quot;&gt;REFERENCES&lt;/span&gt;&lt;/b&gt;&lt;/summary&gt;
  163.  &lt;p&gt;Desai SS, Hentz VR. The treatment of Dupuytren’s disease. J Hand Surg Am. 2011;36:936-942.&amp;nbsp;&lt;/p&gt;&lt;p&gt;Hurst LC, Badalamente MA, Hentz VR, et al; CORD I Study Group. Injectable collagenase clostridium
  164. histolyticum for Dupuytren’s contracture. N Engl J Med. 2009;361:968-979.&amp;nbsp;&lt;/p&gt;&lt;p&gt;Miller MD. Review of Orthopaedics. 5th ed. Philadelphia: Saunders Elsevier; 2008.&amp;nbsp;&lt;/p&gt;&lt;p&gt;Terek RM, Jiranek WA, Goldberg MJ, Wolfe HJ, Alman BA. The expression of platelet-derived growthfactor
  165. gene in Dupuytren’s contracture. J Bone Joint Surg Am. 1995;77:1-9.&lt;/p&gt;
  166. &lt;/details&gt;&lt;/div&gt;</content><link rel='replies' type='application/atom+xml' href='https://medical-phd.blogspot.com/feeds/1479417503835675289/comments/default' title='Post Comments'/><link rel='replies' type='text/html' href='https://medical-phd.blogspot.com/2022/03/dupuytren-disease-case-file.html#comment-form' title='0 Comments'/><link rel='edit' type='application/atom+xml' href='https://www.blogger.com/feeds/3302710210518459416/posts/default/1479417503835675289'/><link rel='self' type='application/atom+xml' href='https://www.blogger.com/feeds/3302710210518459416/posts/default/1479417503835675289'/><link rel='alternate' type='text/html' href='https://medical-phd.blogspot.com/2022/03/dupuytren-disease-case-file.html' title='Dupuytren Disease Case File'/><author><name>Medical Group</name><uri>http://www.blogger.com/profile/01780656800578397393</uri><email>noreply@blogger.com</email><gd:image rel='http://schemas.google.com/g/2005#thumbnail' width='32' height='32' src='//blogger.googleusercontent.com/img/b/R29vZ2xl/AVvXsEiMuROkBDv6yw4SmxGqRA5BPQhOXSwFld-Gufmy4kUqfpoAGQRl7cv-3naew6wjAnCjRFSe_FKX8z7CdE6cFKxxBpnqBuwUTSr-15xd3mFGcr_wJOX2y73G_zJCGND98Q/s220/Final-logo.jpg'/></author><media:thumbnail xmlns:media="http://search.yahoo.com/mrss/" url="https://blogger.googleusercontent.com/img/b/R29vZ2xl/AVvXsEg-jh2dJ52EDQZuQpgx1f630p6EfNQMVc9Ap0-pH8j3oaNkskKRVHelhmlMzp71urdooEJVbKK9GUX-VsfllB3ayXVoNpJysu9eWsY3WiTobggTRp8Xr783bTFs6CR0WBCX0-eGLDIZ61ZusN8Ty9N0Rx30LWEID1j38pwpy-axuS-cLHFVxVJaC_FzHQ/s72-w400-h316-c/Dupuytren-disease-of-the-small-finger.jpg" height="72" width="72"/><thr:total>0</thr:total><georss:featurename>United States</georss:featurename><georss:point>37.09024 -95.712891</georss:point><georss:box>8.780006163821156 -130.869141 65.400473836178847 -60.556641</georss:box></entry><entry><id>tag:blogger.com,1999:blog-3302710210518459416.post-7623815830716173278</id><published>2022-03-26T13:34:00.000-07:00</published><updated>2022-03-26T13:34:04.595-07:00</updated><category scheme="http://www.blogger.com/atom/ns#" term="Case Files"/><category scheme="http://www.blogger.com/atom/ns#" term="Orthopaedic Surgery Case File"/><title type='text'>Cubital Tunnel Syndrome Case File</title><content type='html'>&lt;div class=&quot;scrollbox&quot;&gt;
  167.  
  168. &lt;div style=&quot;text-align: left;&quot;&gt;&lt;b&gt;&lt;span style=&quot;font-size: large;&quot;&gt;Cubital Tunnel Syndrome Case File&lt;/span&gt;&lt;/b&gt;&lt;/div&gt;&lt;div&gt;Eugene C. Toy, MD, Andrew J. Rosenbaum, MD, Timothy T. Roberts, MD, Joshua S. Dines, MD&lt;/div&gt;&lt;div&gt;&lt;br /&gt;&lt;/div&gt;&lt;div&gt;&lt;b&gt;&lt;span style=&quot;color: #017087; font-size: medium;&quot;&gt;CASE 28&lt;/span&gt;&lt;/b&gt;&lt;/div&gt;&lt;div&gt;&lt;div&gt;A 45-year-old right-hand dominant woman presents to the orthopaedic hand clinic with a 6-month history of right hand numbness and tingling. She denies any trauma or inciting incident but reports that she noticed the numbness and tingling after waking up one morning. The symptoms come and go throughout the day and are worse in the night time, waking her up 3 to 4 times a week. Over the past month, her symptoms have gotten progressively worse, with some days of constant numbness. She denies any symptoms in her neck, shoulder, or arm. She denies any pain. She reports feeling weaker in her right hand, especially with opening a bottle or buttoning her shirt. Over the last few months, her right hand has fatigued faster than her left.&lt;/div&gt;&lt;div&gt;&lt;br /&gt;&lt;/div&gt;&lt;div&gt;&lt;span&gt;&amp;nbsp;&amp;nbsp; &amp;nbsp;&lt;/span&gt;On physical exam, she has full active range of motion of her neck, right shoulder, elbow, wrist, and fingers. Focused exam of her right upper extremity reveals no gross deformity or muscle atrophy. She has ulnar deviation of her small ringer. There is no erythema, ecchymosis, or effusions. She localizes her numbness to her ring and small fingers and cannot discern whether the symptoms are dorsal or volar. On strength testing, she has 5/5 strength of her biceps, triceps, wrist extensors/flexors, thumb extensors/flexors, and flexors of her index and middle finger. She has 3/5 strength of her hand intrinsics and the flexors of her ring and small finger. She has normal sensation to light touch but decreased sensation to pinprick over the volar and dorsal ulnar aspect of her hand. She has a 2+ radial pulse and normal Allen test. Elbow, wrist, and hand radiographs are negative for any pathology.&lt;/div&gt;&lt;div&gt;&lt;br /&gt;&lt;/div&gt;&lt;div&gt;&lt;span style=&quot;color: #017087;&quot;&gt;►&lt;/span&gt;&amp;nbsp;What is the most likely diagnosis?&lt;/div&gt;&lt;div&gt;&lt;span style=&quot;color: #017087;&quot;&gt;►&lt;/span&gt;&amp;nbsp;What muscles does this disease process affect?&lt;/div&gt;&lt;div&gt;&lt;span style=&quot;color: #017087;&quot;&gt;►&lt;/span&gt;&amp;nbsp;What is your next diagnostic step?&lt;/div&gt;&lt;div&gt;&lt;span style=&quot;color: #017087;&quot;&gt;►&lt;/span&gt;&amp;nbsp;What are the treatment options?&lt;/div&gt;&lt;/div&gt;&lt;div&gt;&lt;br /&gt;&lt;/div&gt;&lt;div&gt;&lt;br /&gt;&lt;/div&gt;&lt;div&gt;&lt;div&gt;&lt;b&gt;&lt;span style=&quot;font-size: medium;&quot;&gt;ANSWER TO CASE 30:&lt;/span&gt;&lt;/b&gt;&lt;/div&gt;&lt;div&gt;&lt;span style=&quot;background-color: #daebf0; font-size: medium;&quot;&gt;&lt;span&gt;&lt;span style=&quot;background-color: transparent;&quot;&gt;&lt;span style=&quot;color: #017087;&quot;&gt;&lt;b&gt;Cubital Tunnel Syndrome&lt;/b&gt;&lt;/span&gt;&lt;/span&gt;&lt;b&gt;&amp;nbsp;&amp;nbsp;&lt;/b&gt;&lt;/span&gt;&lt;span style=&quot;font-weight: bold;&quot;&gt;&amp;nbsp;&amp;nbsp; &amp;nbsp;&lt;/span&gt;&lt;span style=&quot;font-weight: bold;&quot;&gt;&amp;nbsp;&amp;nbsp; &amp;nbsp;&lt;/span&gt;&lt;span style=&quot;font-weight: bold;&quot;&gt;&amp;nbsp;&amp;nbsp; &amp;nbsp;&lt;/span&gt;&lt;span style=&quot;font-weight: bold;&quot;&gt;&amp;nbsp;&amp;nbsp; &amp;nbsp;&lt;/span&gt;&lt;span style=&quot;font-weight: bold;&quot;&gt;&amp;nbsp;&amp;nbsp; &amp;nbsp;&lt;/span&gt;&lt;span style=&quot;font-weight: bold;&quot;&gt;&amp;nbsp;&amp;nbsp; &amp;nbsp;&lt;/span&gt;&lt;span style=&quot;font-weight: bold;&quot;&gt;&amp;nbsp; &amp;nbsp;&amp;nbsp;&lt;/span&gt;&lt;/span&gt;&lt;/div&gt;&lt;/div&gt;&lt;div&gt;&lt;br /&gt;&lt;/div&gt;&lt;div&gt;&lt;div&gt;&lt;i&gt;Summary&lt;/i&gt;: A 45-year-old right-hand dominant woman presents with 6 months of right hand numbness, tingling, and now weakness. She denies any history of trauma or specific inciting event and states that her symptoms have worsened over the last month. On exam there is no gross deformity or muscle atrophy of her right upper extremity, but weakness of her intrinsics and ring and small finger flexors is observed. She also has decreased sensation to pinprick over the volar and dorsal ulnar aspect of her hand.&lt;/div&gt;&lt;div&gt;&lt;ul style=&quot;text-align: left;&quot;&gt;&lt;li&gt;&lt;b&gt;Most likely diagnosis:&lt;/b&gt; Cubital tunnel syndrome.&lt;/li&gt;&lt;li&gt;&lt;b&gt;Affected muscles:&lt;/b&gt; Flexor carpi ulnaris (FCU), hand intrinsics (lumbricals for ring and small finger, dorsal interossei, palmar interossei), hypothenar muscles (palmaris brevis, abductor digiti minimi, opponens digiti minimi, flexor digiti minimi), flexor digitorum profundus (FDP) for ring and small finger, adductor pollicis, deep head of flexor pollicis brevis.&lt;/li&gt;&lt;li&gt;&lt;b&gt;Next diagnostic step:&lt;/b&gt; Electrodiagnostic testing.&lt;/li&gt;&lt;li&gt;&lt;b&gt;Treatment options:&lt;/b&gt; Nonsurgical options include bracing, activity modifications, and physical therapy, whereas surgery involves ulnar nerve decompression at the cubital tunnel.&lt;/li&gt;&lt;/ul&gt;&lt;/div&gt;&lt;/div&gt;&lt;div&gt;&lt;br /&gt;&lt;/div&gt;&lt;div&gt;&lt;br /&gt;&lt;/div&gt;&lt;div&gt;&lt;div&gt;&lt;b&gt;&lt;span style=&quot;color: #017087; font-size: medium;&quot;&gt;ANALYSIS&lt;/span&gt;&lt;/b&gt;&lt;/div&gt;&lt;div&gt;&lt;b&gt;&lt;span style=&quot;font-size: medium;&quot;&gt;&lt;i&gt;Objectives&lt;/i&gt;&lt;/span&gt;&lt;/b&gt;&lt;/div&gt;&lt;/div&gt;&lt;div&gt;&lt;div&gt;&lt;ol style=&quot;text-align: left;&quot;&gt;&lt;li&gt;Understand the anatomy of the ulnar nerve and the potential sites of compression.&lt;/li&gt;&lt;li&gt;Understand the physical exam findings and electromyelogram (EMG) analysis associated with ulnar nerve entrapment.&lt;/li&gt;&lt;li&gt;Know the treatment options for cubital tunnel syndrome.&lt;/li&gt;&lt;/ol&gt;&lt;/div&gt;&lt;/div&gt;&lt;div&gt;&lt;br /&gt;&lt;/div&gt;&lt;div&gt;&lt;br /&gt;&lt;/div&gt;&lt;div&gt;&lt;b&gt;&lt;i&gt;&lt;span style=&quot;font-size: medium;&quot;&gt;Considerations&lt;/span&gt;&lt;/i&gt;&lt;/b&gt;&lt;/div&gt;&lt;div&gt;&lt;br /&gt;&lt;/div&gt;&lt;div&gt;&lt;div&gt;This 45-year-old woman presents complaining of symptoms concerning for an entrapment neuropathy. The first priority is to perform a thorough history, which will help in narrowing down the possible diagnoses. This patient denies any trauma or inciting event, and her symptoms (right hand numbness, tingling, and weakness), which have become more constant over the last month, typically wax and wane throughout the day and are most severe at night. Given this history, the practitioner must consider the entrapment neuropathies of the upper extremity in the differential diagnosis. This includes carpal tunnel and cubital tunnel syndromes. A focused physical exam of the right upper extremity will further aid the clinician in determining the diagnosis. This includes ruling out cervical spine pathology that can cause similar symptoms, as well as a brachial plexopathy. In this specific case, for which ulnar nerve entrapment is likely, plain radiographs should be obtained and electrodiagnostic testing performed. Treatment options can then be discussed pending confirmation of the diagnosis of cubital tunnel syndrome.&lt;/div&gt;&lt;/div&gt;&lt;div&gt;&lt;br /&gt;&lt;/div&gt;&lt;div&gt;&lt;br /&gt;&lt;/div&gt;&lt;div&gt;&lt;div&gt;&lt;div&gt;&lt;b&gt;&lt;span style=&quot;font-size: medium;&quot;&gt;APPROACH TO:&lt;/span&gt;&lt;/b&gt;&lt;/div&gt;&lt;div&gt;&lt;span style=&quot;background-color: #daebf0; color: #017087; font-size: medium;&quot;&gt;&lt;span&gt;&lt;span style=&quot;background-color: transparent;&quot;&gt;&lt;b&gt;Cubital Tunnel Syndrome&lt;/b&gt;&lt;/span&gt;&lt;b&gt;&amp;nbsp;&lt;/b&gt;&lt;/span&gt;&lt;span style=&quot;font-weight: bold;&quot;&gt;&amp;nbsp;&amp;nbsp; &amp;nbsp;&lt;/span&gt;&lt;span style=&quot;font-weight: bold;&quot;&gt;&amp;nbsp;&amp;nbsp; &amp;nbsp;&lt;/span&gt;&lt;span style=&quot;font-weight: bold;&quot;&gt;&amp;nbsp;&amp;nbsp; &amp;nbsp;&lt;/span&gt;&lt;span style=&quot;font-weight: bold;&quot;&gt;&amp;nbsp;&amp;nbsp; &amp;nbsp;&lt;/span&gt;&lt;span style=&quot;font-weight: bold;&quot;&gt;&amp;nbsp;&amp;nbsp; &amp;nbsp;&lt;/span&gt;&lt;span style=&quot;font-weight: bold;&quot;&gt;&amp;nbsp;&amp;nbsp; &amp;nbsp;&lt;/span&gt;&lt;span style=&quot;font-weight: bold;&quot;&gt;&amp;nbsp;&amp;nbsp; &amp;nbsp;&lt;/span&gt;&lt;span style=&quot;font-weight: bold;&quot;&gt;&amp;nbsp;&amp;nbsp;&amp;nbsp;&lt;/span&gt;&lt;span style=&quot;font-weight: bold;&quot;&gt;&amp;nbsp;&lt;/span&gt;&lt;/span&gt;&lt;/div&gt;&lt;/div&gt;&lt;div&gt;&lt;br /&gt;&lt;/div&gt;&lt;div&gt;&lt;b&gt;&lt;span style=&quot;color: #017087; font-size: medium;&quot;&gt;DEFINITIONS&lt;/span&gt;&lt;/b&gt;&lt;/div&gt;&lt;/div&gt;&lt;div&gt;&lt;br /&gt;&lt;/div&gt;&lt;div&gt;&lt;div&gt;&lt;b&gt;TINEL TEST:&lt;/b&gt; A way to detect irritated nerves. It is performed by lightly tapping or percussing over a given nerve to elicit a sensation of “tingling” or “pins and needles” in the distribution of the nerve.&lt;/div&gt;&lt;div&gt;&lt;b&gt;&lt;br /&gt;&lt;/b&gt;&lt;/div&gt;&lt;div&gt;&lt;b&gt;FROMENT SIGN:&lt;/b&gt; A test for ulnar nerve palsy that specifically assesses the action of the adductor pollicis (ulnar nerve innervated). The patient is asked to hold a piece of paper between the thumb and index finger. Normally, as the examiner attempts to pull the paper away, an individual will be able to maintain a hold on the paper with little or no difficulty. However, the patient with ulnar nerve palsy will flex the thumb via the flexor pollicis longus (innervated by the anterior interosseous branch of the median nerve) to try to maintain a hold on the paper.&lt;/div&gt;&lt;div&gt;&lt;b&gt;&lt;br /&gt;&lt;/b&gt;&lt;/div&gt;&lt;div&gt;&lt;b&gt;WARTENBERG SIGN:&lt;/b&gt; A sign noting the position of abduction and extension assumed by the small finger in the setting of cubital tunnel syndrome.&lt;/div&gt;&lt;/div&gt;&lt;div&gt;&lt;br /&gt;&lt;/div&gt;&lt;div&gt;&lt;br /&gt;&lt;/div&gt;&lt;div&gt;&lt;div&gt;&lt;b&gt;&lt;span style=&quot;color: #017087; font-size: medium;&quot;&gt;CLINICAL APPROACH&lt;/span&gt;&lt;/b&gt;&lt;/div&gt;&lt;div&gt;&lt;br /&gt;&lt;/div&gt;&lt;/div&gt;&lt;div&gt;&lt;div&gt;&lt;i&gt;&lt;span style=&quot;font-size: medium;&quot;&gt;&lt;b&gt;Anatomy&lt;/b&gt;&lt;/span&gt;&lt;/i&gt;&lt;/div&gt;&lt;div&gt;&lt;br /&gt;&lt;/div&gt;&lt;div&gt;Cubital tunnel syndrome, the second most common upper-extremity compressive neuropathy after carpal tunnel syndrome, is a term used to describe symptoms related to ulnar nerve compression and/or traction around the elbow. Cubital tunnel syndrome includes any ulnar neuropathy in the mid-arm to mid-forearm.&lt;/div&gt;&lt;div&gt;&lt;br /&gt;&lt;/div&gt;&lt;div&gt;&lt;span&gt;&amp;nbsp;&amp;nbsp; &amp;nbsp;&lt;/span&gt;The ulnar nerve is the terminal branch of the medial cord of the brachial plexus (C8-T1). The nerve courses between the medial head of the triceps and the brachialis muscle before coursing posterior to the medial epicondyle and entering the cubital tunnel. The tunnel’s anatomic borders are as follows: &lt;b&gt;Anterior, medial epicondyle;&amp;nbsp;&lt;/b&gt;&lt;b&gt;posterior, olecranon; floor, medial collateral ligament; roof, arcuate&amp;nbsp;&lt;/b&gt;&lt;b&gt;ligament.&lt;/b&gt; After leaving the cubital tunnel, the ulnar nerve passes into the forearm between the 2 heads of the FCU and exits under the deep flexor pronator aponeurosis. At this point it lies deep to the FDS and FCU and superficial to the FDP. There are several major sites at which the ulnar nerve often becomes compressed (Figure 30–1):&lt;/div&gt;&lt;div style=&quot;text-align: left;&quot;&gt;&lt;br /&gt;&lt;/div&gt;&lt;div style=&quot;text-align: left;&quot;&gt;1. Arcade of Struthers: Band of fascia connecting the medial head of the triceps with the medial intermuscular septum, approximately 7 to 10 cm proximal to the medial epicondyle&lt;br /&gt;&lt;br /&gt;&lt;/div&gt;&lt;div style=&quot;text-align: left;&quot;&gt;2. Medial intermuscular septum&lt;/div&gt;&lt;div style=&quot;text-align: left;&quot;&gt;&lt;br /&gt;&lt;/div&gt;&lt;div style=&quot;text-align: left;&quot;&gt;3. Medial head of triceps: Can become hypertrophied in bodybuilders&lt;/div&gt;&lt;/div&gt;&lt;div class=&quot;separator&quot; style=&quot;clear: both; text-align: center;&quot;&gt;&lt;br /&gt;&lt;/div&gt;&lt;div class=&quot;separator&quot; style=&quot;clear: both; text-align: center;&quot;&gt;&lt;a href=&quot;https://blogger.googleusercontent.com/img/b/R29vZ2xl/AVvXsEgTJpvkDA_HpZZNWRLgaMGvQQVYbTBWL7kItxVYOelXMrArC_o4nHtyabNPWDkgf3Wx92CCOkklYKppdXZVYjfZAMwbWbUIUsHNqC8GWB9VKqrNNxrQvCc1soemK8L0Uiec-1HQaIpSHiiYKhK9TrXOMTN_GNWW2E6ekSJRhZmcfdxW2_Eum6FMNBgRtg/s664/Elbow-anatomy.jpg&quot; style=&quot;margin-left: 1em; margin-right: 1em;&quot;&gt;&lt;img alt=&quot;Elbow anatomy&quot; border=&quot;0&quot; data-original-height=&quot;538&quot; data-original-width=&quot;664&quot; height=&quot;518&quot; src=&quot;https://blogger.googleusercontent.com/img/b/R29vZ2xl/AVvXsEgTJpvkDA_HpZZNWRLgaMGvQQVYbTBWL7kItxVYOelXMrArC_o4nHtyabNPWDkgf3Wx92CCOkklYKppdXZVYjfZAMwbWbUIUsHNqC8GWB9VKqrNNxrQvCc1soemK8L0Uiec-1HQaIpSHiiYKhK9TrXOMTN_GNWW2E6ekSJRhZmcfdxW2_Eum6FMNBgRtg/w640-h518/Elbow-anatomy.jpg&quot; title=&quot;Elbow anatomy&quot; width=&quot;640&quot; /&gt;&lt;/a&gt;&lt;/div&gt;&lt;div&gt;&lt;div&gt;&lt;b&gt;&lt;span style=&quot;color: #017087;&quot;&gt;Figure 30–1.&lt;/span&gt;&lt;/b&gt; Elbow anatomy. (&lt;b&gt;A&lt;/b&gt;) Anterior view, (&lt;b&gt;B&lt;/b&gt;) lateral view, and (&lt;b&gt;C&lt;/b&gt;) medial view. (Reproduced,&lt;/div&gt;&lt;div&gt;with permission, from Tintinalli J, et al. &lt;i&gt;Tintinalli’s Emergency Medicine: A Comprehensive Study Guide.&lt;/i&gt; 7th ed. New York, NY: McGraw-Hill; 2010:Fig. 267-1.)&lt;/div&gt;&lt;/div&gt;&lt;div&gt;&lt;br /&gt;&lt;/div&gt;&lt;div&gt;&lt;br /&gt;&lt;/div&gt;&lt;div&gt;&lt;div&gt;4. Medial epicondyle: Can cause compressive or traction forces via deformity from a prior supracondylar fracture (Tardy ulnar nerve palsy owing to development of progressive valgus deformity associated with these fractures) or osteophytes in an arthritic elbow&lt;/div&gt;&lt;div&gt;&lt;br /&gt;&lt;/div&gt;&lt;div&gt;5. Arcuate ligament: The roof of the cubital tunnel&lt;/div&gt;&lt;div&gt;&lt;br /&gt;&lt;/div&gt;&lt;div&gt;6. Osborne fascia: This is the proximal fibrous edge of the FCU and most common site of ulnar nerve compression&lt;/div&gt;&lt;div&gt;&lt;br /&gt;&lt;/div&gt;&lt;div&gt;7. Epicondylar groove: A shallow groove increasing risk of ulnar nerve subluxation; a site where external compression can occur from leaning on a flexed elbow for prolonged period of time&lt;/div&gt;&lt;div&gt;&lt;br /&gt;&lt;/div&gt;&lt;div&gt;8. Anconeus epitrochlearis: Accessory muscle arising from medial olecranon and triceps and inserting on the medial epicondyle; seen in 10% of patients undergoing cubital tunnel release&lt;/div&gt;&lt;div&gt;&lt;br /&gt;&lt;/div&gt;&lt;div&gt;9. Deep flexor pronator aponeurosis&lt;/div&gt;&lt;/div&gt;&lt;div&gt;&lt;br /&gt;&lt;/div&gt;&lt;div&gt;&lt;br /&gt;&lt;/div&gt;&lt;div&gt;&lt;div&gt;&lt;i&gt;&lt;span style=&quot;font-size: medium;&quot;&gt;&lt;b&gt;Physical Exam&lt;/b&gt;&lt;/span&gt;&lt;/i&gt;&lt;/div&gt;&lt;div&gt;&lt;br /&gt;&lt;/div&gt;&lt;div&gt;Patients often present with poorly localized numbness and tingling in their hand. At times they are able to localize the symptoms in an ulnar nerve distribution.&lt;/div&gt;&lt;/div&gt;&lt;div&gt;&lt;br /&gt;&lt;/div&gt;&lt;div&gt;&lt;div&gt;The symptoms can be occasional or constant and exacerbated by elbow flexion. With longstanding ulnar nerve compression, patients may have wasting of their intrinsic muscles and hand weakness. An intrinsic minus or claw hand (hyperextension of the metacarpal phalangeal joints and flexion of the proximal and distal interphalangeal joints) may result from longstanding compression. Look for a &lt;b&gt;Wartenberg&amp;nbsp;&lt;/b&gt;&lt;b&gt;sign&lt;/b&gt; (ulnar deviation of the small finger)—patients complain of inability to put their hand in their pocket or their small finger getting caught on things. Patients may have weakness of their pinch and compensate by flexing the thumb interphalangeal joint during pinching (&lt;b&gt;Froment sign&lt;/b&gt;).&lt;/div&gt;&lt;/div&gt;&lt;div&gt;&lt;br /&gt;&lt;/div&gt;&lt;div&gt;&lt;div&gt;&lt;span&gt;&amp;nbsp;&amp;nbsp; &amp;nbsp;&lt;/span&gt;Patients may complain of medial elbow pain and have a positive Tinel test (exacerbation of the ulnar hand numbness by tapping on the ulnar nerve). It is important to do a thorough neck and shoulder exam to rule out nerve compression proximal to the elbow. This includes cervical range of motion to evaluate for associated pain and/or radiculopathy. Range the elbow and evaluate for ulnar nerve subluxation.&amp;nbsp;&lt;b&gt;The functional range of motion of the elbow is 30 to 130 degrees.&lt;/b&gt; The normal carrying angle is 7 to 15 degrees. Sensation in the hand should also be examined.&lt;/div&gt;&lt;/div&gt;&lt;div&gt;&lt;br /&gt;&lt;/div&gt;&lt;div&gt;&lt;div&gt;&lt;span&gt;&amp;nbsp;&amp;nbsp; &amp;nbsp;&lt;/span&gt;Radiographs of the elbow should be evaluated for osseous causes of compression (ie, osteophytes). Magnetic resonance imaging (MRI), although not typically indicated, can identify space-occupying lesions or edema within the nerve. Electrodiagnostic testing, which includes EMG and nerve conduction studies, is the gold standard to evaluate for ulnar neuropathy. Nerve conduction studies are considered positive if the motor conduction velocity across the elbow is less than 52 m/s, increased distal sensory latency of greater than 3.2 ms, and motor latencies of greater than 5.3 ms. Of note, abnormal EMG results are associated with poor surgical outcomes.&lt;/div&gt;&lt;/div&gt;&lt;div&gt;&lt;br /&gt;&lt;/div&gt;&lt;div&gt;&lt;br /&gt;&lt;/div&gt;&lt;div&gt;&lt;b style=&quot;color: #017087;&quot;&gt;&lt;span style=&quot;font-size: medium;&quot;&gt;TREATMENT&lt;/span&gt;&lt;/b&gt;&lt;/div&gt;&lt;div&gt;&lt;br /&gt;&lt;/div&gt;&lt;div&gt;&lt;div&gt;To determine the appropriate treatment for a patient with cubital tunnel syndrome, the following considerations must be taken into account: patient compliance, worker’s compensation, occupation, age, comorbidities, duration of symptoms, and severity of symptoms. Additionally, the orthopaedist must be certain that other causes of the patient’s symptoms have been ruled out and that concurrent carpal tunnel syndrome (present in 40% of patients with cubital tunnel syndrome) is not present, which often requires simultaneous release.&lt;/div&gt;&lt;div&gt;&lt;br /&gt;&lt;/div&gt;&lt;div&gt;&lt;span&gt;&amp;nbsp;&amp;nbsp; &amp;nbsp;&lt;/span&gt;For patients with mild cubital tunnel syndrome, nonsurgical options can alleviate symptoms and help prevent long-term nerve damage. A splint that keeps the elbow at approximately 70 degrees of flexion, especially when worn at night, can alleviate symptom exacerbation. Physical therapy (PT) that works on nerve mobilization and gliding, in addition to activity modification, can also aid in symptom relief. Nonsteroidal anti-inflammatory drugs (NSAIDs) as well as soft elbow padding can also be used.&lt;/div&gt;&lt;div&gt;&lt;br /&gt;&lt;/div&gt;&lt;div&gt;&lt;span&gt;&amp;nbsp;&amp;nbsp; &amp;nbsp;&lt;/span&gt;For patients with constant numbness/tingling, muscle atrophy, muscle weakness, or severe slowing of conduction velocity, surgical intervention is recommended.&amp;nbsp;&lt;b&gt;Surgery includes decompressing the nerve with or without transposition.&lt;/b&gt; This is achieved via in situ decompression (decompressing the nerve and leaving it where it is), subcutaneous transposition (moving the nerve anterior to the medial epicondyle), intramuscular transposition (moving the nerve into an area surrounded by the flexor-pronator muscles), submuscular transposition (moving the nerve under the flexor-pronator muscles), and medial epicondylectomy (removing part of the medial epicondyle). The cubital tunnel can also be released endoscopically.&lt;/div&gt;&lt;/div&gt;&lt;div&gt;&lt;br /&gt;&lt;/div&gt;&lt;div&gt;&lt;div&gt;&lt;span&gt;&amp;nbsp;&amp;nbsp; &amp;nbsp;&lt;/span&gt;&lt;b&gt;Most cases of cubital tunnel syndrome requiring surgical intervention can&amp;nbsp;&lt;/b&gt;&lt;b&gt;be treated with in situ decompression.&lt;/b&gt; There are inherent risks with nerve transposition, including compromised blood supply, nerve scarring, injury to adjacent structures, and a longer incision. Studies have shown that in situ decompression is adequate for most patients and has equivalent outcomes to decompression with transposition. Patients with a hypermobile nerve will likely benefit from moving the nerve to a new location.&lt;/div&gt;&lt;/div&gt;&lt;div&gt;&lt;br /&gt;&lt;/div&gt;&lt;div&gt;&lt;br /&gt;&lt;/div&gt;&lt;div&gt;&lt;b&gt;&lt;span style=&quot;color: #017087; font-size: medium;&quot;&gt;&lt;u&gt;COMPREHENSION QUESTIONS&lt;/u&gt;&lt;/span&gt;&lt;/b&gt;&lt;/div&gt;&lt;div&gt;&lt;br /&gt;&lt;/div&gt;&lt;div&gt;&lt;div&gt;30.1 A 40-year-old left-hand dominant man presents with an 8-month history of left hand numbness, tingling, and now weakness. On exam, you note a positive Froment sign. The patient subsequently undergoes electrodiagnostic testing, which is consistent with cubital tunnel syndrome. The cubital tunnel is formed by which structures?&lt;/div&gt;&lt;/div&gt;&lt;blockquote style=&quot;border: none; margin: 0px 0px 0px 40px; padding: 0px; text-align: left;&quot;&gt;&lt;div&gt;&lt;div&gt;A. Anterior, lateral epicondyle; posterior, olecranon; floor, medial collateral ligament; roof, arcuate ligament&lt;/div&gt;&lt;/div&gt;&lt;div&gt;&lt;div&gt;B. Anterior, medial epicondyle; posterior, olecranon; floor, medial collateral ligament; roof, arcuate ligament&lt;/div&gt;&lt;/div&gt;&lt;div&gt;&lt;div&gt;C. Anterior, lateral epicondyle; posterior, olecranon; floor, lateral collateral ligament; roof, arcuate ligament&lt;/div&gt;&lt;/div&gt;&lt;div&gt;&lt;div&gt;D. Anterior, medial epicondyle; posterior, humerus; floor, medial collateral ligament; roof, arcuate ligament&lt;/div&gt;&lt;/div&gt;&lt;/blockquote&gt;&lt;div&gt;&lt;div&gt;&lt;br /&gt;&lt;/div&gt;&lt;div&gt;30.2 A 50-year-old man complains of numbness and tingling along his right small finger. Elbow flexion reproduces the numbness and tingling. Physical therapy and splinting have failed to relieve the symptoms over the last 3 months. Which of the following is the most appropriate intervention?&lt;/div&gt;&lt;/div&gt;&lt;blockquote style=&quot;border: none; margin: 0px 0px 0px 40px; padding: 0px; text-align: left;&quot;&gt;&lt;div&gt;&lt;div&gt;A. In situ ulnar nerve decompression at the cubital tunnel&lt;/div&gt;&lt;/div&gt;&lt;div&gt;&lt;div&gt;B. Ulnar nerve decompression at the cubital tunnel with anterior submuscular transposition&lt;/div&gt;&lt;/div&gt;&lt;div&gt;&lt;div&gt;C. Ulnar nerve decompression at the cubital tunnel with anterior subcutaneous transposition&lt;/div&gt;&lt;/div&gt;&lt;div&gt;&lt;div&gt;D. Continued physical therapy, splinting, and conservative management&lt;/div&gt;&lt;/div&gt;&lt;/blockquote&gt;&lt;div&gt;&lt;div&gt;&lt;br /&gt;&lt;/div&gt;&lt;div&gt;30.3 An amateur bodybuilder presents with physical examination findings concerning for an ulnar nerve compression neuropathy. Which of the following anatomic locations is more commonly associated with ulnar nerve compression in bodybuilders as compared with the regular population?&lt;/div&gt;&lt;/div&gt;&lt;blockquote style=&quot;border: none; margin: 0px 0px 0px 40px; padding: 0px; text-align: left;&quot;&gt;&lt;div&gt;&lt;div&gt;A. Osborne fascia&lt;/div&gt;&lt;/div&gt;&lt;div&gt;&lt;div&gt;B. Arcuate ligament&lt;/div&gt;&lt;/div&gt;&lt;div&gt;&lt;div&gt;C. Medial epicondyle&lt;/div&gt;&lt;/div&gt;&lt;div&gt;&lt;div&gt;D. Medial head of the triceps&lt;/div&gt;&lt;/div&gt;&lt;/blockquote&gt;&lt;div&gt;&lt;br /&gt;&lt;/div&gt;&lt;div&gt;&lt;br /&gt;&lt;/div&gt;&lt;div&gt;&lt;b&gt;&lt;span style=&quot;color: #017087; font-size: medium;&quot;&gt;&lt;u&gt;ANSWERS&lt;/u&gt;&lt;/span&gt;&lt;/b&gt;&lt;/div&gt;&lt;div&gt;&lt;br /&gt;&lt;/div&gt;&lt;div&gt;&lt;div&gt;30.1 &lt;b&gt;B.&lt;/b&gt; The cubital tunnel is formed by the medial epicondyle anteriorly and the olecranon posteriorly. The medial collateral ligament forms the floor, and the roof consists of the arcuate ligament.&lt;/div&gt;&lt;div&gt;&lt;br /&gt;&lt;/div&gt;&lt;div&gt;30.2 &lt;b&gt;A.&lt;/b&gt; The patient has cubital tunnel syndrome that has failed conservative management. In situ decompression of the ulnar nerve is less invasive and has clinical outcomes equivalent to that of decompression with transposition. Decompression with transposition has also been shown to have higher complication rates.&lt;/div&gt;&lt;div&gt;&lt;br /&gt;&lt;/div&gt;&lt;div&gt;30.3 &lt;b&gt;D.&lt;/b&gt; All of the choices are possible sites of ulnar nerve compression, even in bodybuilders. However, hypertrophy of the medial head of the triceps in this population can cause ulnar neuropathy. This is an uncommon site of compression in the general population.&lt;/div&gt;&lt;/div&gt;&lt;div&gt;&lt;br /&gt;&lt;/div&gt;&lt;div&gt;&lt;b style=&quot;background-color: #4093a5;&quot;&gt;&amp;nbsp; &amp;nbsp;&lt;span style=&quot;font-size: medium;&quot;&gt;&amp;nbsp;&lt;/span&gt;&lt;span style=&quot;color: white;&quot;&gt;&lt;span style=&quot;font-size: medium;&quot;&gt;CLINICAL PEARLS&lt;/span&gt;&lt;span&gt;&amp;nbsp; &amp;nbsp;&amp;nbsp;&lt;/span&gt;&lt;/span&gt;&lt;/b&gt;&lt;/div&gt;&lt;div&gt;&lt;table border=&quot;1&quot; cellpadding=&quot;0&quot; cellspacing=&quot;0&quot; class=&quot;MsoTableGrid&quot; style=&quot;border-collapse: collapse; border: none; mso-border-alt: solid windowtext .5pt; mso-padding-alt: 0in 5.4pt 0in 5.4pt; mso-yfti-tbllook: 1184;&quot;&gt;
  169. &lt;tbody&gt;&lt;tr style=&quot;height: 19.15pt; mso-yfti-firstrow: yes; mso-yfti-irow: 0; mso-yfti-lastrow: yes;&quot;&gt;
  170.  &lt;td style=&quot;background: rgb(221, 237, 239); border-left: solid #017087 1.0pt; border: 1pt solid rgb(64, 147, 165); height: 19.15pt; padding: 0in 5.4pt; width: 433.7pt;&quot; valign=&quot;top&quot; width=&quot;578&quot;&gt;
  171.  &lt;div style=&quot;line-height: normal; margin-bottom: 0in;&quot;&gt;&lt;span style=&quot;color: #017087;&quot;&gt;&lt;br /&gt;&lt;/span&gt;&lt;/div&gt;&lt;div style=&quot;line-height: normal; margin-bottom: 0in;&quot;&gt;&lt;span style=&quot;color: #017087;&quot;&gt;►&lt;/span&gt;&amp;nbsp;Cubital tunnel syndrome includes any ulnar neuropathy in the mid-arm to mid-forearm.&lt;/div&gt;&lt;div style=&quot;line-height: normal; margin-bottom: 0in;&quot;&gt;&lt;span style=&quot;color: #017087;&quot;&gt;&lt;br /&gt;&lt;/span&gt;&lt;/div&gt;&lt;div style=&quot;line-height: normal; margin-bottom: 0in;&quot;&gt;&lt;span style=&quot;color: #017087;&quot;&gt;►&lt;/span&gt;&amp;nbsp;The anatomic borders of the cubital tunnel are as follows: Anterior, medial epicondyle; posterior, olecranon; floor, medial collateral ligament; roof, arcuate ligament.&lt;/div&gt;&lt;div style=&quot;line-height: normal; margin-bottom: 0in;&quot;&gt;&lt;span style=&quot;color: #017087;&quot;&gt;&lt;br /&gt;&lt;/span&gt;&lt;/div&gt;&lt;div style=&quot;line-height: normal; margin-bottom: 0in;&quot;&gt;&lt;span style=&quot;color: #017087;&quot;&gt;►&lt;/span&gt;&amp;nbsp;Nerve conduction studies are considered positive if the motor conduction velocity across the elbow is less than 52 m/s, increased distal sensory latency of greater than 3.2 ms, and motor latencies of greater than 5.3 ms.&lt;/div&gt;&lt;div style=&quot;line-height: normal; margin-bottom: 0in;&quot;&gt;&lt;span style=&quot;color: #017087;&quot;&gt;&lt;br /&gt;&lt;/span&gt;&lt;/div&gt;&lt;div style=&quot;line-height: normal; margin-bottom: 0in;&quot;&gt;&lt;span style=&quot;color: #017087;&quot;&gt;►&lt;/span&gt;&amp;nbsp;Conservative management of cubital tunnel includes splinting, NSAIDs, elbow padding, and PT.&lt;/div&gt;&lt;div style=&quot;line-height: normal; margin-bottom: 0in;&quot;&gt;&lt;span style=&quot;color: #017087;&quot;&gt;&lt;br /&gt;&lt;/span&gt;&lt;/div&gt;&lt;div style=&quot;line-height: normal; margin-bottom: 0in;&quot;&gt;&lt;span style=&quot;color: #017087;&quot;&gt;►&lt;/span&gt;&amp;nbsp;Operative interventions include ulnar nerve decompression with or without transposition and/or medial epicondylectomy.&lt;/div&gt;&lt;div style=&quot;line-height: normal; margin-bottom: 0in;&quot;&gt;&lt;br /&gt;&lt;/div&gt;
  172.  &lt;/td&gt;
  173. &lt;/tr&gt;
  174. &lt;/tbody&gt;&lt;/table&gt;&lt;br /&gt;&lt;/div&gt;
  175.  
  176. &lt;details close=&quot;&quot;&gt;
  177.  &lt;summary&gt;&lt;b&gt;&lt;span style=&quot;color: #00788b;&quot;&gt;REFERENCES&lt;/span&gt;&lt;/b&gt;&lt;/summary&gt;
  178.  &lt;p&gt;Grana W. Medial epicondylitis and cubital tunnel syndrome in the throwing athlete. Clin Sports Med.
  179. 2001;20:541-548. Review.&amp;nbsp;&lt;/p&gt;&lt;p&gt;Palmer BA, Hughes TB. Cubital tunnel syndrome. J Hand Surg Am. 2010;35:153-163.&lt;/p&gt;
  180. &lt;/details&gt;&lt;/div&gt;</content><link rel='replies' type='application/atom+xml' href='https://medical-phd.blogspot.com/feeds/7623815830716173278/comments/default' title='Post Comments'/><link rel='replies' type='text/html' href='https://medical-phd.blogspot.com/2022/03/cubital-tunnel-syndrome-case-file.html#comment-form' title='0 Comments'/><link rel='edit' type='application/atom+xml' href='https://www.blogger.com/feeds/3302710210518459416/posts/default/7623815830716173278'/><link rel='self' type='application/atom+xml' href='https://www.blogger.com/feeds/3302710210518459416/posts/default/7623815830716173278'/><link rel='alternate' type='text/html' href='https://medical-phd.blogspot.com/2022/03/cubital-tunnel-syndrome-case-file.html' title='Cubital Tunnel Syndrome Case File'/><author><name>Medical Group</name><uri>http://www.blogger.com/profile/01780656800578397393</uri><email>noreply@blogger.com</email><gd:image rel='http://schemas.google.com/g/2005#thumbnail' width='32' height='32' src='//blogger.googleusercontent.com/img/b/R29vZ2xl/AVvXsEiMuROkBDv6yw4SmxGqRA5BPQhOXSwFld-Gufmy4kUqfpoAGQRl7cv-3naew6wjAnCjRFSe_FKX8z7CdE6cFKxxBpnqBuwUTSr-15xd3mFGcr_wJOX2y73G_zJCGND98Q/s220/Final-logo.jpg'/></author><media:thumbnail xmlns:media="http://search.yahoo.com/mrss/" url="https://blogger.googleusercontent.com/img/b/R29vZ2xl/AVvXsEgTJpvkDA_HpZZNWRLgaMGvQQVYbTBWL7kItxVYOelXMrArC_o4nHtyabNPWDkgf3Wx92CCOkklYKppdXZVYjfZAMwbWbUIUsHNqC8GWB9VKqrNNxrQvCc1soemK8L0Uiec-1HQaIpSHiiYKhK9TrXOMTN_GNWW2E6ekSJRhZmcfdxW2_Eum6FMNBgRtg/s72-w640-h518-c/Elbow-anatomy.jpg" height="72" width="72"/><thr:total>0</thr:total><georss:featurename>United States</georss:featurename><georss:point>37.09024 -95.712891</georss:point><georss:box>8.780006163821156 -130.869141 65.400473836178847 -60.556641</georss:box></entry><entry><id>tag:blogger.com,1999:blog-3302710210518459416.post-6622004262042826540</id><published>2022-03-25T13:26:00.007-07:00</published><updated>2022-03-25T13:26:54.079-07:00</updated><category scheme="http://www.blogger.com/atom/ns#" term="Case Files"/><category scheme="http://www.blogger.com/atom/ns#" term="Orthopaedic Surgery Case File"/><title type='text'>Carpel Tunnel Syndrome Case File</title><content type='html'>&lt;div class=&quot;scrollbox&quot;&gt;
  181.  
  182. &lt;div&gt;&lt;b&gt;&lt;span style=&quot;font-size: large;&quot;&gt;Carpel Tunnel Syndrome Case File&lt;/span&gt;&lt;/b&gt;&lt;/div&gt;&lt;div&gt;Eugene C. Toy, MD, Andrew J. Rosenbaum, MD, Timothy T. Roberts, MD, Joshua S. Dines, MD&lt;/div&gt;&lt;div&gt;&lt;br /&gt;&lt;/div&gt;&lt;div&gt;&lt;b&gt;&lt;span style=&quot;color: #017087; font-size: medium;&quot;&gt;CASE 29&lt;/span&gt;&lt;/b&gt;&lt;/div&gt;&lt;div&gt;&lt;div&gt;A 47-year-old right-hand dominant female presents to the clinic with a 2-month history of intermittent numbness and tingling in her right hand, specifically the index and middle fingers. She often finds the symptoms worst when using her computer and when driving home. Her symptoms are significantly affecting her career as a journalist. She also complains of pain in the same distribution that wakes her up almost nightly. She denies any history of trauma. Her past medical history is significant only for type 2 diabetes mellitus (DM), which is well controlled with oral agents. She does not smoke and drinks 1 or 2 glasses of wine on occasion, and her body mass index is 32 kg/&lt;span face=&quot;arial, sans-serif&quot; style=&quot;background-color: white; color: #4d5156; font-size: 14px;&quot;&gt;m&lt;/span&gt;&lt;span face=&quot;arial, sans-serif&quot; style=&quot;background-color: white; color: #4d5156; line-height: 0.9; position: relative; top: -0.4em; vertical-align: baseline;&quot;&gt;&lt;span style=&quot;font-size: xx-small;&quot;&gt;2&lt;/span&gt;&lt;/span&gt;. Physical examination reveals a slightly overweight but otherwise healthy-appearing female. Both of her upper extremities appear normal, with no obvious signs of trauma. On her right hand, there is a palpable 2+ radial pulse and a normal Allen test. Two-point discrimination is measured at 8 mm on both the radial and ulnar borders of the index and middle fingers as well as the radial border of the ring finger. There is normal (&amp;lt; 5 mm) 2-point discrimination on the ulnar border of the ring finger and on both sides of the small finger. Sensation in the palm is normal.&lt;/div&gt;&lt;div&gt;&lt;br /&gt;&lt;/div&gt;&lt;div&gt;&lt;span style=&quot;color: #017087;&quot;&gt;►&lt;/span&gt;&amp;nbsp;What is the most likely diagnosis?&lt;/div&gt;&lt;div&gt;&lt;span style=&quot;color: #017087;&quot;&gt;►&lt;/span&gt;&amp;nbsp;How would you confirm the diagnosis?&lt;/div&gt;&lt;div&gt;&lt;span style=&quot;color: #017087;&quot;&gt;►&lt;/span&gt;&amp;nbsp;What is the initial treatment for this condition?&lt;/div&gt;&lt;/div&gt;&lt;div&gt;&lt;br /&gt;&lt;/div&gt;&lt;div&gt;&lt;br /&gt;&lt;/div&gt;&lt;div&gt;&lt;div&gt;&lt;b&gt;&lt;span style=&quot;font-size: medium;&quot;&gt;ANSWER TO CASE 29:&lt;/span&gt;&lt;/b&gt;&lt;/div&gt;&lt;div&gt;&lt;span style=&quot;background-color: #daebf0; font-size: medium;&quot;&gt;&lt;span&gt;&lt;span style=&quot;background-color: transparent;&quot;&gt;&lt;span style=&quot;color: #017087;&quot;&gt;&lt;b&gt;Carpel Tunnel Syndrome&lt;/b&gt;&lt;/span&gt;&lt;/span&gt;&lt;b&gt;&amp;nbsp;&amp;nbsp;&lt;/b&gt;&lt;/span&gt;&lt;span style=&quot;font-weight: bold;&quot;&gt;&amp;nbsp;&amp;nbsp; &amp;nbsp;&lt;/span&gt;&lt;span style=&quot;font-weight: bold;&quot;&gt;&amp;nbsp;&amp;nbsp; &amp;nbsp;&lt;/span&gt;&lt;span style=&quot;font-weight: bold;&quot;&gt;&amp;nbsp;&amp;nbsp; &amp;nbsp;&lt;/span&gt;&lt;span style=&quot;font-weight: bold;&quot;&gt;&amp;nbsp;&amp;nbsp; &amp;nbsp;&lt;/span&gt;&lt;span style=&quot;font-weight: bold;&quot;&gt;&amp;nbsp;&amp;nbsp; &amp;nbsp;&lt;/span&gt;&lt;span style=&quot;font-weight: bold;&quot;&gt;&amp;nbsp;&amp;nbsp; &amp;nbsp;&lt;/span&gt;&lt;span style=&quot;font-weight: bold;&quot;&gt;&amp;nbsp; &amp;nbsp;&amp;nbsp;&lt;/span&gt;&lt;/span&gt;&lt;/div&gt;&lt;/div&gt;&lt;div&gt;&lt;br /&gt;&lt;/div&gt;&lt;div&gt;&lt;div&gt;&lt;i&gt;Summary&lt;/i&gt;: A 47-year-old woman with a history of well-controlled type 2 DM has complaints of worsening numbness and tingling in her right hand for 2 months. She works as a journalist, spending much of her time on a computer. She has no history of trauma and is otherwise healthy. Vital signs are normal, and physical exam is remarkable for decreased 2-point discrimination on her index and middle fingers, as well as the radial side of her ring finger.&lt;/div&gt;&lt;div&gt;&lt;ul style=&quot;text-align: left;&quot;&gt;&lt;li&gt;&lt;b&gt;Most likely diagnosis:&lt;/b&gt; Carpal tunnel syndrome (CTS).&lt;/li&gt;&lt;li&gt;&lt;b&gt;Diagnostic testing:&lt;/b&gt; An electromyogram (EMG) and nerve conduction studies may help confirm the diagnosis. EMG studies may show an increase in motor and/or sensory latency across the wrist. Up to 10% of patients with clinical symptomatology, however, have normal EMG studies.&lt;/li&gt;&lt;li&gt;&lt;b&gt;Initial management:&lt;/b&gt; Nonsteroidal anti-inflammatory drugs (NSAIDs), if not contraindicated, and night splinting are the best initial management strategy.&lt;/li&gt;&lt;/ul&gt;&lt;/div&gt;&lt;/div&gt;&lt;div&gt;&lt;br /&gt;&lt;/div&gt;&lt;div&gt;&lt;br /&gt;&lt;/div&gt;&lt;div&gt;&lt;div&gt;&lt;b&gt;&lt;span style=&quot;color: #017087; font-size: medium;&quot;&gt;ANALYSIS&lt;/span&gt;&lt;/b&gt;&lt;/div&gt;&lt;div&gt;&lt;b&gt;&lt;span style=&quot;font-size: medium;&quot;&gt;&lt;i&gt;Objectives&lt;/i&gt;&lt;/span&gt;&lt;/b&gt;&lt;/div&gt;&lt;/div&gt;&lt;div&gt;&lt;div&gt;&lt;ol style=&quot;text-align: left;&quot;&gt;&lt;li&gt;Understand the pathoanatomy responsible for carpal tunnel syndrome.&lt;/li&gt;&lt;li&gt;Describe the physical exam findings of carpal tunnel syndrome.&lt;/li&gt;&lt;li&gt;Recognize underlying conditions (both physiologic and pathologic) that may predispose one to carpal tunnel syndrome.&lt;/li&gt;&lt;li&gt;Distinguish carpal tunnel syndrome from other diseases or conditions that may mimic its presentation.&lt;/li&gt;&lt;li&gt;Understand the conservative and surgical treatment options for carpal tunnel syndrome and their indications.&lt;/li&gt;&lt;/ol&gt;&lt;/div&gt;&lt;/div&gt;&lt;div&gt;&lt;br /&gt;&lt;/div&gt;&lt;div&gt;&lt;br /&gt;&lt;/div&gt;&lt;div&gt;&lt;div&gt;&lt;b&gt;&lt;i&gt;&lt;span style=&quot;font-size: medium;&quot;&gt;Considerations&lt;/span&gt;&lt;/i&gt;&lt;/b&gt;&lt;/div&gt;&lt;div&gt;&lt;br /&gt;&lt;/div&gt;&lt;div&gt;For any patient with acute neurologic complaints isolated to an extremity, it is important to first rule out trauma as an underlying cause. This patient denies a history of trauma, is without significant medical comorbidities, and appears reliable, so further imaging is unlikely to aid the diagnosis. In patients with complicated, unclear, or unreliable histories, radiographs may help eliminate several causes of neurologic symptoms such as fractures, deformities, congenital abnormalities, masses, and other lesions.&lt;/div&gt;&lt;div&gt;&lt;br /&gt;&lt;/div&gt;&lt;div&gt;&lt;span&gt;&amp;nbsp;&amp;nbsp; &amp;nbsp;&lt;/span&gt;There are other conditions that may mimic carpal tunnel syndrome, but can often be distinguished with a thorough physical exam and history. Traumatic conditions such as a distal radius fracture or scaphoid fracture may present with some symptoms that overlap with those of carpal tunnel syndrome, but would likely be discovered with adequate workup. More proximal nerve pathologies such as&lt;/div&gt;&lt;div&gt;cervical radiculopathy or proximal median nerve compression should have additional findings not found in isolated carpal tunnel syndrome, such as paresthesias in the thenar distribution, and would not have positive test findings on provocative tests aimed at eliciting symptoms within the carpal canal (ie, Phalen, Tinel, and Durkan signs). Cubital tunnel syndrome has similar pathology and presentation to carpal tunnel syndrome, but in a different nerve distribution. If the patient fails conservative management, carpal tunnel release surgery is very effective at alleviating symptoms of carpal tunnel syndrome. If performed early, surgical decompression can prevent permanent denervation of the thenar muscles and halt further deterioration of symptoms. In many operative cases, sensation is restored and symptoms resolve. With regard to operative technique, the results from open and endoscopic surgery are comparable.&lt;/div&gt;&lt;/div&gt;&lt;div&gt;&lt;br /&gt;&lt;/div&gt;&lt;div&gt;&lt;br /&gt;&lt;/div&gt;&lt;div&gt;&lt;div&gt;&lt;div&gt;&lt;b&gt;&lt;span style=&quot;font-size: medium;&quot;&gt;APPROACH TO:&lt;/span&gt;&lt;/b&gt;&lt;/div&gt;&lt;div&gt;&lt;span style=&quot;background-color: #daebf0; color: #017087; font-size: medium;&quot;&gt;&lt;span&gt;&lt;span style=&quot;background-color: transparent;&quot;&gt;&lt;b&gt;Carpal Tunnel Syndrome&lt;/b&gt;&lt;/span&gt;&lt;b&gt;&amp;nbsp;&lt;/b&gt;&lt;/span&gt;&lt;span style=&quot;font-weight: bold;&quot;&gt;&amp;nbsp;&amp;nbsp; &amp;nbsp;&lt;/span&gt;&lt;span style=&quot;font-weight: bold;&quot;&gt;&amp;nbsp;&amp;nbsp; &amp;nbsp;&lt;/span&gt;&lt;span style=&quot;font-weight: bold;&quot;&gt;&amp;nbsp;&amp;nbsp; &amp;nbsp;&lt;/span&gt;&lt;span style=&quot;font-weight: bold;&quot;&gt;&amp;nbsp;&amp;nbsp; &amp;nbsp;&lt;/span&gt;&lt;span style=&quot;font-weight: bold;&quot;&gt;&amp;nbsp;&amp;nbsp; &amp;nbsp;&lt;/span&gt;&lt;span style=&quot;font-weight: bold;&quot;&gt;&amp;nbsp;&amp;nbsp; &amp;nbsp;&lt;/span&gt;&lt;span style=&quot;font-weight: bold;&quot;&gt;&amp;nbsp;&amp;nbsp; &amp;nbsp;&lt;/span&gt;&lt;span style=&quot;font-weight: bold;&quot;&gt;&amp;nbsp;&amp;nbsp;&amp;nbsp;&lt;/span&gt;&lt;span style=&quot;font-weight: bold;&quot;&gt;&amp;nbsp;&lt;/span&gt;&lt;/span&gt;&lt;/div&gt;&lt;/div&gt;&lt;div&gt;&lt;br /&gt;&lt;/div&gt;&lt;div&gt;&lt;b&gt;&lt;span style=&quot;color: #017087; font-size: medium;&quot;&gt;DEFINITIONS&lt;/span&gt;&lt;/b&gt;&lt;/div&gt;&lt;/div&gt;&lt;div&gt;&lt;br /&gt;&lt;/div&gt;&lt;div&gt;&lt;div&gt;&lt;b&gt;CARPAL TUNNEL:&lt;/b&gt; A narrow canal in the volar (palmar) wrist that typically contains the median nerve, superficial and deep finger flexor tendons, and the long flexor tendon of the thumb.&lt;/div&gt;&lt;div&gt;&lt;b&gt;&lt;br /&gt;&lt;/b&gt;&lt;/div&gt;&lt;div&gt;&lt;b&gt;GUYON CANAL: &lt;/b&gt;Also known as the ulnar tunnel, this volar wrist canal is located just ulnar (medial) to the carpal tunnel and contains the ulnar artery and ulnar nerve. Compression within this tunnel can lead to a similar syndrome to carpal tunnel, with symptoms instead found in an ulnar nerve distribution.&lt;/div&gt;&lt;div&gt;&lt;b&gt;&lt;br /&gt;&lt;/b&gt;&lt;/div&gt;&lt;div&gt;&lt;b&gt;MEDIAN NERVE COMPRESSION SYNDROME:&lt;/b&gt; A spectrum of compression neuropathies of the median nerve with varying names and symptoms, depending on the location of pathology. When the nerve is compressed proximally, most commonly between the two heads of the pronator teres muscle, it is called pronator teres syndrome, and is characterized by volar hand and forearm numbness as well as potential weaknesses in the thumb flexors and distal flexors of the index and middle fingers. The median nerve can be compressed in several key locations, the most distal of which is in the carpal tunnel.&lt;/div&gt;&lt;/div&gt;&lt;div&gt;&lt;br /&gt;&lt;/div&gt;&lt;div&gt;&lt;br /&gt;&lt;/div&gt;&lt;div&gt;&lt;b&gt;&lt;span style=&quot;color: #017087; font-size: medium;&quot;&gt;CLINICAL APPROACH&lt;/span&gt;&lt;/b&gt;&lt;/div&gt;&lt;div&gt;&lt;br /&gt;&lt;/div&gt;&lt;div&gt;&lt;div&gt;&lt;b&gt;&lt;i&gt;&lt;span style=&quot;font-size: medium;&quot;&gt;Relevant Anatomy&lt;/span&gt;&lt;/i&gt;&lt;/b&gt;&lt;/div&gt;&lt;div&gt;&lt;br /&gt;&lt;/div&gt;&lt;div&gt;The carpal tunnel can be described exactly as it is named—as a tunnel (or canal) passing through the wrist. Understanding the anatomy of the canal, including its borders and its contents, is key to understanding the basis of carpal tunnel syndrome. The carpal canal is formed by the &lt;b&gt;transverse carpal ligament volarly&lt;/b&gt; (palmar), &lt;b&gt;the scaphoid and trapezium radially,&lt;/b&gt; and the &lt;b&gt;hamate and triquetrum ulnarly.&lt;/b&gt; Within the canal passes the &lt;b&gt;median nerve&lt;/b&gt; and 9 flexor tendons: &lt;b&gt;4 tendons of the flexor&amp;nbsp;&lt;/b&gt;digitorum profundus &lt;b&gt;(FDP), 4 tendons of the flexor&lt;/b&gt; digitorum superficialis (&lt;b&gt;FDS&lt;/b&gt;), and the single tendon of the flexor pollicis longus (&lt;b&gt;FPL&lt;/b&gt;).&lt;/div&gt;&lt;/div&gt;&lt;div&gt;&lt;br /&gt;&lt;/div&gt;&lt;div&gt;&lt;div&gt;&lt;span&gt;&amp;nbsp;&amp;nbsp; &amp;nbsp;&lt;/span&gt;The median nerve is responsible for sensation to the volar (palmar) surface of the thumb, index finger, long finger, and the radial (lateral) half of the ring finger. The&amp;nbsp;&lt;b&gt;sensation of the palm and thenar eminence is normal&lt;/b&gt; in carpal tunnel syndrome because the palmar cutaneous branch of the median nerve arises proximal to the carpal tunnel and crosses the wrist superficial to the transverse carpal ligament to give sensation to the palm. In most instances, the recurrent motor branch of the median nerve comes off the median nerve distal to the nerves coursing within the canal, so that compressive effects on the median nerve may also affect the recurrent motor branch.&lt;/div&gt;&lt;div&gt;&lt;br /&gt;&lt;/div&gt;&lt;div&gt;&lt;span&gt;&amp;nbsp;&amp;nbsp; &amp;nbsp;&lt;/span&gt;Decreasing the effective size of the canal (ie, prolonged or repetitive wrist flexion/ extension, etc) or increasing the relative size of the canal’s contents (ie, flexor tendon inflammation/swelling) can lead to carpal tunnel syndrome. This explains why many patients with carpal tunnel syndrome complain of worsened symptoms at night, as it is thought that most people sleep with their wrists in a flexed position.&lt;/div&gt;&lt;/div&gt;&lt;div&gt;&lt;br /&gt;&lt;/div&gt;&lt;div&gt;&lt;br /&gt;&lt;/div&gt;&lt;div&gt;&lt;div&gt;&lt;b&gt;&lt;i&gt;&lt;span style=&quot;font-size: medium;&quot;&gt;Diagnosis&lt;/span&gt;&lt;/i&gt;&lt;/b&gt;&lt;/div&gt;&lt;div&gt;&lt;br /&gt;&lt;/div&gt;&lt;div&gt;Carpal tunnel syndrome is a &lt;b&gt;clinical diagnosis,&lt;/b&gt; meaning the diagnosis is based primarily on history and physical exam findings, with adjunctive tests such as EMG studies used to confirm—but never make—the diagnosis. Physical exam findings such as &lt;b&gt;decreased sensation and 2-point discrimination in the distribution of&amp;nbsp;&lt;/b&gt;&lt;b&gt;the median nerve&lt;/b&gt; at the fingers with &lt;b&gt;sparing of sensation of the thenar eminence&amp;nbsp;&lt;/b&gt;&lt;b&gt;and palm&lt;/b&gt; are pathognomic for carpal tunnel syndrome. &lt;b&gt;Tinel sign&lt;/b&gt; involves&amp;nbsp;&lt;b&gt;percussion over the median nerve at the wrist, and Phalen sign&lt;/b&gt; involves &lt;b&gt;holding the&amp;nbsp;&lt;/b&gt;&lt;b&gt;wrist in flexion for 1 minute.&lt;/b&gt; Both &lt;b&gt;tests are positive if they result in reproduction&amp;nbsp;&lt;/b&gt;&lt;b&gt;of symptoms.&lt;/b&gt; The &lt;b&gt;Durkan test,&lt;/b&gt; or carpal tunnel compression test, is performed with the examiner holding pressure over the patient’s carpal tunnel, while distracting the patient. Reproduction of symptoms within 30 seconds of compression is considered positive. Although the patient in this case does not describe weakness or atrophy of the thenar musculature, a decrease in pinch/grip strength may occur in more advanced stages of carpal tunnel syndrome.&lt;/div&gt;&lt;/div&gt;&lt;div&gt;&lt;br /&gt;&lt;/div&gt;&lt;div&gt;&lt;div&gt;&lt;span&gt;&amp;nbsp;&amp;nbsp; &amp;nbsp;&lt;/span&gt;EMG studies may help clarify the diagnosis of CTS. Such studies may demonstrate a latency and/or asymmetry between the affected and nonaffected hands. Many normal physiologic and/or pathologic conditions may promote the onset of carpal tunnel syndrome. These include, but are not limited to, pregnancy, type 2 DM, gout, rheumatoid arthritis, and hypothyroidism. The mechanisms by which these conditions cause carpal tunnel syndrome are not fully understood. However, each condition is thought to promote a state of inflammation and thus elevated pressure within the carpal canal. Patients should be evaluated for potential underlying causes of carpal tunnel syndrome. Underlying etiologies, if present, must be addressed prior to operative treatment.&lt;/div&gt;&lt;/div&gt;&lt;div&gt;&lt;br /&gt;&lt;/div&gt;&lt;div&gt;&lt;br /&gt;&lt;/div&gt;&lt;div&gt;&lt;b style=&quot;color: #017087;&quot;&gt;&lt;span style=&quot;font-size: medium;&quot;&gt;TREATMENT&lt;/span&gt;&lt;/b&gt;&lt;/div&gt;&lt;div&gt;&lt;br /&gt;&lt;/div&gt;&lt;div&gt;&lt;div&gt;When carpal tunnel syndrome is diagnosed, the most conservative mode of treatment is attempted first. Nighttime symptoms are often the result of the naturally-flexed position of the resting wrist. Removable splints that hold the wrist in neutral, the position in which the carpal tunnel is least compressed, are usually effective at reducing symptoms. Additionally, anti-inflammatories (NSAIDs) are an effective treatment option. In patients whose symptoms are not sufficiently relieved by these options, corticosteroid injections directly into the carpal tunnel may be an option. Local steroid therapy may provide transient relief; however, symptoms often return. Note that there is prognostic value to such therapies, as patients who respond well to steroid injections have a greater chance of successful relief from surgery.&lt;/div&gt;&lt;/div&gt;&lt;div&gt;&lt;br /&gt;&lt;/div&gt;&lt;div&gt;&lt;div&gt;&lt;span&gt;&amp;nbsp;&amp;nbsp; &amp;nbsp;&lt;/span&gt;The definitive treatment for carpal tunnel syndrome refractory to conservative management is a carpal tunnel release. This involves releasing the transverse carpal ligament using either a direct open or endoscopic technique. Carpal tunnel release is a relatively fast outpatient procedure with generally low complication rates and excellent outcomes.&lt;/div&gt;&lt;/div&gt;&lt;div&gt;&lt;br /&gt;&lt;/div&gt;&lt;div&gt;&lt;br /&gt;&lt;/div&gt;&lt;div&gt;&lt;b&gt;&lt;span style=&quot;color: #017087; font-size: medium;&quot;&gt;&lt;u&gt;COMPREHENSION QUESTIONS&lt;/u&gt;&lt;/span&gt;&lt;/b&gt;&lt;/div&gt;&lt;div&gt;&lt;br /&gt;&lt;/div&gt;&lt;div&gt;&lt;div&gt;29.1 A 43-year-old health care worker complains of nocturnal wrist pain and has significant pain when pressure is applied to her volar wrist for more than 10 seconds. Which of the following structures is not usually contained within the carpal tunnel?&lt;/div&gt;&lt;/div&gt;&lt;blockquote style=&quot;border: none; margin: 0px 0px 0px 40px; padding: 0px; text-align: left;&quot;&gt;&lt;div&gt;&lt;div&gt;A. Tendon(s) of FDP&lt;/div&gt;&lt;/div&gt;&lt;div&gt;&lt;div&gt;B. Tendon(s) of FPL&lt;/div&gt;&lt;/div&gt;&lt;div&gt;&lt;div&gt;C. Palmar cutaneous branch of the median nerve&lt;/div&gt;&lt;/div&gt;&lt;div&gt;&lt;div&gt;D. Tendon(s) of FDS&lt;/div&gt;&lt;/div&gt;&lt;div&gt;&lt;div&gt;E. Motor fibers of the median nerve&lt;/div&gt;&lt;/div&gt;&lt;/blockquote&gt;&lt;div&gt;&lt;div&gt;&lt;br /&gt;&lt;/div&gt;&lt;div&gt;29.2 A 52-year-old retired woman is referred to your clinic with complaints of numbness and tingling in her left (dominant) hand worse at night for the past 3 months. She also complains of significant fatigue and 10-lb weight gain over the same period of time. She denies any medical history and takes no medications. On exam, she is overweight and appears somewhat lethargic, but appears healthy otherwise. On her left hand she has decreased 2-point discrimination on her index and middle fingers. Her grip strength is normal, and she has no thenar atrophy. What is the next best step in management of this patient?&lt;/div&gt;&lt;/div&gt;&lt;blockquote style=&quot;border: none; margin: 0px 0px 0px 40px; padding: 0px; text-align: left;&quot;&gt;&lt;div&gt;&lt;div&gt;A. Left hand carpal tunnel release&lt;/div&gt;&lt;/div&gt;&lt;div&gt;&lt;div&gt;B. EMG/nerve conduction velocity studies of left upper extremity&lt;/div&gt;&lt;/div&gt;&lt;div&gt;&lt;div&gt;C. Cervical spine magnetic resonance imaging (MRI)&lt;/div&gt;&lt;/div&gt;&lt;div&gt;&lt;div&gt;D. Lab studies including complete blood count, basic metabolic panel, thyroid-stimulating hormone, T3, and free T4&lt;/div&gt;&lt;/div&gt;&lt;div&gt;&lt;div&gt;E. Referral for counseling regarding recent changes in her lifestyle&lt;/div&gt;&lt;/div&gt;&lt;/blockquote&gt;&lt;div&gt;&lt;br /&gt;&lt;/div&gt;&lt;div&gt;&lt;div&gt;29.3 A 56-year-old postmenopausal woman is seen in the clinic with complaints of mild numbness and tingling in her right hand, most notably in the index and middle fingers. She takes medication for osteoporosis but is otherwise healthy. Her history is unremarkable except for a wrist fracture after a fall onto her right hand 2 years ago that was fixed with open reduction internal fixation. Her postoperative course was uncomplicated. Physical examination in the office reveals a well-healed surgical incision on the volar aspect of her wrist, normal grip strength, and no sensory deficits. X-rays reveal hardware in adequate alignment, with evidence of bony union at a previous distal radius fracture site. What is the next best step in her management?&lt;/div&gt;&lt;/div&gt;&lt;blockquote style=&quot;border: none; margin: 0px 0px 0px 40px; padding: 0px; text-align: left;&quot;&gt;&lt;div&gt;&lt;div&gt;A. Provision cast immobilization and follow-up x-ray for occult refracture&lt;/div&gt;&lt;/div&gt;&lt;div&gt;&lt;div&gt;B. MRI of right wrist&lt;/div&gt;&lt;/div&gt;&lt;div&gt;&lt;div&gt;C. Emergency surgical release of transverse carpal ligament&lt;/div&gt;&lt;/div&gt;&lt;div&gt;&lt;div&gt;D. NSAID administration with close follow-up&lt;/div&gt;&lt;/div&gt;&lt;div&gt;&lt;div&gt;E. Serum erythrocyte sedimentation rate, C-reactive protein, complete blood count, and wrist aspiration, with cytologic analysis, Gram stain, and culture of aspirate&lt;/div&gt;&lt;/div&gt;&lt;/blockquote&gt;&lt;div&gt;&lt;br /&gt;&lt;/div&gt;&lt;div&gt;&lt;br /&gt;&lt;/div&gt;&lt;div&gt;&lt;b&gt;&lt;span style=&quot;color: #017087; font-size: medium;&quot;&gt;&lt;u&gt;ANSWERS&lt;/u&gt;&lt;/span&gt;&lt;/b&gt;&lt;/div&gt;&lt;div&gt;&lt;br /&gt;&lt;/div&gt;&lt;div&gt;&lt;div&gt;29.1 &lt;b&gt;C. &lt;/b&gt;The palmar cutaneous branch of the median nerve is given off approximately 5 cm proximally to the carpal tunnel in most patients. This explains why patients with carpal tunnel syndrome typically have sparing of sensation of the thenar/palmar aspect of the affected hand. The motor fibers of the median nerve branch off distally to the tunnel, and as a result, patients may exhibit weakness and/or atrophy of the thenar musculature. The 9 flexor tendons of FDP, FDS, and FPL are all contained within the carpal tunnel.&lt;/div&gt;&lt;div&gt;&lt;br /&gt;&lt;/div&gt;&lt;div&gt;29.2 &lt;b&gt;D.&lt;/b&gt; The patient in this question does exhibit symptoms of carpal tunnel syndrome. However, her history and physical exam also point strongly to undiagnosed hypothyroidism. Further workup and treatment of her metabolic condition should be performed before intervention of her carpal tunnel&lt;/div&gt;&lt;div&gt;symptoms.&lt;/div&gt;&lt;div&gt;&lt;br /&gt;&lt;/div&gt;&lt;div&gt;29.3 &lt;b&gt;D.&lt;/b&gt; Distal radius fractures and subsequent fixation are associated with a number of potential complications, including iatrogenic carpal tunnel syndrome of variable severity. In this case, the patient does have postoperative symptoms that are consistent with carpal tunnel syndrome, but they are mild and intermittent. Acute or emergency carpal tunnel release is not indicated. There are no signs of infection or refracture, and cast immobilization may exacerbate her symptoms. MRI of the wrist would not likely provide any new information. The correct choice would be conservative management with close follow-up to monitor for change in symptoms.&lt;/div&gt;&lt;/div&gt;&lt;div&gt;&lt;br /&gt;&lt;/div&gt;&lt;div&gt;&lt;b style=&quot;background-color: #4093a5;&quot;&gt;&amp;nbsp; &amp;nbsp;&lt;span style=&quot;font-size: medium;&quot;&gt;&amp;nbsp;&lt;/span&gt;&lt;span style=&quot;color: white;&quot;&gt;&lt;span style=&quot;font-size: medium;&quot;&gt;CLINICAL PEARLS&lt;/span&gt;&lt;span&gt;&amp;nbsp; &amp;nbsp;&amp;nbsp;&lt;/span&gt;&lt;/span&gt;&lt;/b&gt;&lt;/div&gt;&lt;div&gt;&lt;table border=&quot;1&quot; cellpadding=&quot;0&quot; cellspacing=&quot;0&quot; class=&quot;MsoTableGrid&quot; style=&quot;border-collapse: collapse; border: none; mso-border-alt: solid windowtext .5pt; mso-padding-alt: 0in 5.4pt 0in 5.4pt; mso-yfti-tbllook: 1184;&quot;&gt;
  183. &lt;tbody&gt;&lt;tr style=&quot;height: 19.15pt; mso-yfti-firstrow: yes; mso-yfti-irow: 0; mso-yfti-lastrow: yes;&quot;&gt;
  184.  &lt;td style=&quot;background: rgb(221, 237, 239); border-left: solid #017087 1.0pt; border: 1pt solid rgb(64, 147, 165); height: 19.15pt; padding: 0in 5.4pt; width: 433.7pt;&quot; valign=&quot;top&quot; width=&quot;578&quot;&gt;
  185.  &lt;div style=&quot;line-height: normal; margin-bottom: 0in;&quot;&gt;&lt;span style=&quot;color: #017087;&quot;&gt;&lt;br /&gt;&lt;/span&gt;&lt;/div&gt;&lt;div style=&quot;line-height: normal; margin-bottom: 0in;&quot;&gt;&lt;span style=&quot;color: #017087;&quot;&gt;►&lt;/span&gt;&amp;nbsp;The carpal tunnel is an effectively closed space, containing the median nerve, among other structures. Any reduction in volume of this finite space can lead to increased pressure upon, and thus dysfunction of, the median nerve. This results in carpal tunnel syndrome.&lt;/div&gt;&lt;div style=&quot;line-height: normal; margin-bottom: 0in;&quot;&gt;&lt;span style=&quot;color: #017087;&quot;&gt;&lt;br /&gt;&lt;/span&gt;&lt;/div&gt;&lt;div style=&quot;line-height: normal; margin-bottom: 0in;&quot;&gt;&lt;span style=&quot;color: #017087;&quot;&gt;►&lt;/span&gt;&amp;nbsp;Carpal tunnel syndrome is a clinical diagnosis, made by history and physical exam findings; adjunctive tests such as EMG studies may be used to confirm, but never make, the diagnosis.&lt;/div&gt;&lt;div style=&quot;line-height: normal; margin-bottom: 0in;&quot;&gt;&lt;span style=&quot;color: #017087;&quot;&gt;&lt;br /&gt;&lt;/span&gt;&lt;/div&gt;&lt;div style=&quot;line-height: normal; margin-bottom: 0in;&quot;&gt;&lt;span style=&quot;color: #017087;&quot;&gt;►&lt;/span&gt;&amp;nbsp;Other neurologic conditions or injuries may be differentiated from carpal tunnel syndrome by thorough history and physical exam, including exact localization of motor and sensory symptoms.&lt;/div&gt;&lt;div style=&quot;line-height: normal; margin-bottom: 0in;&quot;&gt;&lt;span style=&quot;color: #017087;&quot;&gt;&lt;br /&gt;&lt;/span&gt;&lt;/div&gt;&lt;div style=&quot;line-height: normal; margin-bottom: 0in;&quot;&gt;&lt;span style=&quot;color: #017087;&quot;&gt;►&lt;/span&gt;&amp;nbsp;Conservative management should always be attempted before operative treatment of CTS. If conservative management fails, surgical carpal tunnel release is performed with either open or endoscopic techniques. Both methods are associated with excellent outcomes.&lt;/div&gt;&lt;div style=&quot;line-height: normal; margin-bottom: 0in;&quot;&gt;&lt;br /&gt;&lt;/div&gt;
  186.  &lt;/td&gt;
  187. &lt;/tr&gt;
  188. &lt;/tbody&gt;&lt;/table&gt;&lt;br /&gt;&lt;/div&gt;
  189.  
  190. &lt;details close=&quot;&quot;&gt;
  191.  &lt;summary&gt;&lt;b&gt;&lt;span style=&quot;color: #00788b;&quot;&gt;REFERENCES&lt;/span&gt;&lt;/b&gt;&lt;/summary&gt;
  192.  &lt;p&gt;American Academy of Orthopaedic Surgeons. Clinical guideline on diagnosis of carpal tunnel syndrome.
  193. 2007. www.aaos.org/Research/guidelines/CTSdiagnosisguide.asp .&amp;nbsp;&lt;/p&gt;&lt;p&gt;American Academy of Orthopaedic Surgeons. Clinical practice guideline on the treatment of carpal
  194. tunnel syndrome. 2008. www.aaos.org/Research/guidelines/CTStreatmentguide.asp .&amp;nbsp;&lt;/p&gt;&lt;p&gt;Bednar MS, Light TR. Disorders of the nerves of the hand. In: HB Skinner, ed. Current Diagnosis and
  195. Treatment in Orthopedics. 4th ed. New York: McGraw-Hill; 2006:559-567.&amp;nbsp;&lt;/p&gt;&lt;p&gt;Bienek T, Kusz D, Cielinski L. Peripheral nerve compression neuropathy after fractures of the distal
  196. radius. J Hand Surg Br. 2006;31:256-260.&amp;nbsp;&lt;/p&gt;&lt;p&gt;Fuller DA, Barrett M, Marburger RK, Hirsch R. Carpal canal pressures after volar plating of distal radius
  197. fractures. J Hand Surg Br. 2006;31:236-239.&lt;/p&gt;
  198. &lt;/details&gt;&lt;/div&gt;</content><link rel='replies' type='application/atom+xml' href='https://medical-phd.blogspot.com/feeds/6622004262042826540/comments/default' title='Post Comments'/><link rel='replies' type='text/html' href='https://medical-phd.blogspot.com/2022/03/carpel-tunnel-syndrome-case-file.html#comment-form' title='0 Comments'/><link rel='edit' type='application/atom+xml' href='https://www.blogger.com/feeds/3302710210518459416/posts/default/6622004262042826540'/><link rel='self' type='application/atom+xml' href='https://www.blogger.com/feeds/3302710210518459416/posts/default/6622004262042826540'/><link rel='alternate' type='text/html' href='https://medical-phd.blogspot.com/2022/03/carpel-tunnel-syndrome-case-file.html' title='Carpel Tunnel Syndrome Case File'/><author><name>Medical Group</name><uri>http://www.blogger.com/profile/01780656800578397393</uri><email>noreply@blogger.com</email><gd:image rel='http://schemas.google.com/g/2005#thumbnail' width='32' height='32' src='//blogger.googleusercontent.com/img/b/R29vZ2xl/AVvXsEiMuROkBDv6yw4SmxGqRA5BPQhOXSwFld-Gufmy4kUqfpoAGQRl7cv-3naew6wjAnCjRFSe_FKX8z7CdE6cFKxxBpnqBuwUTSr-15xd3mFGcr_wJOX2y73G_zJCGND98Q/s220/Final-logo.jpg'/></author><thr:total>0</thr:total><georss:featurename>United States</georss:featurename><georss:point>37.09024 -95.712891</georss:point><georss:box>8.780006163821156 -130.869141 65.400473836178847 -60.556641</georss:box></entry><entry><id>tag:blogger.com,1999:blog-3302710210518459416.post-7499594277683878295</id><published>2022-03-25T12:58:00.003-07:00</published><updated>2022-03-25T12:58:34.628-07:00</updated><category scheme="http://www.blogger.com/atom/ns#" term="Case Files"/><category scheme="http://www.blogger.com/atom/ns#" term="Orthopaedic Surgery Case File"/><title type='text'>Adolescent Idiopathic Scoliosis Case File</title><content type='html'>&lt;div class=&quot;scrollbox&quot;&gt;
  199.  
  200. &lt;div style=&quot;text-align: left;&quot;&gt;&lt;b&gt;&lt;span style=&quot;font-size: large;&quot;&gt;Adolescent Idiopathic Scoliosis Case File&lt;/span&gt;&lt;/b&gt;&lt;/div&gt;&lt;div&gt;Eugene C. Toy, MD, Andrew J. Rosenbaum, MD, Timothy T. Roberts, MD, Joshua S. Dines, MD&lt;/div&gt;&lt;div&gt;&lt;br /&gt;&lt;/div&gt;&lt;div&gt;&lt;b&gt;&lt;span style=&quot;color: #017087; font-size: medium;&quot;&gt;CASE 28&lt;/span&gt;&lt;/b&gt;&lt;/div&gt;&lt;div&gt;&lt;div&gt;A 12-year-old girl is referred to your clinic for concerns of shoulder asymmetry. She states that her friends first noticed that her back was not symmetric when she was changing clothes for basketball practice. She notices occasional back pain that she attributes to carrying a heavy backpack. The review of systems is otherwise unremarkable. Her past medical and developmental history are also noncontributory. She is in the seventh grade and is active in athletics and gymnastics. She experienced menarche 4 months prior and her menses have been regular. On examination, her right shoulder is 2 cm higher than the left, but her posterior iliac wings are of equal and symmetric heights. She has mild tenderness to palpation and slight spasms of the paraspinal musculature of her right-sided thoracic spine. There are no cutaneous lesions in the midline of her back. On forward bending, she has a 7-degree angle of trunk rotation in the thoracic spine, with the right side higher than the left. She has 5/5 strength in bilateral lower extremities throughout all muscle groups and no sensory abnormalities. Knee and ankle jerk reflexes are 2+ and symmetric. She has no sustained clonus or pathologic reflexes.&lt;/div&gt;&lt;div&gt;&lt;br /&gt;&lt;/div&gt;&lt;div&gt;&lt;span style=&quot;color: #017087;&quot;&gt;►&lt;/span&gt;&amp;nbsp;What is the most likely diagnosis?&lt;/div&gt;&lt;div&gt;&lt;span style=&quot;color: #017087;&quot;&gt;►&lt;/span&gt;&amp;nbsp;What is your next diagnostic step?&lt;/div&gt;&lt;div&gt;&lt;span style=&quot;color: #017087;&quot;&gt;►&amp;nbsp;&lt;/span&gt;What are the treatment options?&lt;/div&gt;&lt;/div&gt;&lt;div&gt;&lt;br /&gt;&lt;/div&gt;&lt;div&gt;&lt;br /&gt;&lt;/div&gt;&lt;div&gt;&lt;div&gt;&lt;div&gt;&lt;b&gt;&lt;span style=&quot;font-size: medium;&quot;&gt;ANSWER TO CASE 28:&lt;/span&gt;&lt;/b&gt;&lt;/div&gt;&lt;div&gt;&lt;span style=&quot;background-color: #daebf0; font-size: medium;&quot;&gt;&lt;span&gt;&lt;span style=&quot;background-color: transparent;&quot;&gt;&lt;span style=&quot;color: #017087;&quot;&gt;&lt;b&gt;Adolescent Idiopathic Scoliosis&lt;/b&gt;&lt;/span&gt;&lt;/span&gt;&lt;b&gt;&amp;nbsp;&amp;nbsp;&lt;/b&gt;&lt;/span&gt;&lt;span style=&quot;font-weight: bold;&quot;&gt;&amp;nbsp;&amp;nbsp; &amp;nbsp;&lt;/span&gt;&lt;span style=&quot;font-weight: bold;&quot;&gt;&amp;nbsp;&amp;nbsp; &amp;nbsp;&lt;/span&gt;&lt;span style=&quot;font-weight: bold;&quot;&gt;&amp;nbsp;&amp;nbsp; &amp;nbsp;&lt;/span&gt;&lt;span style=&quot;font-weight: bold;&quot;&gt;&amp;nbsp;&amp;nbsp; &amp;nbsp;&lt;/span&gt;&lt;span style=&quot;font-weight: bold;&quot;&gt;&amp;nbsp;&amp;nbsp; &amp;nbsp;&lt;/span&gt;&lt;span style=&quot;font-weight: bold;&quot;&gt;&amp;nbsp;&amp;nbsp; &amp;nbsp;&lt;/span&gt;&lt;span style=&quot;font-weight: bold;&quot;&gt;&amp;nbsp; &amp;nbsp;&amp;nbsp;&lt;/span&gt;&lt;/span&gt;&lt;/div&gt;&lt;/div&gt;&lt;div&gt;&lt;br /&gt;&lt;/div&gt;&lt;/div&gt;&lt;div&gt;&lt;div&gt;&lt;i&gt;Summary&lt;/i&gt;: A healthy 12-year-old girl presents with mild back pain, shoulder asymmetry, and a 7-degree rotational deformity of her thoracic spine. She has recently experienced menarche and has no other complaints. Neurologic exam is unremarkable.&lt;/div&gt;&lt;div&gt;&lt;ul style=&quot;text-align: left;&quot;&gt;&lt;li&gt;&lt;b&gt;Most likely diagnosis:&lt;/b&gt; Adolescent idiopathic scoliosis (AIS).&lt;/li&gt;&lt;li&gt;&lt;b&gt;Next diagnostic step:&lt;/b&gt; Standing 36-in posteroanterior (PA) and lateral spine radiographs.&lt;/li&gt;&lt;li&gt;&lt;b&gt;Treatment options:&lt;/b&gt; Observation, brace treatment, surgical spinal fusion.&lt;/li&gt;&lt;/ul&gt;&lt;/div&gt;&lt;/div&gt;&lt;div&gt;&lt;br /&gt;&lt;/div&gt;&lt;div&gt;&lt;br /&gt;&lt;/div&gt;&lt;div&gt;&lt;div&gt;&lt;b&gt;&lt;span style=&quot;color: #017087; font-size: medium;&quot;&gt;ANALYSIS&lt;/span&gt;&lt;/b&gt;&lt;/div&gt;&lt;div&gt;&lt;b&gt;&lt;span style=&quot;font-size: medium;&quot;&gt;&lt;i&gt;Objectives&lt;/i&gt;&lt;/span&gt;&lt;/b&gt;&lt;/div&gt;&lt;/div&gt;&lt;div&gt;&lt;ol style=&quot;text-align: left;&quot;&gt;&lt;li&gt;Understand the common presentation and characteristics of patients with AIS.&lt;/li&gt;&lt;li&gt;Recognize pertinent negatives that distinguish idiopathic scoliosis from scoliosis caused by other etiologies.&lt;/li&gt;&lt;li&gt;Consider the factors that lead to the decision making in the treatment algorithm of scoliosis.&lt;/li&gt;&lt;/ol&gt;&lt;/div&gt;&lt;div&gt;&lt;br /&gt;&lt;/div&gt;&lt;div&gt;&lt;br /&gt;&lt;/div&gt;&lt;div&gt;&lt;div&gt;&lt;b&gt;&lt;i&gt;&lt;span style=&quot;font-size: medium;&quot;&gt;Considerations&lt;/span&gt;&lt;/i&gt;&lt;/b&gt;&lt;/div&gt;&lt;div&gt;&lt;br /&gt;&lt;/div&gt;&lt;div&gt;This 12-year-old girl depicts a classic presentation of a patient with AIS, a disease characterized by both abnormal curvature and rotation of the spine from an unknown cause. This is an otherwise healthy adolescent, like most patients with idiopathic scoliosis. This patient is also female, representing the 8:1 female-to-male ratio of patients with scoliosis large enough in magnitude to require treatment. Often, concerns of both patients and parents are not of pain or dysfunction, but of cosmetic appearance. Shoulder, trunk, and flank asymmetry can be very noticeable and can significantly harm the self-image of the patient. The goals of treatment are to prevent long-term complications of cardiovascular and pulmonary dysfunction, which tend to present later with curve progression in adulthood. Surgical intervention in adults carries greater risk of morbidity.&lt;/div&gt;&lt;div&gt;&lt;br /&gt;&lt;/div&gt;&lt;div&gt;&lt;span&gt;&amp;nbsp;&amp;nbsp; &amp;nbsp;&lt;/span&gt;In addition to her deformity, the patient’s complaints of back pain should not be ignored. Although rare, physicians must first rule out potential pathologic causes of scoliosis, as AIS is a &lt;b&gt;diagnosis of exclusion.&lt;/b&gt; Both the history and physical exam, with judicious use of radiologic and laboratory studies, should be used to evaluate for other potential causes of back pain, including osteomyelitis, discitis, spondylolysis, disc herniation, or even skeletal and mesenchymal-derived tumors.&lt;/div&gt;&lt;div&gt;&lt;br /&gt;&lt;/div&gt;&lt;div&gt;&lt;span&gt;&amp;nbsp;&amp;nbsp; &amp;nbsp;&lt;/span&gt;Several red flags may arise in the workup of a patient with scoliosis that warrant further evaluation, including history of fever or constitutional symptoms, unexplained weight loss, neurologic abnormalities, pain unrelated to activity, and/or pain at night. Idiopathic scoliosis is typically characterized by &lt;b&gt;thoracic dextroscoliosis,&amp;nbsp;&lt;/b&gt;or a right-sided thoracic curve. Particular attention should be paid to &lt;b&gt;atypical curve patterns &lt;/b&gt;such as&lt;b&gt; levoscoliosis&lt;/b&gt; (left-sided curve), juvenile-onset deformities (&amp;lt; 10 years of age at time of diagnosis), &lt;b&gt;hyperkyphosis&lt;/b&gt; (a sharp posterior- facing or “hunchback” curve), and large curves without rotational deformities. Spinal deformity can be the initial presenting complaint for patients with central nervous system anomalies such as Arnold-Chiari malformations, syringomyelia, diastematomyelia, tethered cords, or central nervous system tumors. &lt;b&gt;Neuromuscular&amp;nbsp;&lt;/b&gt;&lt;b&gt;scoliosis is the second most common type of scoliosis,&lt;/b&gt; following AIS, and often occurs in patients with cerebral palsy, muscular dystrophy, and other neurologic disorders. Scoliosis may also occur secondary to underlying genetic diseases such as neurofibromatosis, Marfan syndrome, Ehlers-Danlos syndrome, osteogenesis imperfecta, or as secondary to previous spine trauma. On occasion, subclinical neuromuscular diseases such as mild cerebral palsy and Charcot Marie Tooth disease can remain unrecognized into adolescence but be an underlying cause of scoliosis. Alterations in the neurologic exam, unexplained pain, and atypical curve patterns should raise suspicion that the spinal deformity is not idiopathic. In such situations, the physician must obtain further diagnostic imaging such as magnetic resonance imaging (MRI) of the entire spine and/or ultrasound exams to evaluate for common congenital correlates to malformed vertebrae (ie, renal ultrasound, cardiac echocardiogram).&lt;/div&gt;&lt;/div&gt;&lt;div&gt;&lt;br /&gt;&lt;/div&gt;&lt;div&gt;&lt;br /&gt;&lt;/div&gt;&lt;div&gt;&lt;div&gt;&lt;div&gt;&lt;b&gt;&lt;span style=&quot;font-size: medium;&quot;&gt;APPROACH TO:&lt;/span&gt;&lt;/b&gt;&lt;/div&gt;&lt;div&gt;&lt;span style=&quot;background-color: #daebf0; color: #017087; font-size: medium;&quot;&gt;&lt;span&gt;&lt;span style=&quot;background-color: transparent;&quot;&gt;&lt;b&gt;Scoliosis&lt;/b&gt;&lt;/span&gt;&lt;b&gt;&amp;nbsp; &amp;nbsp;&lt;/b&gt;&lt;/span&gt;&lt;span style=&quot;font-weight: bold;&quot;&gt;&amp;nbsp;&amp;nbsp; &amp;nbsp;&lt;/span&gt;&lt;span style=&quot;font-weight: bold;&quot;&gt;&amp;nbsp;&amp;nbsp; &amp;nbsp;&lt;/span&gt;&lt;span style=&quot;font-weight: bold;&quot;&gt;&amp;nbsp;&amp;nbsp; &amp;nbsp;&lt;/span&gt;&lt;span style=&quot;font-weight: bold;&quot;&gt;&amp;nbsp;&amp;nbsp; &amp;nbsp;&lt;/span&gt;&lt;span style=&quot;font-weight: bold;&quot;&gt;&amp;nbsp;&amp;nbsp; &amp;nbsp;&lt;/span&gt;&lt;span style=&quot;font-weight: bold;&quot;&gt;&amp;nbsp;&amp;nbsp; &amp;nbsp;&lt;/span&gt;&lt;span style=&quot;font-weight: bold;&quot;&gt;&amp;nbsp;&amp;nbsp;&amp;nbsp;&lt;/span&gt;&lt;span style=&quot;font-weight: bold;&quot;&gt;&amp;nbsp;&lt;/span&gt;&lt;/span&gt;&lt;/div&gt;&lt;/div&gt;&lt;div&gt;&lt;br /&gt;&lt;/div&gt;&lt;div&gt;&lt;b&gt;&lt;span style=&quot;color: #017087; font-size: medium;&quot;&gt;DEFINITIONS&lt;/span&gt;&lt;/b&gt;&lt;/div&gt;&lt;/div&gt;&lt;div&gt;&lt;br /&gt;&lt;/div&gt;&lt;div&gt;&lt;div&gt;&lt;b&gt;SCOLIOSIS:&lt;/b&gt; Defined by greater than 10 degrees of curvature of the spine in the coronal plane. Although measured on a 2-dimensional x-ray, scoliosis is typically a 3-dimensional process with rotational deformities. It is commonly measured using the Cobb method.&lt;/div&gt;&lt;div&gt;&lt;b&gt;&lt;br /&gt;&lt;/b&gt;&lt;/div&gt;&lt;div&gt;&lt;b&gt;KYPHOSIS:&lt;/b&gt; A deformity in the sagittal plane characterized by apex-posterior curvature of the spine. A small degree of kyphosis is normal and should measure between 20 and 40 degrees in the thoracic spine.&lt;/div&gt;&lt;div&gt;&lt;b&gt;&lt;br /&gt;&lt;/b&gt;&lt;/div&gt;&lt;div&gt;&lt;b&gt;COBB ANGLE:&lt;/b&gt; The angle subtended by lines drawn through vertebral endplates at each point of curve inflection. Cobb angles are typically measured on standing full-length spine x-rays.&lt;/div&gt;&lt;div&gt;&lt;b&gt;&lt;br /&gt;&lt;/b&gt;&lt;/div&gt;&lt;div&gt;&lt;b&gt;PEAK GROWTH VELOCITY:&lt;/b&gt; The skeletal growth spurt usually seen early in the second decade of life. Progression of scoliosis curves is at highest risk during this time.&lt;/div&gt;&lt;div&gt;&lt;b&gt;&lt;br /&gt;&lt;/b&gt;&lt;/div&gt;&lt;div&gt;&lt;b&gt;RISSER SIGN:&lt;/b&gt; The ossification of the iliac apophysis, seen on coronal radiographs. Graded from 0 through 5, based on quartiles of the total length of the iliac crest with ossified from lateral to medial. Ossification is a marker of skeletal maturity and can thus be used to determine the likelihood of curve progression.&lt;/div&gt;&lt;/div&gt;&lt;div&gt;&lt;br /&gt;&lt;/div&gt;&lt;div&gt;&lt;br /&gt;&lt;/div&gt;&lt;div&gt;&lt;b&gt;&lt;span style=&quot;color: #017087; font-size: medium;&quot;&gt;CLINICAL APPROACH&lt;/span&gt;&lt;/b&gt;&lt;/div&gt;&lt;div&gt;&lt;br /&gt;&lt;/div&gt;&lt;div&gt;&lt;div&gt;&lt;b&gt;&lt;i&gt;&lt;span style=&quot;font-size: medium;&quot;&gt;Etiologies&lt;/span&gt;&lt;/i&gt;&lt;/b&gt;&lt;/div&gt;&lt;div&gt;&lt;br /&gt;&lt;/div&gt;&lt;div&gt;Scoliosis is defined by greater than 10 degrees of spinal curvature in the coronal plane; less than 10 degrees of curvature is termed &lt;b&gt;spinal asymmetry.&lt;/b&gt; Although it is typically observed on 2-dimensional x-rays, scoliosis is actually a &lt;b&gt;3-dimensional&amp;nbsp;&lt;/b&gt;&lt;b&gt;rotational deformity&lt;/b&gt; of the vertebral column. Although adolescent idiopathic scoliosis is the most commonly seen presentation of scoliosis, it is a &lt;b&gt;diagnosis of&amp;nbsp;&lt;/b&gt;&lt;b&gt;exclusion. &lt;/b&gt;To diagnose scoliosis, less common, but more insidious, causes must be investigated.&lt;/div&gt;&lt;div&gt;&lt;br /&gt;&lt;/div&gt;&lt;div&gt;&lt;span&gt;&amp;nbsp;&amp;nbsp; &amp;nbsp;&lt;/span&gt;The etiology of scoliosis is best described as multifactorial. Hereditary studies have shown an increased incidence of scoliosis within families, but penetrance is variable. Several genes have been implicated in the etiology for scoliosis, but clinical results are inconclusive for a single gene responsible for the condition. Similarly, hormonal and neuromuscular etiologies have been implicated, but current research has failed to demonstrate conclusive independent causation.&lt;/div&gt;&lt;div&gt;&lt;br /&gt;&lt;/div&gt;&lt;div&gt;&lt;br /&gt;&lt;/div&gt;&lt;div&gt;&lt;b&gt;&lt;i&gt;&lt;span style=&quot;font-size: medium;&quot;&gt;Clinical Presentation of Scoliosis&lt;/span&gt;&lt;/i&gt;&lt;/b&gt;&lt;/div&gt;&lt;div&gt;&lt;br /&gt;&lt;/div&gt;&lt;div&gt;The majority of patients with AIS present with complaints of cosmesis. Only in cases of severe curve magnitude (≥ 80 degrees) is evidence of cardiovascular or pulmonary dysfunction seen in adolescents. Often, patients’ curves have first been observed by classmates, physical education teachers, school nurses, or found incidentally on routine exams by pediatricians. Pain associated with scoliosis is common in adulthood but is rare in adolescence. Complete gestational and birth history as well as neurologic developmental milestone achievement should be reviewed, along with a detailed physical and neurologic exam at the initial visit. Radiographic imaging should be performed at regular intervals to monitor for progression of the curve with consideration given to the cumulative radiation dose to which the patient will be exposed.&lt;/div&gt;&lt;/div&gt;&lt;div&gt;&lt;br /&gt;&lt;/div&gt;&lt;div&gt;&lt;br /&gt;&lt;/div&gt;&lt;div&gt;&lt;b style=&quot;color: #017087;&quot;&gt;&lt;span style=&quot;font-size: medium;&quot;&gt;TREATMENT&lt;/span&gt;&lt;/b&gt;&lt;/div&gt;&lt;div&gt;&lt;br /&gt;&lt;/div&gt;&lt;div&gt;&lt;div&gt;Treatment is defined by the likelihood of curve progression during both adolescence and adulthood. Most AIS patients are healthy and active despite sometimes pronounced deformities. When discussing the natural history of scoliosis with patients and their families, the consequences of curve progression into adulthood and its effects on cardiopulmonary health must be discussed. Such dysfunctions are often absent in the asymptomatic adolescent, but may manifest later in life with significant morbidity.&lt;/div&gt;&lt;div&gt;&lt;br /&gt;&lt;/div&gt;&lt;div&gt;&lt;span&gt;&amp;nbsp;&amp;nbsp; &amp;nbsp;&lt;/span&gt;Generally, AIS patients with curves of less than 20 degrees are managed with clinical observation and intermittent radiographic follow-up if the curve is at high risk of progression. Brace treatment is indicated for AIS patients with curve magnitudes of 25 to 40 degrees and who are still growing. Patients most likely to benefit from bracing include premenarchal girls, girls or boys in or before their peak height velocity, or girls or boys with less than 50% of iliac apophysis ossification (ie, Risser stage of 0, 1, or 2). As stated, curve magnitude increases at the greatest rate during the period of peak growth velocity. Remember to discuss with patients that the realistic&amp;nbsp;&lt;b&gt;goal for brace treatment is not to correct the spinal curvature, but to prevent it&amp;nbsp;&lt;/b&gt;&lt;b&gt;from increasing in magnitude.&lt;/b&gt;&amp;nbsp;The best prognostic indicator for successful scoliosis bracing is the ability to immediately improve the prebracing radiographic curve by 50% when the first brace is applied. Braces must be worn at a minimum of 16 to 23 hours per day, and, not surprisingly, patient compliance is a considerable issue.&lt;/div&gt;&lt;/div&gt;&lt;div&gt;&lt;br /&gt;&lt;/div&gt;&lt;div class=&quot;separator&quot; style=&quot;clear: both; text-align: center;&quot;&gt;&lt;a href=&quot;https://blogger.googleusercontent.com/img/b/R29vZ2xl/AVvXsEgpXxbIleprYfGkEwfiNrmOmg2cs4CiKXdWxRJxyK-EKRzJemmKhSqGjY6VHd0BNhDnL9kzqyPxNn4nPKZO_c0zUW07xweL_6NxuRgINCi-xRKrTPfgOiOl-mSiVCr2gfm9xJBPXJXun-Hbb_R1e9LLE7FXx6nSFcjjTXaxxKBXg-PJ2Dq9PZX6uJFlVQ/s478/Treatment-of-a-scoliotic-curve.jpg&quot; style=&quot;margin-left: 1em; margin-right: 1em;&quot;&gt;&lt;img alt=&quot;Treatment of a scoliotic curve&quot; border=&quot;0&quot; data-original-height=&quot;453&quot; data-original-width=&quot;478&quot; height=&quot;379&quot; src=&quot;https://blogger.googleusercontent.com/img/b/R29vZ2xl/AVvXsEgpXxbIleprYfGkEwfiNrmOmg2cs4CiKXdWxRJxyK-EKRzJemmKhSqGjY6VHd0BNhDnL9kzqyPxNn4nPKZO_c0zUW07xweL_6NxuRgINCi-xRKrTPfgOiOl-mSiVCr2gfm9xJBPXJXun-Hbb_R1e9LLE7FXx6nSFcjjTXaxxKBXg-PJ2Dq9PZX6uJFlVQ/w400-h379/Treatment-of-a-scoliotic-curve.jpg&quot; title=&quot;Treatment of a scoliotic curve&quot; width=&quot;400&quot; /&gt;&lt;/a&gt;&lt;/div&gt;&lt;div&gt;&lt;div&gt;&lt;b&gt;&lt;span style=&quot;color: #017087;&quot;&gt;Figure 28–1.&lt;/span&gt;&lt;/b&gt; Treatment of a scoliotic curve by instrumentation and fusion. (&lt;b&gt;A&lt;/b&gt;) Preoperative view and&lt;/div&gt;&lt;div&gt;(&lt;b&gt;B&lt;/b&gt;) postoperative view. (Reproduced, with permission, from Skinner HB. &lt;i&gt;Current Diagnosis &amp;amp; Treatment in Orthopedics&lt;/i&gt;. 4th ed. New York, NY: McGraw-Hill; 2006:Fig. 11-36.)&lt;/div&gt;&lt;/div&gt;&lt;div&gt;&lt;br /&gt;&lt;/div&gt;&lt;div&gt;&lt;div&gt;&lt;span&gt;&amp;nbsp;&amp;nbsp; &amp;nbsp;&lt;/span&gt;AIS curves greater than 50 degrees are a potential indication for spinal fusion surgery. Natural history studies of AIS suggest the greatest increased risk of progression into adulthood when curves are beyond 50 degrees. The primary goal of AIS surgical treatment is to halt the progression of deformity by achieving a solid fusion of the spine. Additional surgical goals include the sparing of spinal motion segments wherever possible, obtaining a pain-free spine, and correcting as much of the curvature as is safely possible. Surgical spinal fusion typically involves a combination of bone grafting and instrumentation to fix the curved spine into a rigid structure. Surgical approaches may be posterior, anterior, or combined in cases of severe disease. Instrumentation used to hold the fusion may include rods, pedicle screws, hooks, wires, and other implants ( Figure 28–1 ).&lt;/div&gt;&lt;/div&gt;&lt;div&gt;&lt;br /&gt;&lt;/div&gt;&lt;div&gt;&lt;br /&gt;&lt;/div&gt;&lt;div&gt;&lt;div&gt;&lt;b&gt;&lt;i&gt;&lt;span style=&quot;font-size: medium;&quot;&gt;Complications&lt;/span&gt;&lt;/i&gt;&lt;/b&gt;&lt;/div&gt;&lt;div&gt;&lt;br /&gt;&lt;/div&gt;&lt;div&gt;Spinal fusion procedures are often long, painful, and highly morbid procedures with significant blood loss, requiring transfusion and/or the use of intraoperative blood salvage techniques. Early wound infection (1%-2%), pseudoarthrosis (1%-2%), painful prominent hardware, and hardware breakage are described complications. The most devastating but rare complication is iatrogenic neurologic injury. Intraoperative neurologic monitoring may help prevent this. A &lt;b&gt;crankshaft phenomenon&lt;/b&gt; may occur when an immature spine is fused posteriorly, but continues to grow anteriorly, causing the expanding spine to spiral around the posterior fusion. This may be prevented with concomitant anterior fusion.&lt;/div&gt;&lt;/div&gt;&lt;div&gt;&lt;br /&gt;&lt;/div&gt;&lt;div&gt;&lt;br /&gt;&lt;/div&gt;&lt;div&gt;&lt;b&gt;&lt;span style=&quot;color: #017087; font-size: medium;&quot;&gt;&lt;u&gt;COMPREHENSION QUESTIONS&lt;/u&gt;&lt;/span&gt;&lt;/b&gt;&lt;/div&gt;&lt;div&gt;&lt;br /&gt;&lt;/div&gt;&lt;div&gt;&lt;div&gt;28.1 A 10-year-old healthy girl is referred to your office after testing positive for scoliosis during a schoolwide screening. Physical exam is unremarkable except for absence of her abdominal reflex on the right side. Standing 36-in scoliosis radiographs show a 25-degree levoscoliosis curve with minimal rotation and hyperkyphosis. She is Risser 0 developmental stage on x-ray. What is the next step in management?&lt;/div&gt;&lt;/div&gt;&lt;blockquote style=&quot;border: none; margin: 0px 0px 0px 40px; padding: 0px; text-align: left;&quot;&gt;&lt;div&gt;&lt;div&gt;A. Reassure and observe with clinical and radiographic examination in 3 to 4 months&lt;/div&gt;&lt;/div&gt;&lt;div&gt;&lt;div&gt;B. Fitting for a thoracolumbosacral orthosis (TLSO)&lt;/div&gt;&lt;/div&gt;&lt;div&gt;&lt;div&gt;C. Obtain a bone scan&lt;/div&gt;&lt;/div&gt;&lt;div&gt;&lt;div&gt;D. Obtain an MRI of the entire central nervous system&lt;/div&gt;&lt;/div&gt;&lt;div&gt;&lt;div&gt;E. Book patient for spinal fusion after obtaining appropriate consent&lt;/div&gt;&lt;/div&gt;&lt;/blockquote&gt;&lt;div&gt;&lt;div&gt;&lt;br /&gt;&lt;/div&gt;&lt;div&gt;28.2 A 15-year-old male athlete presents for evaluation of shoulder asymmetry pointed out by his football teammates. His mother had never noticed the discrepancy until now but is very concerned. The boy has a 15-degree right-sided trunk rotation on the Adam forward-bending test. His neurologic examination is unremarkable. Thirty-six inch PA standing scoliosis radiographs show a 60-degree right thoracic curve and shoulder elevation 2 cm greater on the right than left. What is the next step in management?&lt;/div&gt;&lt;/div&gt;&lt;blockquote style=&quot;border: none; margin: 0px 0px 0px 40px; padding: 0px; text-align: left;&quot;&gt;&lt;div&gt;&lt;div&gt;A. Trial nonoperative treatment with a thoracolumbosacral orthosis (TLSO)&lt;/div&gt;&lt;/div&gt;&lt;div&gt;&lt;div&gt;B. Reassure and observe patient with repeat clinical and radiographic evaluation in 3 to 4 months&lt;/div&gt;&lt;/div&gt;&lt;div&gt;&lt;div&gt;C. Obtain an MRI of the entire central nervous system&lt;/div&gt;&lt;/div&gt;&lt;div&gt;&lt;div&gt;D. Obtain a bone scan and laboratory studies for evidence of underlying pathology including erythrocyte sedimentation rate, C-reactive protein, and complete blood count&lt;/div&gt;&lt;/div&gt;&lt;div&gt;&lt;div&gt;E. Book patient for spinal fusion after obtaining appropriate consent&lt;/div&gt;&lt;/div&gt;&lt;/blockquote&gt;&lt;div&gt;&lt;div&gt;&lt;br /&gt;&lt;/div&gt;&lt;div&gt;28.3 A 15-year-old girl has asymptomatic adolescent idiopathic scoliosis. Her parents ask you how debilitated she might be compared with her peers who do not have scoliosis. Which of the following is true regarding the natural history of idiopathic scoliosis?&lt;/div&gt;&lt;/div&gt;&lt;blockquote style=&quot;border: none; margin: 0px 0px 0px 40px; padding: 0px; text-align: left;&quot;&gt;&lt;div&gt;&lt;div&gt;A. Difficulty with pregnancy&lt;/div&gt;&lt;/div&gt;&lt;div&gt;&lt;div&gt;B. Increased risk of developing cancer&lt;/div&gt;&lt;/div&gt;&lt;div&gt;&lt;div&gt;C. Acute or chronic back pain&lt;/div&gt;&lt;/div&gt;&lt;div&gt;&lt;div&gt;D. Athletic limitations&lt;/div&gt;&lt;/div&gt;&lt;/blockquote&gt;&lt;div&gt;&lt;br /&gt;&lt;/div&gt;&lt;div&gt;&lt;br /&gt;&lt;/div&gt;&lt;div&gt;&lt;b&gt;&lt;span style=&quot;color: #017087; font-size: medium;&quot;&gt;&lt;u&gt;ANSWERS&lt;/u&gt;&lt;/span&gt;&lt;/b&gt;&lt;/div&gt;&lt;div&gt;&lt;br /&gt;&lt;/div&gt;&lt;div&gt;&lt;div&gt;28.1 &lt;b&gt;D.&lt;/b&gt; Atypical curve patterns require MRI evaluation of the spine and central nervous system for workup of underlying pathology. Bone scans, although useful for diagnosing infection or subacute bony injury, are not sensitive to other potential pathology etiologies. Bracing and observation are inappropriate without first ruling out a pathologic etiology.&lt;/div&gt;&lt;div&gt;&lt;br /&gt;&lt;/div&gt;&lt;div&gt;28.2 &lt;b&gt;E.&lt;/b&gt; Spinal fusion is indicated in curves measuring greater than 50 degrees. Although workup with MRI or laboratory studies is necessary when patients have evidence of pathologic etiology for scoliosis, patients without concerning symptoms do not require such an extensive workup. This patient’s curve is too great in magnitude to be managed with observation or with bracing.&lt;/div&gt;&lt;div&gt;&lt;br /&gt;&lt;/div&gt;&lt;div&gt;28.3 &lt;b&gt;C.&lt;/b&gt; Scoliotic patients have been found to have more acute or chronic back pain, as well as cosmetic concerns. Pregnancy complications, athletic limitations, and an increased cancer risk are not associated with AIS.&lt;/div&gt;&lt;/div&gt;&lt;div&gt;&lt;br /&gt;&lt;/div&gt;&lt;div&gt;&lt;b style=&quot;background-color: #4093a5;&quot;&gt;&amp;nbsp; &amp;nbsp;&lt;span style=&quot;font-size: medium;&quot;&gt;&amp;nbsp;&lt;/span&gt;&lt;span style=&quot;color: white;&quot;&gt;&lt;span style=&quot;font-size: medium;&quot;&gt;CLINICAL PEARLS&lt;/span&gt;&lt;span&gt;&amp;nbsp; &amp;nbsp;&amp;nbsp;&lt;/span&gt;&lt;/span&gt;&lt;/b&gt;&lt;/div&gt;&lt;div&gt;&lt;table border=&quot;1&quot; cellpadding=&quot;0&quot; cellspacing=&quot;0&quot; class=&quot;MsoTableGrid&quot; style=&quot;border-collapse: collapse; border: none; mso-border-alt: solid windowtext .5pt; mso-padding-alt: 0in 5.4pt 0in 5.4pt; mso-yfti-tbllook: 1184;&quot;&gt;
  201. &lt;tbody&gt;&lt;tr style=&quot;height: 19.15pt; mso-yfti-firstrow: yes; mso-yfti-irow: 0; mso-yfti-lastrow: yes;&quot;&gt;
  202.  &lt;td style=&quot;background: rgb(221, 237, 239); border-left: solid #017087 1.0pt; border: 1pt solid rgb(64, 147, 165); height: 19.15pt; padding: 0in 5.4pt; width: 433.7pt;&quot; valign=&quot;top&quot; width=&quot;578&quot;&gt;
  203.  &lt;div style=&quot;line-height: normal; margin-bottom: 0in;&quot;&gt;&lt;span style=&quot;color: #017087;&quot;&gt;&lt;br /&gt;&lt;/span&gt;&lt;/div&gt;&lt;div style=&quot;line-height: normal; margin-bottom: 0in;&quot;&gt;&lt;span style=&quot;color: #017087;&quot;&gt;►&lt;/span&gt;&amp;nbsp;Adolescent idiopathic scoliosis is a diagnosis of exclusion, meaning physicians must rule out potential congenital, neuromuscular, traumatic, or syndromic etiologies before making the diagnosis.&lt;/div&gt;&lt;div style=&quot;line-height: normal; margin-bottom: 0in;&quot;&gt;&lt;span style=&quot;color: #017087;&quot;&gt;&lt;br /&gt;&lt;/span&gt;&lt;/div&gt;&lt;div style=&quot;line-height: normal; margin-bottom: 0in;&quot;&gt;&lt;span style=&quot;color: #017087;&quot;&gt;►&lt;/span&gt;&amp;nbsp;The risk of scoliosis curve progression is greatest during early adolescence when the peak growth velocity occurs.&lt;/div&gt;&lt;div style=&quot;line-height: normal; margin-bottom: 0in;&quot;&gt;&lt;span style=&quot;color: #017087;&quot;&gt;&lt;br /&gt;&lt;/span&gt;&lt;/div&gt;&lt;div style=&quot;line-height: normal; margin-bottom: 0in;&quot;&gt;&lt;span style=&quot;color: #017087;&quot;&gt;►&lt;/span&gt;&amp;nbsp;Among many non-AIS causes of back pain in adolescents are discitis, herniated disks, osteoid osteoma, lymphoma, and spondylolysis.&lt;/div&gt;&lt;div style=&quot;line-height: normal; margin-bottom: 0in;&quot;&gt;&lt;span style=&quot;color: #017087;&quot;&gt;&lt;br /&gt;&lt;/span&gt;&lt;/div&gt;&lt;div style=&quot;line-height: normal; margin-bottom: 0in;&quot;&gt;&lt;span style=&quot;color: #017087;&quot;&gt;►&lt;/span&gt;&amp;nbsp;Scoliosis bracing is indicated for adolescents in their peak height velocity or those who have not yet reached their peak height velocity, as well as patients with significant curve magnitudes (&amp;gt; 25-40 degrees) and those in Risser 0 through 2 stages of skeletal maturity.&lt;/div&gt;&lt;div style=&quot;line-height: normal; margin-bottom: 0in;&quot;&gt;&lt;span style=&quot;color: #017087;&quot;&gt;&lt;br /&gt;&lt;/span&gt;&lt;/div&gt;&lt;div style=&quot;line-height: normal; margin-bottom: 0in;&quot;&gt;&lt;span style=&quot;color: #017087;&quot;&gt;►&lt;/span&gt;&amp;nbsp;Spinal fusion surgery is generally indicated for patients with progressive curves greater than 50 degrees.&lt;/div&gt;&lt;div style=&quot;line-height: normal; margin-bottom: 0in;&quot;&gt;&lt;br /&gt;&lt;/div&gt;
  204.  &lt;/td&gt;
  205. &lt;/tr&gt;
  206. &lt;/tbody&gt;&lt;/table&gt;&lt;br /&gt;&lt;/div&gt;
  207.  
  208. &lt;details close=&quot;&quot;&gt;
  209.  &lt;summary&gt;&lt;b&gt;&lt;span style=&quot;color: #00788b;&quot;&gt;REFERENCES&lt;/span&gt;&lt;/b&gt;&lt;/summary&gt;
  210.  &lt;p&gt;Cobb JR. Outline for the study of scoliosis. In: Thomson JEM, Boount WP, eds. The American Academy
  211. of Orthopaedic Surgeons. Instructional Course Lectures. Ann Arbor, MI: JW Edwards; 1948;5:261-275.&amp;nbsp;&lt;/p&gt;&lt;p&gt;Miller MD, ed. Pediatric orthopaedics. In: Review of Orthopaedics . 5th ed. Philadelphia: Saunders Elsevier;
  212. 2008:198-244.&amp;nbsp;&lt;/p&gt;&lt;p&gt;Sanders JO, Browne RH, Cooney TE, Finegold DN, McConnell SJ, Margraf SA. Correlates of the peak
  213. height velocity in girls with idiopathic scoliosis. Spine. 2006;31:2289-2295.&amp;nbsp;&lt;/p&gt;&lt;p&gt;Song KM, Little DG. Peak height velocity as a maturity indicator for males with idiopathic scoliosis.
  214. J Pediatr Orthop. 2000;20:286-288.&lt;/p&gt;
  215. &lt;/details&gt;&lt;/div&gt;</content><link rel='replies' type='application/atom+xml' href='https://medical-phd.blogspot.com/feeds/7499594277683878295/comments/default' title='Post Comments'/><link rel='replies' type='text/html' href='https://medical-phd.blogspot.com/2022/03/adolescent-idiopathic-scoliosis-case-file.html#comment-form' title='0 Comments'/><link rel='edit' type='application/atom+xml' href='https://www.blogger.com/feeds/3302710210518459416/posts/default/7499594277683878295'/><link rel='self' type='application/atom+xml' href='https://www.blogger.com/feeds/3302710210518459416/posts/default/7499594277683878295'/><link rel='alternate' type='text/html' href='https://medical-phd.blogspot.com/2022/03/adolescent-idiopathic-scoliosis-case-file.html' title='Adolescent Idiopathic Scoliosis Case File'/><author><name>Medical Group</name><uri>http://www.blogger.com/profile/01780656800578397393</uri><email>noreply@blogger.com</email><gd:image rel='http://schemas.google.com/g/2005#thumbnail' width='32' height='32' src='//blogger.googleusercontent.com/img/b/R29vZ2xl/AVvXsEiMuROkBDv6yw4SmxGqRA5BPQhOXSwFld-Gufmy4kUqfpoAGQRl7cv-3naew6wjAnCjRFSe_FKX8z7CdE6cFKxxBpnqBuwUTSr-15xd3mFGcr_wJOX2y73G_zJCGND98Q/s220/Final-logo.jpg'/></author><media:thumbnail xmlns:media="http://search.yahoo.com/mrss/" url="https://blogger.googleusercontent.com/img/b/R29vZ2xl/AVvXsEgpXxbIleprYfGkEwfiNrmOmg2cs4CiKXdWxRJxyK-EKRzJemmKhSqGjY6VHd0BNhDnL9kzqyPxNn4nPKZO_c0zUW07xweL_6NxuRgINCi-xRKrTPfgOiOl-mSiVCr2gfm9xJBPXJXun-Hbb_R1e9LLE7FXx6nSFcjjTXaxxKBXg-PJ2Dq9PZX6uJFlVQ/s72-w400-h379-c/Treatment-of-a-scoliotic-curve.jpg" height="72" width="72"/><thr:total>0</thr:total><georss:featurename>United States</georss:featurename><georss:point>37.09024 -95.712891</georss:point><georss:box>8.780006163821156 -130.869141 65.400473836178847 -60.556641</georss:box></entry><entry><id>tag:blogger.com,1999:blog-3302710210518459416.post-7274488063108359104</id><published>2022-03-25T12:25:00.001-07:00</published><updated>2022-03-25T12:25:10.979-07:00</updated><category scheme="http://www.blogger.com/atom/ns#" term="Case Files"/><category scheme="http://www.blogger.com/atom/ns#" term="Orthopaedic Surgery Case File"/><title type='text'>Lumbar Burst Fracture Case File</title><content type='html'>&lt;div class=&quot;scrollbox&quot;&gt;
  216.  
  217. &lt;div style=&quot;text-align: left;&quot;&gt;&lt;b&gt;&lt;span style=&quot;font-size: large;&quot;&gt;Lumbar Burst Fracture Case File&lt;/span&gt;&lt;/b&gt;&lt;/div&gt;&lt;div&gt;Eugene C. Toy, MD, Andrew J. Rosenbaum, MD, Timothy T. Roberts, MD, Joshua S. Dines, MD&lt;/div&gt;&lt;div&gt;&lt;br /&gt;&lt;/div&gt;&lt;div&gt;&lt;b&gt;&lt;span style=&quot;color: #017087; font-size: medium;&quot;&gt;CASE 27&lt;/span&gt;&lt;/b&gt;&lt;/div&gt;&lt;div&gt;&lt;div&gt;A 34-year-old male construction worker arrives in the emergency department (ED) after a 25-foot fall at a construction site 3 hours ago. The patient landed on his feet and then fell backwards and was unable to stand after the accident. He is complaining of right ankle and low back pain, as well as numbness and tingling in both legs. He has no prior medical history. On exam, the patient is hemodynamically stable with vital signs within normal limits. His right heel is swollen and tender, but the skin is intact. He has midline tenderness in the upper lumbar spine and substantially decreased sensation to light touch in his bilateral proximal thighs and extending distally. Rectal exam demonstrates decreased rectal tone, as well as decreased but intact perianal sensation. Bulbocavernosus reflex is absent, as are his bilateral patella and Achilles reflexes. Plain films and a computed tomography (CT) are acquired by the ED and are shown in Figure 27–1 .&lt;/div&gt;&lt;/div&gt;&lt;div&gt;&lt;br /&gt;&lt;/div&gt;&lt;div class=&quot;separator&quot; style=&quot;clear: both; text-align: center;&quot;&gt;&lt;a href=&quot;https://blogger.googleusercontent.com/img/b/R29vZ2xl/AVvXsEjccPWgwLZGBFsZjdAxV1L8SsSPmZE8btrJh0oqn4rN4eC-UPlsgSXwll2tVjki0BggLhWGCYmBce_ed0CSb5_ubACD_4DdR_OXfT0T0VB9IokiVbZWTI3SWrrFGilRnYm2Avb2ARUuVbLAuhQ1luvfIEcoY9k8vTzMXDcrLEie8aB9HM8Hx-QccESqYw/s699/Lateral-plain-radiographs.jpg&quot; style=&quot;margin-left: 1em; margin-right: 1em;&quot;&gt;&lt;img alt=&quot;Lateral plain radiographs&quot; border=&quot;0&quot; data-original-height=&quot;306&quot; data-original-width=&quot;699&quot; height=&quot;280&quot; src=&quot;https://blogger.googleusercontent.com/img/b/R29vZ2xl/AVvXsEjccPWgwLZGBFsZjdAxV1L8SsSPmZE8btrJh0oqn4rN4eC-UPlsgSXwll2tVjki0BggLhWGCYmBce_ed0CSb5_ubACD_4DdR_OXfT0T0VB9IokiVbZWTI3SWrrFGilRnYm2Avb2ARUuVbLAuhQ1luvfIEcoY9k8vTzMXDcrLEie8aB9HM8Hx-QccESqYw/w640-h280/Lateral-plain-radiographs.jpg&quot; title=&quot;Lateral plain radiographs&quot; width=&quot;640&quot; /&gt;&lt;/a&gt;&lt;/div&gt;&lt;div&gt;&lt;div&gt;&lt;b&gt;&lt;span style=&quot;color: #017087;&quot;&gt;Figure 27–1.&lt;/span&gt;&lt;/b&gt; Lateral plain radiographs ( left ) of the lumbar spine and an axial CT cut ( right ) of the L1&lt;/div&gt;&lt;div&gt;vertebral body. (Courtesy of Timothy T. Roberts, MD)&lt;/div&gt;&lt;/div&gt;&lt;div&gt;&lt;br /&gt;&lt;/div&gt;&lt;div&gt;&lt;div&gt;&lt;span style=&quot;color: #017087;&quot;&gt;►&lt;/span&gt;&amp;nbsp;What type of fracture is seen in Figure 27–1?&lt;/div&gt;&lt;div&gt;&lt;span style=&quot;color: #017087;&quot;&gt;►&lt;/span&gt;&amp;nbsp;What is the most likely mechanism of injury for this fracture?&lt;/div&gt;&lt;div&gt;&lt;span style=&quot;color: #017087;&quot;&gt;►&lt;/span&gt;&amp;nbsp;What is the next diagnostic step?&lt;/div&gt;&lt;/div&gt;&lt;div&gt;&lt;br /&gt;&lt;/div&gt;&lt;div&gt;&lt;br /&gt;&lt;/div&gt;&lt;div&gt;&lt;div&gt;&lt;div&gt;&lt;b&gt;&lt;span style=&quot;font-size: medium;&quot;&gt;ANSWER TO CASE 27:&lt;/span&gt;&lt;/b&gt;&lt;/div&gt;&lt;div&gt;&lt;span style=&quot;background-color: #daebf0; font-size: medium;&quot;&gt;&lt;span&gt;&lt;span style=&quot;background-color: transparent;&quot;&gt;&lt;span style=&quot;color: #017087;&quot;&gt;&lt;b&gt;Lumbar Burst Fracture&lt;/b&gt;&lt;/span&gt;&lt;/span&gt;&lt;b&gt;&amp;nbsp;&amp;nbsp;&lt;/b&gt;&lt;/span&gt;&lt;span style=&quot;font-weight: bold;&quot;&gt;&amp;nbsp;&amp;nbsp; &amp;nbsp;&lt;/span&gt;&lt;span style=&quot;font-weight: bold;&quot;&gt;&amp;nbsp;&amp;nbsp; &amp;nbsp;&lt;/span&gt;&lt;span style=&quot;font-weight: bold;&quot;&gt;&amp;nbsp;&amp;nbsp; &amp;nbsp;&lt;/span&gt;&lt;span style=&quot;font-weight: bold;&quot;&gt;&amp;nbsp;&amp;nbsp; &amp;nbsp;&lt;/span&gt;&lt;span style=&quot;font-weight: bold;&quot;&gt;&amp;nbsp;&amp;nbsp; &amp;nbsp;&lt;/span&gt;&lt;span style=&quot;font-weight: bold;&quot;&gt;&amp;nbsp;&amp;nbsp; &amp;nbsp;&lt;/span&gt;&lt;span style=&quot;font-weight: bold;&quot;&gt;&amp;nbsp; &amp;nbsp;&amp;nbsp;&lt;/span&gt;&lt;/span&gt;&lt;/div&gt;&lt;/div&gt;&lt;div&gt;&lt;br /&gt;&lt;/div&gt;&lt;/div&gt;&lt;div&gt;&lt;div&gt;&lt;i&gt;Summary&lt;/i&gt;: A 34-year-old healthy male construction worker presents to the ED with apparent injuries to his right heel and lower back after a fall from 25 feet. He landed in an upright, standing position. Furthermore, he has evidence of loss of sensation and tingling in his legs and perineal region and substantial weakness in both legs. The lack of a bulbocavernosus reflex (ie, the reflexive contraction of the anal sphincter when the glans penis is pinched or a Foley catheter is pulled) means that the patient is in spinal shock. He was unable to walk after his injury, which may be secondary to spinal cord injury and/or an additional injury to his heel.&lt;/div&gt;&lt;div&gt;&lt;ul style=&quot;text-align: left;&quot;&gt;&lt;li&gt;&lt;b&gt;Spine fracture:&lt;/b&gt; This patient has a lumbar spine burst fracture.&lt;/li&gt;&lt;li&gt;&lt;b&gt;Common mechanism of injury:&lt;/b&gt; Lumbar spine burst fractures are commonly the result of high energy axial loading, most often from a fall from a height or a motor vehicle collision.&lt;/li&gt;&lt;li&gt;&lt;b&gt;Next diagnostic step:&lt;/b&gt; Neurologic changes in the setting of spinal fracture should warrant magnetic resonance imaging (MRI) to evaluate involvement of the spinal cord and/or roots. Given the high-energy fall, this patient will also need a minimum of an anteroposterior (AP) or posteroanterior (PA) chest x-ray, an AP pelvis x-ray, C-spine imaging (x-rays and/or CT scans), and x-rays of the right ankle, given the heel tenderness and swelling on examination.&lt;/li&gt;&lt;/ul&gt;&lt;/div&gt;&lt;/div&gt;&lt;div&gt;&lt;br /&gt;&lt;/div&gt;&lt;div&gt;&lt;br /&gt;&lt;/div&gt;&lt;div&gt;&lt;div&gt;&lt;b&gt;&lt;span style=&quot;color: #017087; font-size: medium;&quot;&gt;ANALYSIS&lt;/span&gt;&lt;/b&gt;&lt;/div&gt;&lt;div&gt;&lt;b&gt;&lt;span style=&quot;font-size: medium;&quot;&gt;&lt;i&gt;Objectives&lt;/i&gt;&lt;/span&gt;&lt;/b&gt;&lt;/div&gt;&lt;/div&gt;&lt;div&gt;&lt;div&gt;&lt;ol style=&quot;text-align: left;&quot;&gt;&lt;li&gt;Learn how to grade spinal cord injury with regard to sensation and muscle strength.&lt;/li&gt;&lt;li&gt;Understand the definition, diagnosis, and prognosis of spinal shock, and understand its role in the management of patients with acute neurologic injuries.&lt;/li&gt;&lt;li&gt;Know the diagnostic approach to acute injuries of the lumbar spine, and identify “red flags” for serious injuries requiring immediate surgical intervention.&lt;/li&gt;&lt;/ol&gt;&lt;/div&gt;&lt;/div&gt;&lt;div&gt;&lt;br /&gt;&lt;/div&gt;&lt;div&gt;&lt;br /&gt;&lt;/div&gt;&lt;div&gt;&lt;div&gt;&lt;i&gt;&lt;b&gt;&lt;span style=&quot;font-size: medium;&quot;&gt;Considerations&lt;/span&gt;&lt;/b&gt;&lt;/i&gt;&lt;/div&gt;&lt;div&gt;&lt;br /&gt;&lt;/div&gt;&lt;div&gt;This 34-year-old male sustained an injury to his lumbar spine, based on both the presence of tenderness in the lumbar region and his motor and sensory examination. Additional injuries must be excluded through imaging of the entire spinal axis. Although there are multiple types of spine injuries that could result in the neurologic deficits seen in this patient, the type of mechanism (axial load) and level of neurologic deficit suggest that he has sustained lumbar burst fracture at or around the region of L1. This is confirmed by the lateral radiograph and axial CT shown in Figure 27–1.&lt;/div&gt;&lt;div&gt;&lt;br /&gt;&lt;/div&gt;&lt;div&gt;&lt;span&gt;&amp;nbsp;&amp;nbsp; &amp;nbsp;&lt;/span&gt;The presence of a neurologic deficit suggests that fracture fragments or displaced soft tissues have compressed or directly injured the spinal cord and/or its roots. This patient’s injury is at the L1 level; the spinal cord typically ends somewhere between the top of L1 and bottom of L2, so this patient could have an injury either to the end of his spinal cord (known as the conus medullaris) or to the nerve roots&lt;/div&gt;&lt;div&gt;traveling to the legs from the conus medullaris (known as the cauda equina). Because the patient demonstrates evidence of spinal shock, however, we can reason that he has likely suffered injury to his conus medullaris. Spinal shock does not occur at injury levels distal to the cord.&lt;/div&gt;&lt;/div&gt;&lt;div&gt;&lt;br /&gt;&lt;/div&gt;&lt;div&gt;&lt;div&gt;&lt;span&gt;&amp;nbsp;&amp;nbsp; &amp;nbsp;&lt;/span&gt;Several other orthopaedic injuries are characteristic of high-energy axial loading mechanisms, including calcaneus fractures, femoral shaft and neck fractures, vertical shear pelvis fractures, and lumbar spine injuries. This patient has a swollen and tender heel, likely consistent with a calcaneus fracture. Further workup and management of this extremity injury is also required.&lt;/div&gt;&lt;/div&gt;&lt;div&gt;&lt;br /&gt;&lt;/div&gt;&lt;div&gt;&lt;br /&gt;&lt;/div&gt;&lt;div&gt;&lt;div&gt;&lt;div&gt;&lt;b&gt;&lt;span style=&quot;font-size: medium;&quot;&gt;APPROACH TO:&lt;/span&gt;&lt;/b&gt;&lt;/div&gt;&lt;div&gt;&lt;span style=&quot;background-color: #daebf0; color: #017087; font-size: medium;&quot;&gt;&lt;span&gt;&lt;span style=&quot;background-color: transparent;&quot;&gt;&lt;b&gt;Lumbar Spine Fracture&lt;/b&gt;&lt;/span&gt;&lt;b&gt;&amp;nbsp; &amp;nbsp;&amp;nbsp;&lt;/b&gt;&lt;/span&gt;&lt;span style=&quot;font-weight: bold;&quot;&gt;&amp;nbsp;&amp;nbsp; &amp;nbsp;&lt;/span&gt;&lt;span style=&quot;font-weight: bold;&quot;&gt;&amp;nbsp;&amp;nbsp; &amp;nbsp;&lt;/span&gt;&lt;span style=&quot;font-weight: bold;&quot;&gt;&amp;nbsp;&amp;nbsp; &amp;nbsp;&lt;/span&gt;&lt;span style=&quot;font-weight: bold;&quot;&gt;&amp;nbsp;&amp;nbsp; &amp;nbsp;&lt;/span&gt;&lt;span style=&quot;font-weight: bold;&quot;&gt;&amp;nbsp;&amp;nbsp; &amp;nbsp;&lt;/span&gt;&lt;span style=&quot;font-weight: bold;&quot;&gt;&amp;nbsp;&amp;nbsp; &amp;nbsp;&lt;/span&gt;&lt;span style=&quot;font-weight: bold;&quot;&gt;&amp;nbsp;&amp;nbsp;&amp;nbsp;&lt;/span&gt;&lt;span style=&quot;font-weight: bold;&quot;&gt;&amp;nbsp;&lt;/span&gt;&lt;/span&gt;&lt;/div&gt;&lt;/div&gt;&lt;div&gt;&lt;br /&gt;&lt;/div&gt;&lt;div&gt;&lt;b&gt;&lt;span style=&quot;color: #017087; font-size: medium;&quot;&gt;DEFINITIONS&lt;/span&gt;&lt;/b&gt;&lt;/div&gt;&lt;/div&gt;&lt;div&gt;&lt;br /&gt;&lt;/div&gt;&lt;div&gt;&lt;div&gt;&lt;b&gt;CONUS MEDULLARIS:&lt;/b&gt; The caudal end of the spinal cord, beneath which nerves to the lower extremities and pelvis travel in their individual roots. The conus medullaris is typically located between L1 and L2, although substantial variation in its location emphasizes the importance of locating this structure for every patient undergoing surgery on the upper lumbar spine to verify its location.&lt;/div&gt;&lt;div&gt;&lt;b&gt;&lt;br /&gt;&lt;/b&gt;&lt;/div&gt;&lt;div&gt;&lt;b&gt;CAUDA EQUINA: &lt;/b&gt;The collection of nerve roots beneath the conus medullaris that transmit signals from the spinal cord to the lower extremities and pelvis. The nerve roots making up the cauda equina are independently mobile and can thus tolerate a greater degree of spinal canal compression before injury when compared with spinal levels adjacent to the spinal cord itself.&lt;/div&gt;&lt;div&gt;&lt;b&gt;&lt;br /&gt;&lt;/b&gt;&lt;/div&gt;&lt;div&gt;&lt;b&gt;BURST FRACTURE:&lt;/b&gt; A type of vertebral fracture characterized by injuries to 2 or 3 “columns” of the vertebrae: the &lt;b&gt;anterior column,&lt;/b&gt; including the anterior two thirds of the vertebral body and the anterior longitudinal ligament; the &lt;b&gt;middle column,&amp;nbsp;&lt;/b&gt;including the posterior third of the vertebral body and the posterior longitudinal ligament; and the &lt;b&gt;posterior column,&lt;/b&gt; including the facet joints, lamina, spinous processes, and interspinous ligaments. Burst fractures may injure the neural elements if the posterior aspect of the vertebral body is pushed backward into the spinal canal. This is known as &lt;b&gt;retropulsion.&lt;/b&gt;&lt;/div&gt;&lt;div&gt;&lt;b&gt;&lt;br /&gt;&lt;/b&gt;&lt;/div&gt;&lt;div&gt;&lt;b&gt;COMPRESSION FRACTURE:&lt;/b&gt; Similar to burst fractures, these axial loading injuries involve fracture of the vertebral body; however, compression fractures only involve the anterior column or anterior portion of the vertebral body. They are inherently more stable than burst fractures and do not typically result in neurologic deficit, as the fracture does not propagate into the spinal canal.&lt;/div&gt;&lt;div&gt;&lt;b&gt;&lt;br /&gt;&lt;/b&gt;&lt;/div&gt;&lt;div&gt;&lt;b&gt;CHANCE FRACTURE:&lt;/b&gt; A spinal injury in which a flexion-distraction mechanism results in injury to all 3 spinal columns. Injuries may propagate through the vertebral bony elements or result in partial or complete ligamentous disruption, with forces instead tearing through the interspinous ligaments and disc space. These injuries have a high incidence of both intra-abdominal and neurologic injuries and are associated with seatbelt restraints during violent deceleration.&lt;/div&gt;&lt;/div&gt;&lt;div&gt;&lt;br /&gt;&lt;/div&gt;&lt;div&gt;&lt;div&gt;&lt;b&gt;SPINAL SHOCK:&lt;/b&gt; A temporary state of paralysis, sensory loss, and complete absence of reflexes below the level of a spinal cord injury. Spinal shock typically lasts 24 to 48 hours and ends when cord-mediated reflexes return, specifically the bulbocavernosus reflex.&lt;/div&gt;&lt;/div&gt;&lt;div&gt;&lt;br /&gt;&lt;/div&gt;&lt;div&gt;&lt;br /&gt;&lt;/div&gt;&lt;div&gt;&lt;b&gt;&lt;span style=&quot;color: #017087; font-size: medium;&quot;&gt;CLINICAL APPROACH&lt;/span&gt;&lt;/b&gt;&lt;/div&gt;&lt;div&gt;&lt;br /&gt;&lt;/div&gt;&lt;div&gt;&lt;div&gt;&lt;b&gt;&lt;i&gt;&lt;span style=&quot;font-size: medium;&quot;&gt;History and Physical Exam&lt;/span&gt;&lt;/i&gt;&lt;/b&gt;&lt;/div&gt;&lt;div&gt;&lt;br /&gt;&lt;/div&gt;&lt;div&gt;Lumbar spine trauma is often the result of high-energy mechanisms, and thus all patients should undergo a complete and thorough trauma evaluation. After the patient has been stabilized—both hemodynamically and physically—a complete history must be obtained, including the timing and mechanism of injury, symptoms immediately after the injury and on arrival to the ED, and whether the patient was moved after the initial injury. The practitioner should be wary of progressive neurologic changes, both since the initial injury and throughout the workup period in the ED. Serial examinations are essential.&lt;/div&gt;&lt;/div&gt;&lt;div&gt;&lt;br /&gt;&lt;/div&gt;&lt;div&gt;&lt;div&gt;&lt;span&gt;&amp;nbsp;&amp;nbsp; &amp;nbsp;&lt;/span&gt;After an efficient history is obtained, physical exam should be performed. With the patient supine, motor strength, graded 0 through 5, is tested in the upper and lower extremities. A sensory exam is then performed to define sensory deficits according to nerve root dermatomes. Instructions on how to perform a complete motor, sensory, and reflex testing on patients with suspected spinal injuries are found in the first section of this text, “The Approach to the Orthopaedic Patient.”&lt;/div&gt;&lt;/div&gt;&lt;div&gt;&lt;br /&gt;&lt;/div&gt;&lt;div&gt;&lt;div&gt;&lt;span&gt;&amp;nbsp;&amp;nbsp; &amp;nbsp;&lt;/span&gt;After neurovascular examination of the extremities, the patient is then rolled onto his or her side so that the entire spine can be palpated for regions of tenderness, crepitus, or step-off, and a rectal exam is performed. Log-roll precautions are employed to prevent further injury to a potentially unstable spine. During the rectal exam, the patient is asked whether they can sense that an exam is being performed, rectal tone is evaluated (absent, decreased, or intact), and the patient is asked to bear down as if he or she is having a bowel movement to test volitional contraction of the external sphincter. Finally, the bulbocavernosus reflex is tested by either pinching the glans penis or pulling on the Foley catheter and noting whether the anal sphincter contracts.&amp;nbsp;&lt;b&gt;If a patient has a spinal cord-level injury, has an absent bulbocavernosus reflex, and&amp;nbsp;&lt;/b&gt;&lt;b&gt;is less than 48 hours from injury, he or she is considered to be in spinal shock.&amp;nbsp;&lt;/b&gt;In such situations, recognize that neurologic deficits may improve or resolve completely. Patients who have a return of this reflex or patients who are more than 48 hours from injury and have complete loss of neurologic function below the level of injury are considered to have a complete spinal cord injury. They are thus unlikely to regain any function below this level. Note that the bulbocavernosus reflex is only relevant for spinal cord-level injuries and not for injuries that affect the cauda equina.&lt;/div&gt;&lt;/div&gt;&lt;div&gt;&lt;br /&gt;&lt;/div&gt;&lt;div&gt;&lt;br /&gt;&lt;/div&gt;&lt;div&gt;&lt;div&gt;&lt;b&gt;&lt;i&gt;&lt;span style=&quot;font-size: medium;&quot;&gt;Imaging&lt;/span&gt;&lt;/i&gt;&lt;/b&gt;&lt;/div&gt;&lt;div&gt;&lt;br /&gt;&lt;/div&gt;&lt;div&gt;After plain radiographs of the entire spine are obtained, areas of potential injury should undergo further imaging. CT is helpful to define bony injury and plays an essential role in surgical planning. MRI is useful for evaluating spinal cord compression or edema, individual nerve roots, and intervertebral discs and assessing for ligamentous and other soft-tissue injuries. MRI is also helpful in evaluating sequelae from direct injuries such as hematoma formation, which may result in cord compression and neurologic injury.&lt;/div&gt;&lt;/div&gt;&lt;div&gt;&lt;br /&gt;&lt;/div&gt;&lt;div&gt;&lt;br /&gt;&lt;/div&gt;&lt;div&gt;&lt;b style=&quot;color: #017087;&quot;&gt;&lt;span style=&quot;font-size: medium;&quot;&gt;TREATMENT&lt;/span&gt;&lt;/b&gt;&lt;/div&gt;&lt;div&gt;&lt;br /&gt;&lt;/div&gt;&lt;div&gt;&lt;div&gt;Initial treatment consists of stabilization, immobilization, pain and spasm control, and possibly administration of steroids. Although its true efficacy is controversial, high-dose intravenous (IV) methylprednisolone may be indicated for patients with nonpenetrating spinal cord injury who have presented within 8 hours of injury. Theoretically, steroids attenuate the deleterious inflammatory processes that lead to secondary nervous injury after primary spinal cord trauma.&lt;/div&gt;&lt;div&gt;&lt;br /&gt;&lt;/div&gt;&lt;div&gt;&lt;span&gt;&amp;nbsp;&amp;nbsp; &amp;nbsp;&lt;/span&gt;The treatment of lumbar fractures depends on the &lt;b&gt;stability of the fracture.&amp;nbsp;&lt;/b&gt;Although the exact definition is disputed, &lt;b&gt;unstable fractures&lt;/b&gt; generally include those with &lt;b&gt;marked neurologic compromise or complete 3-column disruption&amp;nbsp;&lt;/b&gt;(whether bony or ligamentous) or those &lt;b&gt;at risk of significant progression to&amp;nbsp;&lt;/b&gt;&lt;b&gt;deformity.&lt;/b&gt;&lt;/div&gt;&lt;/div&gt;&lt;div&gt;&lt;br /&gt;&lt;/div&gt;&lt;div&gt;&lt;br /&gt;&lt;/div&gt;&lt;div&gt;&lt;div&gt;&lt;b&gt;&lt;i&gt;&lt;span style=&quot;font-size: medium;&quot;&gt;Nonoperative Treatment&lt;/span&gt;&lt;/i&gt;&lt;/b&gt;&lt;/div&gt;&lt;div&gt;&lt;br /&gt;&lt;/div&gt;&lt;div&gt;Patients with compression fractures or burst fractures without neurologic injury and with anatomic or near anatomic alignment may be definitely treated in a well-fitted brace. Compression fractures do not generally need surgical stabilization if they have less than 30 degrees of kyphosis or less than 40% to 50% loss of height anteriorly. Stable burst fractures may be treated with the same criteria and should have no greater than 40% of spinal canal compromise by retropulsed bony fragments. Chance fractures may be treated nonoperatively if they are bony in nature and are without significant deformity (ie, result in &amp;lt; 15 degrees of kyphosis). Purely bony, minimally displaced Chance fractures are amenable to nonoperative treatment because the large surface area of cancellous bone involved in the fracture has a significantly greater chance of healing than their equivalent ligamentous ruptures.&lt;/div&gt;&lt;/div&gt;&lt;div&gt;&lt;br /&gt;&lt;/div&gt;&lt;div&gt;&lt;br /&gt;&lt;/div&gt;&lt;div&gt;&lt;div&gt;&lt;b&gt;&lt;i&gt;&lt;span style=&quot;font-size: medium;&quot;&gt;Operative Treatment&lt;/span&gt;&lt;/i&gt;&lt;/b&gt;&lt;/div&gt;&lt;div&gt;&lt;br /&gt;&lt;/div&gt;&lt;div&gt;Generally, patients with unstable burst fractures must undergo surgical decompression and stabilization. Patients with evidence of a neurologic injury with preservation of some function should undergo urgent decompression of compressed neural elements and stabilization of fractured vertebral levels, preferably within 12 hours of the injury to maximize chance of neurologic recovery. Patients without a neurologic injury may require surgical stabilization if imaging demonstrates unsatisfactory alignment or evidence of instability, as defined previously. Chance fractures are inherently unstable and require surgical stabilization if they are ligamentous in nature, accompanied by neurologic compromise, or result in deformity of greater than 15 degrees of kyphosis. All patients with unstable injuries must be immobilized in a brace until they are able to undergo surgery.&lt;/div&gt;&lt;/div&gt;&lt;div&gt;&lt;br /&gt;&lt;/div&gt;&lt;div&gt;&lt;br /&gt;&lt;/div&gt;&lt;div&gt;&lt;b&gt;&lt;span style=&quot;color: #017087; font-size: medium;&quot;&gt;&lt;u&gt;COMPREHENSION QUESTIONS&lt;/u&gt;&lt;/span&gt;&lt;/b&gt;&lt;/div&gt;&lt;div&gt;&lt;br /&gt;&lt;/div&gt;&lt;div&gt;&lt;div&gt;27.1 A 44-year-old cab driver is involved in a high-speed, head-on collision. In the ED, he is hemodynamically stable and has full neurologic function of his lower extremities. He is complaining of severe lower back pain. Radiographs and CT imaging are acquired of his lumbar spine. Which of the following findings is indicative of an unstable lumbar burst fracture that may need surgical stabilization?&lt;/div&gt;&lt;/div&gt;&lt;blockquote style=&quot;border: none; margin: 0px 0px 0px 40px; padding: 0px; text-align: left;&quot;&gt;&lt;div&gt;&lt;div&gt;A. 50% loss of vertebral height at the level of injury&lt;/div&gt;&lt;/div&gt;&lt;div&gt;&lt;div&gt;B. 20 degrees of kyphosis at the level of injury&lt;/div&gt;&lt;/div&gt;&lt;div&gt;&lt;div&gt;C. 30% of canal compromise&lt;/div&gt;&lt;/div&gt;&lt;div&gt;&lt;div&gt;D. Injury to the anterior and middle spinal columns only&lt;/div&gt;&lt;/div&gt;&lt;div&gt;&lt;div&gt;E. Injury to the posterior column only&lt;/div&gt;&lt;/div&gt;&lt;/blockquote&gt;&lt;div&gt;&lt;div&gt;&lt;br /&gt;&lt;/div&gt;&lt;div&gt;27.2 A 43-year-old woman who fell from a third-floor balcony presents to the ED with bilateral foot numbness as well as 2/5 bilateral extensor hallucis longus (EHL) function and absent plantar flexion. Radiographs demonstrate an L5 burst fracture with 40% retropulsion and 30% loss of height. On further history, the patient states she fell 4 days ago and finally sought care because she was experiencing decreasing sensation in her groin and can no longer void. What is the next most appropriate step in management?&lt;/div&gt;&lt;/div&gt;&lt;blockquote style=&quot;border: none; margin: 0px 0px 0px 40px; padding: 0px; text-align: left;&quot;&gt;&lt;div&gt;&lt;div&gt;A. IV high-dose corticosteroids&lt;/div&gt;&lt;/div&gt;&lt;div&gt;&lt;div&gt;B. Epidural steroid injection&lt;/div&gt;&lt;/div&gt;&lt;div&gt;&lt;div&gt;C. Nonsteroidal anti-inflammatory drugs (NSAIDs), bedrest, and appropriate bracing&lt;/div&gt;&lt;/div&gt;&lt;div&gt;&lt;div&gt;D. Surgical anterior decompression with fusion&lt;/div&gt;&lt;/div&gt;&lt;div&gt;&lt;div&gt;E. Surgical posterior decompressive lumbar laminectomy without fusion&lt;/div&gt;&lt;/div&gt;&lt;/blockquote&gt;&lt;div&gt;&lt;div&gt;&lt;br /&gt;&lt;/div&gt;&lt;div&gt;27.3 A 56-year-old male rock climber falls 8 feet from a ledge and lands on his buttocks. He arrives 6 hours later in the ED and complains of lower back pain. He is completely neurologically intact. Imaging of his lumbar spine shows an L3 burst fracture involving both anterior and middle columns with 30% loss of height anteriorly and minimal retropulsion. He has no other injuries. What is the most appropriate treatment for this patient?&lt;/div&gt;&lt;/div&gt;&lt;blockquote style=&quot;border: none; margin: 0px 0px 0px 40px; padding: 0px; text-align: left;&quot;&gt;&lt;div&gt;&lt;div&gt;A. IV steroids and appropriate bracing&lt;/div&gt;&lt;/div&gt;&lt;div&gt;&lt;div&gt;B. Low-dose narcotics, NSAIDs, and appropriate bracing&lt;/div&gt;&lt;/div&gt;&lt;div&gt;&lt;div&gt;C. Surgical decompression with fusion&lt;/div&gt;&lt;/div&gt;&lt;div&gt;&lt;div&gt;D. Further imaging must be obtained, including an MRI to rule out spinal cord injury&lt;/div&gt;&lt;/div&gt;&lt;div&gt;&lt;div&gt;E. Reassurance and activity modification&lt;/div&gt;&lt;/div&gt;&lt;/blockquote&gt;&lt;div&gt;&lt;br /&gt;&lt;/div&gt;&lt;div&gt;&lt;br /&gt;&lt;/div&gt;&lt;div&gt;&lt;b&gt;&lt;span style=&quot;color: #017087; font-size: medium;&quot;&gt;&lt;u&gt;ANSWERS&lt;/u&gt;&lt;/span&gt;&lt;/b&gt;&lt;/div&gt;&lt;div&gt;&lt;br /&gt;&lt;/div&gt;&lt;div&gt;&lt;div&gt;27.1 &lt;b&gt;A.&lt;/b&gt; Lumbar burst fractures are considered unstable and indicate surgical stabilization if they involve all 3 columns, result in &amp;gt; 40% to 50% loss of height or &amp;gt; 30 degrees of kyphosis, or have &amp;gt; 40% canal compromise. Burst fractures by definition are not exclusive to the posterior elements.&lt;/div&gt;&lt;div&gt;&lt;br /&gt;&lt;/div&gt;&lt;div&gt;27.2 &lt;b&gt;D.&lt;/b&gt; Although there are details suggesting that this patient’s burst fracture may be stable (ie, only 40% retropulsion and only 30% loss of height), her progressive neurologic symptoms are indication for surgical intervention. E is incorrect because it does not address the fracture or provide stabilization, as would an anterior decompression and fusion. Local epidural steroids do not address the fracture. IV steroids are controversial; however, they are not indicated greater than 8 hours from injury.&lt;/div&gt;&lt;div&gt;&lt;br /&gt;&lt;/div&gt;&lt;div&gt;27.3 &lt;b&gt;B.&lt;/b&gt; This patient has a stable burst fracture involving only the anterior and middle columns without significant loss of height or angulation. The appropriate treatment is pain control (narcotics are appropriate given his acute painful injury) and brace immobilization. Because the injury is stable and because he has no neurologic disruption, he does not necessarily need surgical stabilization, nor does he need extensive, expensive imaging such as an MRI. The patient may be reassured and should certainly refrain periodically from rock climbing, but requires a minimum of brace immobilization until the fracture has healed.&lt;/div&gt;&lt;/div&gt;&lt;div&gt;&lt;br /&gt;&lt;/div&gt;&lt;div&gt;&lt;b style=&quot;background-color: #4093a5;&quot;&gt;&amp;nbsp; &amp;nbsp;&lt;span style=&quot;font-size: medium;&quot;&gt;&amp;nbsp;&lt;/span&gt;&lt;span style=&quot;color: white;&quot;&gt;&lt;span style=&quot;font-size: medium;&quot;&gt;CLINICAL PEARLS&lt;/span&gt;&lt;span&gt;&amp;nbsp; &amp;nbsp;&amp;nbsp;&lt;/span&gt;&lt;/span&gt;&lt;/b&gt;&lt;/div&gt;&lt;div&gt;&lt;table border=&quot;1&quot; cellpadding=&quot;0&quot; cellspacing=&quot;0&quot; class=&quot;MsoTableGrid&quot; style=&quot;border-collapse: collapse; border: none; mso-border-alt: solid windowtext .5pt; mso-padding-alt: 0in 5.4pt 0in 5.4pt; mso-yfti-tbllook: 1184;&quot;&gt;
  218. &lt;tbody&gt;&lt;tr style=&quot;height: 19.15pt; mso-yfti-firstrow: yes; mso-yfti-irow: 0; mso-yfti-lastrow: yes;&quot;&gt;
  219.  &lt;td style=&quot;background: rgb(221, 237, 239); border-left: solid #017087 1.0pt; border: 1pt solid rgb(64, 147, 165); height: 19.15pt; padding: 0in 5.4pt; width: 433.7pt;&quot; valign=&quot;top&quot; width=&quot;578&quot;&gt;&lt;div style=&quot;line-height: normal; margin-bottom: 0in;&quot;&gt;&lt;span style=&quot;color: #017087;&quot;&gt;&lt;br /&gt;&lt;/span&gt;&lt;/div&gt;&lt;div style=&quot;line-height: normal; margin-bottom: 0in;&quot;&gt;&lt;span style=&quot;color: #017087;&quot;&gt;►&lt;/span&gt;&amp;nbsp;Patients with injuries secondary to axial-loading mechanisms should be evaluated for additional associated injuries, such as calcaneus fractures and lumbar burst fractures.&lt;/div&gt;&lt;div style=&quot;line-height: normal; margin-bottom: 0in;&quot;&gt;&lt;span style=&quot;color: #017087;&quot;&gt;&lt;br /&gt;&lt;/span&gt;&lt;/div&gt;&lt;div style=&quot;line-height: normal; margin-bottom: 0in;&quot;&gt;&lt;span style=&quot;color: #017087;&quot;&gt;►&lt;/span&gt;&amp;nbsp;A careful physical exam, including testing of the bulbocavernosus reflex, gives valuable information about whether neurologic injury has occurred and whether or not the patient is in spinal shock.&lt;/div&gt;&lt;div style=&quot;line-height: normal; margin-bottom: 0in;&quot;&gt;&lt;span style=&quot;color: #017087;&quot;&gt;&lt;br /&gt;&lt;/span&gt;&lt;/div&gt;&lt;div style=&quot;line-height: normal; margin-bottom: 0in;&quot;&gt;&lt;span style=&quot;color: #017087;&quot;&gt;►&lt;/span&gt;&amp;nbsp;Patients with incomplete neurologic injuries (preservation of motor or sensory function below the level of the injury) should undergo urgent decompression and stabilization of the injury to maximize the likelihood of neurologic improvement or preservation.&lt;/div&gt;&lt;div style=&quot;line-height: normal; margin-bottom: 0in;&quot;&gt;&lt;br /&gt;&lt;/div&gt;
  220.  &lt;/td&gt;
  221. &lt;/tr&gt;
  222. &lt;/tbody&gt;&lt;/table&gt;&lt;br /&gt;&lt;/div&gt;
  223.  
  224. &lt;details close=&quot;&quot;&gt;
  225.  &lt;summary&gt;&lt;b&gt;&lt;span style=&quot;color: #00788b;&quot;&gt;REFERENCES&lt;/span&gt;&lt;/b&gt;&lt;/summary&gt;
  226.  &lt;p&gt;Egol KE, Koval KJ, Zukerman JD, eds. Thoracolumbar spine. In: Handbook of Fractures. 4th ed. Philadelphia:
  227. Lippincott Williams &amp;amp; Wilkins; 2010:123-140.&amp;nbsp;&lt;/p&gt;&lt;p&gt;Mikles MR, Stchur RP, Graziano GP. Posterior instrumentation for thoracolumbar fractures. J Am Acad
  228. Orthop Surg. 2004;12:424-435.&amp;nbsp;&lt;/p&gt;&lt;p&gt;Singh K, Kim D, Vaccaro AR. Thoracic and lumbar spinal injuries. In: Herkowitz HG, Garfin SR,
  229. Eismont FF, Bell GR, Balderson RA, eds. Rothman-Simeone: The Spine. Philadelphia: Saunders
  230. Elsevier; 2006:1132-1156.&lt;/p&gt;
  231. &lt;/details&gt;&lt;/div&gt;</content><link rel='replies' type='application/atom+xml' href='https://medical-phd.blogspot.com/feeds/7274488063108359104/comments/default' title='Post Comments'/><link rel='replies' type='text/html' href='https://medical-phd.blogspot.com/2022/03/lumbar-burst-fracture-case-file.html#comment-form' title='0 Comments'/><link rel='edit' type='application/atom+xml' href='https://www.blogger.com/feeds/3302710210518459416/posts/default/7274488063108359104'/><link rel='self' type='application/atom+xml' href='https://www.blogger.com/feeds/3302710210518459416/posts/default/7274488063108359104'/><link rel='alternate' type='text/html' href='https://medical-phd.blogspot.com/2022/03/lumbar-burst-fracture-case-file.html' title='Lumbar Burst Fracture Case File'/><author><name>Medical Group</name><uri>http://www.blogger.com/profile/01780656800578397393</uri><email>noreply@blogger.com</email><gd:image rel='http://schemas.google.com/g/2005#thumbnail' width='32' height='32' src='//blogger.googleusercontent.com/img/b/R29vZ2xl/AVvXsEiMuROkBDv6yw4SmxGqRA5BPQhOXSwFld-Gufmy4kUqfpoAGQRl7cv-3naew6wjAnCjRFSe_FKX8z7CdE6cFKxxBpnqBuwUTSr-15xd3mFGcr_wJOX2y73G_zJCGND98Q/s220/Final-logo.jpg'/></author><media:thumbnail xmlns:media="http://search.yahoo.com/mrss/" url="https://blogger.googleusercontent.com/img/b/R29vZ2xl/AVvXsEjccPWgwLZGBFsZjdAxV1L8SsSPmZE8btrJh0oqn4rN4eC-UPlsgSXwll2tVjki0BggLhWGCYmBce_ed0CSb5_ubACD_4DdR_OXfT0T0VB9IokiVbZWTI3SWrrFGilRnYm2Avb2ARUuVbLAuhQ1luvfIEcoY9k8vTzMXDcrLEie8aB9HM8Hx-QccESqYw/s72-w640-h280-c/Lateral-plain-radiographs.jpg" height="72" width="72"/><thr:total>0</thr:total><georss:featurename>United States</georss:featurename><georss:point>37.09024 -95.712891</georss:point><georss:box>8.780006163821156 -130.869141 65.400473836178847 -60.556641</georss:box></entry><entry><id>tag:blogger.com,1999:blog-3302710210518459416.post-2643894504727485743</id><published>2022-03-25T11:44:00.004-07:00</published><updated>2022-03-25T11:44:29.190-07:00</updated><category scheme="http://www.blogger.com/atom/ns#" term="Case Files"/><category scheme="http://www.blogger.com/atom/ns#" term="Orthopaedic Surgery Case File"/><title type='text'>Cauda Equina Syndrome Case File</title><content type='html'>&lt;div class=&quot;scrollbox&quot;&gt;
  232.  
  233. &lt;div&gt;&lt;b&gt;&lt;span style=&quot;font-size: large;&quot;&gt;Cauda Equina Syndrome Case File&lt;/span&gt;&lt;/b&gt;&lt;/div&gt;&lt;div&gt;Eugene C. Toy, MD, Andrew J. Rosenbaum, MD, Timothy T. Roberts, MD, Joshua S. Dines, MD&lt;/div&gt;&lt;div&gt;&lt;br /&gt;&lt;/div&gt;&lt;div&gt;&lt;b&gt;&lt;span style=&quot;color: #017087; font-size: medium;&quot;&gt;CASE 26&lt;/span&gt;&lt;/b&gt;&lt;/div&gt;&lt;div&gt;&lt;div&gt;A 43-year-old obese man presents to the emergency department (ED) with complaints of 12 hours of right leg pain and “tingling” in his left foot. Yesterday he was helping his neighbor carry a sofa up a flight of stairs when he experienced sudden “lightning-like” pain in his back that radiated down his legs. He went home and took acetaminophen before bed. This morning, he awoke with worsening back and left leg pain, and he was startled to notice weakness in his left foot and ankle. Now, in the ED, he reports an almost complete inability to move his left foot and a burning, constant pain throughout both legs. Physical exam reveals decreased motor strength in his left lower extremity, notably 2/5 extensor hallucis longus and foot dorsiflexor strength. He is unable to plantar flex this foot. His left hip and knee extensors are 5/5 in strength, as are all major muscular groups of his right lower and bilateral upper extremities. He has normal patellar reflexes bilaterally, but an Achilles reflex is only present on his right side. Sensation testing demonstrates decreased sensitivity to light touch and pin-prick stimulation in the perianal regions, perineum, and posterior thighs, bilaterally. The dorsum and plantar aspects of the left foot are similarly insensate. Digital rectal examination demonstrates decreased tone, and the patient is surprised to learn that his underwear is damp with urine. Bladder scan demonstrates a retained volume of 1100 mL. Vascular examination reveals warm and well-perfused skin with palpable dorsalis pedis pulses bilaterally.&lt;/div&gt;&lt;div&gt;&lt;span style=&quot;color: #017087;&quot;&gt;&lt;br /&gt;&lt;/span&gt;&lt;/div&gt;&lt;div&gt;&lt;span style=&quot;color: #017087;&quot;&gt;►&lt;/span&gt;&amp;nbsp;What is the most likely diagnosis?&lt;/div&gt;&lt;div&gt;&lt;span style=&quot;color: #017087;&quot;&gt;►&lt;/span&gt;&amp;nbsp;What are some of the most common causes of this condition?&lt;/div&gt;&lt;div&gt;&lt;span style=&quot;color: #017087;&quot;&gt;►&lt;/span&gt;&amp;nbsp;What are the next steps in the management of this patient?&lt;/div&gt;&lt;/div&gt;&lt;div&gt;&lt;br /&gt;&lt;/div&gt;&lt;div&gt;&lt;br /&gt;&lt;/div&gt;&lt;div&gt;&lt;div&gt;&lt;div&gt;&lt;b&gt;&lt;span style=&quot;font-size: medium;&quot;&gt;ANSWER TO CASE 26:&lt;/span&gt;&lt;/b&gt;&lt;/div&gt;&lt;div&gt;&lt;span style=&quot;background-color: #daebf0; font-size: medium;&quot;&gt;&lt;span&gt;&lt;span style=&quot;background-color: transparent;&quot;&gt;&lt;span style=&quot;color: #017087;&quot;&gt;&lt;b&gt;Cauda Equina Syndrome&lt;/b&gt;&lt;/span&gt;&lt;/span&gt;&lt;b&gt;&amp;nbsp;&amp;nbsp;&lt;/b&gt;&lt;/span&gt;&lt;span style=&quot;font-weight: bold;&quot;&gt;&amp;nbsp;&amp;nbsp; &amp;nbsp;&lt;/span&gt;&lt;span style=&quot;font-weight: bold;&quot;&gt;&amp;nbsp;&amp;nbsp; &amp;nbsp;&lt;/span&gt;&lt;span style=&quot;font-weight: bold;&quot;&gt;&amp;nbsp;&amp;nbsp; &amp;nbsp;&lt;/span&gt;&lt;span style=&quot;font-weight: bold;&quot;&gt;&amp;nbsp;&amp;nbsp; &amp;nbsp;&lt;/span&gt;&lt;span style=&quot;font-weight: bold;&quot;&gt;&amp;nbsp;&amp;nbsp; &amp;nbsp;&lt;/span&gt;&lt;span style=&quot;font-weight: bold;&quot;&gt;&amp;nbsp;&amp;nbsp; &amp;nbsp;&lt;/span&gt;&lt;span style=&quot;font-weight: bold;&quot;&gt;&amp;nbsp; &amp;nbsp;&amp;nbsp;&lt;/span&gt;&lt;/span&gt;&lt;/div&gt;&lt;/div&gt;&lt;div&gt;&lt;br /&gt;&lt;/div&gt;&lt;/div&gt;&lt;div&gt;&lt;div&gt;&lt;i&gt;Summary&lt;/i&gt;: This is a 43-year-old man who presents with a 12-hour history of progressively worsening lumbosacral back and left leg pain, weakness, and sensory loss that began after he tried to lift a heavy object. He is experiencing bilateral lower extremity and perianal paresthesias, with significant weakness in his left lower extremity, diminished left-sided reflexes, decreased rectal tone, and urinary retention.&lt;/div&gt;&lt;div&gt;&lt;ul style=&quot;text-align: left;&quot;&gt;&lt;li&gt;&lt;b&gt;Most likely diagnosis:&lt;/b&gt; Cauda equina syndrome (CES) secondary to herniated nucleus pulposis (HNP), most likely at the L4-5 level.&lt;/li&gt;&lt;li&gt;&lt;b&gt;Most common causes:&lt;/b&gt; Compression of the cauda equina secondary to lumbar disc herniation, primary tumors or metastatic disease, infections, stenosis, hematomas, and ischemic insult.&lt;/li&gt;&lt;li&gt;&lt;b&gt;Management of CES:&lt;/b&gt; Pain control, placement of urinary catheter, MRI of lumbosacral spine to confirm diagnosis and to identify region of compression, and, if positive, emergency surgical decompression of cauda equina.&lt;/li&gt;&lt;/ul&gt;&lt;/div&gt;&lt;/div&gt;&lt;div&gt;&lt;br /&gt;&lt;/div&gt;&lt;div&gt;&lt;br /&gt;&lt;/div&gt;&lt;div&gt;&lt;div&gt;&lt;b&gt;&lt;span style=&quot;color: #017087; font-size: medium;&quot;&gt;ANALYSIS&lt;/span&gt;&lt;/b&gt;&lt;/div&gt;&lt;div&gt;&lt;b&gt;&lt;span style=&quot;font-size: medium;&quot;&gt;&lt;i&gt;Objectives&lt;/i&gt;&lt;/span&gt;&lt;/b&gt;&lt;/div&gt;&lt;/div&gt;&lt;div&gt;&lt;div&gt;&lt;ol style=&quot;text-align: left;&quot;&gt;&lt;li&gt;Recognize the presentation of cauda equina syndrome (CES).&lt;/li&gt;&lt;li&gt;Understand the anatomic and pathophysiologic bases of this condition.&lt;/li&gt;&lt;li&gt;Be familiar with the management of CES and understand its outcomes.&lt;/li&gt;&lt;/ol&gt;&lt;/div&gt;&lt;/div&gt;&lt;div&gt;&lt;br /&gt;&lt;/div&gt;&lt;div&gt;&lt;br /&gt;&lt;/div&gt;&lt;div&gt;&lt;div&gt;&lt;b&gt;&lt;i&gt;&lt;span style=&quot;font-size: medium;&quot;&gt;Considerations&lt;/span&gt;&lt;/i&gt;&lt;/b&gt;&lt;/div&gt;&lt;div&gt;&lt;br /&gt;&lt;/div&gt;&lt;div&gt;Low back pain is one of the most common complaints in EDs and primary care offices in the United States. Although back pain is disconcerting, even debilitating, to patients, very few patients presenting with acute-onset pain—0.05% by some estimates—require urgent operative intervention. CES is one such urgent indication. This patient’s symptoms, although mild at first, rapidly developed into a syndrome including sciatica, lower-extremity sensorimotor loss, and bowel and bladder dysfunction. This rapid progression of symptoms demands urgent and careful attention by knowledgeable physicians, as &lt;b&gt;delayed intervention can result&amp;nbsp;&lt;/b&gt;&lt;b&gt;in devastating consequences if left untreated for greater than 24 to 48 hours.&lt;/b&gt;&lt;/div&gt;&lt;div&gt;&lt;br /&gt;&lt;/div&gt;&lt;div&gt;&lt;span&gt;&amp;nbsp;&amp;nbsp; &amp;nbsp;&lt;/span&gt;After adequate pain relief, a complete and thorough medical history should assess for factors that make the patient prone to cauda equina, such as a history of symptoms of radicular disease, previous spinal surgery or injections, trauma, infection, the use of anticoagulants, or previous spine pathology. Potential spine fractures should be ruled out, because this patient’s symptoms began as he was undergoing significant axial loads that may have caused vertebral fractures with cord compression. Next, a thorough physical exam should be performed and must include the often-neglected assessment of sensation, motor, and reflexive function of the perianal and rectal regions, including evaluating the presence or absence of normal tone. For detailed instruction on performing the spinal exam, please refer to the “Approach to the Orthopaedic Patient” section, located near the beginning of this text.&lt;/div&gt;&lt;/div&gt;&lt;div&gt;&lt;br /&gt;&lt;/div&gt;&lt;div class=&quot;separator&quot; style=&quot;clear: both; text-align: center;&quot;&gt;&lt;a href=&quot;https://blogger.googleusercontent.com/img/b/R29vZ2xl/AVvXsEi5ZFcZ5X52mNPdN-2Ih9j67Y19Uqr2hp9JJlKizaTxFgr1KhVROlsx_mFh68kaZwZRymmU2iYe2nNP7p2VD8e5MR93gkNcZFND8kuD1V9nhQTmKX4BtUfArS7Y0cFdwHp6OMcgfOAKhDIUffzIP_n9zQ4-AXfA24L_uEMdeA3yVZsTFuPPYAF3EA-p6w/s699/Cauda-Equina-Syndrome.jpg&quot; style=&quot;margin-left: 1em; margin-right: 1em;&quot;&gt;&lt;img alt=&quot;Cauda Equina Syndrome&quot; border=&quot;0&quot; data-original-height=&quot;346&quot; data-original-width=&quot;699&quot; height=&quot;317&quot; src=&quot;https://blogger.googleusercontent.com/img/b/R29vZ2xl/AVvXsEi5ZFcZ5X52mNPdN-2Ih9j67Y19Uqr2hp9JJlKizaTxFgr1KhVROlsx_mFh68kaZwZRymmU2iYe2nNP7p2VD8e5MR93gkNcZFND8kuD1V9nhQTmKX4BtUfArS7Y0cFdwHp6OMcgfOAKhDIUffzIP_n9zQ4-AXfA24L_uEMdeA3yVZsTFuPPYAF3EA-p6w/w640-h317/Cauda-Equina-Syndrome.jpg&quot; title=&quot;Cauda Equina Syndrome&quot; width=&quot;640&quot; /&gt;&lt;/a&gt;&lt;/div&gt;&lt;div&gt;&lt;div&gt;&lt;b&gt;&lt;span style=&quot;color: #017087;&quot;&gt;Figure 26–1.&lt;/span&gt;&lt;/b&gt; Axial (&lt;i&gt;left&lt;/i&gt;) and sagittal (&lt;i&gt;right&lt;/i&gt;) slices of T2-weighted spinal MRI of large, centrally herniated disc at L4-5. (Courtesy of Timothy T. Roberts, MD)&lt;/div&gt;&lt;/div&gt;&lt;div&gt;&lt;br /&gt;&lt;/div&gt;&lt;div&gt;&lt;div&gt;&lt;span&gt;&amp;nbsp;&amp;nbsp; &amp;nbsp;&lt;/span&gt;The next step in this patient’s workup includes urgent acquisition of appropriate imaging, either through magnetic resonance imaging (MRI) of the lumbosacral spine or computed tomography (CT) myelography in the presence of contraindications to MRI. &lt;b&gt;MRI serves to elucidate the cause and location of the cauda equina&amp;nbsp;&lt;/b&gt;&lt;b&gt;compression, as well as to guide the surgical approach (anterior or posterior)&amp;nbsp;&lt;/b&gt;&lt;b&gt;and the need for potential arthrodesis (in the setting of instability attributable to&amp;nbsp;&lt;/b&gt;&lt;b&gt;trauma, infection, or tumor).&lt;/b&gt; In this case, axial and sagittal T2-weighted images, seen in Figure 26–1 , demonstrate a large, posteriorly herniated disc at the L4-5 level extruding paracentrally and laterally on the left side. Roots caudal to this level show varied levels of dysfunction consistent with midline compression of the descending lumbosacral roots. Plans should be made for urgent passage to the operating room, as permanent neurologic damage may result from delayed decompression of the cauda equina.&lt;/div&gt;&lt;/div&gt;&lt;div&gt;&lt;br /&gt;&lt;/div&gt;&lt;div&gt;&lt;br /&gt;&lt;/div&gt;&lt;div&gt;&lt;div&gt;&lt;div&gt;&lt;div&gt;&lt;b&gt;&lt;span style=&quot;font-size: medium;&quot;&gt;APPROACH TO:&lt;/span&gt;&lt;/b&gt;&lt;/div&gt;&lt;div&gt;&lt;span style=&quot;background-color: #daebf0; color: #017087; font-size: medium;&quot;&gt;&lt;span&gt;&lt;span style=&quot;background-color: transparent;&quot;&gt;&lt;b&gt;Cauda Equina Syndrome&lt;/b&gt;&lt;/span&gt;&lt;b&gt;&amp;nbsp;&lt;/b&gt;&lt;/span&gt;&lt;span style=&quot;font-weight: bold;&quot;&gt;&amp;nbsp;&amp;nbsp; &amp;nbsp;&lt;/span&gt;&lt;span style=&quot;font-weight: bold;&quot;&gt;&amp;nbsp;&amp;nbsp; &amp;nbsp;&lt;/span&gt;&lt;span style=&quot;font-weight: bold;&quot;&gt;&amp;nbsp;&amp;nbsp; &amp;nbsp;&lt;/span&gt;&lt;span style=&quot;font-weight: bold;&quot;&gt;&amp;nbsp;&amp;nbsp; &amp;nbsp;&lt;/span&gt;&lt;span style=&quot;font-weight: bold;&quot;&gt;&amp;nbsp;&amp;nbsp; &amp;nbsp;&lt;/span&gt;&lt;span style=&quot;font-weight: bold;&quot;&gt;&amp;nbsp;&amp;nbsp; &amp;nbsp;&lt;/span&gt;&lt;span style=&quot;font-weight: bold;&quot;&gt;&amp;nbsp;&amp;nbsp; &amp;nbsp;&lt;/span&gt;&lt;span style=&quot;font-weight: bold;&quot;&gt;&amp;nbsp;&amp;nbsp;&amp;nbsp;&lt;/span&gt;&lt;span style=&quot;font-weight: bold;&quot;&gt;&amp;nbsp;&lt;/span&gt;&lt;/span&gt;&lt;/div&gt;&lt;/div&gt;&lt;div&gt;&lt;br /&gt;&lt;/div&gt;&lt;div&gt;&lt;b&gt;&lt;span style=&quot;color: #017087; font-size: medium;&quot;&gt;DEFINITIONS&lt;/span&gt;&lt;/b&gt;&lt;/div&gt;&lt;/div&gt;&lt;div&gt;&lt;br /&gt;&lt;/div&gt;&lt;/div&gt;&lt;div&gt;&lt;div&gt;&lt;b&gt;SCIATICA:&lt;/b&gt; A referred pain syndrome in the buttock, leg, and/or foot distributions of the sciatic nerve, caused by compression of either the sciatic nerve itself, or its individual lumbosacral roots.&lt;/div&gt;&lt;div&gt;&lt;b&gt;&lt;br /&gt;&lt;/b&gt;&lt;/div&gt;&lt;div&gt;&lt;b&gt;ANAL WINK REFLEX:&lt;/b&gt; A normal reflex of anal sphincter contraction, elicited by stroking the skin lateral to the anus. Absence of this reflex may suggest dysfunction of the S2-4 nerve roots.&lt;/div&gt;&lt;/div&gt;&lt;div&gt;&lt;br /&gt;&lt;/div&gt;&lt;div&gt;&lt;div&gt;&lt;b&gt;LOWER MOTOR NEURON LESION: &lt;/b&gt;Any lesion that disrupts the function of the motor nerve fibers between the anterior horn of the spinal cord and its innervated muscles. Characterized by hypotonia, hypo- or areflexia, flaccid paralysis or weakness, fibrillations, and fasciculations. By contrast, upper motor neuron lesions affect motor nerve pathways between the cerebral motor cortex and the spinal cord and are characterized by hypertonia, hyperreflexia, and spastic paralysis.&lt;/div&gt;&lt;/div&gt;&lt;div&gt;&lt;br /&gt;&lt;/div&gt;&lt;div&gt;&lt;br /&gt;&lt;/div&gt;&lt;div&gt;&lt;b&gt;&lt;span style=&quot;color: #017087; font-size: medium;&quot;&gt;CLINICAL APPROACH&lt;/span&gt;&lt;/b&gt;&lt;/div&gt;&lt;div&gt;&lt;br /&gt;&lt;/div&gt;&lt;div&gt;&lt;div&gt;CES is an uncommon and relatively rare condition that accounts for fewer than 1 in 2000 patients who present with lower back pain. This condition is associated with a large space-occupying lesion within the lumbosacral spine that impinges on the loose terminal roots of the spinal nerves within the spinal canal. These roots form the cauda equina, or “horse’s tail” in Latin, so named for their likeness to the equine appendage.&amp;nbsp;&lt;b&gt;Diagnostic criteria for CES include 1 or more of the following: (1) bladder&amp;nbsp;&lt;/b&gt;&lt;b&gt;and/or bowel dysfunction—typically retention in its early stages, (2) reduced sensation&amp;nbsp;&lt;/b&gt;&lt;b&gt;in the saddle area, or (3) sexual dysfunction, with possible motor/sensory/&amp;nbsp;&lt;/b&gt;&lt;b&gt;reflex deficit in the lower limb. &lt;/b&gt;Signs and symptoms of CES develop in less than&amp;nbsp;&lt;b&gt;24 hours&lt;/b&gt; from a given insult in more than 85% of reported cases. Nerve root compression in CES most commonly results from a herniated lumbar disc, but is thought to represent only 1% to 6% of all operative lumbar HNP cases. Risk factors for HNP include obesity, male sex, age greater than 40 years, and history of spine disorders. Less common etiologies of CES include compression by a tumorous growth, infection and abscess formation, spinal stenosis, hematoma formation, and inflammation.&lt;/div&gt;&lt;/div&gt;&lt;div&gt;&lt;br /&gt;&lt;/div&gt;&lt;div&gt;&lt;br /&gt;&lt;/div&gt;&lt;div&gt;&lt;div&gt;&lt;b&gt;&lt;i&gt;&lt;span style=&quot;font-size: medium;&quot;&gt;Anatomy and Pathophysiology&lt;/span&gt;&lt;/i&gt;&lt;/b&gt;&lt;/div&gt;&lt;div&gt;&lt;br /&gt;&lt;/div&gt;&lt;div&gt;The pathophysiology of CES is complex and not fully understood. In adults, the spinal cord terminates between the L1 and L2 vertebrae body, below which the cauda equina descend as a collection of peripheral nerve roots to the L1 to coccygeal levels. These roots are thought to be especially vulnerable to compression because they are only protected by a single layer of endoneurium, whereas most other peripheral nerves and indeed the spinal cord itself are surrounded by an epineurium, perineurium, and endoneurium. Compression and damage to the cauda equina can lead to diffuse &lt;b&gt;lower motor neuron lesions,&lt;/b&gt; resulting in uni- or bilateral sciatica, lower extremity weakness, saddle anesthesia (resulting from insult to the S2-4 roots), and bowel and bladder dysfunction (also primarily the S2-4 roots).&lt;/div&gt;&lt;div&gt;&lt;br /&gt;&lt;/div&gt;&lt;div&gt;&lt;span&gt;&amp;nbsp;&amp;nbsp; &amp;nbsp;&lt;/span&gt;Patients presenting with back pain and bowel or bladder dysfunction should raise immediate flags for the possibility of CES. Neurogenic bladder dysfunction, characterized by urinary retention more often than incontinence, is an essential symptom of CES. In normal urinary bladder physiology, the detrusor urinae muscle is responsible for contracting the bladder during voiding and is controlled by the parasympathetic nervous system (PNS) via S2, S3, and S4 nerve roots. During urinary voiding, the PNS directs detrusor contraction with simultaneous relaxation of the internal urinary sphincter. When damage to the S2-4 nerve roots occurs, the bladder cannot contract or release through the sphincter, and thus overflow incontinence develops. Postvoid residual volume should be obtained in patients in whom CES is suspected.&lt;/div&gt;&lt;/div&gt;&lt;div&gt;&lt;br /&gt;&lt;/div&gt;&lt;div&gt;&lt;div&gt;&lt;span&gt;&amp;nbsp;&amp;nbsp; &amp;nbsp;&lt;/span&gt;The S2-4 nerve roots also provide sensation to the saddle region, including the perineum, buttocks, and posteromedial thighs. A complete rectal examination, including testing for an &lt;b&gt;anal wink reflex and a bulbocavernosus reflex,&lt;/b&gt; should be performed on CES patients. Patients presenting with both saddle anesthesia and urinary incontinence—suggestive of extensive insult to the sacral roots—have been found to have poor prognosis with regard to long-term bladder function.&lt;/div&gt;&lt;/div&gt;&lt;div&gt;&lt;br /&gt;&lt;/div&gt;&lt;div&gt;&lt;br /&gt;&lt;/div&gt;&lt;div&gt;&lt;div&gt;&lt;b&gt;&lt;i&gt;&lt;span style=&quot;font-size: medium;&quot;&gt;Radiologic Evaluation&lt;/span&gt;&lt;/i&gt;&lt;/b&gt;&lt;/div&gt;&lt;div&gt;&lt;br /&gt;&lt;/div&gt;&lt;div&gt;After a thorough history and physical exam, urgent diagnostic imaging should be performed. Plain radiographs are of limited value in the diagnosis of CES, but may be useful to evaluate for alternate pathology such as a fracture, dislocation or subluxation, tumor, or infection. Diagnosing lumbar disc herniation in CES requires a CT or MRI. &lt;b&gt;MRI is the gold standard&lt;/b&gt; for evaluating patients because it allows for detailed visualization of the spinal canal, disc spaces, nerve roots, and visualization of any potential space-occupying lesion within the canal or foramina. If MRI is contraindicated or unavailable, CT myelogram is the next best study to evaluate for CES. In addition to confirming the diagnosis, advanced imaging is helpful for preoperative planning for decompression.&lt;/div&gt;&lt;/div&gt;&lt;div&gt;&lt;br /&gt;&lt;/div&gt;&lt;div&gt;&lt;br /&gt;&lt;/div&gt;&lt;div&gt;&lt;b style=&quot;color: #017087;&quot;&gt;&lt;span style=&quot;font-size: medium;&quot;&gt;TREATMENT AND OUTCOMES&lt;/span&gt;&lt;/b&gt;&lt;/div&gt;&lt;div&gt;&lt;br /&gt;&lt;/div&gt;&lt;div&gt;&lt;div&gt;In patients without absolute medical contraindications, CES is treated with emergent surgical exploration and nerve root decompression. A variety of surgical procedures and techniques are performed in the treatment of CES, ranging from minimally invasive micro-discectomies to extensive multiple-level bilateral laminectomies, discectomies, and occasionally arthrodeses. No significant evidence, however, supports the superiority of any one procedure. It is generally thought that surgical decompression should occur within 24 to 48 hours of CES diagnosis to prevent further neurologic decline and to improve chances of recovery. &lt;b&gt;There is evidence to suggest that&amp;nbsp;&lt;/b&gt;&lt;b&gt;patients treated more than 48 hours after diagnosis had significantly decreased&amp;nbsp;&lt;/b&gt;&lt;b&gt;odds of recovering complete sensory, motor, sexual, urinary, and rectal function.&lt;/b&gt;&lt;/div&gt;&lt;/div&gt;&lt;div&gt;&lt;br /&gt;&lt;/div&gt;&lt;div&gt;&lt;br /&gt;&lt;/div&gt;&lt;div&gt;&lt;b&gt;&lt;span style=&quot;color: #017087; font-size: medium;&quot;&gt;&lt;u&gt;COMPREHENSION QUESTIONS&lt;/u&gt;&lt;/span&gt;&lt;/b&gt;&lt;/div&gt;&lt;div&gt;&lt;br /&gt;&lt;/div&gt;&lt;div&gt;&lt;div&gt;26.1 A 63-year-old woman with a history of metastatic lung cancer, diabetes, and bradycardia requiring a permanent pacemaker presents with 24 hours of urinary retention and progressive bilateral lower extremity weakness, beginning spontaneously. What is the next step in management?&lt;/div&gt;&lt;/div&gt;&lt;blockquote style=&quot;border: none; margin: 0px 0px 0px 40px; padding: 0px; text-align: left;&quot;&gt;&lt;div&gt;&lt;div&gt;A. Metastatic workup including CT of chest and preparations for biopsy, if applicable&lt;/div&gt;&lt;/div&gt;&lt;div&gt;&lt;div&gt;B. High-dose intravenous methylprednisolone × 24 hours and short-term bedrest&lt;/div&gt;&lt;/div&gt;&lt;div&gt;&lt;div&gt;C. Administration of nonsteroidal anti-inflammatory drugs (NSAIDs), acetaminophen, activity modification, physical therapy, and close follow-up&lt;/div&gt;&lt;/div&gt;&lt;div&gt;&lt;div&gt;D. Emergency CT myelogram of the lumbosacral spine&lt;/div&gt;&lt;/div&gt;&lt;div&gt;&lt;div&gt;E. Emergency MRI of chest with gadolinium contrast&lt;/div&gt;&lt;/div&gt;&lt;/blockquote&gt;&lt;div&gt;&lt;br /&gt;&lt;/div&gt;&lt;div&gt;&lt;div&gt;26.2 A 55-year-old man presents with low back pain and right “foot drop” 72 hours after straining his back trying to shovel snow from his driveway. He denies bowel or bladder changes. Sensation is intact throughout his bilateral lower extremities and perianal region, with the exception of some numbness to light touch across the dorsum of his right foot. He has a weak extensor hallucis longus and tibialis anterior (3/5) on his right side, but otherwise full strength in his bilateral low extremities. Straight leg raise is positive on his right side, sending “electric shock” sciatic pain down to his foot. He takes warfarin for atrial fibrillation, and his international normalized ratio (INR) is 2.5. He has no additional symptoms, nor medical history. What is the next step in the management of this patient?&lt;/div&gt;&lt;/div&gt;&lt;blockquote style=&quot;border: none; margin: 0px 0px 0px 40px; padding: 0px; text-align: left;&quot;&gt;&lt;div&gt;&lt;div&gt;A. High-dose intravenous methylprednisolone for 24 hours&lt;/div&gt;&lt;/div&gt;&lt;div&gt;&lt;div&gt;B. Emergency MRI, if not contraindicated, of lumbosacral spine&lt;/div&gt;&lt;/div&gt;&lt;div&gt;&lt;div&gt;C. NSAIDs, acetaminophen, activity modification, and physical therapy&lt;/div&gt;&lt;/div&gt;&lt;div&gt;&lt;div&gt;D. Urgent reversal of INR with vitamin K and/or fresh-frozen plasma and preoperative planning including making the patient nothing by mouth (NPO), acquiring preoperative laboratory studies, chest x-ray, and electrocardiogram (ECG)&lt;/div&gt;&lt;/div&gt;&lt;div&gt;&lt;div&gt;E. Epidural steroid injection&lt;/div&gt;&lt;/div&gt;&lt;/blockquote&gt;&lt;div&gt;&lt;br /&gt;&lt;/div&gt;&lt;div&gt;&lt;div&gt;26.3 A 44-year-old woman suffers a fall while rock climbing, landing on her buttocks and falling forward. Despite prolonged airlift to the ED, she is hemodynamically stable. She complains of bilateral pain in her legs, distal to her knees. She has profound weakness in her bilaterally extensor hallucis longi and gastrocsoleus complexes and has marked saddle anesthesia. MRI shows a large, midline herniated disc, compressing each of the traversing nerve roots and entire cauda equina below its level, but sparing the exiting nerve roots. Which disc is most likely involved in this injury?&lt;/div&gt;&lt;/div&gt;&lt;blockquote style=&quot;border: none; margin: 0px 0px 0px 40px; padding: 0px; text-align: left;&quot;&gt;&lt;div&gt;&lt;div&gt;A. L2-3&lt;/div&gt;&lt;/div&gt;&lt;div&gt;&lt;div&gt;B. L3-4&lt;/div&gt;&lt;/div&gt;&lt;div&gt;&lt;div&gt;C. L4-5&lt;/div&gt;&lt;/div&gt;&lt;div&gt;&lt;div&gt;D. L5-S1&lt;/div&gt;&lt;/div&gt;&lt;div&gt;&lt;div&gt;E. S1-2&lt;/div&gt;&lt;/div&gt;&lt;/blockquote&gt;&lt;div&gt;&lt;br /&gt;&lt;/div&gt;&lt;div&gt;&lt;br /&gt;&lt;/div&gt;&lt;div&gt;&lt;b&gt;&lt;span style=&quot;color: #017087; font-size: medium;&quot;&gt;&lt;u&gt;ANSWERS&lt;/u&gt;&lt;/span&gt;&lt;/b&gt;&lt;/div&gt;&lt;div&gt;&lt;br /&gt;&lt;/div&gt;&lt;div&gt;&lt;div&gt;26.1 &lt;b&gt;B.&lt;/b&gt; This patient has acute symptoms of CES. Although MRI of the lumbosacral spine is the ideal study to confirm this diagnosis, this patient’s pacemaker may be a contraindication to MRI. In such situations, CT myelography is the next best option. When performed for CES evaluation, MRIs should focus on the lumbosacral spine, not the chest and abdomen, and gadolinium contrast is usually unnecessary. Although steroids and/or NSAIDs and physical may play a role in treatment of acute HNP, they are not the first line in the treatment of CES. Likewise, this patient may require metastatic workup, but this is not the priority in the acute management of CES.&lt;/div&gt;&lt;/div&gt;&lt;div&gt;&lt;br /&gt;&lt;/div&gt;&lt;div&gt;&lt;div&gt;26.2 &lt;b&gt;C.&lt;/b&gt; This patient’s history and presentation is consistent with an isolated, acutely HNP at L4-5. Although his motor and sensory deficits are concerning, the majority of patients will experience resolution of symptoms with nonoperative conservative management including NSAIDs, acetaminophen, physical therapy, and activity modification. Although an MRI may be helpful in making this diagnosis, it is not emergently indicated, as this patient does not have evidence of CES or other pathology that would require emergency, extensive workup, or surgical intervention. Epidural steroid injection is not the first line of treatment for an acute HNP, but may offer some relief in patients with refractory symptoms. Generally, epidural injection should not be performed with a significantly elevated INR.&lt;/div&gt;&lt;div&gt;&lt;br /&gt;&lt;/div&gt;&lt;div&gt;26.3 &lt;b&gt;C.&lt;/b&gt; This patient has weak extensor hallucis longi (L5 nerve root), weak gastrocsoleus complexes (S1), and saddle anesthesia (S2-4). She has normal function above this level, suggesting that the L5 nerve roots and those below are affected. An injury at L4-5 that spares the exiting roots (L4) but affects the traversing roots (L5) and those below (S1-5) would be most consistent with these symptoms. Remember the sacral spine does not have interbody discs, as it is fused.&lt;/div&gt;&lt;/div&gt;&lt;div&gt;&lt;br /&gt;&lt;/div&gt;&lt;div&gt;&lt;b style=&quot;background-color: #4093a5;&quot;&gt;&amp;nbsp; &amp;nbsp;&lt;span style=&quot;font-size: medium;&quot;&gt;&amp;nbsp;&lt;/span&gt;&lt;span style=&quot;color: white;&quot;&gt;&lt;span style=&quot;font-size: medium;&quot;&gt;CLINICAL PEARLS&lt;/span&gt;&lt;span&gt;&amp;nbsp; &amp;nbsp;&amp;nbsp;&lt;/span&gt;&lt;/span&gt;&lt;/b&gt;&lt;/div&gt;&lt;div&gt;&lt;table border=&quot;1&quot; cellpadding=&quot;0&quot; cellspacing=&quot;0&quot; class=&quot;MsoTableGrid&quot; style=&quot;border-collapse: collapse; border: none; mso-border-alt: solid windowtext .5pt; mso-padding-alt: 0in 5.4pt 0in 5.4pt; mso-yfti-tbllook: 1184;&quot;&gt;
  234. &lt;tbody&gt;&lt;tr style=&quot;height: 19.15pt; mso-yfti-firstrow: yes; mso-yfti-irow: 0; mso-yfti-lastrow: yes;&quot;&gt;
  235.  &lt;td style=&quot;background: rgb(221, 237, 239); border-left: solid #017087 1.0pt; border: 1pt solid rgb(64, 147, 165); height: 19.15pt; padding: 0in 5.4pt; width: 433.7pt;&quot; valign=&quot;top&quot; width=&quot;578&quot;&gt;&lt;div style=&quot;line-height: normal; margin-bottom: 0in;&quot;&gt;&lt;br /&gt;&lt;/div&gt;&lt;div style=&quot;line-height: normal; margin-bottom: 0in;&quot;&gt;&lt;span style=&quot;color: #017087;&quot;&gt;►&lt;/span&gt;&amp;nbsp;CES is defined by a characteristic cluster of symptoms, including low back pain, sciatica, lower extremity sensorimotor loss, and bowel and bladder dysfunction.&lt;/div&gt;&lt;div style=&quot;line-height: normal; margin-bottom: 0in;&quot;&gt;&lt;span style=&quot;color: #017087;&quot;&gt;&lt;br /&gt;&lt;/span&gt;&lt;/div&gt;&lt;div style=&quot;line-height: normal; margin-bottom: 0in;&quot;&gt;&lt;span style=&quot;color: #017087;&quot;&gt;►&lt;/span&gt;&amp;nbsp;CES is treated with urgent surgical decompression, unless there is an absolute contraindication to surgery.&lt;/div&gt;&lt;div style=&quot;line-height: normal; margin-bottom: 0in;&quot;&gt;&lt;span style=&quot;color: #017087;&quot;&gt;&lt;br /&gt;&lt;/span&gt;&lt;/div&gt;&lt;div style=&quot;line-height: normal; margin-bottom: 0in;&quot;&gt;&lt;span style=&quot;color: #017087;&quot;&gt;►&lt;/span&gt;&amp;nbsp;CES is a surgical emergency! Preparations for urgent operative intervention should begin as soon as the diagnosis is suspected.&lt;/div&gt;&lt;div style=&quot;line-height: normal; margin-bottom: 0in;&quot;&gt;&lt;br /&gt;&lt;/div&gt;
  236.  &lt;/td&gt;
  237. &lt;/tr&gt;
  238. &lt;/tbody&gt;&lt;/table&gt;&lt;br /&gt;&lt;/div&gt;
  239.  
  240. &lt;details close=&quot;&quot;&gt;
  241.  &lt;summary&gt;&lt;b&gt;&lt;span style=&quot;color: #00788b;&quot;&gt;REFERENCES&lt;/span&gt;&lt;/b&gt;&lt;/summary&gt;
  242.  &lt;p&gt;Flynn, JM, ed. Lumbar degenerative disease. In: Orthopaedic Knowledge Update: Ten . Rosemont, IL:
  243. American Academy of Orthopaedic Surgeons; 2011:599-610.&amp;nbsp;&lt;/p&gt;&lt;p&gt;Spector LR, Madigan L, Rhyne A, Darden B, Kim D. Cauda equina syndrome. J Am Acad Orthop Surg.
  244. 2008;16:471-479.&lt;/p&gt;
  245. &lt;/details&gt;&lt;/div&gt;</content><link rel='replies' type='application/atom+xml' href='https://medical-phd.blogspot.com/feeds/2643894504727485743/comments/default' title='Post Comments'/><link rel='replies' type='text/html' href='https://medical-phd.blogspot.com/2022/03/cauda-equina-syndrome-case-file.html#comment-form' title='0 Comments'/><link rel='edit' type='application/atom+xml' href='https://www.blogger.com/feeds/3302710210518459416/posts/default/2643894504727485743'/><link rel='self' type='application/atom+xml' href='https://www.blogger.com/feeds/3302710210518459416/posts/default/2643894504727485743'/><link rel='alternate' type='text/html' href='https://medical-phd.blogspot.com/2022/03/cauda-equina-syndrome-case-file.html' title='Cauda Equina Syndrome Case File'/><author><name>Medical Group</name><uri>http://www.blogger.com/profile/01780656800578397393</uri><email>noreply@blogger.com</email><gd:image rel='http://schemas.google.com/g/2005#thumbnail' width='32' height='32' src='//blogger.googleusercontent.com/img/b/R29vZ2xl/AVvXsEiMuROkBDv6yw4SmxGqRA5BPQhOXSwFld-Gufmy4kUqfpoAGQRl7cv-3naew6wjAnCjRFSe_FKX8z7CdE6cFKxxBpnqBuwUTSr-15xd3mFGcr_wJOX2y73G_zJCGND98Q/s220/Final-logo.jpg'/></author><media:thumbnail xmlns:media="http://search.yahoo.com/mrss/" url="https://blogger.googleusercontent.com/img/b/R29vZ2xl/AVvXsEi5ZFcZ5X52mNPdN-2Ih9j67Y19Uqr2hp9JJlKizaTxFgr1KhVROlsx_mFh68kaZwZRymmU2iYe2nNP7p2VD8e5MR93gkNcZFND8kuD1V9nhQTmKX4BtUfArS7Y0cFdwHp6OMcgfOAKhDIUffzIP_n9zQ4-AXfA24L_uEMdeA3yVZsTFuPPYAF3EA-p6w/s72-w640-h317-c/Cauda-Equina-Syndrome.jpg" height="72" width="72"/><thr:total>0</thr:total><georss:featurename>United States</georss:featurename><georss:point>37.09024 -95.712891</georss:point><georss:box>8.780006163821156 -130.869141 65.400473836178847 -60.556641</georss:box></entry><entry><id>tag:blogger.com,1999:blog-3302710210518459416.post-7131228201821791542</id><published>2022-03-25T10:08:00.006-07:00</published><updated>2022-03-25T10:08:38.321-07:00</updated><category scheme="http://www.blogger.com/atom/ns#" term="Case Files"/><category scheme="http://www.blogger.com/atom/ns#" term="Orthopaedic Surgery Case File"/><title type='text'>Herniated Nucleus Pulposus of the Cervical Spine Case File</title><content type='html'>&lt;div class=&quot;scrollbox&quot;&gt;
  246.  
  247. &lt;div style=&quot;text-align: left;&quot;&gt;&lt;div&gt;&lt;b&gt;&lt;span style=&quot;font-size: large;&quot;&gt;Herniated Nucleus Pulposus of the Cervical Spine Case File&lt;/span&gt;&lt;/b&gt;&lt;/div&gt;&lt;div&gt;Eugene C. Toy, MD, Andrew J. Rosenbaum, MD, Timothy T. Roberts, MD, Joshua S. Dines, MD&lt;/div&gt;&lt;div&gt;&lt;br /&gt;&lt;/div&gt;&lt;div&gt;&lt;b&gt;&lt;span style=&quot;color: #017087; font-size: medium;&quot;&gt;CASE 25&lt;/span&gt;&lt;/b&gt;&lt;/div&gt;&lt;div&gt;&lt;div&gt;A 45-year-old musician comes to the office complaining of a 3-month history of neck pain that radiates into his right arm. It is “electric” in nature, spontaneously sending a shooting sensation down his arm. Last week, he noted weakness in his right arm with elbow flexion. His pain is relatively constant and is accompanied by some loss of sensation in his thumb and index finger. He is finding it difficult to play the piano with his right hand. The patient gains some relief by lying down, but states that occasional bouts of “shooting” pain are interfering with his sleep. He denies other systemic complaints, such as fevers and chills. His past medical and surgical history is unremarkable except for a 20-pack-year smoking history.&lt;/div&gt;&lt;div&gt;&lt;br /&gt;&lt;/div&gt;&lt;div&gt;&lt;span&gt;&amp;nbsp;&amp;nbsp; &amp;nbsp;&lt;/span&gt;On physical exam, gait testing demonstrates a fluid, nonataxic, nonantalgic gait. Neck range of motion is decreased with lateral bending and axial rotation to the right side, and neck extension is uncomfortable. Right shoulder range of motion is pain-free. Neurologic testing demonstrates marked weakness with resisted elbow flexion (4/5) and shoulder abduction and forward flexion (4/5) with full strength in all other right upper extremity muscle groups. He has a decreased bicipital reflex as compared with the unaffected side. Capillary refill is brisk. No lesions, ecchymosis, or erythema is present in the arm or hand. Examination of the left upper extremity and both lower extremities are within normal limits. Plain radiographs of the cervical spine and right shoulder are obtained and show no acute pathology, revealing only mild degenerative changes.&lt;/div&gt;&lt;div&gt;&lt;span style=&quot;color: #017087;&quot;&gt;&lt;br /&gt;&lt;/span&gt;&lt;/div&gt;&lt;div&gt;&lt;span style=&quot;color: #017087;&quot;&gt;►&lt;/span&gt;&amp;nbsp;What is the most likely diagnosis?&lt;/div&gt;&lt;div&gt;&lt;span style=&quot;color: #017087;&quot;&gt;►&lt;/span&gt;&amp;nbsp;What is the next step in therapy?&lt;/div&gt;&lt;/div&gt;&lt;div&gt;&lt;br /&gt;&lt;/div&gt;&lt;div&gt;&lt;br /&gt;&lt;/div&gt;&lt;div&gt;&lt;div&gt;&lt;div&gt;&lt;b&gt;&lt;span style=&quot;font-size: medium;&quot;&gt;ANSWER TO CASE 25:&lt;/span&gt;&lt;/b&gt;&lt;/div&gt;&lt;div&gt;&lt;span style=&quot;background-color: #daebf0; font-size: medium;&quot;&gt;&lt;span&gt;&lt;span style=&quot;background-color: transparent;&quot;&gt;&lt;span style=&quot;color: #017087;&quot;&gt;&lt;b&gt;Herniated Nucleus Pulposus of the Cervical Spine&lt;/b&gt;&lt;/span&gt;&lt;/span&gt;&lt;b&gt;&amp;nbsp;&amp;nbsp;&lt;/b&gt;&lt;/span&gt;&lt;span style=&quot;font-weight: bold;&quot;&gt;&amp;nbsp;&amp;nbsp; &amp;nbsp;&lt;/span&gt;&lt;span style=&quot;font-weight: bold;&quot;&gt;&amp;nbsp;&amp;nbsp; &amp;nbsp;&lt;/span&gt;&lt;span style=&quot;font-weight: bold;&quot;&gt;&amp;nbsp;&amp;nbsp; &amp;nbsp;&lt;/span&gt;&lt;span style=&quot;font-weight: bold;&quot;&gt;&amp;nbsp;&amp;nbsp; &amp;nbsp;&lt;/span&gt;&lt;span style=&quot;font-weight: bold;&quot;&gt;&amp;nbsp;&amp;nbsp; &amp;nbsp;&lt;/span&gt;&lt;span style=&quot;font-weight: bold;&quot;&gt;&amp;nbsp;&amp;nbsp; &amp;nbsp;&lt;/span&gt;&lt;span style=&quot;font-weight: bold;&quot;&gt;&amp;nbsp; &amp;nbsp;&amp;nbsp;&lt;/span&gt;&lt;/span&gt;&lt;/div&gt;&lt;/div&gt;&lt;div&gt;&lt;br /&gt;&lt;/div&gt;&lt;/div&gt;&lt;div&gt;&lt;div&gt;&lt;i&gt;Summary&lt;/i&gt;: A 45-year-old male smoker presents with 3 months of electric-like pains beginning in his neck and radiating down his right arm. He has weakness in his biceps. Radiographs reveal only mild cervical spondylosis.&lt;/div&gt;&lt;div&gt;&lt;ul style=&quot;text-align: left;&quot;&gt;&lt;li&gt;&lt;b&gt;Most likely diagnosis:&lt;/b&gt; Cervical radiculopathy due to a herniated nucleus pulposus (HNP).&lt;/li&gt;&lt;li&gt;&lt;b&gt;Next step in therapy:&lt;/b&gt; Nonoperative (conservative) management with overthe- counter medications including acetaminophen and nonsteroidal antiinflammatory drugs (NSAIDs) and possibly muscle relaxants. Short courses of oral corticosteroids, such as methylprednisolone (Medrol) dose packs, are also an option. Brief courses of low-dose narcotics for breakthrough pain may be prescribed, and recommendations for stretching exercises and application of cold and/or warm compresses may be given. Smoking cessation is indicated, as tobacco and nicotine use has been associated with the pathogenesis of degenerative changes in the disk and HNP.&lt;/li&gt;&lt;/ul&gt;&lt;/div&gt;&lt;/div&gt;&lt;div&gt;&lt;br /&gt;&lt;/div&gt;&lt;div&gt;&lt;br /&gt;&lt;/div&gt;&lt;div&gt;&lt;div&gt;&lt;b&gt;&lt;span style=&quot;color: #017087; font-size: medium;&quot;&gt;ANALYSIS&lt;/span&gt;&lt;/b&gt;&lt;/div&gt;&lt;div&gt;&lt;b&gt;&lt;span style=&quot;font-size: medium;&quot;&gt;&lt;i&gt;Objectives&lt;/i&gt;&lt;/span&gt;&lt;/b&gt;&lt;/div&gt;&lt;/div&gt;&lt;div&gt;&lt;div&gt;&lt;ol style=&quot;text-align: left;&quot;&gt;&lt;li&gt;Develop a diagnostic approach to cervical neck pain.&lt;/li&gt;&lt;li&gt;Understand the workup for radiculopathic pain.&lt;/li&gt;&lt;li&gt;Be familiar with the natural history of HNP and the indications for surgery.&lt;/li&gt;&lt;/ol&gt;&lt;/div&gt;&lt;/div&gt;&lt;div&gt;&lt;br /&gt;&lt;/div&gt;&lt;div&gt;&lt;br /&gt;&lt;/div&gt;&lt;div&gt;&lt;div&gt;&lt;b&gt;&lt;i&gt;&lt;span style=&quot;font-size: medium;&quot;&gt;Considerations&lt;/span&gt;&lt;/i&gt;&lt;/b&gt;&lt;/div&gt;&lt;div&gt;&lt;br /&gt;&lt;/div&gt;&lt;div&gt;This 45-year-old man developed neck pain insidiously. Although he has slight weakness, he does not have any symptoms requiring urgent intervention. A careful history eliminates the possibilities of radiculopathy as secondary to trauma, infection, or other inflammatory neoplastic processes. The physician must be sure to localize the pathology to the cervical spine, as ipsilateral shoulder and upperextremity pathologies such as brachial plexus injury or peripheral nerve entrapment may present similarly. On physical exam, strength, sensation, and reflexes are tested in all applicable myotomal and dermatomal distributions of the bilateral upper and lower extremities. This patient exhibits mild weakness consistent with a right-sided C6 nerve root pathology. Special tests, like the Spurling test, help diagnose nerve root compression at the cervical level. Plain radiographs, although not always necessary on first presentation, are relatively inexpensive and painless and are a simple and convenient way to evaluate for processes other than an HNP, such as fracture or facet arthropathy.&lt;/div&gt;&lt;div&gt;&lt;br /&gt;&lt;/div&gt;&lt;div&gt;&lt;span&gt;&amp;nbsp;&amp;nbsp; &amp;nbsp;&lt;/span&gt;Given this patient’s history, radiographs consistent with only mild degenerative changes and physical exam findings that include weakness with resisted elbow flexion, positive Spurling test, and blunted biceps reflex, he likely has a C6 radiculopathy due to a C5-6 HNP. There is no clear consensus on whether further imaging is necessary in this patient. Indeed the great majority of patients with spinal radiculopathies experience a spontaneous resolution of symptoms with little more than over-the-counter therapies. Because this patient has yet to try NSAIDs and other conservative treatments, he may be trialed on such therapies before further workup is performed. Should the patient experience a dramatic worsening of symptoms or simply a failure to improve on this regimen, magnetic resonance imaging (MRI) of his cervical spine should be obtained.&lt;/div&gt;&lt;/div&gt;&lt;div&gt;&lt;br /&gt;&lt;/div&gt;&lt;div&gt;&lt;br /&gt;&lt;/div&gt;&lt;div&gt;&lt;div&gt;&lt;div&gt;&lt;div&gt;&lt;b&gt;&lt;span style=&quot;font-size: medium;&quot;&gt;APPROACH TO:&lt;/span&gt;&lt;/b&gt;&lt;/div&gt;&lt;div&gt;&lt;span style=&quot;background-color: #daebf0; color: #017087; font-size: medium;&quot;&gt;&lt;span&gt;&lt;span style=&quot;background-color: transparent;&quot;&gt;&lt;b&gt;Cervical Radiculopathy&lt;/b&gt;&lt;/span&gt;&lt;b&gt;&amp;nbsp;&lt;/b&gt;&lt;/span&gt;&lt;span style=&quot;font-weight: bold;&quot;&gt;&amp;nbsp;&amp;nbsp; &amp;nbsp;&lt;/span&gt;&lt;span style=&quot;font-weight: bold;&quot;&gt;&amp;nbsp;&amp;nbsp; &amp;nbsp;&lt;/span&gt;&lt;span style=&quot;font-weight: bold;&quot;&gt;&amp;nbsp;&amp;nbsp; &amp;nbsp;&lt;/span&gt;&lt;span style=&quot;font-weight: bold;&quot;&gt;&amp;nbsp;&amp;nbsp; &amp;nbsp;&lt;/span&gt;&lt;span style=&quot;font-weight: bold;&quot;&gt;&amp;nbsp;&amp;nbsp; &amp;nbsp;&lt;/span&gt;&lt;span style=&quot;font-weight: bold;&quot;&gt;&amp;nbsp;&amp;nbsp; &amp;nbsp;&lt;/span&gt;&lt;span style=&quot;font-weight: bold;&quot;&gt;&amp;nbsp;&amp;nbsp; &amp;nbsp;&lt;/span&gt;&lt;span style=&quot;font-weight: bold;&quot;&gt;&amp;nbsp;&amp;nbsp;&amp;nbsp;&lt;/span&gt;&lt;span style=&quot;font-weight: bold;&quot;&gt;&amp;nbsp;&lt;/span&gt;&lt;/span&gt;&lt;/div&gt;&lt;/div&gt;&lt;div&gt;&lt;br /&gt;&lt;/div&gt;&lt;div&gt;&lt;b&gt;&lt;span style=&quot;color: #017087; font-size: medium;&quot;&gt;DEFINITIONS&lt;/span&gt;&lt;/b&gt;&lt;/div&gt;&lt;/div&gt;&lt;div&gt;&lt;br /&gt;&lt;/div&gt;&lt;/div&gt;&lt;div&gt;&lt;div&gt;&lt;b&gt;RADICULOPATHY:&lt;/b&gt; Clinical findings of sensory or motor dysfunction in the distribution of a specific nerve root. Radiculopathies are typically caused by nerve root compression.&lt;/div&gt;&lt;div&gt;&lt;b&gt;&lt;br /&gt;&lt;/b&gt;&lt;/div&gt;&lt;div&gt;&lt;b&gt;HOFFMAN SIGN: &lt;/b&gt;A physical exam finding suggestive of upper motor neuron dysfunction in the upper extremity. Hoffman sign is elicited by repeatedly “flicking” the tip of the patient’s index or middle finger and observing for reactive flexion of the thumb or fingers. It is analogous to the Babinski sign of the lower extremities.&lt;/div&gt;&lt;div&gt;&lt;b&gt;&lt;br /&gt;&lt;/b&gt;&lt;/div&gt;&lt;div&gt;&lt;b&gt;SPURLING SIGN: &lt;/b&gt;A physical exam finding elicited with neck extension and rotation toward the affected side. The test is positive when the pattern of radicular pain into the upper extremity is reproduced. Spurling sign is highly suggestive of a cervical spine etiology for upper-extremity pain and should be negative in situations in which radicular-like symptoms are caused by brachial plexus or peripheral nerve compressive etiologies.&lt;/div&gt;&lt;/div&gt;&lt;div&gt;&lt;br /&gt;&lt;/div&gt;&lt;div&gt;&lt;br /&gt;&lt;/div&gt;&lt;div&gt;&lt;b&gt;&lt;span style=&quot;color: #017087; font-size: medium;&quot;&gt;CLINICAL APPROACH&lt;/span&gt;&lt;/b&gt;&lt;/div&gt;&lt;div&gt;&lt;br /&gt;&lt;/div&gt;&lt;div&gt;&lt;div&gt;&lt;b&gt;&lt;i&gt;&lt;span style=&quot;font-size: medium;&quot;&gt;Anatomy and Pathophysiology&lt;/span&gt;&lt;/i&gt;&lt;/b&gt;&lt;/div&gt;&lt;div&gt;&lt;br /&gt;&lt;/div&gt;&lt;div&gt;The most common disk level of the cervical spine responsible for radiculopathic pain is C6-7, which typically manifests as symptoms attributable to C7 nerve root dysfunction. A disk herniation at the C5-6 level is the second most common location in the cervical spine.&lt;/div&gt;&lt;div&gt;&lt;br /&gt;&lt;/div&gt;&lt;div&gt;&lt;span&gt;&amp;nbsp;&amp;nbsp; &amp;nbsp;&lt;/span&gt;Cervical radiculopathy is thought to result from several mechanisms. Most commonly, acute nerve compression occurs from direct nerve root impingement by herniated soft disc material. Typical locations for HNP in the axial plane are posterolateral to the disc space and medial to the uncovertebral joint. Here, the disc-restraining posterior longitudinal ligament and annulus fibrosis are absent, and there is little to restrain an expanding disc when it is placed under pressure. Herniations here typically impinge on the exiting nerve root (ie, the C7 root at C6-7 and the C6 root at C5-6). A second mechanism for cervical nerve root compression may result from chronic disc height loss and hypertrophy of the uncovertebral joint, which is consistent with aging and degenerative disease. Discal height loss may be accompanied by bulging of the disc material or uncovertebral hypertrophy posteriorly against the spinal canal and nerve roots, leading to compression. Similar height loss and degeneration may cause stenosis within the neuroforamina, leading to compression of the exiting nerve root. At least in the acute setting, herniated disc material is thought to be inflammatory in nature and has been associated with the production of inflammatory mediators including interleukin (IL)-1 and IL-6, substance P, bradykinin, tumor necrosis factor α, and prostaglandins.&lt;/div&gt;&lt;/div&gt;&lt;div&gt;&lt;br /&gt;&lt;/div&gt;&lt;div&gt;&lt;br /&gt;&lt;/div&gt;&lt;div&gt;&lt;div&gt;&lt;b&gt;&lt;i&gt;&lt;span style=&quot;font-size: medium;&quot;&gt;History and Physical Exam&lt;/span&gt;&lt;/i&gt;&lt;/b&gt;&lt;/div&gt;&lt;div&gt;&lt;br /&gt;&lt;/div&gt;&lt;div&gt;A thorough history and physical exam should be performed, with special attention to the origin of the patient’s symptoms. A history of traumatic onset may suggest fracture or traumatic disc herniation. A history of neoplastic processes or systemic inflammatory states may be suggestive of nerve compression secondary to tumor growth, pathologic fracture, or epidural abscess. Patients may complain of fever, chills, weight loss, or nocturnal malaise in the setting of radicular pain and neurologic dysfunction. In such cases, workup should include advanced imaging of the cervical spine (typically an MRI) and laboratory studies such as white blood cell count, erythrocyte sedimentation rate, and C-reactive protein. If lesions are suggested or suspected, a more generalized exam for metastatic disease involving computed tomography (CT) or nuclear scanning may be required. Common symptoms consistent with HNP of the cervical spine include complaints of occipital headaches; neck pain, worse with motion; and pain in the neck, shoulder, and arms. Root compression often manifests with paresthesias in the affected extremity. It is important to realize, however, that there is a significant overlap and variance in the dermatomal distributions of nerve roots. Therefore, paresthesias associated with single-nerve root compression may not always follow the textbook dermatomal maps.&lt;/div&gt;&lt;div&gt;&lt;br /&gt;&lt;/div&gt;&lt;div&gt;&lt;span&gt;&amp;nbsp;&amp;nbsp; &amp;nbsp;&lt;/span&gt;On exam, motor strength is graded on a 0 to 5 scale. Sensory testing should evaluate for both dorsal column and spinothalamic tract function by evaluation of positional and pain/temperature sensations, respectively. Evidence of upper motor neuropathy should be ruled out. For detailed instruction on performing the spinal exam, please refer to the “Approach to the Orthopaedic Patient” section, located near the beginning of this text.&lt;/div&gt;&lt;/div&gt;&lt;div&gt;&lt;br /&gt;&lt;/div&gt;&lt;div&gt;&lt;br /&gt;&lt;/div&gt;&lt;div&gt;&lt;div&gt;&lt;i&gt;&lt;span style=&quot;font-size: medium;&quot;&gt;&lt;b&gt;Differential Diagnosis&lt;/b&gt;&lt;/span&gt;&lt;/i&gt;&lt;/div&gt;&lt;div&gt;&lt;br /&gt;&lt;/div&gt;&lt;div&gt;Before a clinical diagnosis of cervical radiculopathy can be made, several other disease processes must be considered and then ruled out. Most notably, brachial plexus injuries and peripheral nerve entrapment syndromes of the upper extremity may closely resemble the sensory and motor symptomatology of nerve root compression. Other sources of radicular-like symptoms include degenerative changes, stenosis, and space-occupying lesions, such as abscesses or tumors. The presence of upper motor neuron signs (suggested by a positive Hoffman sign or gait changes) would point to either a cervical stenosis causing cord compression or other neurologic conditions such as amyotrophic lateral sclerosis.&lt;/div&gt;&lt;/div&gt;&lt;div&gt;&lt;br /&gt;&lt;/div&gt;&lt;div&gt;&lt;br /&gt;&lt;/div&gt;&lt;div&gt;&lt;div&gt;&lt;b&gt;&lt;i&gt;&lt;span style=&quot;font-size: medium;&quot;&gt;Imaging&lt;/span&gt;&lt;/i&gt;&lt;/b&gt;&lt;/div&gt;&lt;div&gt;&lt;br /&gt;&lt;/div&gt;&lt;div&gt;Once fractures and other pathology are ruled out by plain x-ray, MRI may be considered a next step in the further workup of cervical radicular pain. It must be stated again, however, that because the great majority of these patients experience a spontaneous resolution of symptoms, not every patient presenting with radicular symptoms needs to undergo urgent MRI. &lt;b&gt;In patients with unremitting pain that&amp;nbsp;&lt;/b&gt;&lt;b&gt;has failed conservative management, continued or progressive weakness, neurologic&amp;nbsp;&lt;/b&gt;&lt;b&gt;decompensation, or bowel or bladder changes, MRI is the gold standard to&amp;nbsp;&lt;/b&gt;&lt;b&gt;evaluate spinal cord and root compression in the absence of contraindications.&amp;nbsp;&lt;/b&gt;On T2-weighted MRI, cerebrospinal fluid is bright and clearly distinguishes areas of compression and patency. In patients for whom MRI is contraindicated, most commonly those with pacemakers or interventions for intracerebral aneurysms, CT myelography may offer a clear visualization of the spinal cord and roots.&lt;/div&gt;&lt;/div&gt;&lt;div&gt;&lt;br /&gt;&lt;/div&gt;&lt;div&gt;&lt;br /&gt;&lt;/div&gt;&lt;div&gt;&lt;b style=&quot;color: #017087;&quot;&gt;&lt;span style=&quot;font-size: medium;&quot;&gt;TREATMENT&lt;/span&gt;&lt;/b&gt;&lt;/div&gt;&lt;div&gt;&lt;br /&gt;&lt;/div&gt;&lt;div&gt;&lt;div&gt;&lt;b&gt;&lt;i&gt;&lt;span style=&quot;font-size: medium;&quot;&gt;Nonoperative Treatment&lt;/span&gt;&lt;/i&gt;&lt;/b&gt;&lt;/div&gt;&lt;div&gt;&lt;br /&gt;&lt;/div&gt;&lt;div&gt;Acute cervical radiculopathy typically has a self-limited course. In fact, 75% of these patients improve without surgical intervention. &lt;b&gt;The first line against cervical&amp;nbsp;&lt;/b&gt;&lt;b&gt;radiculopathic pain should be NSAIDs and/or acetaminophen.&lt;/b&gt; NSAIDs play a distinct role in attenuating the painful local inflammatory response resulting from disc herniations and their inflammatory effects on nerve roots and may decrease pressure around affected nerve roots. Low-dose narcotic analgesics, although not commonly recommended for chronic treatment, may be necessary to control acute pain that is refractory to NSAIDs and acetaminophen. Adjunctive treatment with muscle relaxants such as cyclobenzaprine may help decrease associated painful muscle spasms and may decrease narcotic requirements in certain patients. &lt;b&gt;Although current literature&amp;nbsp;&lt;/b&gt;&lt;b&gt;fails to demonstrate their clear efficacy, brief courses of oral corticosteroids&amp;nbsp;&lt;/b&gt;&lt;b&gt;are sometimes prescribed to decrease inflammatory responses to HNP.&amp;nbsp;&lt;/b&gt;Adjunctive therapies such as physical therapy, massage therapy, and the application of heat or ice packs are not strongly supported by the literature, but may be used if helpful to the patient.&lt;/div&gt;&lt;div&gt;&lt;br /&gt;&lt;/div&gt;&lt;div&gt;&lt;span&gt;&amp;nbsp;&amp;nbsp; &amp;nbsp;&lt;/span&gt;When conservative measures fail, epidural steroid injections may offer some transient relief but have not been shown to alter the natural history of the disease process. Localized steroid injections may have an anti-inflammatory effect on irritated nerve roots and may play a role in the inhibition of pain pathways. The true efficacy of epidural injections is debated and varies greatly between individuals.&lt;/div&gt;&lt;/div&gt;&lt;div&gt;&lt;br /&gt;&lt;/div&gt;&lt;div&gt;&lt;br /&gt;&lt;/div&gt;&lt;div&gt;&lt;div&gt;&lt;b&gt;&lt;i&gt;&lt;span style=&quot;font-size: medium;&quot;&gt;Surgical Treatment&lt;/span&gt;&lt;/i&gt;&lt;/b&gt;&lt;/div&gt;&lt;div&gt;&lt;br /&gt;&lt;/div&gt;&lt;div&gt;When nonsurgical treatment fails to relieve symptoms, or if neurologic deficits progressively worsen, surgical decompression may be necessary. &lt;b&gt;As a general rule, surgical&amp;nbsp;&lt;/b&gt;&lt;b&gt;approaches to the cervical spine should be directed “where the pathology&amp;nbsp;&lt;/b&gt;&lt;b&gt;is.”&lt;/b&gt; Because herniation originates in the disc space, an anterior surgical approach is preferred in the setting of central and posterolateral anterior compressive pathology. Anterior cervical discectomy and fusion (ACDF) is the most common surgical treatment for cervical HNP. For isolated single-level nerve root compression, posterior approaches to the cervical spine may be used if the pathology is accessible through the neural foramen. Posterior laminoforaminotomy relieves pressure on the nerve root by unroofing the neuroforamen and allowing the root to decompress. This technique preserves motion between vertebral levels because the disc is left intact, and thus fusion of vertebral segments is unnecessary. Surgical outcomes for relief of radicular pain range from 80% to 90% with either anterior or posterior approaches.&lt;/div&gt;&lt;/div&gt;&lt;div&gt;&lt;br /&gt;&lt;/div&gt;&lt;div&gt;&lt;div&gt;&lt;span&gt;&amp;nbsp;&amp;nbsp; &amp;nbsp;&lt;/span&gt;Lastly, a relatively new and controversial technique in the treatment of singlelevel disease is cervical disc arthroplasty (replacement). Although limited in both application and supporting data at this time, this promising procedure may offer the best of both worlds: preserving motion between segments while allowing for a thorough discectomy and decompression.&lt;/div&gt;&lt;/div&gt;&lt;div&gt;&lt;br /&gt;&lt;/div&gt;&lt;div&gt;&lt;br /&gt;&lt;/div&gt;&lt;div&gt;&lt;div&gt;&lt;b&gt;&lt;i&gt;&lt;span style=&quot;font-size: medium;&quot;&gt;Complications&lt;/span&gt;&lt;/i&gt;&lt;/b&gt;&lt;/div&gt;&lt;div&gt;&lt;br /&gt;&lt;/div&gt;&lt;div&gt;Although ACDF procedures typically have good outcomes, they are not without potential complications. The anterior approach to the cervical spine puts several vulnerable neurovascular structures at risk, including the recurrent laryngeal nerve (1% risk of injury with ACDF) and, to a lesser degree, the hypoglossal nerve, vagal nerve, and the carotid arteries. Horner syndrome is “classically” characterized by ptosis, anhidrosis, and miosis and results from injury to the sympathetic chain, located laterally to the longus colli muscles at the C6 level. Finally, pseudarthrosis, or the failure of bony fusion, may occur at rates as high as 5% to 10% for singlelevel fusions. Rates dramatically increase to 30% to 40% for multiple-level fusions, but few pseudoarthroses are actually symptomatic or require revision work. Tobacco and chronic corticosteroid use is a significant risk factor for the development of pseudoarthrosis.&lt;/div&gt;&lt;/div&gt;&lt;div&gt;&lt;br /&gt;&lt;/div&gt;&lt;div&gt;&lt;br /&gt;&lt;/div&gt;&lt;div&gt;&lt;b&gt;&lt;span style=&quot;color: #017087; font-size: medium;&quot;&gt;&lt;u&gt;COMPREHENSION QUESTIONS&lt;/u&gt;&lt;/span&gt;&lt;/b&gt;&lt;/div&gt;&lt;div&gt;&lt;br /&gt;&lt;/div&gt;&lt;div&gt;&lt;div&gt;25.1 A 56-year-old female bodybuilder presents to your clinic after straining her neck during an overhead clean and jerk contest. She complains of numbness in her middle finger and weakness in her elbow extension. Which of the following findings would most likely be present in this patient?&lt;/div&gt;&lt;/div&gt;&lt;/div&gt;&lt;blockquote style=&quot;border: none; margin: 0px 0px 0px 40px; padding: 0px; text-align: left;&quot;&gt;&lt;div style=&quot;text-align: left;&quot;&gt;&lt;div&gt;&lt;div&gt;A. Elbow flexion weakness&lt;/div&gt;&lt;/div&gt;&lt;/div&gt;&lt;div style=&quot;text-align: left;&quot;&gt;&lt;div&gt;&lt;div&gt;B. Finger grip weakness&lt;/div&gt;&lt;/div&gt;&lt;/div&gt;&lt;div style=&quot;text-align: left;&quot;&gt;&lt;div&gt;&lt;div&gt;C. Decreased biceps reflex&lt;/div&gt;&lt;/div&gt;&lt;/div&gt;&lt;div style=&quot;text-align: left;&quot;&gt;&lt;div&gt;&lt;div&gt;D. Decreased sensation over the ring and small fingers&lt;/div&gt;&lt;/div&gt;&lt;/div&gt;&lt;div style=&quot;text-align: left;&quot;&gt;&lt;div&gt;&lt;div&gt;E. Decreased triceps reflex&lt;/div&gt;&lt;/div&gt;&lt;/div&gt;&lt;/blockquote&gt;&lt;div style=&quot;text-align: left;&quot;&gt;&lt;div&gt;&lt;br /&gt;&lt;/div&gt;&lt;div&gt;&lt;div&gt;25.2 A 49-year-old downhill skier “tweaks” his neck while loading heavy ski equipment atop his family SUV. He complains of 3 days of small finger numbness and difficulty gripping his ski poles. Which of the following nerve roots is most likely entrapped?&lt;/div&gt;&lt;/div&gt;&lt;/div&gt;&lt;blockquote style=&quot;border: none; margin: 0px 0px 0px 40px; padding: 0px; text-align: left;&quot;&gt;&lt;div style=&quot;text-align: left;&quot;&gt;&lt;div&gt;&lt;div&gt;A. C5&lt;/div&gt;&lt;/div&gt;&lt;/div&gt;&lt;div style=&quot;text-align: left;&quot;&gt;&lt;div&gt;&lt;div&gt;B. C6&lt;/div&gt;&lt;/div&gt;&lt;/div&gt;&lt;div style=&quot;text-align: left;&quot;&gt;&lt;div&gt;&lt;div&gt;C. C7&lt;/div&gt;&lt;/div&gt;&lt;/div&gt;&lt;div style=&quot;text-align: left;&quot;&gt;&lt;div&gt;&lt;div&gt;D. C8&lt;/div&gt;&lt;/div&gt;&lt;/div&gt;&lt;div style=&quot;text-align: left;&quot;&gt;&lt;div&gt;&lt;div&gt;E. T1&lt;/div&gt;&lt;/div&gt;&lt;/div&gt;&lt;/blockquote&gt;&lt;div style=&quot;text-align: left;&quot;&gt;&lt;div&gt;&lt;br /&gt;&lt;/div&gt;&lt;div&gt;&lt;div&gt;25.3 A 42-year-old construction worker has experienced 8 weeks of unremitting left biceps weakness, wrist extensor weakness, and both pain and numbness in his index finger and thumb. His pain is especially bad when turning his head to the left. He has tried oral steroids, opiates, NSAIDs, acetaminophen, acupuncture, and a local steroid injection 3 weeks ago; all have failed to provide adequate relieve. He states his biceps and wrist weakness is getting worse, as he is now unable to lift even a glass of water with his left arm. MRI confirms HNP at the left-sided C5-6 neuro foramen. What is the next best step in the management of this patient?&lt;/div&gt;&lt;/div&gt;&lt;/div&gt;&lt;blockquote style=&quot;border: none; margin: 0px 0px 0px 40px; padding: 0px; text-align: left;&quot;&gt;&lt;div style=&quot;text-align: left;&quot;&gt;&lt;div&gt;&lt;div&gt;A. Repeat corticosteroid injection at C5-6 level&lt;/div&gt;&lt;/div&gt;&lt;/div&gt;&lt;div style=&quot;text-align: left;&quot;&gt;&lt;div&gt;&lt;div&gt;B. Anterior cervical discectomy and fusion (ACDF) of C5-6&lt;/div&gt;&lt;/div&gt;&lt;/div&gt;&lt;div style=&quot;text-align: left;&quot;&gt;&lt;div&gt;&lt;div&gt;C. Increasing oral steroid dosing&lt;/div&gt;&lt;/div&gt;&lt;/div&gt;&lt;div style=&quot;text-align: left;&quot;&gt;&lt;div&gt;&lt;div&gt;D. Prescribing a hard collar for neck immobilization&lt;/div&gt;&lt;/div&gt;&lt;/div&gt;&lt;div style=&quot;text-align: left;&quot;&gt;&lt;div&gt;&lt;div&gt;E. CT scan and/or bone scan to rule out fracture or other injury&lt;/div&gt;&lt;/div&gt;&lt;/div&gt;&lt;/blockquote&gt;&lt;div style=&quot;text-align: left;&quot;&gt;&lt;div&gt;&lt;br /&gt;&lt;/div&gt;&lt;div&gt;&lt;b&gt;&lt;span style=&quot;color: #017087; font-size: medium;&quot;&gt;&lt;u&gt;&lt;br /&gt;&lt;/u&gt;&lt;/span&gt;&lt;/b&gt;&lt;/div&gt;&lt;div&gt;&lt;b&gt;&lt;span style=&quot;color: #017087; font-size: medium;&quot;&gt;&lt;u&gt;ANSWERS&lt;/u&gt;&lt;/span&gt;&lt;/b&gt;&lt;/div&gt;&lt;div&gt;&lt;br /&gt;&lt;/div&gt;&lt;div&gt;&lt;div&gt;25.1 &lt;b&gt;E.&lt;/b&gt; This patient has a C6-C7 HNP affecting the C7 nerve root, which is classically responsible for elbow extension and wrist flexion. The triceps reflex is most commonly associated with C7 nerve root pathology, making E the correct answer choice.&lt;/div&gt;&lt;div&gt;&lt;br /&gt;&lt;/div&gt;&lt;div&gt;25.2 &lt;b&gt;D.&lt;/b&gt; Grip strength and sensation to the small and sometimes ring fingers are classically attributed to the C8 nerve root, most often affected by C7-T1 disc herniations.&lt;/div&gt;&lt;div&gt;&lt;br /&gt;&lt;/div&gt;&lt;div&gt;25.3 &lt;b&gt;B.&lt;/b&gt; ACDF of C5-6 disc with fusion is an effective and highly successful procedure for patients with clearly defined symptomatology and imaging consistent with pathology. This patient has progressive neurologic changes consistent with a well-defined dermo/myotomal distribution. He has failed conservative therapy; increasing steroid dosing and even additional steroid injection are unlikely to help. Further imaging is unnecessary given the consistency between his symptoms and MRI and a lack of suspicion for further injury.&lt;/div&gt;&lt;/div&gt;&lt;div&gt;&lt;br /&gt;&lt;/div&gt;&lt;div&gt;&lt;b style=&quot;background-color: #4093a5;&quot;&gt;&amp;nbsp; &amp;nbsp;&lt;span style=&quot;font-size: medium;&quot;&gt;&amp;nbsp;&lt;/span&gt;&lt;span style=&quot;color: white;&quot;&gt;&lt;span style=&quot;font-size: medium;&quot;&gt;CLINICAL PEARLS&lt;/span&gt;&lt;span&gt;&amp;nbsp; &amp;nbsp;&amp;nbsp;&lt;/span&gt;&lt;/span&gt;&lt;/b&gt;&lt;/div&gt;&lt;div&gt;&lt;table border=&quot;1&quot; cellpadding=&quot;0&quot; cellspacing=&quot;0&quot; class=&quot;MsoTableGrid&quot; style=&quot;border-collapse: collapse; border: none; mso-border-alt: solid windowtext .5pt; mso-padding-alt: 0in 5.4pt 0in 5.4pt; mso-yfti-tbllook: 1184;&quot;&gt;
  248. &lt;tbody&gt;&lt;tr style=&quot;height: 19.15pt; mso-yfti-firstrow: yes; mso-yfti-irow: 0; mso-yfti-lastrow: yes;&quot;&gt;
  249.  &lt;td style=&quot;background: rgb(221, 237, 239); border-left: solid #017087 1.0pt; border: 1pt solid rgb(64, 147, 165); height: 19.15pt; padding: 0in 5.4pt; width: 433.7pt;&quot; valign=&quot;top&quot; width=&quot;578&quot;&gt;&lt;div style=&quot;line-height: normal; margin-bottom: 0in;&quot;&gt;&lt;span style=&quot;color: #017087;&quot;&gt;&lt;br /&gt;&lt;/span&gt;&lt;/div&gt;&lt;div style=&quot;line-height: normal; margin-bottom: 0in;&quot;&gt;&lt;span style=&quot;color: #017087;&quot;&gt;►&lt;/span&gt;&amp;nbsp;More than 75% of patients with cervical radiculopathic pain improve without surgical intervention.&lt;/div&gt;&lt;div style=&quot;line-height: normal; margin-bottom: 0in;&quot;&gt;&lt;span style=&quot;color: #017087;&quot;&gt;&lt;br /&gt;&lt;/span&gt;&lt;/div&gt;&lt;div style=&quot;line-height: normal; margin-bottom: 0in;&quot;&gt;&lt;span style=&quot;color: #017087;&quot;&gt;►&lt;/span&gt;&amp;nbsp;NSAIDs and acetaminophen are the mainstays and first line against cervical&lt;/div&gt;&lt;div style=&quot;line-height: normal; margin-bottom: 0in;&quot;&gt;radiculopathy.&lt;/div&gt;&lt;div style=&quot;line-height: normal; margin-bottom: 0in;&quot;&gt;&lt;span style=&quot;color: #017087;&quot;&gt;&lt;br /&gt;&lt;/span&gt;&lt;/div&gt;&lt;div style=&quot;line-height: normal; margin-bottom: 0in;&quot;&gt;&lt;span style=&quot;color: #017087;&quot;&gt;►&lt;/span&gt;&amp;nbsp;Beware of treating MRI findings without a careful clinical correlation. MRI has high rate of false positives, with almost 30% of normal, asymptomatic individuals &amp;gt; 40 years of age positive for HNP or foraminal stenosis on MRI.&lt;/div&gt;&lt;div style=&quot;line-height: normal; margin-bottom: 0in;&quot;&gt;&lt;br /&gt;&lt;/div&gt;
  250.  &lt;/td&gt;
  251. &lt;/tr&gt;
  252. &lt;/tbody&gt;&lt;/table&gt;&lt;br /&gt;&lt;/div&gt;&lt;/div&gt;
  253.  
  254. &lt;details close=&quot;&quot;&gt;
  255.  &lt;summary&gt;&lt;b&gt;&lt;span style=&quot;color: #00788b;&quot;&gt;REFERENCES&lt;/span&gt;&lt;/b&gt;&lt;/summary&gt;
  256.  &lt;p&gt;Nordin M, Carragee EJ, Hogg-Johnson S, et al, for the Bone and Joint Decade 2000-2010 Task Force on
  257. Neck Pain and Its Associated Disorders. Assessment of neck pain and its associated disorders: results
  258. of the Bone and Joint Decade 2000-2010 Task Force on Neck Pain and Its Associated Disorders.
  259. Spine . 2008;33(4 suppl):S101-S122.&amp;nbsp;&lt;/p&gt;&lt;p&gt;Rhee JM, Yoon T, Riew D. Cervical radiculopathy. J Am Acad Orthop Surg. 2007;15:486-494.&amp;nbsp;&lt;/p&gt;&lt;p&gt;Shelerud RA, Paynter KS. Rarer causes of radiculopathy: spinal tumors, infections, and other unusual
  260. causes. Phys Med Rehabil Clin N Am . 2002;13:645-696.&amp;nbsp;&lt;/p&gt;&lt;p&gt;Tong HC, Haig AJ, Yamakawa K. The Spurling test and cervical radiculopathy. Spine . 2002;27:156-159.&lt;/p&gt;
  261. &lt;/details&gt;&lt;/div&gt;</content><link rel='replies' type='application/atom+xml' href='https://medical-phd.blogspot.com/feeds/7131228201821791542/comments/default' title='Post Comments'/><link rel='replies' type='text/html' href='https://medical-phd.blogspot.com/2022/03/herniated-nucleus-pulposus-of-the-cervical-spine-case-file.html#comment-form' title='0 Comments'/><link rel='edit' type='application/atom+xml' href='https://www.blogger.com/feeds/3302710210518459416/posts/default/7131228201821791542'/><link rel='self' type='application/atom+xml' href='https://www.blogger.com/feeds/3302710210518459416/posts/default/7131228201821791542'/><link rel='alternate' type='text/html' href='https://medical-phd.blogspot.com/2022/03/herniated-nucleus-pulposus-of-the-cervical-spine-case-file.html' title='Herniated Nucleus Pulposus of the Cervical Spine Case File'/><author><name>Medical Group</name><uri>http://www.blogger.com/profile/01780656800578397393</uri><email>noreply@blogger.com</email><gd:image rel='http://schemas.google.com/g/2005#thumbnail' width='32' height='32' src='//blogger.googleusercontent.com/img/b/R29vZ2xl/AVvXsEiMuROkBDv6yw4SmxGqRA5BPQhOXSwFld-Gufmy4kUqfpoAGQRl7cv-3naew6wjAnCjRFSe_FKX8z7CdE6cFKxxBpnqBuwUTSr-15xd3mFGcr_wJOX2y73G_zJCGND98Q/s220/Final-logo.jpg'/></author><thr:total>0</thr:total><georss:featurename>United States</georss:featurename><georss:point>37.09024 -95.712891</georss:point><georss:box>8.780006163821156 -130.869141 65.400473836178847 -60.556641</georss:box></entry><entry><id>tag:blogger.com,1999:blog-3302710210518459416.post-1557492615936300481</id><published>2022-03-25T07:58:00.004-07:00</published><updated>2022-03-25T07:58:29.281-07:00</updated><category scheme="http://www.blogger.com/atom/ns#" term="Case Files"/><category scheme="http://www.blogger.com/atom/ns#" term="Orthopaedic Surgery Case File"/><title type='text'>Ulnar Collateral Ligament Injury of the Elbow Case File</title><content type='html'>&lt;div class=&quot;scrollbox&quot;&gt;
  262.  
  263. &lt;div style=&quot;text-align: left;&quot;&gt;&lt;b&gt;&lt;span style=&quot;font-size: large;&quot;&gt;Ulnar Collateral Ligament Injury of the Elbow Case File&lt;/span&gt;&lt;/b&gt;&lt;/div&gt;&lt;div&gt;Eugene C. Toy, MD, Andrew J. Rosenbaum, MD, Timothy T. Roberts, MD, Joshua S. Dines, MD&lt;/div&gt;&lt;div&gt;&lt;br /&gt;&lt;/div&gt;&lt;div&gt;&lt;b&gt;&lt;span style=&quot;color: #017087; font-size: medium;&quot;&gt;CASE 24&lt;/span&gt;&lt;/b&gt;&lt;/div&gt;&lt;div&gt;&lt;div&gt;A 21-year-old right-hand dominant college baseball pitcher presents to your office with a 1-month history of right elbow pain. He reports that the pain is localized to the medial side of the elbow and that his symptoms are exacerbated by any overhead throwing activities. He has recently noticed a significant change in throwing accuracy and states that his throwing velocity has significantly diminished (98 mph to 86 mph). He denies any associated mechanical symptoms such as locking or catching within the elbow; however, he does report occasional numbness and tingling in his ring and small fingers. Physical examination demonstrates focal tenderness over the medial epicondyle as well as the flexor-pronator mass. He has approximately 5 degrees less elbow extension as compared with the contralateral elbow. He has significant pain with valgus stress applied to the elbow.&lt;/div&gt;&lt;div&gt;&lt;br /&gt;&lt;/div&gt;&lt;div&gt;&lt;span style=&quot;color: #017087;&quot;&gt;►&lt;/span&gt;&amp;nbsp;What is your most likely diagnosis?&lt;/div&gt;&lt;div&gt;&lt;span style=&quot;color: #017087;&quot;&gt;►&lt;/span&gt;&amp;nbsp;What is your next diagnostic step?&lt;/div&gt;&lt;div&gt;&lt;span style=&quot;color: #017087;&quot;&gt;►&lt;/span&gt;&amp;nbsp;What is the next step in therapy?&lt;/div&gt;&lt;/div&gt;&lt;div&gt;&lt;br /&gt;&lt;/div&gt;&lt;div&gt;&lt;br /&gt;&lt;/div&gt;&lt;div&gt;&lt;div&gt;&lt;div&gt;&lt;b&gt;&lt;span style=&quot;font-size: medium;&quot;&gt;ANSWER TO CASE 24:&lt;/span&gt;&lt;/b&gt;&lt;/div&gt;&lt;div&gt;&lt;span style=&quot;background-color: #daebf0; font-size: medium;&quot;&gt;&lt;span&gt;&lt;span style=&quot;background-color: transparent;&quot;&gt;&lt;span style=&quot;color: #017087;&quot;&gt;&lt;b&gt;Ulnar Collateral Ligament Injury of the Elbow&lt;/b&gt;&lt;/span&gt;&lt;/span&gt;&lt;b&gt;&amp;nbsp;&amp;nbsp;&lt;/b&gt;&lt;/span&gt;&lt;span style=&quot;font-weight: bold;&quot;&gt;&amp;nbsp;&amp;nbsp; &amp;nbsp;&lt;/span&gt;&lt;span style=&quot;font-weight: bold;&quot;&gt;&amp;nbsp;&amp;nbsp; &amp;nbsp;&lt;/span&gt;&lt;span style=&quot;font-weight: bold;&quot;&gt;&amp;nbsp;&amp;nbsp; &amp;nbsp;&lt;/span&gt;&lt;span style=&quot;font-weight: bold;&quot;&gt;&amp;nbsp;&amp;nbsp; &amp;nbsp;&lt;/span&gt;&lt;span style=&quot;font-weight: bold;&quot;&gt;&amp;nbsp;&amp;nbsp; &amp;nbsp;&lt;/span&gt;&lt;span style=&quot;font-weight: bold;&quot;&gt;&amp;nbsp;&amp;nbsp; &amp;nbsp;&lt;/span&gt;&lt;span style=&quot;font-weight: bold;&quot;&gt;&amp;nbsp; &amp;nbsp;&amp;nbsp;&lt;/span&gt;&lt;/span&gt;&lt;/div&gt;&lt;/div&gt;&lt;div&gt;&lt;br /&gt;&lt;/div&gt;&lt;/div&gt;&lt;div&gt;&lt;div&gt;&lt;i&gt;Summary&lt;/i&gt;: A 21-year-old right-hand dominant college baseball pitcher presents with a 1-month history of right elbow pain during overhead throwing activities and complaints of decreased athletic performance. On examination, he has focal tenderness over the medial epicondyle and the flexor pronator mass, as well as reproducible pain with a valgus stress applied to the right elbow.&lt;/div&gt;&lt;div&gt;&lt;ul style=&quot;text-align: left;&quot;&gt;&lt;li&gt;&lt;b&gt;Most likely diagnosis:&lt;/b&gt; Ulnar collateral ligament (UCL) tear.&lt;/li&gt;&lt;li&gt;&lt;b&gt;Next diagnostic step:&lt;/b&gt; Standard radiographs with or without “throwers series.”&lt;/li&gt;&lt;li&gt;&lt;b&gt;Next step in therapy:&lt;/b&gt; UCL reconstruction.&lt;/li&gt;&lt;/ul&gt;&lt;/div&gt;&lt;/div&gt;&lt;div&gt;&lt;br /&gt;&lt;/div&gt;&lt;div&gt;&lt;br /&gt;&lt;/div&gt;&lt;div&gt;&lt;div&gt;&lt;b&gt;&lt;span style=&quot;color: #017087; font-size: medium;&quot;&gt;ANALYSIS&lt;/span&gt;&lt;/b&gt;&lt;/div&gt;&lt;div&gt;&lt;b&gt;&lt;span style=&quot;font-size: medium;&quot;&gt;&lt;i&gt;Objectives&lt;/i&gt;&lt;/span&gt;&lt;/b&gt;&lt;/div&gt;&lt;/div&gt;&lt;div&gt;&lt;div&gt;&lt;ol style=&quot;text-align: left;&quot;&gt;&lt;li&gt;Review relevant elbow anatomy and biomechanics as it pertains to the throwing athlete and recognize common associated pathology in athletes who present with UCL tears.&lt;/li&gt;&lt;li&gt;Discuss a diagnostic approach to UCL injury in throwing athletes, including common physical exam findings and the specific role of standard and advanced imaging techniques.&lt;/li&gt;&lt;li&gt;Be familiar with the available treatment modalities and the primary principles of nonoperative (physical therapy) and operative management (UCL reconstruction).&lt;/li&gt;&lt;/ol&gt;&lt;/div&gt;&lt;/div&gt;&lt;div&gt;&lt;br /&gt;&lt;/div&gt;&lt;div&gt;&lt;br /&gt;&lt;/div&gt;&lt;div&gt;&lt;div&gt;&lt;i&gt;&lt;span style=&quot;font-size: medium;&quot;&gt;&lt;b&gt;Considerations&lt;/b&gt;&lt;/span&gt;&lt;/i&gt;&lt;/div&gt;&lt;div&gt;&lt;br /&gt;&lt;/div&gt;&lt;div&gt;The UCL is composed of 3 distinct bundles, namely the anterior bundle, the posterior bundle, and the oblique bundle (ie, transverse ligament). The &lt;b&gt;anterior bundle&lt;/b&gt; serves as the &lt;b&gt;primary stabilizer to valgus stress&lt;/b&gt; when the elbow is in a flexed position. To understand the contribution of the UCL to elbow stability during throwing, it is important to be familiar with the &lt;b&gt;6 phases of the baseball pitch.&lt;/b&gt; These highly coordinated phases include the (1) wind-up, (2) early arm-cocking, (3) late arm-cocking, (4) acceleration, (5) deceleration, and (6) follow-through phases. Valgus force generated at the elbow during the throwing motion is &lt;b&gt;highest during the late cocking and early acceleration phases.&lt;/b&gt;&lt;/div&gt;&lt;div&gt;&lt;br /&gt;&lt;/div&gt;&lt;div&gt;&lt;span&gt;&amp;nbsp;&amp;nbsp; &amp;nbsp;&lt;/span&gt;Throwing athletes place significant functional demands on the UCL, and repetitive overuse can result in chronic injury to the ligament. These injuries were once considered career-ending; however, improved understanding of elbow biomechanics and throwing kinematics has resulted in successful clinical outcomes in the majority of patients who undergo surgical reconstruction of the UCL. The 21-year-old baseball pitcher in this clinical scenario developed a UCL tear from repetitive valgus force placed on the UCL during pitching. Overhead throwing can produce forces that &lt;b&gt;exceed the ligament’s tensile strength,&lt;/b&gt; thus resulting in ligament rupture and debilitating medial elbow pain. UCL reconstruction is the principal procedure to restore valgus stability and relieve medial elbow pain in the setting of UCL injury.&lt;/div&gt;&lt;/div&gt;&lt;div&gt;&lt;br /&gt;&lt;/div&gt;&lt;div&gt;&lt;br /&gt;&lt;/div&gt;&lt;div&gt;&lt;div&gt;&lt;div&gt;&lt;b&gt;&lt;span style=&quot;font-size: medium;&quot;&gt;APPROACH TO:&lt;/span&gt;&lt;/b&gt;&lt;/div&gt;&lt;div&gt;&lt;span style=&quot;background-color: #daebf0; color: #017087; font-size: medium;&quot;&gt;&lt;span&gt;&lt;span style=&quot;background-color: transparent;&quot;&gt;&lt;b&gt;UCL Injury of the Elbow&lt;/b&gt;&lt;/span&gt;&lt;b&gt;&amp;nbsp;&lt;/b&gt;&lt;/span&gt;&lt;span style=&quot;font-weight: bold;&quot;&gt;&amp;nbsp;&amp;nbsp; &amp;nbsp;&lt;/span&gt;&lt;span style=&quot;font-weight: bold;&quot;&gt;&amp;nbsp;&amp;nbsp; &amp;nbsp;&lt;/span&gt;&lt;span style=&quot;font-weight: bold;&quot;&gt;&amp;nbsp;&amp;nbsp; &amp;nbsp;&lt;/span&gt;&lt;span style=&quot;font-weight: bold;&quot;&gt;&amp;nbsp;&amp;nbsp; &amp;nbsp;&lt;/span&gt;&lt;span style=&quot;font-weight: bold;&quot;&gt;&amp;nbsp;&amp;nbsp; &amp;nbsp;&lt;/span&gt;&lt;span style=&quot;font-weight: bold;&quot;&gt;&amp;nbsp;&amp;nbsp; &amp;nbsp;&lt;/span&gt;&lt;span style=&quot;font-weight: bold;&quot;&gt;&amp;nbsp;&amp;nbsp; &amp;nbsp;&lt;/span&gt;&lt;span style=&quot;font-weight: bold;&quot;&gt;&amp;nbsp;&amp;nbsp;&amp;nbsp;&lt;/span&gt;&lt;span style=&quot;font-weight: bold;&quot;&gt;&amp;nbsp;&lt;/span&gt;&lt;/span&gt;&lt;/div&gt;&lt;/div&gt;&lt;div&gt;&lt;br /&gt;&lt;/div&gt;&lt;div&gt;&lt;b&gt;&lt;span style=&quot;color: #017087; font-size: medium;&quot;&gt;DEFINITIONS&lt;/span&gt;&lt;/b&gt;&lt;/div&gt;&lt;/div&gt;&lt;div&gt;&lt;br /&gt;&lt;/div&gt;&lt;div&gt;&lt;div&gt;&lt;b&gt;LIGAMENT:&lt;/b&gt; A dense connective tissue structure comprising primarily type I collagen with the primary function of restricting joint motion (ie, stabilize joints).&lt;/div&gt;&lt;div&gt;&lt;b&gt;&lt;br /&gt;&lt;/b&gt;&lt;/div&gt;&lt;div&gt;&lt;b&gt;VALGUS STRESS:&lt;/b&gt; Refers to a force at the elbow that is produced with outward angulation (ie, away from the body’s midline) of the distal segment of the arm.&lt;/div&gt;&lt;div&gt;&lt;b&gt;&lt;br /&gt;&lt;/b&gt;&lt;/div&gt;&lt;div&gt;&lt;b&gt;TENSILE STRENGTH:&lt;/b&gt; A measure of a material’s ability to resist a specific force that tends to tear it apart. It is expressed as the maximum stress/force that the material can withstand without disruption.&lt;/div&gt;&lt;/div&gt;&lt;div&gt;&lt;br /&gt;&lt;/div&gt;&lt;div&gt;&lt;br /&gt;&lt;/div&gt;&lt;div&gt;&lt;b&gt;&lt;span style=&quot;color: #017087; font-size: medium;&quot;&gt;CLINICAL APPROACH&lt;/span&gt;&lt;/b&gt;&lt;/div&gt;&lt;div&gt;&lt;br /&gt;&lt;/div&gt;&lt;div&gt;&lt;div&gt;When obtaining a history from a throwing athlete who presents with medial elbow pain, it is important to obtain specific information regarding the athlete’s sport (baseball, softball, javelin, etc.), level of participation, and any recent changes in the athlete’s training routine (ie, increase in pitch count, types of pitches thrown). In general, patients can present with an acute tear of the ligament, characterized by the sudden onset of medial elbow pain accompanied by a &lt;b&gt;“popping”&lt;/b&gt; sensation, or with chronic overuse injuries, which are typically characterized by persistent bouts of pain during pitching and complaints of decreased throwing accuracy, velocity, and stamina. &lt;b&gt;Most patients report significant pain during the late cocking and&amp;nbsp;&lt;/b&gt;&lt;b&gt;early acceleration phases of throwing, as it represents the phase at which the&amp;nbsp;&lt;/b&gt;&lt;b&gt;ligament experiences the greatest amount of valgus stress.&lt;/b&gt; Mechanical symptoms such as locking or catching during elbow motion can represent associated pathology, including olecranon osteophytes or intraarticular loose bodies. Ulnar nerve pathology typically manifests as specific neurologic complaints, including numbness and tingling that radiates to the ring and small fingers.&lt;/div&gt;&lt;div&gt;&lt;br /&gt;&lt;/div&gt;&lt;div&gt;&lt;span&gt;&amp;nbsp;&amp;nbsp; &amp;nbsp;&lt;/span&gt;Physical exam should begin with inspection of the elbow to detect the presence of an effusion. If a UCL injury is suspected, the ligament should be palpated along its anatomic course from the medial epicondyle (origin) to the sublime tubercle (insertion) located on the ulna. Any tenderness within this area can represent significant injury to the ligament or the flexor-pronator musculature. Both passive and active range of motion should be compared with the opposite extremity, as it is common to observe slight elbow flexion contractures in the dominant extremity. Several physical examination tests can be used to specifically test the integrity of the UCL. The&amp;nbsp;&lt;b&gt;valgus stress test &lt;/b&gt;is performed with the arm stabilized against the examiner and the elbow flexed to 30 degrees ( Figure 24–1 ). Valgus stress is then gently applied at the elbow to detect any abnormal widening at the ulnohumeral joint and/or reproduction of painful symptoms. The &lt;b&gt;“milking maneuver”&lt;/b&gt; is performed with the elbow flexed beyond 90 degrees, as the contralateral arm is used to grasp the thumb and generate a valgus force at the elbow ( Figure 24–2 ). If the patient reports pain with this test, a UCL injury should be suspected. The &lt;b&gt;moving valgus stress test&lt;/b&gt; is performed with the examiner applying a valgus torque at the elbow in full flexion and quickly extending the elbow to produce a shear force ( Figures 24–3A and 24–3B ). This test is highly sensitive (100%) and specific (75%) for UCL injury.&lt;/div&gt;&lt;/div&gt;&lt;div&gt;&lt;br /&gt;&lt;/div&gt;&lt;div class=&quot;separator&quot; style=&quot;clear: both; text-align: center;&quot;&gt;&lt;a href=&quot;https://blogger.googleusercontent.com/img/b/R29vZ2xl/AVvXsEgpHgXJ0duUh16Ll2fipaHWDGu98IUo2AbcfurQ9SvG65id91qWh3eXFR7lk977FM4nKmGqQ-2Q_E2gvMBwduWt-LqZTHBrjbcbQk3nRhDj9eFHDUaKGRkC46NILqa0gKx-N0FFhpzoCWhXgGobFqbkRuyxlVRnP9rTGR33PWKcZX0euNH-AtGrm8SoMQ/s549/valgus-stress-test.jpg&quot; style=&quot;margin-left: 1em; margin-right: 1em;&quot;&gt;&lt;img alt=&quot;valgus stress test&quot; border=&quot;0&quot; data-original-height=&quot;366&quot; data-original-width=&quot;549&quot; height=&quot;266&quot; src=&quot;https://blogger.googleusercontent.com/img/b/R29vZ2xl/AVvXsEgpHgXJ0duUh16Ll2fipaHWDGu98IUo2AbcfurQ9SvG65id91qWh3eXFR7lk977FM4nKmGqQ-2Q_E2gvMBwduWt-LqZTHBrjbcbQk3nRhDj9eFHDUaKGRkC46NILqa0gKx-N0FFhpzoCWhXgGobFqbkRuyxlVRnP9rTGR33PWKcZX0euNH-AtGrm8SoMQ/w400-h266/valgus-stress-test.jpg&quot; title=&quot;valgus stress test&quot; width=&quot;400&quot; /&gt;&lt;/a&gt;&lt;/div&gt;&lt;div&gt;&lt;div&gt;&lt;b&gt;&lt;span style=&quot;color: #017087;&quot;&gt;Figure 24–1.&amp;nbsp;&lt;/span&gt;&lt;/b&gt;The valgus stress test is performed with the arm stabilized against the examiner and&lt;/div&gt;&lt;div&gt;the elbow flexed to 30 degrees. Valgus stress is then gently applied at the elbow to detect any&lt;/div&gt;&lt;div&gt;abnormal widening at the ulnohumeral joint and/or reproduction of painful symptoms. (Courtesy of&lt;/div&gt;&lt;div&gt;Christopher Dodson, MD)&lt;/div&gt;&lt;/div&gt;&lt;div&gt;&lt;br /&gt;&lt;/div&gt;&lt;div class=&quot;separator&quot; style=&quot;clear: both; text-align: center;&quot;&gt;&lt;a href=&quot;https://blogger.googleusercontent.com/img/b/R29vZ2xl/AVvXsEiXkTvPQJ2zzjBoEqm9_GcUBM2bDR4hS8Nmi8oAnHJRJkRunG1xf9hrUINPZAnB_KOpgtXtuzKpFzpnT8QAsRdmyC_DGB64_pr5GXeAeIxTSIuwkJarDq8sWkgosozAPnpXel_B-fXLiHSp5Ahlwt2VrM--l_Bp4X27qwdmvUBCDrFXZYlQoBNKpHm7gA/s594/valgus-force-at-the-elbow.jpg&quot; style=&quot;margin-left: 1em; margin-right: 1em;&quot;&gt;&lt;img alt=&quot;valgus force at the elbow&quot; border=&quot;0&quot; data-original-height=&quot;594&quot; data-original-width=&quot;415&quot; height=&quot;400&quot; src=&quot;https://blogger.googleusercontent.com/img/b/R29vZ2xl/AVvXsEiXkTvPQJ2zzjBoEqm9_GcUBM2bDR4hS8Nmi8oAnHJRJkRunG1xf9hrUINPZAnB_KOpgtXtuzKpFzpnT8QAsRdmyC_DGB64_pr5GXeAeIxTSIuwkJarDq8sWkgosozAPnpXel_B-fXLiHSp5Ahlwt2VrM--l_Bp4X27qwdmvUBCDrFXZYlQoBNKpHm7gA/w280-h400/valgus-force-at-the-elbow.jpg&quot; title=&quot;valgus force at the elbow&quot; width=&quot;280&quot; /&gt;&lt;/a&gt;&lt;/div&gt;&lt;div&gt;&lt;div&gt;&lt;b&gt;&lt;span style=&quot;color: #017087;&quot;&gt;Figure 24–2.&amp;nbsp;&lt;/span&gt;&lt;/b&gt;The “milking maneuver” is performed with the elbow flexed beyond 90 degrees, as the&lt;/div&gt;&lt;div&gt;contralateral arm is used to grasp the thumb and generate a valgus force at the elbow. If the patient&lt;/div&gt;&lt;div&gt;reports pain with this test, a UCL injury should be suspected. (Courtesy of Christopher Dodson, MD)&lt;/div&gt;&lt;/div&gt;&lt;div&gt;&lt;br /&gt;&lt;/div&gt;&lt;div class=&quot;separator&quot; style=&quot;clear: both; text-align: center;&quot;&gt;&lt;a href=&quot;https://blogger.googleusercontent.com/img/b/R29vZ2xl/AVvXsEgQ44iSh4XzOHC6T6o_Vk4Sb4-GxghTgdstavt8xqLlVc4h3XUcsmD76rJY8cmbYnsW-3RX0KzFzpRnnebSHzTbyHBpIEisJi1z4etlURhmBxjodbOYDluIrIofDXkqS6RtiY_ORYx_CU8FsTIOwNJF-ps13wN6szZtzT787e8M-I26H6ngswdMYqfmKw/s567/moving-valgus-stress-test.jpg&quot; style=&quot;margin-left: 1em; margin-right: 1em;&quot;&gt;&lt;img alt=&quot;moving valgus stress test&quot; border=&quot;0&quot; data-original-height=&quot;567&quot; data-original-width=&quot;549&quot; height=&quot;400&quot; src=&quot;https://blogger.googleusercontent.com/img/b/R29vZ2xl/AVvXsEgQ44iSh4XzOHC6T6o_Vk4Sb4-GxghTgdstavt8xqLlVc4h3XUcsmD76rJY8cmbYnsW-3RX0KzFzpRnnebSHzTbyHBpIEisJi1z4etlURhmBxjodbOYDluIrIofDXkqS6RtiY_ORYx_CU8FsTIOwNJF-ps13wN6szZtzT787e8M-I26H6ngswdMYqfmKw/w388-h400/moving-valgus-stress-test.jpg&quot; title=&quot;moving valgus stress test&quot; width=&quot;388&quot; /&gt;&lt;/a&gt;&lt;/div&gt;&lt;div&gt;&lt;div&gt;&lt;b&gt;&lt;span style=&quot;color: #017087;&quot;&gt;Figure 24–3.&lt;/span&gt;&lt;/b&gt;&amp;nbsp;The moving valgus stress test is performed with the examiner (&lt;b&gt;A&lt;/b&gt;) applying a valgus&lt;/div&gt;&lt;div&gt;torque at the elbow in full flexion and (&lt;b&gt;B&lt;/b&gt;) quickly extending the elbow to produce a shear force.&lt;/div&gt;&lt;div&gt;This test is highly sensitive (100%) and specific (75%) for UCL injury. (Courtesy of Christopher&lt;/div&gt;&lt;div&gt;Dodson, MD)&lt;/div&gt;&lt;/div&gt;&lt;div&gt;&lt;br /&gt;&lt;/div&gt;&lt;div&gt;&lt;br /&gt;&lt;/div&gt;&lt;div&gt;&lt;div&gt;&lt;span&gt;&amp;nbsp;&amp;nbsp; &amp;nbsp;&lt;/span&gt;Several basic and advanced imaging studies can be used to facilitate accurate diagnosis and guide proper treatment in any athlete who presents with a suspected UCL tear. Routine diagnostic imaging should begin with standard AP and lateral radiographs of the elbow. A &lt;b&gt;“throwers series,”&lt;/b&gt; which includes two oblique radiographic views in internal and external rotation and an oblique axial view with the elbow in 110 degrees of flexion, can also be obtained to look for subtle associated pathology (ie, osteophyte formation, radiocapitellar osteochondritis dissecans, intraarticular loose bodies). Lastly, stress AP radiographs (x-rays obtained with valgus stress applied to the elbow) of both the injured and uninjured elbows can be used to detect any subtle differences in ulnohumeral widening secondary to UCL disruption. Advanced imaging, such as magnetic resonance imaging (MRI), can help identify ligament thickening from chronic overuse injury or more obvious full-thickness tears. Magnetic resonance arthrography obtained with intraarticular injection of gadolinium can aid in the diagnosis of partial undersurface tears of the ligament.&lt;/div&gt;&lt;/div&gt;&lt;div&gt;&lt;br /&gt;&lt;/div&gt;&lt;div&gt;&lt;div&gt;&lt;span&gt;&amp;nbsp;&amp;nbsp; &amp;nbsp;&lt;/span&gt;Some UCL tears can successfully be managed with a structured rehabilitation protocol. Early phases of rehabilitation include strict rest from all throwing activities and reduction of pain and inflammation with ice and nonsteroidal antiinflammatory drugs (NSAID). Gradual flexor-pronator strengthening exercises are incorporated, along with a progressive throwing program once all painful symptoms have resolved.&lt;/div&gt;&lt;/div&gt;&lt;div&gt;&lt;br /&gt;&lt;/div&gt;&lt;div&gt;&lt;div&gt;&lt;span&gt;&amp;nbsp;&amp;nbsp; &amp;nbsp;&lt;/span&gt;Surgical reconstruction of the UCL is indicated when patients demonstrate continued symptoms after a trial of structured rehabilitation. UCL reconstruction should only be performed in high-demand, throwing athletes who are willing to participate in an extensive postoperative rehabilitation program. When indicated, reconstruction involves securing a tendon graft to the humerus and ulna. Several techniques have been described, all of which provide good to excellent results in approximately 85% of cases. Common techniques used include the &lt;b&gt;modified Jobe&amp;nbsp;&lt;/b&gt;&lt;b&gt;technique &lt;/b&gt;and the&lt;b&gt; docking technique.&lt;/b&gt; It is clear that a muscle-splitting approach and minimal surgical manipulation of the ulnar nerve can limit postoperative morbidity and complications.&lt;/div&gt;&lt;/div&gt;&lt;div&gt;&lt;br /&gt;&lt;/div&gt;&lt;div&gt;&lt;div&gt;&lt;span&gt;&amp;nbsp;&amp;nbsp; &amp;nbsp;&lt;/span&gt;The main complications to beware of include damage to the ulnar or medial antebrachial cutaneous nerve during surgical dissection, ulnar or epicondylar fracture during tunnel drilling, and postoperative elbow stiffness from inadequate rehabilitation. In general, athletes can expect to return to competitive throwing 12 months after surgery if the shoulder, elbow, and forearm are pain-free and full range of motion has been restored.&lt;/div&gt;&lt;/div&gt;&lt;div&gt;&lt;br /&gt;&lt;/div&gt;&lt;div&gt;&lt;br /&gt;&lt;/div&gt;&lt;div&gt;&lt;b&gt;&lt;span style=&quot;color: #017087; font-size: medium;&quot;&gt;&lt;u&gt;COMPREHENSION QUESTIONS&lt;/u&gt;&lt;/span&gt;&lt;/b&gt;&lt;/div&gt;&lt;div&gt;&lt;br /&gt;&lt;/div&gt;&lt;div&gt;&lt;div&gt;24.1 A 19-year-old college javelin thrower presents with medial elbow pain during overhead throwing. He is diagnosed with a UCL tear after physical examination and MRI. What anatomic structure is the primary stabilizer to valgus stress at the elbow?&lt;/div&gt;&lt;/div&gt;&lt;blockquote style=&quot;border: none; margin: 0px 0px 0px 40px; padding: 0px; text-align: left;&quot;&gt;&lt;div&gt;&lt;div&gt;A. Anterior bundle of the UCL&lt;/div&gt;&lt;/div&gt;&lt;div&gt;&lt;div&gt;B. Transverse ligament&lt;/div&gt;&lt;/div&gt;&lt;div&gt;&lt;div&gt;C. Radial head&lt;/div&gt;&lt;/div&gt;&lt;div&gt;&lt;div&gt;D. Flexor pronator mass&lt;/div&gt;&lt;/div&gt;&lt;/blockquote&gt;&lt;div&gt;&lt;div&gt;&lt;br /&gt;&lt;/div&gt;&lt;div&gt;24.2 A 17-year-old baseball pitcher presents with medial elbow pain during overhead throwing. He is diagnosed with a UCL tear after physical examination and MRI. During what phase of throwing is he most likely to complain of medial elbow pain?&lt;/div&gt;&lt;/div&gt;&lt;blockquote style=&quot;border: none; margin: 0px 0px 0px 40px; padding: 0px; text-align: left;&quot;&gt;&lt;div&gt;&lt;div&gt;A. Follow-through&lt;/div&gt;&lt;/div&gt;&lt;div&gt;&lt;div&gt;B. Deceleration&lt;/div&gt;&lt;/div&gt;&lt;div&gt;&lt;div&gt;C. Acceleration&lt;/div&gt;&lt;/div&gt;&lt;div&gt;&lt;div&gt;D. Wind-up&lt;/div&gt;&lt;/div&gt;&lt;/blockquote&gt;&lt;div&gt;&lt;br /&gt;&lt;/div&gt;&lt;div&gt;&lt;div&gt;24.3 A 21-year-old javelin thrower is diagnosed with a full-thickness UCL tear. On presentation, he complains of numbness and tingling in the ring and small fingers. Which muscle group would he be most likely to have motor weakness in as well?&lt;/div&gt;&lt;/div&gt;&lt;blockquote style=&quot;border: none; margin: 0px 0px 0px 40px; padding: 0px; text-align: left;&quot;&gt;&lt;div&gt;&lt;div&gt;A. Brachioradialis&lt;/div&gt;&lt;/div&gt;&lt;div&gt;&lt;div&gt;B. Extensor carpi ulnaris&lt;/div&gt;&lt;/div&gt;&lt;div&gt;&lt;div&gt;C. Abductor digiti minimi&lt;/div&gt;&lt;/div&gt;&lt;div&gt;&lt;div&gt;D. Palmaris longus&lt;/div&gt;&lt;/div&gt;&lt;/blockquote&gt;&lt;div&gt;&lt;br /&gt;&lt;/div&gt;&lt;div&gt;&lt;br /&gt;&lt;/div&gt;&lt;div&gt;&lt;b&gt;&lt;span style=&quot;color: #017087; font-size: medium;&quot;&gt;&lt;u&gt;ANSWERS&lt;/u&gt;&lt;/span&gt;&lt;/b&gt;&lt;/div&gt;&lt;div&gt;&lt;br /&gt;&lt;/div&gt;&lt;div&gt;&lt;div&gt;24.1 &lt;b&gt;A&lt;/b&gt;. The anterior bundle of the UCL is the primary stabilizer to valgus stress at the elbow.&lt;/div&gt;&lt;div&gt;&lt;br /&gt;&lt;/div&gt;&lt;div&gt;24.2 &lt;b&gt;C.&lt;/b&gt; Throwing athletes typically complain of medial elbow pain during the acceleration phase of throwing. Valgus force generated at the elbow during the throwing motion is highest during the acceleration phase.&lt;/div&gt;&lt;div&gt;&lt;br /&gt;&lt;/div&gt;&lt;div&gt;24.3 &lt;b&gt;C.&lt;/b&gt; The patient is experiencing ulnar nerve paresthesias. If the patient had associated motor symptoms, they would manifest as weakness within the abductor digiti minimi. The other answer choices are not muscles innervated by the ulnar nerve.&lt;/div&gt;&lt;/div&gt;&lt;div&gt;&lt;br /&gt;&lt;/div&gt;&lt;div&gt;&lt;b style=&quot;background-color: #4093a5;&quot;&gt;&amp;nbsp; &amp;nbsp;&lt;span style=&quot;font-size: medium;&quot;&gt;&amp;nbsp;&lt;/span&gt;&lt;span style=&quot;color: white;&quot;&gt;&lt;span style=&quot;font-size: medium;&quot;&gt;CLINICAL PEARLS&lt;/span&gt;&lt;span&gt;&amp;nbsp; &amp;nbsp;&amp;nbsp;&lt;/span&gt;&lt;/span&gt;&lt;/b&gt;&lt;/div&gt;&lt;div&gt;&lt;table border=&quot;1&quot; cellpadding=&quot;0&quot; cellspacing=&quot;0&quot; class=&quot;MsoTableGrid&quot; style=&quot;border-collapse: collapse; border: none; mso-border-alt: solid windowtext .5pt; mso-padding-alt: 0in 5.4pt 0in 5.4pt; mso-yfti-tbllook: 1184;&quot;&gt;
  264. &lt;tbody&gt;&lt;tr style=&quot;height: 19.15pt; mso-yfti-firstrow: yes; mso-yfti-irow: 0; mso-yfti-lastrow: yes;&quot;&gt;
  265.  &lt;td style=&quot;background: rgb(221, 237, 239); border-left: solid #017087 1.0pt; border: 1pt solid rgb(64, 147, 165); height: 19.15pt; padding: 0in 5.4pt; width: 433.7pt;&quot; valign=&quot;top&quot; width=&quot;578&quot;&gt;
  266.  &lt;div style=&quot;line-height: normal; margin-bottom: 0in;&quot;&gt;&lt;span style=&quot;color: #017087;&quot;&gt;&lt;br /&gt;&lt;/span&gt;&lt;/div&gt;&lt;div style=&quot;line-height: normal; margin-bottom: 0in;&quot;&gt;&lt;span style=&quot;color: #017087;&quot;&gt;►&amp;nbsp;&lt;/span&gt;The anterior bundle of the UCL is the primary stabilizer to valgus stress at the elbow.&lt;/div&gt;&lt;div style=&quot;line-height: normal; margin-bottom: 0in;&quot;&gt;&lt;span style=&quot;color: #017087;&quot;&gt;&lt;br /&gt;&lt;/span&gt;&lt;/div&gt;&lt;div style=&quot;line-height: normal; margin-bottom: 0in;&quot;&gt;&lt;span style=&quot;color: #017087;&quot;&gt;►&lt;/span&gt;&amp;nbsp;Patients with UCL injuries typically report medial elbow pain during the acceleration phase of throwing.&lt;/div&gt;&lt;div style=&quot;line-height: normal; margin-bottom: 0in;&quot;&gt;&lt;span style=&quot;color: #017087;&quot;&gt;&lt;br /&gt;&lt;/span&gt;&lt;/div&gt;&lt;div style=&quot;line-height: normal; margin-bottom: 0in;&quot;&gt;&lt;span style=&quot;color: #017087;&quot;&gt;►&lt;/span&gt;&amp;nbsp;The most sensitive and specific clinical test for UCL injury is the moving valgus stress test.&lt;/div&gt;&lt;div style=&quot;line-height: normal; margin-bottom: 0in;&quot;&gt;&lt;span style=&quot;color: #017087;&quot;&gt;&lt;br /&gt;&lt;/span&gt;&lt;/div&gt;&lt;div style=&quot;line-height: normal; margin-bottom: 0in;&quot;&gt;&lt;span style=&quot;color: #017087;&quot;&gt;►&lt;/span&gt;&amp;nbsp;A muscle-splitting surgical approach and minimal manipulation of the ulnar nerve can limit postoperative morbidity to the flexor pronator mass and the ulnar nerve.&lt;/div&gt;&lt;div style=&quot;line-height: normal; margin-bottom: 0in;&quot;&gt;&lt;span style=&quot;color: #017087;&quot;&gt;&lt;br /&gt;&lt;/span&gt;&lt;/div&gt;&lt;div style=&quot;line-height: normal; margin-bottom: 0in;&quot;&gt;&lt;span style=&quot;color: #017087;&quot;&gt;►&lt;/span&gt;&amp;nbsp;Ulnar nerve transposition can be performed in conjunction with UCL reconstruction if patients demonstrate significant pathology related to the nerve (ulnar nerve subluxation or motor weakness).&lt;/div&gt;&lt;div style=&quot;line-height: normal; margin-bottom: 0in;&quot;&gt;&lt;br /&gt;&lt;/div&gt;
  267.  &lt;/td&gt;
  268. &lt;/tr&gt;
  269. &lt;/tbody&gt;&lt;/table&gt;&lt;br /&gt;&lt;/div&gt;
  270.  
  271. &lt;details close=&quot;&quot;&gt;
  272.  &lt;summary&gt;&lt;b&gt;&lt;span style=&quot;color: #00788b;&quot;&gt;REFERENCES&lt;/span&gt;&lt;/b&gt;&lt;/summary&gt;
  273.  &lt;p&gt;Fleisig GS, Andrews JR, Dillma CJ, Escamilla RF. Kinetics of baseball pitching with implications about
  274. injury mechanisms. Am J Sports Med. 1995;23:233-239.&amp;nbsp;&lt;/p&gt;&lt;p&gt;Hariri S, Safran MR. Ulnar collateral ligament injury in the overhead athlete. Clin Sports Med.
  275. 2010;29:619-644.&amp;nbsp;&lt;/p&gt;&lt;p&gt;Jobe FW, Stark H, Lombardo SJ. Reconstruction of the ulnar collateral ligament in athletes. J Bone Joint
  276. Surg [Am]. 1986;68A:1158-1163.&amp;nbsp;&lt;/p&gt;&lt;p&gt;Williams RJ, Urquhart ER, Altchek DW. Medial collateral ligament tears in the throwing athlete. Instr
  277. Course Lect. 2004;53:579-586.&lt;/p&gt;
  278. &lt;/details&gt;&lt;/div&gt;</content><link rel='replies' type='application/atom+xml' href='https://medical-phd.blogspot.com/feeds/1557492615936300481/comments/default' title='Post Comments'/><link rel='replies' type='text/html' href='https://medical-phd.blogspot.com/2022/03/ulnar-collateral-ligament-injury-of-the-elbow-case-file.html#comment-form' title='0 Comments'/><link rel='edit' type='application/atom+xml' href='https://www.blogger.com/feeds/3302710210518459416/posts/default/1557492615936300481'/><link rel='self' type='application/atom+xml' href='https://www.blogger.com/feeds/3302710210518459416/posts/default/1557492615936300481'/><link rel='alternate' type='text/html' href='https://medical-phd.blogspot.com/2022/03/ulnar-collateral-ligament-injury-of-the-elbow-case-file.html' title='Ulnar Collateral Ligament Injury of the Elbow Case File'/><author><name>Medical Group</name><uri>http://www.blogger.com/profile/01780656800578397393</uri><email>noreply@blogger.com</email><gd:image rel='http://schemas.google.com/g/2005#thumbnail' width='32' height='32' src='//blogger.googleusercontent.com/img/b/R29vZ2xl/AVvXsEiMuROkBDv6yw4SmxGqRA5BPQhOXSwFld-Gufmy4kUqfpoAGQRl7cv-3naew6wjAnCjRFSe_FKX8z7CdE6cFKxxBpnqBuwUTSr-15xd3mFGcr_wJOX2y73G_zJCGND98Q/s220/Final-logo.jpg'/></author><media:thumbnail xmlns:media="http://search.yahoo.com/mrss/" url="https://blogger.googleusercontent.com/img/b/R29vZ2xl/AVvXsEgpHgXJ0duUh16Ll2fipaHWDGu98IUo2AbcfurQ9SvG65id91qWh3eXFR7lk977FM4nKmGqQ-2Q_E2gvMBwduWt-LqZTHBrjbcbQk3nRhDj9eFHDUaKGRkC46NILqa0gKx-N0FFhpzoCWhXgGobFqbkRuyxlVRnP9rTGR33PWKcZX0euNH-AtGrm8SoMQ/s72-w400-h266-c/valgus-stress-test.jpg" height="72" width="72"/><thr:total>0</thr:total><georss:featurename>United States</georss:featurename><georss:point>37.09024 -95.712891</georss:point><georss:box>8.780006163821156 -130.869141 65.400473836178847 -60.556641</georss:box></entry><entry><id>tag:blogger.com,1999:blog-3302710210518459416.post-9106962394899765164</id><published>2022-03-25T05:30:00.007-07:00</published><updated>2022-03-25T05:30:52.762-07:00</updated><category scheme="http://www.blogger.com/atom/ns#" term="Case Files"/><category scheme="http://www.blogger.com/atom/ns#" term="Orthopaedic Surgery Case File"/><title type='text'>Meniscal Tears Case File</title><content type='html'>&lt;div class=&quot;scrollbox&quot;&gt;
  279.  
  280. &lt;div style=&quot;text-align: left;&quot;&gt;&lt;b&gt;&lt;span style=&quot;font-size: large;&quot;&gt;Meniscal Tears Case File&lt;/span&gt;&lt;/b&gt;&lt;/div&gt;&lt;div&gt;Eugene C. Toy, MD, Andrew J. Rosenbaum, MD, Timothy T. Roberts, MD, Joshua S. Dines, MD&lt;/div&gt;&lt;div&gt;&lt;br /&gt;&lt;/div&gt;&lt;div&gt;&lt;b&gt;&lt;span style=&quot;color: #017087; font-size: medium;&quot;&gt;CASE 23&lt;/span&gt;&lt;/b&gt;&lt;/div&gt;&lt;div&gt;&lt;div&gt;A 28-year-old otherwise healthy man presents to the office complaining of pain and swelling in his right knee. He reports that he first noticed pain when he pivoted off of the right leg during a pickup football game 2 days earlier. He was able to finish playing the game. He did not recall an audible “pop” or immediate swelling. The effusion developed later that evening. He denies any sense of instability. The patient denies any other medical or surgical history. On physical exam of the right knee, there are no abrasions or ecchymosis. There is a mild to moderate effusion and tenderness to palpation along the medial joint line. There is no pes anserine, lateral joint line, or other bony tenderness. He has complete, but painful, range of motion. He is stable to varus and valgus stress at 0 and 30 degrees. He has negative Lachman, anterior, and posterior drawer tests. McMurray and Apley tests are positive. He is neurovascularly intact distally.&lt;/div&gt;&lt;div&gt;&lt;span style=&quot;color: #017087;&quot;&gt;&lt;br /&gt;&lt;/span&gt;&lt;/div&gt;&lt;div&gt;&lt;span style=&quot;color: #017087;&quot;&gt;►&lt;/span&gt;&amp;nbsp;What is your most likely diagnosis?&lt;/div&gt;&lt;div&gt;&lt;span style=&quot;color: #017087;&quot;&gt;►&lt;/span&gt;&amp;nbsp;What should be included in the differential diagnosis?&lt;/div&gt;&lt;div&gt;&lt;span style=&quot;color: #017087;&quot;&gt;►&lt;/span&gt;&amp;nbsp;What is your next diagnostic step?&lt;/div&gt;&lt;div&gt;&lt;span style=&quot;color: #017087;&quot;&gt;►&lt;/span&gt;&amp;nbsp;What is the next step in therapy?&lt;/div&gt;&lt;/div&gt;&lt;div&gt;&lt;br /&gt;&lt;/div&gt;&lt;div&gt;&lt;br /&gt;&lt;/div&gt;&lt;div&gt;&lt;div&gt;&lt;b&gt;&lt;span style=&quot;font-size: medium;&quot;&gt;ANSWER TO CASE 23:&lt;/span&gt;&lt;/b&gt;&lt;/div&gt;&lt;div&gt;&lt;span style=&quot;background-color: #daebf0; font-size: medium;&quot;&gt;&lt;span&gt;&lt;span style=&quot;background-color: transparent;&quot;&gt;&lt;span style=&quot;color: #017087;&quot;&gt;&lt;b&gt;Meniscal Tears&lt;/b&gt;&lt;/span&gt;&lt;/span&gt;&lt;b&gt;&amp;nbsp;&amp;nbsp;&lt;/b&gt;&lt;/span&gt;&lt;span style=&quot;font-weight: bold;&quot;&gt;&amp;nbsp;&amp;nbsp; &amp;nbsp;&lt;/span&gt;&lt;span style=&quot;font-weight: bold;&quot;&gt;&amp;nbsp;&amp;nbsp; &amp;nbsp;&lt;/span&gt;&lt;span style=&quot;font-weight: bold;&quot;&gt;&amp;nbsp;&amp;nbsp; &amp;nbsp;&lt;/span&gt;&lt;span style=&quot;font-weight: bold;&quot;&gt;&amp;nbsp;&amp;nbsp; &amp;nbsp;&lt;/span&gt;&lt;span style=&quot;font-weight: bold;&quot;&gt;&amp;nbsp;&amp;nbsp; &amp;nbsp;&lt;/span&gt;&lt;span style=&quot;font-weight: bold;&quot;&gt;&amp;nbsp;&amp;nbsp; &amp;nbsp;&lt;/span&gt;&lt;span style=&quot;font-weight: bold;&quot;&gt;&amp;nbsp; &amp;nbsp;&amp;nbsp;&lt;/span&gt;&lt;/span&gt;&lt;/div&gt;&lt;/div&gt;&lt;div&gt;&lt;br /&gt;&lt;/div&gt;&lt;div&gt;&lt;div&gt;&lt;i&gt;Summary&lt;/i&gt;: A 28-year-old healthy man who sustained a right knee twisting injury during a pickup football game presents with right knee pain. His exam is significant for a moderate effusion, medial joint line tenderness, positive McMurray and Apley tests, and a stable ligamentous exam.&lt;/div&gt;&lt;div&gt;&lt;ul style=&quot;text-align: left;&quot;&gt;&lt;li&gt;&lt;b&gt;Most likely diagnosis:&lt;/b&gt; Tear of the medial meniscus.&lt;/li&gt;&lt;li&gt;&lt;b&gt;Differential diagnosis:&lt;/b&gt; Osteoarthritis, loose body, patellar subluxation or dislocation, osteochondritis dissecans, articular cartilage lesions, tibial plateau fractures, ligamentous injury, pes anserine bursitis, and fat pad impingement syndrome.&lt;/li&gt;&lt;li&gt;&lt;b&gt;Next diagnostic step:&lt;/b&gt; Plain radiographs of the knee, followed by magnetic resonance imaging (MRI).&lt;/li&gt;&lt;li&gt;&lt;b&gt;Next step in therapy:&lt;/b&gt; Arthroscopic partial meniscectomy, with resection to a stable, smooth rim of meniscus. Meniscal repair is also a possibility, depending on the location of the tear.&lt;/li&gt;&lt;/ul&gt;&lt;/div&gt;&lt;/div&gt;&lt;div&gt;&lt;br /&gt;&lt;/div&gt;&lt;div&gt;&lt;br /&gt;&lt;/div&gt;&lt;div&gt;&lt;div&gt;&lt;b&gt;&lt;span style=&quot;color: #017087; font-size: medium;&quot;&gt;ANALYSIS&lt;/span&gt;&lt;/b&gt;&lt;/div&gt;&lt;div&gt;&lt;b&gt;&lt;span style=&quot;font-size: medium;&quot;&gt;&lt;i&gt;Objectives&lt;/i&gt;&lt;/span&gt;&lt;/b&gt;&lt;/div&gt;&lt;/div&gt;&lt;div&gt;&lt;div&gt;&lt;ol style=&quot;text-align: left;&quot;&gt;&lt;li&gt;Understand the typical patient demographics and mechanism of injury associated with meniscal tears.&lt;/li&gt;&lt;li&gt;Be familiar with the treatment options for meniscal pathology.&lt;/li&gt;&lt;/ol&gt;&lt;/div&gt;&lt;/div&gt;&lt;div&gt;&lt;br /&gt;&lt;/div&gt;&lt;div&gt;&lt;br /&gt;&lt;/div&gt;&lt;div&gt;&lt;div&gt;&lt;i&gt;&lt;span style=&quot;font-size: medium;&quot;&gt;&lt;b&gt;Considerations&lt;/b&gt;&lt;/span&gt;&lt;/i&gt;&lt;/div&gt;&lt;div&gt;&lt;br /&gt;&lt;/div&gt;&lt;div&gt;This is a 28-year-old man who presents after an injury to his right knee sustained during a pickup football game. The history and physical exam are very typical for meniscal pathology. This includes his twisting mechanism of injury, pain, lateonset joint effusion (versus a sudden-onset effusion, which may indicate isolated or concomitant anterior cruciate ligament tear), and medial joint line tenderness. Additionally, 2 meniscus-specific tests, the McMurray and Apley tests, are positive ( Figure 23–1 ). Although the patient’s history and examination are consistent with an acute meniscal injury, the orthopaedist must also be familiar with patients presenting with symptoms indicative of the more chronic “degenerative” and “complex” meniscal tears. In the setting of a meniscal tear, radiographs and potentially MRI should be obtained. Treatment, which consists of either operative or more conservative measures, is patient specific, with multiple factors determining the ideal approach.&lt;/div&gt;&lt;/div&gt;&lt;div&gt;&lt;br /&gt;&lt;/div&gt;&lt;div class=&quot;separator&quot; style=&quot;clear: both; text-align: center;&quot;&gt;&lt;a href=&quot;https://blogger.googleusercontent.com/img/b/R29vZ2xl/AVvXsEgAIoowDVW89v37Nedp2vIoliCp_WhZ26Os2lDoB0gDcUnaYQJRhNyQLhFk52UuyhxUHm5SlXgsQoMNBP-uZU9ZMoB7sTz2MdPFzfgnVtAzMK0F3L3ErwB6rnpxdxxq_NE0wPE68S6XgwwzSTEveDZoipWAfLv4sbza_JHK-unM_onFsDmo33AeSOWojQ/s701/Tests-for-tear-of-the-medial-meniscus.jpg&quot; style=&quot;margin-left: 1em; margin-right: 1em;&quot;&gt;&lt;img alt=&quot;Tests for tear of the medial meniscus&quot; border=&quot;0&quot; data-original-height=&quot;305&quot; data-original-width=&quot;701&quot; height=&quot;278&quot; src=&quot;https://blogger.googleusercontent.com/img/b/R29vZ2xl/AVvXsEgAIoowDVW89v37Nedp2vIoliCp_WhZ26Os2lDoB0gDcUnaYQJRhNyQLhFk52UuyhxUHm5SlXgsQoMNBP-uZU9ZMoB7sTz2MdPFzfgnVtAzMK0F3L3ErwB6rnpxdxxq_NE0wPE68S6XgwwzSTEveDZoipWAfLv4sbza_JHK-unM_onFsDmo33AeSOWojQ/w640-h278/Tests-for-tear-of-the-medial-meniscus.jpg&quot; title=&quot;Tests for tear of the medial meniscus&quot; width=&quot;640&quot; /&gt;&lt;/a&gt;&lt;/div&gt;&lt;div&gt;&lt;div&gt;&lt;b&gt;&lt;span style=&quot;color: #017087;&quot;&gt;Figure 23–1. &lt;/span&gt;&lt;/b&gt;Tests for tear of the medial meniscus. (&lt;b&gt;A&lt;/b&gt;) McMurray test. (&lt;b&gt;B&lt;/b&gt;) Apley test. (&lt;b&gt;C&lt;/b&gt;) Childress&lt;/div&gt;&lt;div&gt;test. (Reproduced, with permission, from LeBlond RF, DeGowin RL, Brown DD. &lt;i&gt;DeGowin’s Diagnostic Examination&lt;/i&gt;. 9th ed. New York, NY: McGraw-Hill; 2009:Fig. 13-24.)&lt;/div&gt;&lt;/div&gt;&lt;div&gt;&lt;br /&gt;&lt;/div&gt;&lt;div&gt;&lt;br /&gt;&lt;/div&gt;&lt;div&gt;&lt;div&gt;&lt;div&gt;&lt;b&gt;&lt;span style=&quot;font-size: medium;&quot;&gt;APPROACH TO:&lt;/span&gt;&lt;/b&gt;&lt;/div&gt;&lt;div&gt;&lt;span style=&quot;background-color: #daebf0; color: #017087; font-size: medium;&quot;&gt;&lt;span&gt;&lt;span style=&quot;background-color: transparent;&quot;&gt;&lt;b&gt;Meniscal Tears&lt;/b&gt;&lt;/span&gt;&lt;b&gt;&amp;nbsp;&lt;/b&gt;&lt;/span&gt;&lt;span style=&quot;font-weight: bold;&quot;&gt;&amp;nbsp;&amp;nbsp; &amp;nbsp;&lt;/span&gt;&lt;span style=&quot;font-weight: bold;&quot;&gt;&amp;nbsp;&amp;nbsp; &amp;nbsp;&lt;/span&gt;&lt;span style=&quot;font-weight: bold;&quot;&gt;&amp;nbsp;&amp;nbsp; &amp;nbsp;&lt;/span&gt;&lt;span style=&quot;font-weight: bold;&quot;&gt;&amp;nbsp;&amp;nbsp; &amp;nbsp;&lt;/span&gt;&lt;span style=&quot;font-weight: bold;&quot;&gt;&amp;nbsp;&amp;nbsp; &amp;nbsp;&lt;/span&gt;&lt;span style=&quot;font-weight: bold;&quot;&gt;&amp;nbsp;&amp;nbsp; &amp;nbsp;&lt;/span&gt;&lt;span style=&quot;font-weight: bold;&quot;&gt;&amp;nbsp;&amp;nbsp; &amp;nbsp;&lt;/span&gt;&lt;span style=&quot;font-weight: bold;&quot;&gt;&amp;nbsp;&amp;nbsp;&amp;nbsp;&lt;/span&gt;&lt;span style=&quot;font-weight: bold;&quot;&gt;&amp;nbsp;&lt;/span&gt;&lt;/span&gt;&lt;/div&gt;&lt;/div&gt;&lt;div&gt;&lt;br /&gt;&lt;/div&gt;&lt;div&gt;&lt;b&gt;&lt;span style=&quot;color: #017087; font-size: medium;&quot;&gt;DEFINITIONS&lt;/span&gt;&lt;/b&gt;&lt;/div&gt;&lt;/div&gt;&lt;div&gt;&lt;br /&gt;&lt;/div&gt;&lt;div&gt;&lt;div&gt;&lt;b&gt;MENISCI (MEDIAL AND LATERAL):&lt;/b&gt; Fibrocartilaginous (predominantly type I collagen) structures within the knee joint that increase contact area, distribute load (50%-70% in extension and 85%-90% in flexion), absorb shock, and play a role in joint stability and proprioception. Removing the entire meniscus decreases the overall knee contact area and subsequently results in increased stress throughout the knee, pain, and degenerative joint disease. The medial meniscus is larger and more oblong than the lateral meniscus. It is also an important secondary stabilizer to anterior tibial translation and is thus essential for stability in an anterior cruciate ligament (ACL)-deficient knee.&lt;/div&gt;&lt;div&gt;&lt;b&gt;&lt;br /&gt;&lt;/b&gt;&lt;/div&gt;&lt;div&gt;&lt;b&gt;VASCULAR ZONES:&lt;/b&gt; The meniscus is divided into thirds based on vascularity. The peripheral third is the most vascularized and deemed the “red-red” zone. The middle third, referred to as the “red-white” zone, has an intermediate vascularity. The inner third, or the “white-white” zone, is avascular.&lt;/div&gt;&lt;div&gt;&lt;b&gt;&lt;br /&gt;&lt;/b&gt;&lt;/div&gt;&lt;div&gt;&lt;b&gt;MCMURRAY TEST:&lt;/b&gt; Flexion and rotation of the knee. To test the medial meniscus, the examiner flexes the knee, places a hand on the medial or posteromedial joint line of the knee, and then brings the knee from flexion to extension while externally rotating the leg. To test the lateral meniscus, the examiner places a hand on the lateral joint line and applies an internal rotation force to the leg. A positive test is found with a palpable “clunk” at the joint line and occurs as the torn meniscus displaces. It is the most specific test (98%) for a meniscal tear but is only 15% sensitive.&lt;/div&gt;&lt;div&gt;&lt;b&gt;&lt;br /&gt;&lt;/b&gt;&lt;/div&gt;&lt;div&gt;&lt;b&gt;APLEY TEST:&lt;/b&gt; The patient is prone with the knee flexed to 90 degrees. A downward compressive force is applied by the examiner through the lower leg while laterally rotating the lower leg. Pain during this maneuver is indicative of a meniscal injury.&lt;/div&gt;&lt;/div&gt;&lt;div&gt;&lt;br /&gt;&lt;/div&gt;&lt;div&gt;&lt;br /&gt;&lt;/div&gt;&lt;div&gt;&lt;b&gt;&lt;span style=&quot;color: #017087; font-size: medium;&quot;&gt;CLINICAL APPROACH&lt;/span&gt;&lt;/b&gt;&lt;/div&gt;&lt;div&gt;&lt;br /&gt;&lt;/div&gt;&lt;div&gt;&lt;div&gt;&lt;b&gt;&lt;i&gt;&lt;span style=&quot;font-size: medium;&quot;&gt;Etiology&lt;/span&gt;&lt;/i&gt;&lt;/b&gt;&lt;/div&gt;&lt;div&gt;&lt;br /&gt;&lt;/div&gt;&lt;div&gt;Meniscal tears have an incidence of 60 to 70 cases per 100,000 people per year and occur with a male predominance of approximately 3:1. &lt;b&gt;Tears can be divided into 2&amp;nbsp;&lt;/b&gt;&lt;b&gt;etiologies: those occurring in younger patients and caused by an acute, twisting,&amp;nbsp;&lt;/b&gt;&lt;b&gt;or rotational force and those seen in older patients with menisci vulnerable to&amp;nbsp;&lt;/b&gt;&lt;b&gt;injury secondary to underlying degenerative changes of the knee.&lt;/b&gt;&lt;/div&gt;&lt;div&gt;&lt;br /&gt;&lt;/div&gt;&lt;div&gt;&lt;b&gt;&lt;i&gt;&lt;span style=&quot;font-size: medium;&quot;&gt;Diagnosis&lt;/span&gt;&lt;/i&gt;&lt;/b&gt;&lt;/div&gt;&lt;div&gt;&lt;br /&gt;&lt;/div&gt;&lt;div&gt;A thorough history and physical exam is the first step in diagnosis, and patients with meniscal pathology may describe an acute injury or a more indolent onset of symptoms. Patients may have recurring knee effusions, medial or lateral joint line tenderness, posterior knee pain, and/or mechanical symptoms, such as locking and clicking.&lt;/div&gt;&lt;div&gt;&lt;br /&gt;&lt;/div&gt;&lt;div&gt;&lt;span&gt;&amp;nbsp;&amp;nbsp; &amp;nbsp;&lt;/span&gt;The physical exam includes observing the patient’s gait, knee alignment, symmetry, and use of assistive devices. Palpation and evaluation of the patella for tenderness and laxity, palpation of the medial and lateral joint lines, the pes anserinus, medial and lateral collateral ligaments, tibial tubercle, and Gerdy tubercle should also be done. Range of motion (both hip and knee) must be assessed and compared with the contralateral side, as should a ligamentous (ACL, posterior cruciate ligament, medial collateral ligament, lateral collateral ligament) exam. &lt;b&gt;Of&amp;nbsp;&lt;/b&gt;&lt;b&gt;note, a torn lateral meniscus is the most commonly associated meniscal injury in&amp;nbsp;&lt;/b&gt;&lt;b&gt;the setting of an acute ACL tear, whereas the medial meniscus is most commonly&amp;nbsp;&lt;/b&gt;&lt;b&gt;torn in the setting of a knee with a chronic ACL deficiency.&lt;/b&gt; Crepitus or locking should also be noted.&lt;/div&gt;&lt;div&gt;&lt;br /&gt;&lt;/div&gt;&lt;div&gt;&lt;span&gt;&amp;nbsp;&amp;nbsp; &amp;nbsp;&lt;/span&gt;Two tests commonly used for the evaluation of meniscus-specific pathology are the &lt;b&gt;McMurray and Apley tests&lt;/b&gt; ( Figure 23–1 ). Both have a high specificity but poor sensitivity. Joint line tenderness remains the most sensitive test (74%) for a meniscal injury. &lt;b&gt;Childress sign (Squat test)&lt;/b&gt; can also be used and is positive when the patient, who attempts to squat and walk like a duck, feels pain, is unable to squat all the way down, and feels a snap or click from the knee (Figure 23–1).&lt;/div&gt;&lt;div&gt;&lt;br /&gt;&lt;/div&gt;&lt;div&gt;&lt;span&gt;&amp;nbsp;&amp;nbsp; &amp;nbsp;&lt;/span&gt;Plain radiographs should be the first imaging obtained. This includes bilateral weightbearing posteroanterior (PA) views, 45-degree flexion PA views, 20 degrees of flexion in patellofemoral view, and a lateral radiograph. MRI is used to confirm the physical exam findings concerning for a meniscal tear and will identify the tear pattern and displacement (Figure 23–2). It has an accuracy of approximately 95% in diagnosing tears.&lt;/div&gt;&lt;/div&gt;&lt;div&gt;&lt;br /&gt;&lt;/div&gt;&lt;div&gt;&lt;br /&gt;&lt;/div&gt;&lt;div&gt;&lt;b style=&quot;color: #017087;&quot;&gt;&lt;span style=&quot;font-size: medium;&quot;&gt;TREATMENT&lt;/span&gt;&lt;/b&gt;&lt;/div&gt;&lt;div&gt;&lt;br /&gt;&lt;/div&gt;&lt;div&gt;&lt;div&gt;Treatment of meniscal tears is patient- and tear-specific and varies from nonsurgical, conservative measures to resection, repair, or transplant. &lt;b&gt;Nonsurgical treatment&amp;nbsp;&lt;/b&gt;&lt;b&gt;includes ice, nonsteroidal anti-inflammatory drugs (NSAIDs), intraarticular corticosteroid&amp;nbsp;&lt;/b&gt;&lt;b&gt;injections, and physical therapy.&lt;/b&gt; Indications include asymptomatic, partial-thickness tears less than 5 to 10 mm in length.&lt;/div&gt;&lt;/div&gt;&lt;div&gt;&lt;br /&gt;&lt;/div&gt;&lt;div class=&quot;separator&quot; style=&quot;clear: both; text-align: center;&quot;&gt;&lt;a href=&quot;https://blogger.googleusercontent.com/img/b/R29vZ2xl/AVvXsEi-nkAyRKtMD9O3EThL5mdOkaf-56HnAkRAJi67uoRGgsbUnMnKO6YBe0YKrRDIb8h276HqVUOfoIGE-8eqkqSNjXDcG4zJD9MnxYjZll51XdWQWt0uBtVPpYpJoiAnjOWu4mv1vttWZJCCXJOuY9blXnMPmRMNYjTIvuaDX5ag9ywMGf5rus7W1c2u_w/s500/Medial-meniscal-tear-on-the-MRI-frontal-view.jpg&quot; style=&quot;margin-left: 1em; margin-right: 1em;&quot;&gt;&lt;img alt=&quot;Medial meniscal tear on the MRI frontal view&quot; border=&quot;0&quot; data-original-height=&quot;409&quot; data-original-width=&quot;500&quot; height=&quot;524&quot; src=&quot;https://blogger.googleusercontent.com/img/b/R29vZ2xl/AVvXsEi-nkAyRKtMD9O3EThL5mdOkaf-56HnAkRAJi67uoRGgsbUnMnKO6YBe0YKrRDIb8h276HqVUOfoIGE-8eqkqSNjXDcG4zJD9MnxYjZll51XdWQWt0uBtVPpYpJoiAnjOWu4mv1vttWZJCCXJOuY9blXnMPmRMNYjTIvuaDX5ag9ywMGf5rus7W1c2u_w/w640-h524/Medial-meniscal-tear-on-the-MRI-frontal-view.jpg&quot; title=&quot;Medial meniscal tear on the MRI frontal view&quot; width=&quot;640&quot; /&gt;&lt;/a&gt;&lt;/div&gt;&lt;div&gt;&lt;div&gt;&lt;b&gt;&lt;span style=&quot;color: #017087;&quot;&gt;Figure 23–2.&lt;/span&gt;&lt;/b&gt; Medial meniscal tear on the MRI frontal view, seen as a small white line through the&lt;/div&gt;&lt;div&gt;black meniscus. (Courtesy of Heidi Chumley, MD)&lt;/div&gt;&lt;/div&gt;&lt;div&gt;&lt;br /&gt;&lt;/div&gt;&lt;div&gt;&lt;br /&gt;&lt;/div&gt;&lt;div&gt;&lt;div&gt;&lt;span&gt;&amp;nbsp;&amp;nbsp; &amp;nbsp;&lt;/span&gt;Surgical options include meniscal repair, open and arthroscopic partial meniscectomy, and meniscal transplant.&lt;b&gt; Meniscal repair is reserved for acute tears in&amp;nbsp;&lt;/b&gt;&lt;b&gt;the “red-red” zone of a nondegenerative meniscus. Arthroscopic partial meniscectomy&amp;nbsp;&lt;/b&gt;&lt;b&gt;is the current standard of care for most tears and can relieve pain and&amp;nbsp;&lt;/b&gt;&lt;b&gt;eliminate mechanical blocks to motion while preserving as much healthy tissue as&amp;nbsp;&lt;/b&gt;&lt;b&gt;possible.&lt;/b&gt; Indications include symptomatic radial or longitudinal tears, patients who have failed nonoperative management, displaced bucket-handle tears, tears creating a mechanical block, and symptomatic discoid lateral meniscus. Patients with severe degenerative joint disease may be candidates for knee replacement. Meniscal allograft transplant is indicated in a select group of young patients who have failed prior partial or total meniscectomy with minimal degenerative changes.&lt;/div&gt;&lt;/div&gt;&lt;div&gt;&lt;br /&gt;&lt;/div&gt;&lt;div&gt;&lt;br /&gt;&lt;/div&gt;&lt;div&gt;&lt;div&gt;&lt;b&gt;&lt;i&gt;&lt;span style=&quot;font-size: medium;&quot;&gt;Complications&lt;/span&gt;&lt;/i&gt;&lt;/b&gt;&lt;/div&gt;&lt;div&gt;&lt;br /&gt;&lt;/div&gt;&lt;div&gt;Because the meniscus is responsible for the distribution of load and shear forces across the knee, the initial meniscal injury as well as its management (ie, partial or total meniscectomy) can increase the contact pressures between the femur and tibia, accelerating the progression of degenerative joint disease. Additionally, repairs can fail and infections can occur, either postoperatively or after an intraarticular injection.&lt;/div&gt;&lt;/div&gt;&lt;div&gt;&lt;br /&gt;&lt;/div&gt;&lt;div&gt;&lt;br /&gt;&lt;/div&gt;&lt;div&gt;&lt;b&gt;&lt;span style=&quot;color: #017087; font-size: medium;&quot;&gt;&lt;u&gt;COMPREHENSION QUESTIONS&lt;/u&gt;&lt;/span&gt;&lt;/b&gt;&lt;/div&gt;&lt;div&gt;&lt;br /&gt;&lt;/div&gt;&lt;div&gt;&lt;div&gt;23.1 A college football player reports pain along the medial joint line with intermittent locking and an inability to fully extend the knee after a twisting injury to his right knee. He has a negative Lachman exam and a positive McMurray test. What is the most likely diagnosis?&lt;/div&gt;&lt;/div&gt;&lt;blockquote style=&quot;border: none; margin: 0px 0px 0px 40px; padding: 0px; text-align: left;&quot;&gt;&lt;div&gt;&lt;div&gt;A. Longitudinal vertical lateral meniscus tear&lt;/div&gt;&lt;/div&gt;&lt;div&gt;&lt;div&gt;B. ACL injury&lt;/div&gt;&lt;/div&gt;&lt;div&gt;&lt;div&gt;C. PCL injury&lt;/div&gt;&lt;/div&gt;&lt;div&gt;&lt;div&gt;D. Bucket-handle medial meniscus tear&lt;/div&gt;&lt;/div&gt;&lt;div&gt;&lt;div&gt;E. None of the above&lt;/div&gt;&lt;/div&gt;&lt;/blockquote&gt;&lt;div&gt;&lt;div&gt;&lt;br /&gt;&lt;/div&gt;&lt;div&gt;23.2 Which of the following physical examination maneuvers is the most sensitive in the diagnosis of a medial meniscus tear?&lt;/div&gt;&lt;/div&gt;&lt;blockquote style=&quot;border: none; margin: 0px 0px 0px 40px; padding: 0px; text-align: left;&quot;&gt;&lt;div&gt;&lt;div&gt;A. Apley test&lt;/div&gt;&lt;/div&gt;&lt;div&gt;&lt;div&gt;B. McMurray test&lt;/div&gt;&lt;/div&gt;&lt;div&gt;&lt;div&gt;C. Medial joint line tenderness&lt;/div&gt;&lt;/div&gt;&lt;div&gt;&lt;div&gt;D. Lachman test&lt;/div&gt;&lt;/div&gt;&lt;div&gt;&lt;div&gt;E. Pain with valgus stress&lt;/div&gt;&lt;/div&gt;&lt;/blockquote&gt;&lt;div&gt;&lt;div&gt;&lt;br /&gt;&lt;/div&gt;&lt;div&gt;23.3 A 20-year-old soccer player sustained a twisting injury to his knee approximately 1 month ago. He complains of continued medial-sided knee pain with occasional locking and catching. MRI findings are consistent with a tear of the medial meniscus. What physical exam finding is classically seen with this injury?&lt;/div&gt;&lt;/div&gt;&lt;blockquote style=&quot;border: none; margin: 0px 0px 0px 40px; padding: 0px; text-align: left;&quot;&gt;&lt;div&gt;&lt;div&gt;A. Positive anterior drawer test&lt;/div&gt;&lt;/div&gt;&lt;div&gt;&lt;div&gt;B. No end point with varus stressing of the knee&lt;/div&gt;&lt;/div&gt;&lt;div&gt;&lt;div&gt;C. Palpable pop when bringing the flexed knee to extension while internally rotating the leg&lt;/div&gt;&lt;/div&gt;&lt;div&gt;&lt;div&gt;D. Palpable pop when bringing the flexed knee to extension while externally rotating the leg&lt;/div&gt;&lt;/div&gt;&lt;/blockquote&gt;&lt;div&gt;&lt;br /&gt;&lt;/div&gt;&lt;div&gt;&lt;br /&gt;&lt;/div&gt;&lt;div&gt;&lt;b&gt;&lt;span style=&quot;color: #017087; font-size: medium;&quot;&gt;&lt;u&gt;ANSWERS&lt;/u&gt;&lt;/span&gt;&lt;/b&gt;&lt;/div&gt;&lt;div&gt;&lt;br /&gt;&lt;/div&gt;&lt;div&gt;&lt;div&gt;23.1 &lt;b&gt;D.&lt;/b&gt; The patient has a bucket-handle medial meniscus tear after an acute injury. Such tears are associated with knee locking and a block to full extension. These tears are more common in young patients, often associated with ACL tears, and are the most commonly missed meniscal tear on MRI.&lt;/div&gt;&lt;div&gt;&lt;br /&gt;&lt;/div&gt;&lt;div&gt;23.2 &lt;b&gt;C.&lt;/b&gt; Although the McMurray and Apley grind tests are commonly used to aid in diagnosis, joint line tenderness to palpation has been shown to be the most sensitive physical exam finding for meniscal injury.&lt;/div&gt;&lt;/div&gt;&lt;div&gt;&lt;div&gt;&lt;br /&gt;&lt;/div&gt;&lt;div&gt;23.3 &lt;b&gt;D.&lt;/b&gt; A positive McMurray test in the setting of a medial meniscus tear is the presence of a palpable pop as the knee is brought from a flexed to extended position while externally rotating the leg. C describes a positive McMurray test for a lateral meniscus tear. A and B do not describe tests for meniscal pathology, but instead the anterior cruciate ligament and the lateral collateral ligament, respectively.&lt;/div&gt;&lt;/div&gt;&lt;div&gt;&lt;br /&gt;&lt;/div&gt;&lt;div&gt;&lt;b style=&quot;background-color: #4093a5;&quot;&gt;&amp;nbsp; &amp;nbsp;&lt;span style=&quot;font-size: medium;&quot;&gt;&amp;nbsp;&lt;/span&gt;&lt;span style=&quot;color: white;&quot;&gt;&lt;span style=&quot;font-size: medium;&quot;&gt;CLINICAL PEARLS&lt;/span&gt;&lt;span&gt;&amp;nbsp; &amp;nbsp;&amp;nbsp;&lt;/span&gt;&lt;/span&gt;&lt;/b&gt;&lt;/div&gt;&lt;div&gt;&lt;table border=&quot;1&quot; cellpadding=&quot;0&quot; cellspacing=&quot;0&quot; class=&quot;MsoTableGrid&quot; style=&quot;border-collapse: collapse; border: none; mso-border-alt: solid windowtext .5pt; mso-padding-alt: 0in 5.4pt 0in 5.4pt; mso-yfti-tbllook: 1184;&quot;&gt;
  281. &lt;tbody&gt;&lt;tr style=&quot;height: 19.15pt; mso-yfti-firstrow: yes; mso-yfti-irow: 0; mso-yfti-lastrow: yes;&quot;&gt;
  282.  &lt;td style=&quot;background: rgb(221, 237, 239); border-left: solid #017087 1.0pt; border: 1pt solid rgb(64, 147, 165); height: 19.15pt; padding: 0in 5.4pt; width: 433.7pt;&quot; valign=&quot;top&quot; width=&quot;578&quot;&gt;
  283.  &lt;div style=&quot;line-height: normal; margin-bottom: 0in;&quot;&gt;&lt;span style=&quot;color: #017087;&quot;&gt;&lt;br /&gt;&lt;/span&gt;&lt;/div&gt;&lt;div style=&quot;line-height: normal; margin-bottom: 0in;&quot;&gt;&lt;span style=&quot;color: #017087;&quot;&gt;►&lt;/span&gt;&amp;nbsp;Meniscal tears are most accurately diagnosed by history and physical exam, with joint line tenderness being the most sensitive indicator of a meniscal tear.&lt;/div&gt;&lt;div style=&quot;line-height: normal; margin-bottom: 0in;&quot;&gt;&lt;span style=&quot;color: #017087;&quot;&gt;&lt;br /&gt;&lt;/span&gt;&lt;/div&gt;&lt;div style=&quot;line-height: normal; margin-bottom: 0in;&quot;&gt;&lt;span style=&quot;color: #017087;&quot;&gt;►&lt;/span&gt;&amp;nbsp;Two tests commonly used for the evaluation of meniscus-specific pathology are the McMurray and Apley tests, both of which have a high specificity and low sensitivity.&lt;/div&gt;&lt;div style=&quot;line-height: normal; margin-bottom: 0in;&quot;&gt;&lt;span style=&quot;color: #017087;&quot;&gt;&lt;br /&gt;&lt;/span&gt;&lt;/div&gt;&lt;div style=&quot;line-height: normal; margin-bottom: 0in;&quot;&gt;&lt;span style=&quot;color: #017087;&quot;&gt;►&lt;/span&gt;&amp;nbsp;MRI has an accuracy of approximately 95% in diagnosing tears. Asymptomatic, partial-thickness tears less than 5 to 10 mm in length are typically treated nonoperatively.&lt;/div&gt;&lt;div style=&quot;line-height: normal; margin-bottom: 0in;&quot;&gt;&lt;span style=&quot;color: #017087;&quot;&gt;&lt;br /&gt;&lt;/span&gt;&lt;/div&gt;&lt;div style=&quot;line-height: normal; margin-bottom: 0in;&quot;&gt;&lt;span style=&quot;color: #017087;&quot;&gt;►&amp;nbsp;&lt;/span&gt;Indications for arthroscopic partial meniscectomy include symptomatic radial or longitudinal tears, patients who have failed nonoperative management, displaced bucket-handle tears, tears creating a mechanical block, and symptomatic discoid lateral meniscus.&lt;/div&gt;&lt;div style=&quot;line-height: normal; margin-bottom: 0in;&quot;&gt;&lt;br /&gt;&lt;/div&gt;
  284.  &lt;/td&gt;
  285. &lt;/tr&gt;
  286. &lt;/tbody&gt;&lt;/table&gt;&lt;br /&gt;&lt;/div&gt;
  287.  
  288. &lt;details close=&quot;&quot;&gt;
  289.  &lt;summary&gt;&lt;b&gt;&lt;span style=&quot;color: #00788b;&quot;&gt;REFERENCES&lt;/span&gt;&lt;/b&gt;&lt;/summary&gt;
  290.  &lt;p&gt;Douglas JA, Sgaglione NA. Meniscal injuries. In: Schepsis AA, Busconi BD, eds. Orthopaedic Surgery
  291. Essentials: Sports Medicine. Philadelphia, PA: Lippincott Williams &amp;amp; Wilkins; 2006.&amp;nbsp;&lt;/p&gt;&lt;p&gt;Greis PE, Bardana DD, Holmstrom MC, Bruks RT. Meniscal injury: I. Basic science and evaluation. J Am
  292. Acad Orthop Surg. 2002;10:168-176.&amp;nbsp;&lt;/p&gt;&lt;p&gt;Ryzewicz M, Peterson B, Siparsky PN, Bartz RL. The diagnosis of meniscus tears: the role of MRI and
  293. clinical examination. Clin Orthop Relat Res. 2007;455:124-133.&lt;/p&gt;
  294. &lt;/details&gt;&lt;/div&gt;</content><link rel='replies' type='application/atom+xml' href='https://medical-phd.blogspot.com/feeds/9106962394899765164/comments/default' title='Post Comments'/><link rel='replies' type='text/html' href='https://medical-phd.blogspot.com/2022/03/meniscal-tears-case-file.html#comment-form' title='0 Comments'/><link rel='edit' type='application/atom+xml' href='https://www.blogger.com/feeds/3302710210518459416/posts/default/9106962394899765164'/><link rel='self' type='application/atom+xml' href='https://www.blogger.com/feeds/3302710210518459416/posts/default/9106962394899765164'/><link rel='alternate' type='text/html' href='https://medical-phd.blogspot.com/2022/03/meniscal-tears-case-file.html' title='Meniscal Tears Case File'/><author><name>Medical Group</name><uri>http://www.blogger.com/profile/01780656800578397393</uri><email>noreply@blogger.com</email><gd:image rel='http://schemas.google.com/g/2005#thumbnail' width='32' height='32' src='//blogger.googleusercontent.com/img/b/R29vZ2xl/AVvXsEiMuROkBDv6yw4SmxGqRA5BPQhOXSwFld-Gufmy4kUqfpoAGQRl7cv-3naew6wjAnCjRFSe_FKX8z7CdE6cFKxxBpnqBuwUTSr-15xd3mFGcr_wJOX2y73G_zJCGND98Q/s220/Final-logo.jpg'/></author><media:thumbnail xmlns:media="http://search.yahoo.com/mrss/" url="https://blogger.googleusercontent.com/img/b/R29vZ2xl/AVvXsEgAIoowDVW89v37Nedp2vIoliCp_WhZ26Os2lDoB0gDcUnaYQJRhNyQLhFk52UuyhxUHm5SlXgsQoMNBP-uZU9ZMoB7sTz2MdPFzfgnVtAzMK0F3L3ErwB6rnpxdxxq_NE0wPE68S6XgwwzSTEveDZoipWAfLv4sbza_JHK-unM_onFsDmo33AeSOWojQ/s72-w640-h278-c/Tests-for-tear-of-the-medial-meniscus.jpg" height="72" width="72"/><thr:total>0</thr:total></entry><entry><id>tag:blogger.com,1999:blog-3302710210518459416.post-3300321162947111263</id><published>2022-03-25T04:35:00.000-07:00</published><updated>2022-03-25T04:35:01.156-07:00</updated><category scheme="http://www.blogger.com/atom/ns#" term="Case Files"/><category scheme="http://www.blogger.com/atom/ns#" term="Orthopaedic Surgery Case File"/><title type='text'>Anterior Cruciate Ligament Reconstruction Case File</title><content type='html'>&lt;div class=&quot;scrollbox&quot;&gt;
  295.  
  296. &lt;div style=&quot;text-align: left;&quot;&gt;&lt;b&gt;&lt;span style=&quot;font-size: large;&quot;&gt;Anterior Cruciate Ligament Reconstruction Case File&lt;/span&gt;&lt;/b&gt;&lt;/div&gt;&lt;div&gt;Eugene C. Toy, MD, Andrew J. Rosenbaum, MD, Timothy T. Roberts, MD, Joshua S. Dines, MD&lt;/div&gt;&lt;div&gt;&lt;br /&gt;&lt;/div&gt;&lt;div&gt;&lt;b&gt;&lt;span style=&quot;color: #017087; font-size: medium;&quot;&gt;CASE 22&lt;/span&gt;&lt;/b&gt;&lt;/div&gt;&lt;div&gt;&lt;div&gt;An 18-year-old female athlete presents after injuring her right knee during a soccer game. She states that she was running for a ball downfield when she planted her right foot to cut inside a defender, heard a “pop,” and fell to the ground in pain. She was unable to leave the field on her own power. Over the next 10 to 15 minutes, her right knee swelled considerably compared with her left. She was placed into a knee immobilizer by her trainer and presents to your office the day after the injury.&lt;/div&gt;&lt;div&gt;&lt;br /&gt;&lt;/div&gt;&lt;div&gt;&lt;span&gt;&amp;nbsp;&amp;nbsp; &amp;nbsp;&lt;/span&gt;On physical exam, the patient has a significant effusion and holds her knee in approximately 10 degrees of flexion; she is unable to flex past 90 degrees. She is uncomfortable and guarding throughout your examination. Her knee is aspirated, and 40 mL of blood is extracted. She is more comfortable after this and her range of motion is improved. She has no joint line tenderness. Her knee is stable to varus and valgus stress. On Lachman exam, there is significant anterior translation compared with her left, uninjured knee without a firm end point. Anterior drawer testing reveals increased tibial translation, whereas posterior drawer testing shows symmetric posterior translation.&lt;/div&gt;&lt;div&gt;&lt;br /&gt;&lt;/div&gt;&lt;div&gt;&lt;span style=&quot;color: #017087;&quot;&gt;►&lt;/span&gt;&amp;nbsp;What is the most likely diagnosis?&lt;/div&gt;&lt;div&gt;&lt;span style=&quot;color: #017087;&quot;&gt;►&lt;/span&gt;&amp;nbsp;What physical exam finding confirms your diagnosis?&lt;/div&gt;&lt;div&gt;&lt;span style=&quot;color: #017087;&quot;&gt;►&lt;/span&gt;&amp;nbsp;What imaging study should be ordered to confirm your diagnosis?&lt;/div&gt;&lt;div&gt;&lt;span style=&quot;color: #017087;&quot;&gt;►&lt;/span&gt;&amp;nbsp;What is the most appropriate treatment for this patient?&lt;/div&gt;&lt;/div&gt;&lt;div&gt;&lt;br /&gt;&lt;/div&gt;&lt;div&gt;&lt;br /&gt;&lt;/div&gt;&lt;div&gt;&lt;div&gt;&lt;b&gt;&lt;span style=&quot;font-size: medium;&quot;&gt;ANSWER TO CASE 22:&lt;/span&gt;&lt;/b&gt;&lt;/div&gt;&lt;div&gt;&lt;span style=&quot;background-color: #daebf0; font-size: medium;&quot;&gt;&lt;span&gt;&lt;span style=&quot;background-color: transparent;&quot;&gt;&lt;span style=&quot;color: #017087;&quot;&gt;&lt;b&gt;Anterior Cruciate Ligament Reconstruction&lt;/b&gt;&lt;/span&gt;&lt;/span&gt;&lt;b&gt;&amp;nbsp;&amp;nbsp;&lt;/b&gt;&lt;/span&gt;&lt;span style=&quot;font-weight: bold;&quot;&gt;&amp;nbsp;&amp;nbsp; &amp;nbsp;&lt;/span&gt;&lt;span style=&quot;font-weight: bold;&quot;&gt;&amp;nbsp;&amp;nbsp; &amp;nbsp;&lt;/span&gt;&lt;span style=&quot;font-weight: bold;&quot;&gt;&amp;nbsp;&amp;nbsp; &amp;nbsp;&lt;/span&gt;&lt;span style=&quot;font-weight: bold;&quot;&gt;&amp;nbsp;&amp;nbsp; &amp;nbsp;&lt;/span&gt;&lt;span style=&quot;font-weight: bold;&quot;&gt;&amp;nbsp;&amp;nbsp; &amp;nbsp;&lt;/span&gt;&lt;span style=&quot;font-weight: bold;&quot;&gt;&amp;nbsp;&amp;nbsp; &amp;nbsp;&lt;/span&gt;&lt;span style=&quot;font-weight: bold;&quot;&gt;&amp;nbsp; &amp;nbsp;&amp;nbsp;&lt;/span&gt;&lt;/span&gt;&lt;/div&gt;&lt;/div&gt;&lt;div&gt;&lt;br /&gt;&lt;/div&gt;&lt;div&gt;&lt;div&gt;&lt;i&gt;Summary&lt;/i&gt;: An 18-year-old female soccer player presents 1 day after injuring her right knee after a noncontact pivoting injury during a soccer game. She had immediate pain and quickly developed an effusion. She was unable to leave the field under her own power. Arthrocentesis produced a large hemarthrosis. She has no joint line tenderness. Lachman and anterior drawer tests are positive. Her knee is stable to varus and valgus stress.&lt;/div&gt;&lt;div&gt;&lt;ul style=&quot;text-align: left;&quot;&gt;&lt;li&gt;&lt;b&gt;Most likely diagnosis:&lt;/b&gt; Anterior cruciate ligament (ACL) rupture.&lt;/li&gt;&lt;li&gt;&lt;b&gt;Confirmatory physical finding:&lt;/b&gt; Positive Lachman test.&lt;/li&gt;&lt;li&gt;&lt;b&gt;Imaging:&lt;/b&gt; Knee x-rays followed by magnetic resonance imaging (MRI).&lt;/li&gt;&lt;li&gt;&lt;b&gt;Treatment:&lt;/b&gt; ACL reconstruction after regaining knee motion.&lt;/li&gt;&lt;/ul&gt;&lt;/div&gt;&lt;/div&gt;&lt;div&gt;&lt;br /&gt;&lt;/div&gt;&lt;div&gt;&lt;br /&gt;&lt;/div&gt;&lt;div&gt;&lt;div&gt;&lt;b&gt;&lt;span style=&quot;color: #017087; font-size: medium;&quot;&gt;ANALYSIS&lt;/span&gt;&lt;/b&gt;&lt;/div&gt;&lt;div&gt;&lt;b&gt;&lt;span style=&quot;font-size: medium;&quot;&gt;&lt;i&gt;Objectives&lt;/i&gt;&lt;/span&gt;&lt;/b&gt;&lt;/div&gt;&lt;/div&gt;&lt;div&gt;&lt;div&gt;&lt;ol style=&quot;text-align: left;&quot;&gt;&lt;li&gt;Know the anatomy and function of the ACL.&lt;/li&gt;&lt;li&gt;Understand the pertinent physical exam findings, differential diagnosis, and confirmatory imaging modalities for ACL injury.&lt;/li&gt;&lt;li&gt;Be familiar with the rationale for nonoperative and operative treatment of ACL injuries.&lt;/li&gt;&lt;/ol&gt;&lt;/div&gt;&lt;/div&gt;&lt;div&gt;&lt;br /&gt;&lt;/div&gt;&lt;div&gt;&lt;br /&gt;&lt;/div&gt;&lt;div&gt;&lt;div&gt;&lt;i&gt;&lt;b&gt;&lt;span style=&quot;font-size: medium;&quot;&gt;Considerations&lt;/span&gt;&lt;/b&gt;&lt;/i&gt;&lt;/div&gt;&lt;div&gt;&lt;br /&gt;&lt;/div&gt;&lt;div&gt;This 18-year-old female athlete presents with concern for an ACL injury. The mechanism, a noncontact pivoting injury with the foot firmly planted on the ground, is consistent with this. The “pop” she heard, coupled with the rapid development of a joint effusion, also supports this diagnosis. In the office it is important to obtain a reliable physical exam, which can be impeded by a hemarthrosis, as seen in this case. Joint effusions are painful, limit both active and passive range of motion, and complicate the practitioner’s ability to accurately perform a complete exam of the knee. After aspiration, the patient’s range of motion improves, and Lachman and anterior drawer tests are found to be positive. The next step in the workup includes plain radiographs and an MRI. The ACL tear must be confirmed and associated fractures and other injuries ruled out. After this, a treatment plan can be established.&lt;/div&gt;&lt;/div&gt;&lt;div&gt;&lt;br /&gt;&lt;/div&gt;&lt;div&gt;&lt;br /&gt;&lt;/div&gt;&lt;div&gt;&lt;div&gt;&lt;b&gt;&lt;span style=&quot;font-size: medium;&quot;&gt;APPROACH TO:&lt;/span&gt;&lt;/b&gt;&lt;/div&gt;&lt;div&gt;&lt;span style=&quot;background-color: #daebf0; color: #017087; font-size: medium;&quot;&gt;&lt;span&gt;&lt;span style=&quot;background-color: transparent;&quot;&gt;&lt;b&gt;Anterior Cruciate Ligament Reconstruction&lt;/b&gt;&lt;/span&gt;&lt;b&gt;&amp;nbsp;&lt;/b&gt;&lt;/span&gt;&lt;span style=&quot;font-weight: bold;&quot;&gt;&amp;nbsp;&amp;nbsp; &amp;nbsp;&lt;/span&gt;&lt;span style=&quot;font-weight: bold;&quot;&gt;&amp;nbsp;&amp;nbsp; &amp;nbsp;&lt;/span&gt;&lt;span style=&quot;font-weight: bold;&quot;&gt;&amp;nbsp;&amp;nbsp; &amp;nbsp;&lt;/span&gt;&lt;span style=&quot;font-weight: bold;&quot;&gt;&amp;nbsp;&amp;nbsp; &amp;nbsp;&lt;/span&gt;&lt;span style=&quot;font-weight: bold;&quot;&gt;&amp;nbsp;&amp;nbsp; &amp;nbsp;&lt;/span&gt;&lt;span style=&quot;font-weight: bold;&quot;&gt;&amp;nbsp;&amp;nbsp; &amp;nbsp;&lt;/span&gt;&lt;span style=&quot;font-weight: bold;&quot;&gt;&amp;nbsp;&amp;nbsp; &amp;nbsp;&lt;/span&gt;&lt;span style=&quot;font-weight: bold;&quot;&gt;&amp;nbsp;&amp;nbsp;&amp;nbsp;&lt;/span&gt;&lt;span style=&quot;font-weight: bold;&quot;&gt;&amp;nbsp;&lt;/span&gt;&lt;/span&gt;&lt;/div&gt;&lt;/div&gt;&lt;div&gt;&lt;br /&gt;&lt;/div&gt;&lt;div&gt;&lt;b&gt;&lt;span style=&quot;color: #017087; font-size: medium;&quot;&gt;DEFINITIONS&lt;/span&gt;&lt;/b&gt;&lt;/div&gt;&lt;div&gt;&lt;br /&gt;&lt;/div&gt;&lt;div&gt;&lt;div&gt;&lt;b&gt;LACHMAN TEST:&lt;/b&gt; The most reliable and clinically sensitive physical exam test for detecting an ACL tear. To perform the test, place the patient supine on the exam table with the knee in approximately 30 degrees flexion. The examiner should place one hand behind the tibia with the thumb on the tibial tuberosity. The other hand is placed on the patient’s thigh. As the tibia is pulled anteriorly, an intact ACL should prevent forward translation on the femur. When the ACL is intact, a firm end point is observed. Findings consistent with a positive Lachman test include anterior tibial translation with a soft end point, more than 2 mm of anterior translation compared with the contralateral knee, and greater than 10 mm of total anterior translation.&lt;/div&gt;&lt;div&gt;&lt;b&gt;&lt;br /&gt;&lt;/b&gt;&lt;/div&gt;&lt;div&gt;&lt;b&gt;ANTERIOR DRAWER TEST:&lt;/b&gt; Test performed to evaluate for ACL injury. The patient is placed supine with the hip flexed to 45 degrees and the knee to 90 degrees. The examiner should sit on the patient’s feet and grasp the tibia, pulling it forward (or backward in the posterior drawer test, which is used to diagnose posterior cruciate ligament [PCL] injury). Increased forward tibial translation and end point laxity suggest a ruptured ACL (increased posterior translation suggests a PCL injury).&lt;/div&gt;&lt;div&gt;&lt;b&gt;&lt;br /&gt;&lt;/b&gt;&lt;/div&gt;&lt;div&gt;&lt;b&gt;PIVOT SHIFT TEST: &lt;/b&gt;Test used to assess rotational stability of the knee. It is performed by applying slight distal traction on the extended leg with a valgus and internal rotation force then applied. In this position, the tibia will be subluxated anteriorly in an ACL-deficient knee. The knee is then flexed to 30 degrees, at which time the iliotibial band transitions from a knee extensor to flexor, helping to reduce the subluxated tibia. General anesthesia is often required to perform this test in the acute setting because of pain and guarding.&lt;/div&gt;&lt;/div&gt;&lt;div&gt;&lt;br /&gt;&lt;/div&gt;&lt;div&gt;&lt;br /&gt;&lt;/div&gt;&lt;div&gt;&lt;b&gt;&lt;span style=&quot;color: #017087; font-size: medium;&quot;&gt;CLINICAL APPROACH&lt;/span&gt;&lt;/b&gt;&lt;/div&gt;&lt;div&gt;&lt;br /&gt;&lt;/div&gt;&lt;div&gt;&lt;div&gt;&lt;i&gt;&lt;span style=&quot;font-size: medium;&quot;&gt;&lt;b&gt;Epidemiology&lt;/b&gt;&lt;/span&gt;&lt;/i&gt;&lt;/div&gt;&lt;div&gt;&lt;br /&gt;&lt;/div&gt;&lt;div&gt;ACL injuries are highly publicized and researched. They comprise nearly half of all knee ligament injuries and are more common in the young, athletic population, with 70% occurring secondary to sporting activities. Skiing and soccer have the highest risk of ACL rupture, with female athletes being 2 to 8 times more likely to tear their ACL than men.&lt;/div&gt;&lt;div&gt;&lt;i&gt;&lt;b&gt;&lt;span style=&quot;font-size: medium;&quot;&gt;&lt;br /&gt;&lt;/span&gt;&lt;/b&gt;&lt;/i&gt;&lt;/div&gt;&lt;div&gt;&lt;i&gt;&lt;b&gt;&lt;span style=&quot;font-size: medium;&quot;&gt;Anatomy and Biomechanics&lt;/span&gt;&lt;/b&gt;&lt;/i&gt;&lt;/div&gt;&lt;div&gt;&lt;br /&gt;&lt;/div&gt;&lt;div&gt;The ACL originates from the medial wall of the lateral femoral condyle and travels obliquely, inserting on the tibia in the anterior aspect of the intercondylar eminence of the tibia ( Figure 22–1 ). This location is adjacent to the anterior insertion of the lateral meniscus. &lt;b&gt;It is made up of 2 distinct components, the anteromedial (AM)&amp;nbsp;&lt;/b&gt;&lt;b&gt;and posterolateral (PL) bundles, which are named for their tibial insertions.&lt;/b&gt; The ligament is intraarticular but extrasynovial and receives its blood supply from the middle geniculate artery. The ACL is approximately 33-mm long and 11 mm in diameter. It can resist a load of approximately 2200 newtons.&lt;/div&gt;&lt;/div&gt;&lt;div&gt;&lt;br /&gt;&lt;/div&gt;&lt;div class=&quot;separator&quot; style=&quot;clear: both; text-align: center;&quot;&gt;&lt;a href=&quot;https://blogger.googleusercontent.com/img/b/R29vZ2xl/AVvXsEjPQaZtedoDvmNKkkoP1QPCxMhysQx0siynBYKFZxtsoe55ZqEljikCSCIDT86lipo4VKsa8q7NeTrf7jCAiz9PEIlwtnEtPOcog5S6cHdzZtIPGpn8Erhsc_GvmYtfB3h75L2MJk5VjsDX6NxI1guatLvNFr4kQZFVpBv4g7ITFAKd8X0L_tX5taERrw/s778/right-knee-joint-with-the-joint-capsule.jpg&quot; style=&quot;margin-left: 1em; margin-right: 1em;&quot;&gt;&lt;img alt=&quot;right knee joint with the joint capsule&quot; border=&quot;0&quot; data-original-height=&quot;528&quot; data-original-width=&quot;778&quot; height=&quot;434&quot; src=&quot;https://blogger.googleusercontent.com/img/b/R29vZ2xl/AVvXsEjPQaZtedoDvmNKkkoP1QPCxMhysQx0siynBYKFZxtsoe55ZqEljikCSCIDT86lipo4VKsa8q7NeTrf7jCAiz9PEIlwtnEtPOcog5S6cHdzZtIPGpn8Erhsc_GvmYtfB3h75L2MJk5VjsDX6NxI1guatLvNFr4kQZFVpBv4g7ITFAKd8X0L_tX5taERrw/w640-h434/right-knee-joint-with-the-joint-capsule.jpg&quot; title=&quot;right knee joint with the joint capsule&quot; width=&quot;640&quot; /&gt;&lt;/a&gt;&lt;/div&gt;&lt;br /&gt;&lt;div&gt;&lt;div class=&quot;separator&quot; style=&quot;clear: both; text-align: center;&quot;&gt;&lt;a href=&quot;https://blogger.googleusercontent.com/img/b/R29vZ2xl/AVvXsEj3iNSERzbh8eRGf4EO68W3uDkP4AvoblWci4p5tXPTiSrQgkIlCsJxnArTK8LjSDQftIrSARtdBMR2vGstNRcyCxi8ZaT9_CoOscWrlwSLAXH-Lbm2icBL3SeUnx4k0rYS08XE1_l52Y-1UXXUHfBczt0hGbUbJv688sZzgPLoikkjWbEG0ALVu6voLQ/s680/superior-views-of-the-right-knee-joint.jpg&quot; style=&quot;margin-left: 1em; margin-right: 1em;&quot;&gt;&lt;img alt=&quot;superior views of the right knee joint&quot; border=&quot;0&quot; data-original-height=&quot;408&quot; data-original-width=&quot;680&quot; height=&quot;384&quot; src=&quot;https://blogger.googleusercontent.com/img/b/R29vZ2xl/AVvXsEj3iNSERzbh8eRGf4EO68W3uDkP4AvoblWci4p5tXPTiSrQgkIlCsJxnArTK8LjSDQftIrSARtdBMR2vGstNRcyCxi8ZaT9_CoOscWrlwSLAXH-Lbm2icBL3SeUnx4k0rYS08XE1_l52Y-1UXXUHfBczt0hGbUbJv688sZzgPLoikkjWbEG0ALVu6voLQ/w640-h384/superior-views-of-the-right-knee-joint.jpg&quot; title=&quot;superior views of the right knee joint&quot; width=&quot;640&quot; /&gt;&lt;/a&gt;&lt;/div&gt;&lt;/div&gt;&lt;div&gt;&lt;div&gt;&lt;b&gt;&lt;span style=&quot;color: #017087;&quot;&gt;Figure 22–1.&lt;/span&gt;&lt;/b&gt; (&lt;b&gt;A&lt;/b&gt;) Anterior view of the right knee joint with the joint capsule open showing the patella&lt;/div&gt;&lt;div&gt;reflected inferiorly. (&lt;b&gt;B&lt;/b&gt;) Posterior and (&lt;b&gt;C&lt;/b&gt;) superior views of the right knee joint. (Reproduced, with&lt;/div&gt;&lt;div&gt;permission, from Morton DA, Foreman KB, Albertine KH. &lt;i&gt;The Big Picture: Gross Anatomy&lt;/i&gt;. New York, NY: McGraw-Hill; 2011:Fig. 36-5.)&lt;/div&gt;&lt;/div&gt;&lt;div&gt;&lt;br /&gt;&lt;/div&gt;&lt;div&gt;&lt;div&gt;The primary function of the ACL is to restrict anterior translation of the tibia relative to the femur. Secondarily, it provides a restraint to tibial rotation and varus or valgus stress. &lt;b&gt;The AM bundle is the larger of the 2, is tighter in flexion,&amp;nbsp;&lt;/b&gt;and provides the majority of the restraint against anterior translation of the tibia on the femur, particularly at mid-ranges of knee flexion. &lt;b&gt;The PL bundle is more&amp;nbsp;&lt;/b&gt;&lt;b&gt;taut in extension and plays a greater role in countering rotational forces.&lt;/b&gt;&lt;/div&gt;&lt;/div&gt;&lt;div&gt;&lt;br /&gt;&lt;/div&gt;&lt;div&gt;&lt;br /&gt;&lt;/div&gt;&lt;div&gt;&lt;div&gt;&lt;i&gt;&lt;span style=&quot;font-size: medium;&quot;&gt;&lt;b&gt;Differential Diagnosis&lt;/b&gt;&lt;/span&gt;&lt;/i&gt;&lt;/div&gt;&lt;div&gt;&lt;br /&gt;&lt;/div&gt;&lt;div&gt;The history given by the patient of an injury occurring during a cutting move, the audible “popping” or tearing sensation, and large postinjury effusion represent a classic presentation of an ACL tear. &lt;b&gt;Included in the differential diagnosis are patellar&amp;nbsp;&lt;/b&gt;&lt;b&gt;dislocation, meniscal injury, posterior cruciate ligament injury, and osteochondral&amp;nbsp;&lt;/b&gt;fracture. Most of these diagnoses can be confirmed with a thorough physical exam. An arthrocentesis of the affected knee can also add information, as the presence of fat within the fluid suggests a fracture, as opposed to a pure hemarthrosis, which is seen in a multitude of knee injuries.&lt;/div&gt;&lt;/div&gt;&lt;div&gt;&lt;br /&gt;&lt;/div&gt;&lt;div&gt;&lt;br /&gt;&lt;/div&gt;&lt;div&gt;&lt;div&gt;&lt;b&gt;&lt;i&gt;&lt;span style=&quot;font-size: medium;&quot;&gt;Physical Exam&lt;/span&gt;&lt;/i&gt;&lt;/b&gt;&lt;/div&gt;&lt;div&gt;&lt;br /&gt;&lt;/div&gt;&lt;div&gt;Physical exam of any joint should begin with simple inspection. Range of motion is assessed, and lack of extension can suggest a locked knee from a meniscal tear or loose body. The joint lines are palpated for presence of pain that would suggest meniscal injury. Varus and valgus stability is assessed at 0 degrees and 30 degrees to confirm the integrity of the medial and lateral collateral ligaments. The ACL is examined using the &lt;b&gt;Lachman test.&lt;/b&gt; This is performed by attempting to translate the proximal tibia anteriorly with respect to the distal femur with the knee held in 30 degrees of flexion. This tests the translation of the tibia as compared with the noninjured knee as well as evaluating for a firm end point. The &lt;b&gt;anterior and&amp;nbsp;&lt;/b&gt;&lt;b&gt;posterior drawer tests&lt;/b&gt; examine the ACL and PCL, respectively. The knee is flexed to 90 degrees, and translation anteriorly (anterior drawer) of the proximal tibia assesses the ACL, whereas posterior translation (posterior drawer) assesses the PCL. Care must be taken to recognize any posterior sag of the tibia before performing the anterior drawer test, as this would indicate a PCL injury and ultimately lead to a false-positive anterior drawer. The &lt;b&gt;pivot shift test&lt;/b&gt; is another sensitive test for ACL rupture and assesses the secondary rotatory restraint of the ACL. In the acute setting, this is quite painful and is most often used intraoperatively during an exam under anesthesia.&lt;/div&gt;&lt;/div&gt;&lt;div&gt;&lt;br /&gt;&lt;/div&gt;&lt;div&gt;&lt;br /&gt;&lt;/div&gt;&lt;div&gt;&lt;div&gt;&lt;b&gt;&lt;i&gt;&lt;span style=&quot;font-size: medium;&quot;&gt;Imaging&lt;/span&gt;&lt;/i&gt;&lt;/b&gt;&lt;/div&gt;&lt;div&gt;&lt;br /&gt;&lt;/div&gt;&lt;div&gt;Anteroposterior (AP) and lateral views of the knee should be the first imaging studies obtained to rule out any associated fractures. This includes tibial eminence fractures, which can mimic a ligamentous injury, and the &lt;b&gt;“Segond fracture,”&lt;/b&gt; which is a lateral capsular avulsion fracture that is considered pathognomonic for an underlying ACL injury. Merchant, or “sunrise,” views are special views of the patella that evaluate the bony architecture of the patellofemoral articulation for fracture or dislocation.&lt;/div&gt;&lt;div&gt;&lt;br /&gt;&lt;/div&gt;&lt;div&gt;&lt;span&gt;&amp;nbsp;&amp;nbsp; &amp;nbsp;&lt;/span&gt;After initial radiographic evaluation, &lt;b&gt;MRI of the injured knee is highly sensitive&amp;nbsp;&lt;/b&gt;&lt;b&gt;and specific in confirming injury to intraarticular structures&lt;/b&gt; ( Figures 22–2A and 22–2B ). In addition to visualizing the ligaments, the MRI allows for evaluation for chondral injuries and bone bruises, which can also be indicative of ligamentous injury. The menisci may also be evaluated for injury with an MRI.&lt;/div&gt;&lt;/div&gt;&lt;div&gt;&lt;br /&gt;&lt;/div&gt;&lt;div class=&quot;separator&quot; style=&quot;clear: both; text-align: center;&quot;&gt;&lt;a href=&quot;https://blogger.googleusercontent.com/img/b/R29vZ2xl/AVvXsEjq1ZgDj_vzta7N-toUzUKwDdtAq6VkYyHpmtOaRMmGpJ8DTTn6OnD9GIaE5blCP2miNWm66Y1r5u1dKAwg7JsmnxIbiWIya7m23bJPKgNUgQ-KQYseqF8SPb7hS0hWneFltB2VKUs0R8mEf3wWuFjNAi7-UCa13nn24v7XhukNlKvhRdRNjxLcZjfF1A/s470/Sagittal-T2-weighted-fat-saturated-MRI.jpg&quot; style=&quot;margin-left: 1em; margin-right: 1em;&quot;&gt;&lt;img alt=&quot;Sagittal T2-weighted fat-saturated MRI&quot; border=&quot;0&quot; data-original-height=&quot;470&quot; data-original-width=&quot;367&quot; height=&quot;640&quot; src=&quot;https://blogger.googleusercontent.com/img/b/R29vZ2xl/AVvXsEjq1ZgDj_vzta7N-toUzUKwDdtAq6VkYyHpmtOaRMmGpJ8DTTn6OnD9GIaE5blCP2miNWm66Y1r5u1dKAwg7JsmnxIbiWIya7m23bJPKgNUgQ-KQYseqF8SPb7hS0hWneFltB2VKUs0R8mEf3wWuFjNAi7-UCa13nn24v7XhukNlKvhRdRNjxLcZjfF1A/w500-h640/Sagittal-T2-weighted-fat-saturated-MRI.jpg&quot; title=&quot;Sagittal T2-weighted fat-saturated MRI&quot; width=&quot;500&quot; /&gt;&lt;/a&gt;&lt;/div&gt;&lt;br /&gt;&lt;div&gt;&lt;div class=&quot;separator&quot; style=&quot;clear: both; text-align: center;&quot;&gt;&lt;a href=&quot;https://blogger.googleusercontent.com/img/b/R29vZ2xl/AVvXsEjVYlLZtj0awpD8WLQTHrrccb4tMCTGGxm6B8mCbk6F_W6Vh4tCX2Si7jLwS56fGDzGVtMIPEWHkPOQfgsfpJX8dUFK9SczKw49TgRq-6gv-Dqe_Q5PYIUNdNmnl5d78U2bqn_SyefcaJ56wTO2kSfvi6LBEht8LzVv_i2I_x9zzygKUQim79zdMLdU3Q/s483/Lateral-T1-weighted-MRI.jpg&quot; style=&quot;margin-left: 1em; margin-right: 1em;&quot;&gt;&lt;img alt=&quot;Lateral T1-weighted MRI&quot; border=&quot;0&quot; data-original-height=&quot;483&quot; data-original-width=&quot;460&quot; height=&quot;400&quot; src=&quot;https://blogger.googleusercontent.com/img/b/R29vZ2xl/AVvXsEjVYlLZtj0awpD8WLQTHrrccb4tMCTGGxm6B8mCbk6F_W6Vh4tCX2Si7jLwS56fGDzGVtMIPEWHkPOQfgsfpJX8dUFK9SczKw49TgRq-6gv-Dqe_Q5PYIUNdNmnl5d78U2bqn_SyefcaJ56wTO2kSfvi6LBEht8LzVv_i2I_x9zzygKUQim79zdMLdU3Q/w381-h400/Lateral-T1-weighted-MRI.jpg&quot; title=&quot;Lateral T1-weighted MRI&quot; width=&quot;381&quot; /&gt;&lt;/a&gt;&lt;/div&gt;&lt;/div&gt;&lt;div&gt;&lt;div&gt;&lt;b&gt;&lt;span style=&quot;color: #017087;&quot;&gt;Figure 22–2.&lt;/span&gt;&lt;/b&gt; (&lt;b&gt;A&lt;/b&gt;) Sagittal T2-weighted fat-saturated MRI of the knee showing a normal ACL ( arrow ).&lt;/div&gt;&lt;div&gt;Notice the fascicular arrangement. (Reproduced, with permission, from Chen MYM, Pope TL, Ott&lt;/div&gt;&lt;div&gt;DJ. &lt;i&gt;Basic Radiology&lt;/i&gt;. 2nd ed. New York, NY: McGraw-Hill; 2011:Fig. 7-20.) (&lt;b&gt;B&lt;/b&gt;) Lateral T1-weighted&lt;/div&gt;&lt;div&gt;MRI showing an acute rupture of the anterior cruciate ligament (&lt;i&gt;arrow&lt;/i&gt;). (Reproduced, with permission, from Doherty GM. &lt;i&gt;Current Diagnosis &amp;amp; Treatment: Surgery&lt;/i&gt;. 13th ed. New York, NY: McGraw-Hill; 2010:Fig. 40-30.)&lt;/div&gt;&lt;/div&gt;&lt;div&gt;&lt;br /&gt;&lt;/div&gt;&lt;div&gt;&lt;br /&gt;&lt;/div&gt;&lt;div&gt;&lt;b style=&quot;color: #017087;&quot;&gt;&lt;span style=&quot;font-size: medium;&quot;&gt;TREATMENT&lt;/span&gt;&lt;/b&gt;&lt;/div&gt;&lt;div&gt;&lt;br /&gt;&lt;/div&gt;&lt;div&gt;&lt;div&gt;&lt;i&gt;&lt;span style=&quot;font-size: medium;&quot;&gt;&lt;b&gt;Rationale for Treatment&lt;/b&gt;&lt;/span&gt;&lt;/i&gt;&lt;/div&gt;&lt;div&gt;&lt;br /&gt;&lt;/div&gt;&lt;div&gt;ACL-deficient patients complain of their affected knees frequently “giving out.” This recurrent instability prevents the majority of these patients from reaching their preinjury level of activity and places them at an increased risk for meniscal and chondral injuries. The development of arthritis in ACL-deficient knees compared with those that undergo reconstruction is controversial, as some authors have found that the patients &lt;i&gt;with&lt;/i&gt; ACL-reconstructed knees have more long-term degenerative changes.&lt;/div&gt;&lt;div&gt;&lt;br /&gt;&lt;/div&gt;&lt;div&gt;&lt;span&gt;&amp;nbsp;&amp;nbsp; &amp;nbsp;&lt;/span&gt;Factors influencing the treatment of ACL-deficient knees include patient age, functional demand, instability, expectations, and associated injuries. Many orthopaedic surgeons also prefer that patients regain knee range of motion before performing a reconstructive procedure. &lt;b&gt;In general, patients younger than 30 years of&amp;nbsp;&lt;/b&gt;&lt;b&gt;age should undergo surgical reconstruction, whereas those older than 30 years&amp;nbsp;&lt;/b&gt;&lt;b&gt;of age should undergo an initial period of rehabilitation followed by reevaluation.&amp;nbsp;&lt;/b&gt;In the latter setting, reconstruction is indicated in the presence of recurrent instability. Repair of concurrent meniscus tears, cartilage lesions, and/or associated ligamentous injuries often mandates ACL reconstruction, either at the same time or as a later staged procedure, to attain good outcomes. For example, meniscus repairs in ACL-deficient knees do not heal and therefore should be done in conjunction with ACL reconstruction. Additionally, posterolateral corner (PLC) and posteromedial corner (PMC) injuries that are undiagnosed or untreated in the setting of an ACL tear are common causes of ACL reconstruction failure.&lt;/div&gt;&lt;/div&gt;&lt;div&gt;&lt;br /&gt;&lt;/div&gt;&lt;div&gt;&lt;br /&gt;&lt;/div&gt;&lt;div&gt;&lt;div&gt;&lt;i&gt;&lt;span style=&quot;font-size: medium;&quot;&gt;&lt;b&gt;Nonoperative Treatment&lt;/b&gt;&lt;/span&gt;&lt;/i&gt;&lt;/div&gt;&lt;div&gt;&lt;br /&gt;&lt;/div&gt;&lt;div&gt;Nonoperative treatment of ACL injuries involves activity modification, bracing, and physical therapy. Activities that require cutting and jumping, such as basketball and soccer, should be replaced with straight-ahead activities, such as biking and running.&amp;nbsp;&lt;b&gt;Braces do not replace a native, intact ACL.&lt;/b&gt; However, there are specific ACL braces that can provide symptomatic comfort. Physical therapy focuses on hamstring strengthening and can have good functional outcomes.&lt;/div&gt;&lt;/div&gt;&lt;div&gt;&lt;br /&gt;&lt;/div&gt;&lt;div&gt;&lt;br /&gt;&lt;/div&gt;&lt;div&gt;&lt;div&gt;&lt;b&gt;&lt;i&gt;&lt;span style=&quot;font-size: medium;&quot;&gt;ACL Reconstruction&lt;/span&gt;&lt;/i&gt;&lt;/b&gt;&lt;/div&gt;&lt;div&gt;&lt;br /&gt;&lt;/div&gt;&lt;div&gt;Early efforts for the treatment of the ACL-deficient knee focused on primary repair of the ligament. However, mid- and long-term outcomes were poor, which led to the development of a variety of intra- and extraarticular autologous soft tissue and synthetic augmentation procedures. These treatments were also found to have poor long-term outcomes and increased complications. There are several theories regarding why primary repair was ineffective and pertain to the ACL’s location within the knee joint. As an intraarticular structure, synovial fluid prevents the formation of a fibrin blood clot that bridges the tear. The torn ends of the ligament subsequently fibrose, and end-to-end healing does not occur.&lt;/div&gt;&lt;/div&gt;&lt;div&gt;&lt;br /&gt;&lt;/div&gt;&lt;div&gt;&lt;div&gt;&lt;span&gt;&amp;nbsp;&amp;nbsp; &amp;nbsp;&lt;/span&gt;Because primary ACL repair is ineffective, current operative treatment involves arthroscopically assisted reconstruction in which bone tunnels are placed at the ACL’s tibial insertion and at the femoral origin. A soft tissue autograft or allograft is ultimately passed through these tunnels. &lt;b&gt;Classically, a bone-patellar tendon-bone&amp;nbsp;&lt;/b&gt;&lt;b&gt;autograft has been used for primary reconstructive procedures, with allograft&amp;nbsp;&lt;/b&gt;&lt;b&gt;reserved for older patients, multi-ligamentously injured knees, and revision&amp;nbsp;&lt;/b&gt;&lt;b&gt;cases.&lt;/b&gt; Although autografts are healthy, living tissue, they risk donor site morbidity. Conversely, allografts do not cause donor site morbidity but have variable tissue quality and carry the low risk of disease transmission (ie, hepatitis C and human immunodeficiency virus). More recently, quadrupled hamstring autografts have gained favor with the advancement of fixation techniques. This involves harvesting of the distal gracilis and semitendinosus tendons at their pes anserine insertion and doubling them over themselves to create 4 strands. This technique is inherently stronger than a bone-patellar tendon-bone graft, but its initial fixation to bone is not as strong. Other graft options include Achilles tendon and tibialis anterior tendon allograft. Some surgeons perform a double-bundle reconstruction, as there is thought that this more closely simulates normal knee kinematics. However, doublebundle reconstruction is a significantly more complex procedure, and more clinical data must be presented proving its superiority over the more classic, single-bundle reconstruction techniques before it is deemed the standard of care.&lt;/div&gt;&lt;/div&gt;&lt;div&gt;&lt;br /&gt;&lt;/div&gt;&lt;div&gt;&lt;br /&gt;&lt;/div&gt;&lt;div&gt;&lt;div&gt;&lt;i&gt;&lt;span style=&quot;font-size: medium;&quot;&gt;&lt;b&gt;Complications&lt;/b&gt;&lt;/span&gt;&lt;/i&gt;&lt;/div&gt;&lt;div&gt;&lt;br /&gt;&lt;/div&gt;&lt;div&gt;The goal of ACL reconstruction is restoration of normal knee kinematics. &lt;b&gt;However, one&amp;nbsp;&lt;/b&gt;&lt;b&gt;of the most common complications after ACL reconstruction is loss of motion—&amp;nbsp;&lt;/b&gt;&lt;b&gt;specifically, development of a flexion contracture.&lt;/b&gt; The causes of this include arthrofibrosis, cyclops lesions, improper graft tensioning and placement, and inadequate postoperative extension bracing. A &lt;b&gt;cyclops lesion&lt;/b&gt; is a hypertrophied remnant of the native ACL on the tibia, which results in the inability to attain full extension and may also cause anterior knee pain. These patients often present 4 to 6 months after reconstruction with an inability to achieve full extension. Arthroscopy is diagnostic and the lesion can be excised, which usually resolves the problem. Aberrant graft positioning may lead to loss of motion and early graft failure. A graft placed anteriorly to its normal axis of rotation will make it tight in flexion and thus limit it. A graft placed too posterior will limit terminal extension, as it becomes too tight in extension.&lt;/div&gt;&lt;div&gt;&lt;br /&gt;&lt;/div&gt;&lt;div&gt;&lt;span&gt;&amp;nbsp;&amp;nbsp; &amp;nbsp;&lt;/span&gt;Other complications of ACL reconstruction include infection, graft donor site morbidity, and the development of complex regional pain syndrome, formerly known as reflex sympathetic dystrophy.&lt;/div&gt;&lt;/div&gt;&lt;div&gt;&lt;br /&gt;&lt;/div&gt;&lt;div&gt;&lt;br /&gt;&lt;/div&gt;&lt;div&gt;&lt;div&gt;&lt;i&gt;&lt;span style=&quot;font-size: medium;&quot;&gt;&lt;b&gt;Postoperative Care and Rehabilitation&lt;/b&gt;&lt;/span&gt;&lt;/i&gt;&lt;/div&gt;&lt;div&gt;&lt;br /&gt;&lt;/div&gt;&lt;div&gt;The goal of postoperative rehabilitation is to reduce swelling, maintain patellar mobility to limit anterior knee pain, and regain full range of motion of the knee. Strengthening exercises revolve around the quadriceps, which atrophy quickly after surgery, and the hamstrings. Isometric hamstring and quadriceps contractions are appropriate and do not place excessive stress on the graft. Closed chain exercises (foot planted) should be emphasized. Open chain quadriceps strengthening must be avoided, in addition to isokinetic quadriceps strengthening during early rehabilitation. Joint motion promotes healing, and full passive extension should be allowed early in rehabilitation. Most patients are placed in a knee brace locked in extension postoperatively and are allowed to bear weight on it as tolerated, with crutches for comfort. In general, return to full sporting activities is not advised until at least 5 months after surgery. Pain and swelling must have subsided, full range of motion achieved, and quadriceps and hamstring strength close to that of the normal knee (80% and 90%, respectively). It is important to recognize that specific rehabilitation protocols vary from surgeon to surgeon and that these are only general guidelines.&lt;/div&gt;&lt;/div&gt;&lt;div&gt;&lt;br /&gt;&lt;/div&gt;&lt;div&gt;&lt;br /&gt;&lt;/div&gt;&lt;div&gt;&lt;b&gt;&lt;span style=&quot;color: #017087; font-size: medium;&quot;&gt;&lt;u&gt;COMPREHENSION QUESTIONS&lt;/u&gt;&lt;/span&gt;&lt;/b&gt;&lt;/div&gt;&lt;div&gt;&lt;br /&gt;&lt;/div&gt;&lt;div&gt;&lt;div&gt;22.1 A 22-year-old male basketball player presents with a swollen, painful left knee. He was on a fast break and as he began to jump for an alley-oop, he felt his knee give out and he collapsed to the floor. What is the most likely diagnosis?&lt;/div&gt;&lt;/div&gt;&lt;blockquote style=&quot;border: none; margin: 0px 0px 0px 40px; padding: 0px; text-align: left;&quot;&gt;&lt;div&gt;&lt;div&gt;A. Patello-femoral dislocation&lt;/div&gt;&lt;/div&gt;&lt;div&gt;&lt;div&gt;B. Medial collateral ligament injury&lt;/div&gt;&lt;/div&gt;&lt;div&gt;&lt;div&gt;C. Hamstring strain&lt;/div&gt;&lt;/div&gt;&lt;div&gt;&lt;div&gt;D. Anterior cruciate ligament injury&lt;/div&gt;&lt;/div&gt;&lt;div&gt;&lt;div&gt;E. Posterior cruciate ligament injury&lt;/div&gt;&lt;/div&gt;&lt;/blockquote&gt;&lt;div&gt;&lt;div&gt;&lt;br /&gt;&lt;/div&gt;&lt;div&gt;22.2 A 28-year-old female professional skier presents with an exquisitely tender, swollen right knee after a crash during a training run for the alpine downhill. She is unable to bear weight. Knee radiographs reveal an avulsion fracture off of the lateral tibial plateau, but are otherwise unremarkable. What is the most likely positive physical exam finding?&lt;/div&gt;&lt;/div&gt;&lt;blockquote style=&quot;border: none; margin: 0px 0px 0px 40px; padding: 0px; text-align: left;&quot;&gt;&lt;div&gt;&lt;div&gt;A. Posterior drawer&lt;/div&gt;&lt;/div&gt;&lt;div&gt;&lt;div&gt;B. Lachman test&lt;/div&gt;&lt;/div&gt;&lt;div&gt;&lt;div&gt;C. McMurray test&lt;/div&gt;&lt;/div&gt;&lt;div&gt;&lt;div&gt;D. Patellar apprehension&lt;/div&gt;&lt;/div&gt;&lt;div&gt;&lt;div&gt;E. Exam will be unremarkable&lt;/div&gt;&lt;/div&gt;&lt;/blockquote&gt;&lt;div&gt;&lt;div&gt;&lt;br /&gt;&lt;/div&gt;&lt;div&gt;22.3 Nonoperative rehabilitation for an ACL-deficient knee should include which of the following?&lt;/div&gt;&lt;/div&gt;&lt;blockquote style=&quot;border: none; margin: 0px 0px 0px 40px; padding: 0px; text-align: left;&quot;&gt;&lt;div&gt;&lt;div&gt;A. Quadriceps strengthening&lt;/div&gt;&lt;/div&gt;&lt;div&gt;&lt;div&gt;B. Straight leg raises&lt;/div&gt;&lt;/div&gt;&lt;div&gt;&lt;div&gt;C. Leg presses&lt;/div&gt;&lt;/div&gt;&lt;div&gt;&lt;div&gt;D. Short distance sprints&lt;/div&gt;&lt;/div&gt;&lt;div&gt;&lt;div&gt;E. Hamstring strengthening&lt;/div&gt;&lt;/div&gt;&lt;/blockquote&gt;&lt;div&gt;&lt;div&gt;&lt;br /&gt;&lt;/div&gt;&lt;div&gt;22.4 Where is the primary restraint to anterior translation of the tibia at mid-ranges of knee flexion?&lt;/div&gt;&lt;/div&gt;&lt;blockquote style=&quot;border: none; margin: 0px 0px 0px 40px; padding: 0px; text-align: left;&quot;&gt;&lt;div&gt;&lt;div&gt;A. Posteromedial bundle of the ACL&lt;/div&gt;&lt;/div&gt;&lt;div&gt;&lt;div&gt;B. Posterolateral bundle of the ACL&lt;/div&gt;&lt;/div&gt;&lt;div&gt;&lt;div&gt;C. Anteromedial bundle of the ACL&lt;/div&gt;&lt;/div&gt;&lt;div&gt;&lt;div&gt;D. Anterolateral bundle of the ACL&lt;/div&gt;&lt;/div&gt;&lt;div&gt;&lt;div&gt;E. Both bundles function identically throughout all knee ranges&lt;/div&gt;&lt;/div&gt;&lt;/blockquote&gt;&lt;div&gt;&lt;br /&gt;&lt;/div&gt;&lt;div&gt;&lt;br /&gt;&lt;/div&gt;&lt;div&gt;&lt;b&gt;&lt;span style=&quot;color: #017087; font-size: medium;&quot;&gt;&lt;u&gt;ANSWERS&lt;/u&gt;&lt;/span&gt;&lt;/b&gt;&lt;/div&gt;&lt;div&gt;&lt;br /&gt;&lt;/div&gt;&lt;div&gt;&lt;div&gt;22.1 &lt;b&gt;D.&lt;/b&gt; The history provided describes a noncontact injury to the knee during an acceleration-deceleration/pivoting move with the patient experiencing a feeling of giving way in his knee. The player quickly accumulates a knee effusion and has significant pain. These findings are characteristic descriptions of an ACL injury. The Lachman test is diagnostic for this injury, and an MRI would confirm the diagnosis.&lt;/div&gt;&lt;div&gt;&lt;br /&gt;&lt;/div&gt;&lt;div&gt;22.2 &lt;b&gt;B.&lt;/b&gt; Skiing, along with soccer, is strongly associated with ACL injury. The patient presents after a crash with an effusion and inability to bear weight. Knee radiographs demonstrate a lateral capsular avulsion fracture (Segond fracture). This is pathognomonic for an ACL injury. The Lachman test, performed by translating the tibia anteriorly with the knee in 30 degrees of flexion and assessing for a firm end point, is highly sensitive for an ACL injury.&lt;/div&gt;&lt;div&gt;&lt;br /&gt;&lt;/div&gt;&lt;div&gt;22.3 &lt;b&gt;E.&lt;/b&gt; When treating a patient with an ACL-deficient knee nonoperatively, rehabilitation should consist of aggressive hamstring strengthening.&lt;/div&gt;&lt;div&gt;&lt;br /&gt;&lt;/div&gt;&lt;div&gt;22.4 &lt;b&gt;C.&lt;/b&gt; The ACL is the primary restraint to anterior translation of the tibia with respect to the femur. It consists of 2 bundles, the anteromedial and posterolateral bundles. The anteromedial is the largest of the 2 bundles and functions as the primary restraint to anterior translation in the mid-ranges of knee flexion.&lt;/div&gt;&lt;/div&gt;&lt;div&gt;&lt;br /&gt;&lt;/div&gt;&lt;div&gt;&lt;b style=&quot;background-color: #4093a5;&quot;&gt;&amp;nbsp; &amp;nbsp;&lt;span style=&quot;font-size: medium;&quot;&gt;&amp;nbsp;&lt;/span&gt;&lt;span style=&quot;color: white;&quot;&gt;&lt;span style=&quot;font-size: medium;&quot;&gt;CLINICAL PEARLS&lt;/span&gt;&lt;span&gt;&amp;nbsp; &amp;nbsp;&amp;nbsp;&lt;/span&gt;&lt;/span&gt;&lt;/b&gt;&lt;/div&gt;&lt;div&gt;&lt;table border=&quot;1&quot; cellpadding=&quot;0&quot; cellspacing=&quot;0&quot; class=&quot;MsoTableGrid&quot; style=&quot;border-collapse: collapse; border: none; mso-border-alt: solid windowtext .5pt; mso-padding-alt: 0in 5.4pt 0in 5.4pt; mso-yfti-tbllook: 1184;&quot;&gt;
  297. &lt;tbody&gt;&lt;tr style=&quot;height: 19.15pt; mso-yfti-firstrow: yes; mso-yfti-irow: 0; mso-yfti-lastrow: yes;&quot;&gt;
  298.  &lt;td style=&quot;background: rgb(221, 237, 239); border-left: solid #017087 1.0pt; border: 1pt solid rgb(64, 147, 165); height: 19.15pt; padding: 0in 5.4pt; width: 433.7pt;&quot; valign=&quot;top&quot; width=&quot;578&quot;&gt;
  299.  &lt;div style=&quot;line-height: normal; margin-bottom: 0in;&quot;&gt;&lt;span style=&quot;color: #017087;&quot;&gt;&lt;br /&gt;&lt;/span&gt;&lt;/div&gt;&lt;div style=&quot;line-height: normal; margin-bottom: 0in;&quot;&gt;&lt;span style=&quot;color: #017087;&quot;&gt;►&lt;/span&gt;&amp;nbsp;The ACL originates from the medial wall of the lateral femoral condyle and travels obliquely, inserting on the tibia just anterior to and between the intercondylar eminences of the tibia.&lt;/div&gt;&lt;div style=&quot;line-height: normal; margin-bottom: 0in;&quot;&gt;&lt;span style=&quot;color: #017087;&quot;&gt;&lt;br /&gt;&lt;/span&gt;&lt;/div&gt;&lt;div style=&quot;line-height: normal; margin-bottom: 0in;&quot;&gt;&lt;span style=&quot;color: #017087;&quot;&gt;►&lt;/span&gt;&amp;nbsp;The primary role of the ACL is restraining anterior translation of the tibia with respect to the femur. It is a secondary restraint to tibial rotation.&lt;/div&gt;&lt;div style=&quot;line-height: normal; margin-bottom: 0in;&quot;&gt;&lt;span style=&quot;color: #017087;&quot;&gt;&lt;br /&gt;&lt;/span&gt;&lt;/div&gt;&lt;div style=&quot;line-height: normal; margin-bottom: 0in;&quot;&gt;&lt;span style=&quot;color: #017087;&quot;&gt;►&lt;/span&gt;&amp;nbsp;ACL tears are caused by noncontact pivoting injuries with the foot firmly planted on the ground. They are commonly seen in soccer players and skiers.&lt;/div&gt;&lt;div style=&quot;line-height: normal; margin-bottom: 0in;&quot;&gt;&lt;span style=&quot;color: #017087;&quot;&gt;&lt;br /&gt;&lt;/span&gt;&lt;/div&gt;&lt;div style=&quot;line-height: normal; margin-bottom: 0in;&quot;&gt;&lt;span style=&quot;color: #017087;&quot;&gt;►&lt;/span&gt;&amp;nbsp;Nonoperative treatment is typically reserved for older, sedentary patients and is based on a rehabilitation protocol encouraging aggressive hamstring strengthening, bracing, and activity modifications.&lt;/div&gt;&lt;div style=&quot;line-height: normal; margin-bottom: 0in;&quot;&gt;&lt;span style=&quot;color: #017087;&quot;&gt;&lt;br /&gt;&lt;/span&gt;&lt;/div&gt;&lt;div style=&quot;line-height: normal; margin-bottom: 0in;&quot;&gt;&lt;span style=&quot;color: #017087;&quot;&gt;►&lt;/span&gt;&amp;nbsp;The goal of ACL reconstruction is restoration of normal knee kinematics.&lt;/div&gt;&lt;div style=&quot;line-height: normal; margin-bottom: 0in;&quot;&gt;&lt;span style=&quot;color: #017087;&quot;&gt;&lt;br /&gt;&lt;/span&gt;&lt;/div&gt;&lt;div style=&quot;line-height: normal; margin-bottom: 0in;&quot;&gt;&lt;span style=&quot;color: #017087;&quot;&gt;►&lt;/span&gt;&amp;nbsp;ACL reconstruction is the treatment of choice for young, active patients.&lt;/div&gt;&lt;div style=&quot;line-height: normal; margin-bottom: 0in;&quot;&gt;&lt;br /&gt;&lt;/div&gt;
  300.  &lt;/td&gt;
  301. &lt;/tr&gt;
  302. &lt;/tbody&gt;&lt;/table&gt;&lt;br /&gt;&lt;/div&gt;
  303.  
  304. &lt;details close=&quot;&quot;&gt;
  305.  &lt;summary&gt;&lt;b&gt;&lt;span style=&quot;color: #00788b;&quot;&gt;REFERENCES&lt;/span&gt;&lt;/b&gt;&lt;/summary&gt;
  306.  &lt;p&gt;Arnoczky SP. Anatomy of the anterior cruciate ligament. Clin Orthop. 1983;172:19-25.&amp;nbsp;&lt;/p&gt;&lt;p&gt;Beynnon BD, Johnson RJ, Abate JA, et al. Treatment of anterior cruciate ligament injuries: Part 1. Am J
  307. Sports Med. 2005;33:1579-1602.&amp;nbsp;&lt;/p&gt;&lt;p&gt;Beynnon BD, Johnson RJ, Abate JA, et al. Treatment of anterior cruciate ligament injuries: Part 2. Am J
  308. Sports Med. 2005;33:1751-1767.&amp;nbsp;&lt;/p&gt;&lt;p&gt;Boden BP, Sheehan FT, Torg JS, Hewett, TE. Noncontact anterior cruciate ligament injuries: mechanisms
  309. and risk factors. J Am Acad Orthop Surg . 2010;18:520-527.&amp;nbsp;&lt;/p&gt;&lt;p&gt;Honkamp NJ, Fu FH, et al. Anterior cruciate ligament injuries. In: DeLee J et al, ed. DeLee &amp;amp; Drez’s
  310. Orthopaedic Sports Medicine. 3rd ed. Philadelphia: Saunders Elsevier; 2010:1644-1676.&lt;/p&gt;
  311. &lt;/details&gt;&lt;/div&gt;</content><link rel='replies' type='application/atom+xml' href='https://medical-phd.blogspot.com/feeds/3300321162947111263/comments/default' title='Post Comments'/><link rel='replies' type='text/html' href='https://medical-phd.blogspot.com/2022/03/anterior-cruciate-ligament-reconstruction-case-file.html#comment-form' title='0 Comments'/><link rel='edit' type='application/atom+xml' href='https://www.blogger.com/feeds/3302710210518459416/posts/default/3300321162947111263'/><link rel='self' type='application/atom+xml' href='https://www.blogger.com/feeds/3302710210518459416/posts/default/3300321162947111263'/><link rel='alternate' type='text/html' href='https://medical-phd.blogspot.com/2022/03/anterior-cruciate-ligament-reconstruction-case-file.html' title='Anterior Cruciate Ligament Reconstruction Case File'/><author><name>Medical Group</name><uri>http://www.blogger.com/profile/01780656800578397393</uri><email>noreply@blogger.com</email><gd:image rel='http://schemas.google.com/g/2005#thumbnail' width='32' height='32' src='//blogger.googleusercontent.com/img/b/R29vZ2xl/AVvXsEiMuROkBDv6yw4SmxGqRA5BPQhOXSwFld-Gufmy4kUqfpoAGQRl7cv-3naew6wjAnCjRFSe_FKX8z7CdE6cFKxxBpnqBuwUTSr-15xd3mFGcr_wJOX2y73G_zJCGND98Q/s220/Final-logo.jpg'/></author><media:thumbnail xmlns:media="http://search.yahoo.com/mrss/" url="https://blogger.googleusercontent.com/img/b/R29vZ2xl/AVvXsEjPQaZtedoDvmNKkkoP1QPCxMhysQx0siynBYKFZxtsoe55ZqEljikCSCIDT86lipo4VKsa8q7NeTrf7jCAiz9PEIlwtnEtPOcog5S6cHdzZtIPGpn8Erhsc_GvmYtfB3h75L2MJk5VjsDX6NxI1guatLvNFr4kQZFVpBv4g7ITFAKd8X0L_tX5taERrw/s72-w640-h434-c/right-knee-joint-with-the-joint-capsule.jpg" height="72" width="72"/><thr:total>0</thr:total></entry><entry><id>tag:blogger.com,1999:blog-3302710210518459416.post-483106974847837394</id><published>2022-03-24T17:02:00.006-07:00</published><updated>2022-03-24T17:02:48.991-07:00</updated><category scheme="http://www.blogger.com/atom/ns#" term="Case Files"/><category scheme="http://www.blogger.com/atom/ns#" term="Orthopaedic Surgery Case File"/><title type='text'>Rotator Cuff Injury Case File</title><content type='html'>&lt;div class=&quot;scrollbox&quot;&gt;
  312.  
  313. &lt;div style=&quot;text-align: left;&quot;&gt;&lt;b&gt;&lt;span style=&quot;font-size: large;&quot;&gt;Rotator Cuff Injury Case File&lt;/span&gt;&lt;/b&gt;&lt;/div&gt;&lt;div&gt;Eugene C. Toy, MD, Andrew J. Rosenbaum, MD, Timothy T. Roberts, MD, Joshua S. Dines, MD&lt;/div&gt;&lt;div&gt;&lt;br /&gt;&lt;/div&gt;&lt;div&gt;&lt;b&gt;&lt;span style=&quot;color: #017087; font-size: medium;&quot;&gt;CASE 21&lt;/span&gt;&lt;/b&gt;&lt;/div&gt;&lt;div&gt;&lt;div&gt;An otherwise healthy, right-hand-dominant, 25-year-old man is seen in the emergency department after slipping on the front steps of his house 2 days ago. He states that he landed on an outstretched right upper extremity and immediately began experiencing shoulder pain. He has been unable to move his shoulder since the fall and has had his arm in a sling that a friend gave him. On physical exam, you note no ecchymosis, abrasions, or any other deformity. However, shoulder range of motion, especially internal rotation, is significantly limited. Anteroposterior, axillary, and scapular-Y radiographs of his right shoulder reveal no abnormalities. The patient is most concerned about his ability to return to his competitive tennis league by next week, as he is playing for the club championship.&lt;/div&gt;&lt;div&gt;&lt;br /&gt;&lt;/div&gt;&lt;div&gt;&lt;span style=&quot;color: #017087;&quot;&gt;►&lt;/span&gt;&amp;nbsp;What is the most likely diagnosis?&lt;/div&gt;&lt;div&gt;&lt;span style=&quot;color: #017087;&quot;&gt;►&lt;/span&gt;&amp;nbsp;What are the imaging studies of choice for evaluating this patient’s problem?&lt;/div&gt;&lt;div&gt;&lt;span style=&quot;color: #017087;&quot;&gt;►&lt;/span&gt;&amp;nbsp;What is the most appropriate treatment for this patient?&lt;/div&gt;&lt;/div&gt;&lt;div&gt;&lt;br /&gt;&lt;/div&gt;&lt;div&gt;&lt;br /&gt;&lt;/div&gt;&lt;div&gt;&lt;div&gt;&lt;b&gt;&lt;span style=&quot;font-size: medium;&quot;&gt;ANSWER TO CASE 21:&lt;/span&gt;&lt;/b&gt;&lt;/div&gt;&lt;div&gt;&lt;span style=&quot;background-color: #daebf0; font-size: medium;&quot;&gt;&lt;span&gt;&lt;span style=&quot;background-color: transparent;&quot;&gt;&lt;span style=&quot;color: #017087;&quot;&gt;&lt;b&gt;Rotator Cuff Injury&lt;/b&gt;&lt;/span&gt;&lt;/span&gt;&lt;b&gt;&amp;nbsp;&amp;nbsp;&lt;/b&gt;&lt;/span&gt;&lt;span style=&quot;font-weight: bold;&quot;&gt;&amp;nbsp;&amp;nbsp; &amp;nbsp;&lt;/span&gt;&lt;span style=&quot;font-weight: bold;&quot;&gt;&amp;nbsp;&amp;nbsp; &amp;nbsp;&lt;/span&gt;&lt;span style=&quot;font-weight: bold;&quot;&gt;&amp;nbsp;&amp;nbsp; &amp;nbsp;&lt;/span&gt;&lt;span style=&quot;font-weight: bold;&quot;&gt;&amp;nbsp;&amp;nbsp; &amp;nbsp;&lt;/span&gt;&lt;span style=&quot;font-weight: bold;&quot;&gt;&amp;nbsp;&amp;nbsp; &amp;nbsp;&lt;/span&gt;&lt;span style=&quot;font-weight: bold;&quot;&gt;&amp;nbsp;&amp;nbsp; &amp;nbsp;&lt;/span&gt;&lt;span style=&quot;font-weight: bold;&quot;&gt;&amp;nbsp; &amp;nbsp;&amp;nbsp;&lt;/span&gt;&lt;/span&gt;&lt;/div&gt;&lt;/div&gt;&lt;div&gt;&lt;br /&gt;&lt;/div&gt;&lt;div&gt;&lt;div&gt;&lt;i&gt;Summary&lt;/i&gt;: An otherwise healthy and active 25-year-old man slips and lands on his right shoulder. He is unable to raise his arm and has limited active range of motion. Neurovascular examination is normal. Radiographs are negative for a fracture and dislocation.&lt;/div&gt;&lt;div&gt;&lt;ul style=&quot;text-align: left;&quot;&gt;&lt;li&gt;&lt;b&gt;Most likely diagnosis:&lt;/b&gt; Acute tear of the rotator cuff.&lt;/li&gt;&lt;li&gt;&lt;b&gt;Imaging studies of choice:&lt;/b&gt; Plain radiographs must always be obtained first in the setting of acute trauma to the shoulder, as has been done here. When rotator cuff tear (RCT) is suspected, magnetic resonance imaging (MRI) should also be performed. It is extremely accurate (93%-100%) in detecting full-thickness tears and can evaluate tear size, tendon retraction, muscle atrophy, and related intraarticular pathology. However, it cannot be used in patients with pacemakers, aneurysm clips, metal in the eye, or other metal implants within the body.&lt;/li&gt;&lt;li&gt;&lt;b&gt;Treatment:&lt;/b&gt; The appropriate treatment for an acute, full-thickness RCT in an otherwise healthy and physically active young patient is primary repair.&lt;/li&gt;&lt;/ul&gt;&lt;/div&gt;&lt;/div&gt;&lt;div&gt;&lt;br /&gt;&lt;/div&gt;&lt;div&gt;&lt;br /&gt;&lt;/div&gt;&lt;div&gt;&lt;div&gt;&lt;b&gt;&lt;span style=&quot;color: #017087; font-size: medium;&quot;&gt;ANALYSIS&lt;/span&gt;&lt;/b&gt;&lt;/div&gt;&lt;div&gt;&lt;b&gt;&lt;span style=&quot;font-size: medium;&quot;&gt;Objectives&lt;/span&gt;&lt;/b&gt;&lt;/div&gt;&lt;/div&gt;&lt;div&gt;&lt;div&gt;&lt;ol style=&quot;text-align: left;&quot;&gt;&lt;li&gt;Understand rotator cuff anatomy.&lt;/li&gt;&lt;li&gt;Properly diagnose a rotator cuff tear.&lt;/li&gt;&lt;li&gt;Be familiar with treatment options for rotator cuff tears.&lt;/li&gt;&lt;/ol&gt;&lt;/div&gt;&lt;/div&gt;&lt;div&gt;&lt;br /&gt;&lt;/div&gt;&lt;div&gt;&lt;br /&gt;&lt;/div&gt;&lt;div&gt;&lt;div&gt;&lt;i&gt;&lt;span style=&quot;font-size: medium;&quot;&gt;Considerations&lt;/span&gt;&lt;/i&gt;&lt;/div&gt;&lt;div&gt;&lt;br /&gt;&lt;/div&gt;&lt;div&gt;This 45-year-old man sustained an injury to his right shoulder after slipping on his front steps. He presents with pain and weakness of the right shoulder in the setting of normal radiographs. This constellation of findings is concerning for an acute RCT.&lt;/div&gt;&lt;div&gt;&lt;br /&gt;&lt;/div&gt;&lt;div&gt;&lt;span&gt;&amp;nbsp;&amp;nbsp; &amp;nbsp;&lt;/span&gt;A complete history and physical exam of the neck and involved extremity must be done. Although active range of motion is typically decreased because of pain and/or weakness, passive range of motion is expected to be normal. Shoulder strength in elevation, abduction, external rotation, and internal rotation should be assessed, and specific provocative maneuvers can be performed to evaluate the individual muscles of the rotator cuff ( Table 21–1 , Figure 21–1 ).&lt;/div&gt;&lt;div&gt;&lt;br /&gt;&lt;/div&gt;&lt;div&gt;&lt;span&gt;&amp;nbsp;&amp;nbsp; &amp;nbsp;&lt;/span&gt;An MRI, which is the gold standard for diagnosis of RCTs, should be obtained (Figure 21–2). However, if a contraindication to MRI exists, an arthrogram or ultrasound can be done. MRI in this patient will confirm an RCT. Given the patient’s described mechanism of a hyperabduction/external rotation injury during a fall, he has likely torn his subscapularis tendon.&lt;/div&gt;&lt;div&gt;&lt;br /&gt;&lt;/div&gt;&lt;div&gt;&lt;span&gt;&amp;nbsp;&amp;nbsp; &amp;nbsp;&lt;/span&gt;Although nonoperative management is appropriate for some people, this patient will benefit from a rotator cuff repair, as he is young and active.&lt;/div&gt;&lt;/div&gt;&lt;div&gt;&lt;br /&gt;&lt;/div&gt;&lt;div class=&quot;separator&quot; style=&quot;clear: both; text-align: center;&quot;&gt;&lt;a href=&quot;https://blogger.googleusercontent.com/img/b/R29vZ2xl/AVvXsEhJMD9EDdnNS7c_BthHZb4OrKduqx_kNw8426tXdXHs5lnTRcOARAxdJGLG3ZyZsIGC4dZfJso5QM3jIYEY0Pip7jWbyDCo8LiAzLbZp11TF-hNjQoEkX8ELBWMvE3SqNs5M4t-ojkk-4saEirPkcZSlqt8F1UZ7ZA_nnNPb7ZRHUSkDKx0924vpyVcDg/s771/evaluate-specific-muscles-of-the-rotator-cuff.jpg&quot; style=&quot;margin-left: 1em; margin-right: 1em;&quot;&gt;&lt;img alt=&quot;evaluate specific muscles of the rotator cuff&quot; border=&quot;0&quot; data-original-height=&quot;625&quot; data-original-width=&quot;771&quot; height=&quot;518&quot; src=&quot;https://blogger.googleusercontent.com/img/b/R29vZ2xl/AVvXsEhJMD9EDdnNS7c_BthHZb4OrKduqx_kNw8426tXdXHs5lnTRcOARAxdJGLG3ZyZsIGC4dZfJso5QM3jIYEY0Pip7jWbyDCo8LiAzLbZp11TF-hNjQoEkX8ELBWMvE3SqNs5M4t-ojkk-4saEirPkcZSlqt8F1UZ7ZA_nnNPb7ZRHUSkDKx0924vpyVcDg/w640-h518/evaluate-specific-muscles-of-the-rotator-cuff.jpg&quot; title=&quot;evaluate specific muscles of the rotator cuff&quot; width=&quot;640&quot; /&gt;&lt;/a&gt;&lt;/div&gt;&lt;div&gt;&lt;br /&gt;&lt;/div&gt;&lt;div class=&quot;separator&quot; style=&quot;clear: both; text-align: center;&quot;&gt;&lt;a href=&quot;https://blogger.googleusercontent.com/img/b/R29vZ2xl/AVvXsEhsdjWqjCesP4E-W_nWP9m6BFhoX0hWnet90h4nHPi6Zf9wlmb2Hws-YkZo34jXAviRB7JGKH_2vnd7WfDViMndwZ5aHkz1gtoc_24CzSFYHqj4h2mSZ4pzzvIM91ZRhcBj2hNzSDHn6GWYe6HROAzBsKzP3HbQQCz5YNG3Zv3LNeqxUNsUvaOA5K_Slg/s500/supraspinatus-pathology.jpg&quot; style=&quot;margin-left: 1em; margin-right: 1em;&quot;&gt;&lt;img alt=&quot;supraspinatus pathology&quot; border=&quot;0&quot; data-original-height=&quot;441&quot; data-original-width=&quot;500&quot; height=&quot;353&quot; src=&quot;https://blogger.googleusercontent.com/img/b/R29vZ2xl/AVvXsEhsdjWqjCesP4E-W_nWP9m6BFhoX0hWnet90h4nHPi6Zf9wlmb2Hws-YkZo34jXAviRB7JGKH_2vnd7WfDViMndwZ5aHkz1gtoc_24CzSFYHqj4h2mSZ4pzzvIM91ZRhcBj2hNzSDHn6GWYe6HROAzBsKzP3HbQQCz5YNG3Zv3LNeqxUNsUvaOA5K_Slg/w400-h353/supraspinatus-pathology.jpg&quot; title=&quot;supraspinatus pathology&quot; width=&quot;400&quot; /&gt;&lt;/a&gt;&lt;/div&gt;&lt;div&gt;&lt;div&gt;&lt;b&gt;&lt;span style=&quot;color: #017087;&quot;&gt;Figure 21–1.&lt;/span&gt;&lt;/b&gt; The empty can test, which evaluates supraspinatus pathology. (Reproduced, with permission, from Tintinalli J, et al. &lt;i&gt;Tintinalli’s Emergency Medicine: A Comprehensive Study Guide&lt;/i&gt;. 7th ed. New York, NY: McGraw-Hill; 2010:Fig. 277-5.)&lt;/div&gt;&lt;/div&gt;&lt;div&gt;&lt;br /&gt;&lt;/div&gt;&lt;div class=&quot;separator&quot; style=&quot;clear: both; text-align: center;&quot;&gt;&lt;a href=&quot;https://blogger.googleusercontent.com/img/b/R29vZ2xl/AVvXsEiZbmfMNjO3TstVxLvE60-4wBzOEWMVtElTD2jVkM4gRKnWhR1DcqXkmtdnJzQN_tJuSODrqXOpr7LXAf5AD_fuIgcnJVzkgic0o8w_DPSvPQ2iTmzUa857ho3WjWD2rZK9UB8o5SJSAQV5OEyYSn2ZXQ5khco82qvSP3_2yigxcaftdxwgre_poyzxJg/s499/MRI-coronal-image-of-the-shoulder.jpg&quot; style=&quot;margin-left: 1em; margin-right: 1em;&quot;&gt;&lt;img alt=&quot;MRI coronal image of the shoulder&quot; border=&quot;0&quot; data-original-height=&quot;423&quot; data-original-width=&quot;499&quot; height=&quot;339&quot; src=&quot;https://blogger.googleusercontent.com/img/b/R29vZ2xl/AVvXsEiZbmfMNjO3TstVxLvE60-4wBzOEWMVtElTD2jVkM4gRKnWhR1DcqXkmtdnJzQN_tJuSODrqXOpr7LXAf5AD_fuIgcnJVzkgic0o8w_DPSvPQ2iTmzUa857ho3WjWD2rZK9UB8o5SJSAQV5OEyYSn2ZXQ5khco82qvSP3_2yigxcaftdxwgre_poyzxJg/w400-h339/MRI-coronal-image-of-the-shoulder.jpg&quot; title=&quot;MRI coronal image of the shoulder&quot; width=&quot;400&quot; /&gt;&lt;/a&gt;&lt;/div&gt;&lt;div&gt;&lt;div&gt;&lt;b&gt;&lt;span style=&quot;color: #017087;&quot;&gt;Figure 21–2.&lt;/span&gt;&lt;/b&gt;&amp;nbsp;Rotator cuff tear. MRI coronal image of the shoulder reveals a tear in the supraspinatus&lt;/div&gt;&lt;div&gt;tendon (&lt;i&gt;arrow&lt;/i&gt;) with edema (&lt;i&gt;arrowhead&lt;/i&gt;). (Reproduced, with permission, from Tintinalli J, et al. &lt;i&gt;Tintinalli’s Emergency Medicine: A Comprehensive Study Guide&lt;/i&gt;. 7th ed. New York, NY: McGraw-Hill; 2010:Fig. 277-7.)&lt;/div&gt;&lt;/div&gt;&lt;div&gt;&lt;br /&gt;&lt;/div&gt;&lt;div&gt;&lt;br /&gt;&lt;/div&gt;&lt;div&gt;&lt;div&gt;&lt;div&gt;&lt;b&gt;&lt;span style=&quot;font-size: medium;&quot;&gt;APPROACH TO:&lt;/span&gt;&lt;/b&gt;&lt;/div&gt;&lt;div&gt;&lt;span style=&quot;background-color: #daebf0; color: #017087; font-size: medium;&quot;&gt;&lt;span&gt;&lt;span style=&quot;background-color: transparent;&quot;&gt;&lt;b&gt;Rotator Cuff Tears&lt;/b&gt;&lt;/span&gt;&lt;b&gt;&amp;nbsp;&lt;/b&gt;&lt;/span&gt;&lt;span style=&quot;font-weight: bold;&quot;&gt;&amp;nbsp;&amp;nbsp; &amp;nbsp;&lt;/span&gt;&lt;span style=&quot;font-weight: bold;&quot;&gt;&amp;nbsp;&amp;nbsp; &amp;nbsp;&lt;/span&gt;&lt;span style=&quot;font-weight: bold;&quot;&gt;&amp;nbsp;&amp;nbsp; &amp;nbsp;&lt;/span&gt;&lt;span style=&quot;font-weight: bold;&quot;&gt;&amp;nbsp;&amp;nbsp; &amp;nbsp;&lt;/span&gt;&lt;span style=&quot;font-weight: bold;&quot;&gt;&amp;nbsp;&amp;nbsp; &amp;nbsp;&lt;/span&gt;&lt;span style=&quot;font-weight: bold;&quot;&gt;&amp;nbsp;&amp;nbsp; &amp;nbsp;&lt;/span&gt;&lt;span style=&quot;font-weight: bold;&quot;&gt;&amp;nbsp;&amp;nbsp; &amp;nbsp;&lt;/span&gt;&lt;span style=&quot;font-weight: bold;&quot;&gt;&amp;nbsp;&amp;nbsp; &lt;/span&gt;&lt;span style=&quot;font-weight: bold;&quot;&gt;&amp;nbsp;&lt;/span&gt;&lt;/span&gt;&lt;/div&gt;&lt;/div&gt;&lt;div&gt;&lt;br /&gt;&lt;/div&gt;&lt;div&gt;&lt;b&gt;&lt;span style=&quot;color: #017087; font-size: medium;&quot;&gt;DEFINITIONS&lt;/span&gt;&lt;/b&gt;&lt;/div&gt;&lt;/div&gt;&lt;div&gt;&lt;br /&gt;&lt;/div&gt;&lt;div&gt;&lt;div&gt;&lt;b&gt;ROTATOR CUFF:&lt;/b&gt; A confluence of 4 muscles (supraspinatus, infraspinatus, teres minor, and subscapularis) that arise from the scapula and insert on the humeral head. The main functions of the rotator cuff are to stabilize the glenohumeral joint and to rotate the humerus outward.&lt;/div&gt;&lt;div&gt;&lt;b&gt;&lt;br /&gt;&lt;/b&gt;&lt;/div&gt;&lt;div&gt;&lt;b&gt;ROTATOR CUFF TEAR:&lt;/b&gt; A common cause of shoulder pain that can involve an individual tendon or a combination of tendons. The supraspinatus is the most commonly torn rotator cuff tendon.&lt;/div&gt;&lt;div&gt;&lt;b&gt;&lt;br /&gt;&lt;/b&gt;&lt;/div&gt;&lt;div&gt;&lt;b&gt;ROTATOR CUFF ARTHROPATHY:&lt;/b&gt; A condition of the shoulder in which glenohumeral arthritis develops secondary to a chronic, irreparable rotator cuff tear. It is typically seen in older individuals with loss of shoulder motion and strength.&lt;/div&gt;&lt;div&gt;&lt;b&gt;&lt;br /&gt;&lt;/b&gt;&lt;/div&gt;&lt;div&gt;&lt;b&gt;SUBACROMIAL IMPINGEMENT:&lt;/b&gt; Term used to describe the process and continuum by which rotator cuff tendons become irritated and inflamed as they pass through the subacromial space. This is secondary to impingement of the humeral head and rotator cuff beneath the coracoacromial (CA) arch of the shoulder. With the arm in neutral position, the greater tuberosity (where the supraspinatus inserts) lies anterior to the CA arch. With forward flexion and internal rotation, the subacromial bursa and supraspinatus tendon become entrapped between the anterior acromion/coracoid and greater tuberosity.&lt;/div&gt;&lt;/div&gt;&lt;div&gt;&lt;br /&gt;&lt;/div&gt;&lt;div class=&quot;separator&quot; style=&quot;clear: both; text-align: center;&quot;&gt;&lt;a href=&quot;https://blogger.googleusercontent.com/img/b/R29vZ2xl/AVvXsEjM79UAxYNPqiA0Xh8CCVuUthdKl6uVLMgQmF6Fem89A0_u01Fxqe9MoGwvnqyM4P0SLIL08uaaIC4RKjXl04lGbOsLcONGBAmsF79D_92xyHDvmjEhWQxFUVv7TdlCWVerAVxU5cfOa9zr8dzcIjXEpfAY9I6J-t4_n6iPAVkNMDWnFxDbkQpbDLTlKg/s553/rotator-cuff-muscles.jpg&quot; style=&quot;margin-left: 1em; margin-right: 1em;&quot;&gt;&lt;img alt=&quot;rotator cuff muscles&quot; border=&quot;0&quot; data-original-height=&quot;441&quot; data-original-width=&quot;553&quot; height=&quot;319&quot; src=&quot;https://blogger.googleusercontent.com/img/b/R29vZ2xl/AVvXsEjM79UAxYNPqiA0Xh8CCVuUthdKl6uVLMgQmF6Fem89A0_u01Fxqe9MoGwvnqyM4P0SLIL08uaaIC4RKjXl04lGbOsLcONGBAmsF79D_92xyHDvmjEhWQxFUVv7TdlCWVerAVxU5cfOa9zr8dzcIjXEpfAY9I6J-t4_n6iPAVkNMDWnFxDbkQpbDLTlKg/w400-h319/rotator-cuff-muscles.jpg&quot; title=&quot;rotator cuff muscles&quot; width=&quot;400&quot; /&gt;&lt;/a&gt;&lt;/div&gt;&lt;div&gt;&lt;div&gt;&lt;b&gt;&lt;span style=&quot;color: #017087;&quot;&gt;Figure 21–3.&lt;/span&gt;&lt;/b&gt; Posterior view of the shoulder illustrating rotator cuff muscles. (Reproduced, with permission, from Tintinalli J, et al. &lt;i&gt;Tintinalli’s Emergency Medicine: A Comprehensive Study Guide&lt;/i&gt;. 7th ed. New York, NY: McGraw-Hill; 2010:Fig. 277-1.)&lt;/div&gt;&lt;/div&gt;&lt;div&gt;&lt;br /&gt;&lt;/div&gt;&lt;div&gt;&lt;br /&gt;&lt;/div&gt;&lt;div&gt;&lt;b&gt;&lt;span style=&quot;color: #017087; font-size: medium;&quot;&gt;CLINICAL APPROACH&lt;/span&gt;&lt;/b&gt;&lt;/div&gt;&lt;div&gt;&lt;br /&gt;&lt;/div&gt;&lt;div&gt;&lt;div&gt;&lt;i&gt;&lt;span style=&quot;font-size: medium;&quot;&gt;Shoulder Anatomy and Biomechanics&lt;/span&gt;&lt;/i&gt;&lt;/div&gt;&lt;div&gt;&lt;br /&gt;&lt;/div&gt;&lt;div&gt;The shoulder complex is comprised of the glenohumeral (GH), sternoclavicular (SC), acromioclavicular (AC), and scapulothoracic (ST) joints ( Figures 21–3 and 21–4 ).&amp;nbsp;&lt;b&gt;There is a 2:1 ratio of shoulder motion between the GH and ST joints (ie, 180&amp;nbsp;&lt;/b&gt;&lt;b&gt;degrees of abduction consists of 120 degrees of GH motion and 60 degrees of ST&amp;nbsp;&lt;/b&gt;&lt;b&gt;motion).&lt;/b&gt; Both static stabilizers (ie, bony structures, labrum, joint capsule, ligaments) and dynamic stabilizers (ie, rotator cuff, periscapular muscles) help in maintaining congruity and providing GH joint stability. Static stabilizers can function in the setting of intrinsic muscle damage and neuromuscular injury, whereas the dynamic stabilizers cannot. Table 21–2 outlines the muscles vital to shoulder function.&lt;/div&gt;&lt;/div&gt;&lt;div&gt;&lt;br /&gt;&lt;/div&gt;&lt;div class=&quot;separator&quot; style=&quot;clear: both; text-align: center;&quot;&gt;&lt;a href=&quot;https://blogger.googleusercontent.com/img/b/R29vZ2xl/AVvXsEhEKd0N23mSAFwGi0JrQRh7QmEukN3YQX8hLRQKmM8dqnCC7DXsBjAZFDPBUEjBLqI6njfPESEiC_3UScVTkWn2WIARIr0Sg6IkLfnjn_QzZ37970iROkm1lBugHsuPJyWr-NuVsH9VOEAajgyp-E7ffCiskHZqsvs_JHmPpnn45HenJ4lPuqFv5zWUcg/s546/supraspinatus-muscle.jpg&quot; style=&quot;margin-left: 1em; margin-right: 1em;&quot;&gt;&lt;img alt=&quot;supraspinatus muscle&quot; border=&quot;0&quot; data-original-height=&quot;362&quot; data-original-width=&quot;546&quot; height=&quot;265&quot; src=&quot;https://blogger.googleusercontent.com/img/b/R29vZ2xl/AVvXsEhEKd0N23mSAFwGi0JrQRh7QmEukN3YQX8hLRQKmM8dqnCC7DXsBjAZFDPBUEjBLqI6njfPESEiC_3UScVTkWn2WIARIr0Sg6IkLfnjn_QzZ37970iROkm1lBugHsuPJyWr-NuVsH9VOEAajgyp-E7ffCiskHZqsvs_JHmPpnn45HenJ4lPuqFv5zWUcg/w400-h265/supraspinatus-muscle.jpg&quot; title=&quot;supraspinatus muscle&quot; width=&quot;400&quot; /&gt;&lt;/a&gt;&lt;/div&gt;&lt;div&gt;&lt;div&gt;&lt;b&gt;&lt;span style=&quot;color: #017087;&quot;&gt;Figure 21–4.&lt;/span&gt;&lt;/b&gt; Anterior view of the shoulder illustrating the supraspinatus muscle and the long head&lt;/div&gt;&lt;div&gt;of the biceps. (Reproduced, with permission, from Tintinalli J, et al. &lt;i&gt;Tintinalli’s Emergency Medicine: A Comprehensive Study Guide&lt;/i&gt;. 7th ed. New York, NY: McGraw-Hill; 2010:Fig. 277-2.)&lt;/div&gt;&lt;/div&gt;&lt;div&gt;&lt;br /&gt;&lt;/div&gt;&lt;div&gt;&lt;br /&gt;&lt;/div&gt;&lt;div class=&quot;separator&quot; style=&quot;clear: both; text-align: center;&quot;&gt;&lt;a href=&quot;https://blogger.googleusercontent.com/img/b/R29vZ2xl/AVvXsEhH86jphvgr0u7Irpn2_zOIb7Qj8BX7vvZnNJ8OoFuELacsVVHthp8uFVBNIFpTKTjIwK2rvobES0cIEHbKyi_sZTGyXqS9LgsXXOORl_l5OHU_6YWJ2VydDC5-buBRto1Ss2_KJTv_t0g-4MV45F814BCu5FtDo6ngMdeUJ3_2CIJzOBcckf1Vzq8YpQ/s841/rotator-cuff-and-glenohumeral-joint.jpg&quot; style=&quot;margin-left: 1em; margin-right: 1em;&quot;&gt;&lt;img alt=&quot;rotator cuff and glenohumeral joint&quot; border=&quot;0&quot; data-original-height=&quot;841&quot; data-original-width=&quot;772&quot; height=&quot;640&quot; src=&quot;https://blogger.googleusercontent.com/img/b/R29vZ2xl/AVvXsEhH86jphvgr0u7Irpn2_zOIb7Qj8BX7vvZnNJ8OoFuELacsVVHthp8uFVBNIFpTKTjIwK2rvobES0cIEHbKyi_sZTGyXqS9LgsXXOORl_l5OHU_6YWJ2VydDC5-buBRto1Ss2_KJTv_t0g-4MV45F814BCu5FtDo6ngMdeUJ3_2CIJzOBcckf1Vzq8YpQ/w588-h640/rotator-cuff-and-glenohumeral-joint.jpg&quot; title=&quot;rotator cuff and glenohumeral joint&quot; width=&quot;588&quot; /&gt;&lt;/a&gt;&lt;/div&gt;&lt;div&gt;&lt;br /&gt;&lt;/div&gt;&lt;div&gt;&lt;div&gt;&lt;b&gt;&lt;span&gt;&amp;nbsp;&amp;nbsp; &amp;nbsp;&lt;/span&gt;The rotator cuff helps to maintain a stable fulcrum for shoulder motion via its&amp;nbsp;&lt;/b&gt;&lt;b&gt;role as a dynamic stabilizer of the glenohumeral joint.&lt;/b&gt; The cuff balances the force couples in the coronal and transverse planes. In the coronal plane, the inferior rotator cuff (infraspinatus, teres minor, subscapularis) balances the support moment created by the deltoid. In the transverse plane, the anterior cuff (subscapularis) functions to balance the moment created by the posterior cuff (infraspinatus and teres minor).&lt;/div&gt;&lt;/div&gt;&lt;div&gt;&lt;br /&gt;&lt;/div&gt;&lt;div&gt;&lt;br /&gt;&lt;/div&gt;&lt;div&gt;&lt;div&gt;&lt;i&gt;&lt;span style=&quot;font-size: medium;&quot;&gt;Etiology&lt;/span&gt;&lt;/i&gt;&lt;/div&gt;&lt;div&gt;&lt;br /&gt;&lt;/div&gt;&lt;div&gt;The incidence of RCTs ranges from 5% to 40% and increases with age. Of note, 55% of asymptomatic patients ≥ 60 years of age will have rotator cuff pathology evident on MRI. &lt;b&gt;RCTs should be thought of as part of a continuum of disease&amp;nbsp;&lt;/b&gt;&lt;b&gt;involving the shoulder.&lt;/b&gt; The continuum includes subacromial impingement, subcoracoid impingement, calcific tendonitis, rotator cuff tears, and rotator cuff arthropathy.&lt;/div&gt;&lt;/div&gt;&lt;div&gt;&lt;br /&gt;&lt;/div&gt;&lt;div&gt;&lt;div&gt;&lt;span&gt;&amp;nbsp;&amp;nbsp; &amp;nbsp;&lt;/span&gt;Tears can be acute, iatrogenic, or secondary to chronic degenerative changes. Acute avulsion injuries include subscapularis tears in younger patients after a fall and supraspinatus, infraspinatus, and/or teres minor tears in those older than 40 years after a shoulder dislocation. Overhead throwing athletes are susceptible to cuff tears secondary to the high tensile forces that the rotator cuff is subjected to during the deceleration phase of throwing. Iatrogenic injuries are commonly seen after repair failure of the subscapularis tendon after an open anterior shoulder surgery. Chronic degenerative tears are seen in older patients and typically involve the supraspinatus, infraspinatus, and/or teres minor.&lt;/div&gt;&lt;/div&gt;&lt;div&gt;&lt;br /&gt;&lt;/div&gt;&lt;div&gt;&lt;br /&gt;&lt;/div&gt;&lt;div&gt;&lt;div&gt;&lt;i&gt;&lt;span style=&quot;font-size: medium;&quot;&gt;Presentation and Diagnosis&lt;/span&gt;&lt;/i&gt;&lt;/div&gt;&lt;div&gt;&lt;br /&gt;&lt;/div&gt;&lt;div&gt;Patients with RCT present either after an acute trauma and inability to move the arm and shoulder (ie, shoulder dislocation or fall) or with the insidious onset of symptoms such as &lt;b&gt;night pain&lt;/b&gt; and &lt;b&gt;difficulty performing overhead activites.&amp;nbsp;&lt;/b&gt;Physical examination of the shoulder should begin with inspection of the skin for scars, atrophy, swelling, droop, and scapular winging. Palpation of the bony prominences and muscles of the shoulder girdle should be done next. The patient should then be placed supine, at which time active and passive range of motion of both shoulders is assessed. Planes of motion to be evaluated include forward elevation (150-180 degrees is considered normal), abduction, internal rotation to vertebral height (T4-8 is considered normal), internal rotation at 90 degrees of abduction, external rotation at side, and external rotation at 90 degrees of abduction. A neurovascular exam of the entire upper extremity should also be done, as should the specific tests that isolate the individual muscles of the rotator cuff ( Table 21–1 ).&lt;/div&gt;&lt;/div&gt;&lt;div&gt;&lt;br /&gt;&lt;/div&gt;&lt;div&gt;&lt;div&gt;&lt;span&gt;&amp;nbsp;&amp;nbsp; &amp;nbsp;&lt;/span&gt;Imaging studies are ordered based on the findings from the history and physical exam, with plain radiographs the first ones obtained. &lt;b&gt;Findings on plain x-ray&amp;nbsp;&lt;/b&gt;&lt;b&gt;associated with rotator cuff pathology include calcific tendonitis, proximal&amp;nbsp;&lt;/b&gt;&lt;b&gt;migration of the humerus (seen with chronic RCT), or a hooked acromion.&amp;nbsp;&lt;/b&gt;&lt;b&gt;MRI, when obtained, evaluates muscle quality and the tear size, tear shape,&amp;nbsp;&lt;/b&gt;&lt;b&gt;and degree of tendon retraction. Other findings on MRI consistent with RCT&amp;nbsp;&lt;/b&gt;&lt;b&gt;include muscle atrophy, medial biceps tendon subluxation (indicative of a subscapularis&amp;nbsp;&lt;/b&gt;&lt;b&gt;tear), and cysts in the humeral head (seen in the majority of patients&amp;nbsp;&lt;/b&gt;&lt;b&gt;with a chronic RCT).&lt;/b&gt;&lt;/div&gt;&lt;/div&gt;&lt;div&gt;&lt;br /&gt;&lt;/div&gt;&lt;div&gt;&lt;br /&gt;&lt;/div&gt;&lt;div&gt;&lt;div&gt;&lt;i&gt;&lt;span style=&quot;font-size: medium;&quot;&gt;Classification&lt;/span&gt;&lt;/i&gt;&lt;/div&gt;&lt;div&gt;&lt;br /&gt;&lt;/div&gt;&lt;div&gt;Classification of cuff tears can be done based on anatomical location, tear size, amount of cuff atrophy, tear shape, thickness, and chronicity. &lt;b&gt;Partial-thickness&amp;nbsp;&lt;/b&gt;tears often appear as fraying of an intact tendon, whereas &lt;b&gt;full-thickness&lt;/b&gt; tears are through-and-through. These can be small pin-point defects, large buttonhole tears (the tendon still remains attached to the humeral head and thus retains function), or tears in which the tendon is completely detached from the humeral head. When classified by chronicity, they can be described as &lt;b&gt;acute&lt;/b&gt; (due to a sudden, powerful movement or trauma), &lt;b&gt;subacute&lt;/b&gt;, or &lt;b&gt;chronic&lt;/b&gt; (develops over a longer period of time, seen in individuals who frequently participate in overhead activities).&lt;/div&gt;&lt;/div&gt;&lt;div&gt;&lt;br /&gt;&lt;/div&gt;&lt;div&gt;&lt;br /&gt;&lt;/div&gt;&lt;div&gt;&lt;b style=&quot;color: #017087;&quot;&gt;&lt;span style=&quot;font-size: medium;&quot;&gt;TREATMENT&lt;/span&gt;&lt;/b&gt;&lt;/div&gt;&lt;div&gt;&lt;br /&gt;&lt;/div&gt;&lt;div&gt;&lt;div&gt;Rotator cuff tears can be managed via both operative and nonoperative means. The patient’s age, activity level, overall health, cuff status (ie, size and age of tear, amount of retraction, muscle quality), and presence of GH arthritis must be considered when determining the best treatment for a given patient. &lt;b&gt;In general, young,&amp;nbsp;&lt;/b&gt;&lt;b&gt;active patients presenting with an acute tear and a primary complaint of weakness&amp;nbsp;&lt;/b&gt;&lt;b&gt;are best treated with early surgical repair, whereas older patients complaining of&amp;nbsp;&lt;/b&gt;&lt;b&gt;pain in the setting of chronic, degenerative tears are most responsive to nonoperative&amp;nbsp;&lt;/b&gt;&lt;b&gt;treatment.&lt;/b&gt;&lt;/div&gt;&lt;/div&gt;&lt;div&gt;&lt;br /&gt;&lt;/div&gt;&lt;div&gt;&lt;div&gt;&lt;span&gt;&amp;nbsp;&amp;nbsp; &amp;nbsp;&lt;/span&gt;Nonoperative treatment includes nonsteroidal anti-inflammatory drugs, (NSAIDs), activity modifications, subacromial corticosteroid injections, and physical therapy. Corticosteroid injections into the subacromial space are done with failure of other conservative modalities for greater than 4 to 6 weeks. The goal of these injections is to decrease pain. They may be repeated after several months if initially effective. &lt;b&gt;However, the patient should receive no more than 3 injections&amp;nbsp;&lt;/b&gt;&lt;b&gt;per year.&lt;/b&gt; Physical therapy focuses on aggressive rotator cuff and periscapular muscle strengthening.&lt;/div&gt;&lt;/div&gt;&lt;div&gt;&lt;br /&gt;&lt;/div&gt;&lt;div&gt;&lt;div&gt;&lt;span&gt;&amp;nbsp;&amp;nbsp; &amp;nbsp;&lt;/span&gt;Operative repair of an RCT can be done through either open or arthroscopic approaches, which have been shown to have equivalent results. With both techniques, the goal is to restore the native tendon(s) insertional footprint area on the tuberosities. It is thought that a larger, more anatomic footprint improves healing and the mechanical strength of the repair. Biologic healing of the cuff is estimated to take approximately 8 to 12 weeks; this is the rate-limiting step for recovery. Anatomic footprint restoration is often achieved with double-row suture techniques. Of note, animal models have &lt;i&gt;failed &lt;/i&gt;to show increased repair strength with the addition of a trough in the greater tuberosity.&lt;/div&gt;&lt;/div&gt;&lt;div&gt;&lt;br /&gt;&lt;/div&gt;&lt;div&gt;&lt;div&gt;&lt;span&gt;&amp;nbsp;&amp;nbsp; &amp;nbsp;&lt;/span&gt;Complications after rotator cuff repair include recurrence, deltoid detachment (with open procedures), acromiclavicular joint pain, axillary nerve injury, and suprascapular nerve injury. &lt;b&gt;Worker’s compensation patients report worse outcomes with higher postoperative disability and lower satisfaction.&lt;/b&gt;&lt;/div&gt;&lt;/div&gt;&lt;div&gt;&lt;br /&gt;&lt;/div&gt;&lt;div&gt;&lt;div&gt;&lt;span&gt;&amp;nbsp;&amp;nbsp; &amp;nbsp;&lt;/span&gt;There are several procedures that may be performed at the time of rotator cuff repair or as independent operations before repair. Subacromial decompression is done if impingement is thought to have contributed to tendon irritation and tearing. AC joint resection is performed via distal clavicle excision in patients whose AC joint is tender to palpation and painful. A long head of the biceps tenotomy (detachment of the tendon origin from the labrum or glenoid) or tenodesis (releasing of the tendon origin and suturing it to the proximal humerus) is done in patients with a symptomatic biceps tendon.&lt;/div&gt;&lt;/div&gt;&lt;div&gt;&lt;br /&gt;&lt;/div&gt;&lt;div&gt;&lt;div&gt;&lt;span&gt;&amp;nbsp;&amp;nbsp; &amp;nbsp;&lt;/span&gt;Irreparable and massive rotator cuff tears require special consideration. In patients with this but an otherwise normal GH joint, tendon transfer can be performed with the goal of restoring overhead function. For irreparable or chronic subscapularis tears, pectoralis major transfer is performed. For large tears of the supraspinatus and infraspinatus, a latissimus dorsi transfer is performed. In this procedure, the latissimus is attached to the cuff muscles, subscapularis, and greater tuberosity. The given extremity is then immobilized in a brace for 6 weeks in 45 degrees of abduction and 30 degrees of external rotation. &lt;b&gt;Of note, the best candidate for a latissimus dorsi&amp;nbsp;&lt;/b&gt;&lt;b&gt;transfer is a young laborer.&lt;/b&gt;&lt;/div&gt;&lt;/div&gt;&lt;div&gt;&lt;br /&gt;&lt;/div&gt;&lt;div&gt;&lt;br /&gt;&lt;/div&gt;&lt;div&gt;&lt;b&gt;&lt;span style=&quot;color: #017087; font-size: medium;&quot;&gt;&lt;u&gt;COMPREHENSION QUESTIONS&lt;/u&gt;&lt;/span&gt;&lt;/b&gt;&lt;/div&gt;&lt;div&gt;&lt;br /&gt;&lt;/div&gt;&lt;div&gt;&lt;div&gt;21.1 A 45-year-old man has a fall from his motorcycle and is complaining of shoulder pain. Radiographs are negative for fracture and dislocation. On exam, you note a positive lift-off test. What is the most likely diagnosis?&lt;/div&gt;&lt;/div&gt;&lt;blockquote style=&quot;border: none; margin: 0px 0px 0px 40px; padding: 0px; text-align: left;&quot;&gt;&lt;div&gt;&lt;div&gt;A. Teres minor tear&lt;/div&gt;&lt;/div&gt;&lt;div&gt;&lt;div&gt;B. Supraspinatus tear&lt;/div&gt;&lt;/div&gt;&lt;div&gt;&lt;div&gt;C. Infraspinatus tear&lt;/div&gt;&lt;/div&gt;&lt;div&gt;&lt;div&gt;D. Subscapularis tear&lt;/div&gt;&lt;/div&gt;&lt;/blockquote&gt;&lt;div&gt;&lt;div&gt;&lt;br /&gt;&lt;/div&gt;&lt;div&gt;21.2 An 80-year-old man sustained an anterior shoulder dislocation 2 weeks ago. He was reduced in the emergency department acutely and has been treated in a sling by his primary care physician. Radiographs after reduction appear normal. He now presents with severe pain and weakness of his shoulder. What is the most likely diagnosis?&lt;/div&gt;&lt;/div&gt;&lt;blockquote style=&quot;border: none; margin: 0px 0px 0px 40px; padding: 0px; text-align: left;&quot;&gt;&lt;div&gt;&lt;div&gt;A. Continued anterior subluxation of the humerus&lt;/div&gt;&lt;/div&gt;&lt;div&gt;&lt;div&gt;B. Rotator cuff tear&lt;/div&gt;&lt;/div&gt;&lt;div&gt;&lt;div&gt;C. Cervical spine radiculopathy&lt;/div&gt;&lt;/div&gt;&lt;div&gt;&lt;div&gt;D. Proximal humerus fracture&lt;/div&gt;&lt;/div&gt;&lt;/blockquote&gt;&lt;div&gt;&lt;div&gt;&lt;br /&gt;&lt;/div&gt;&lt;div&gt;21.3 A 65-year-old woman presents to the clinic complaining of worsening pain in her left shoulder that is most severe at night. She is an avid swimmer and states that she has been unable to swim for the last 3 weeks because of the pain. She denies any recent trauma and has brought plain radiographs of her shoulder in 3 views (ordered by her internist) with her, which are negative. You suspect a torn rotator cuff and order an MRI. However, the patient tells you that she cannot have one, due to her pacemaker. Which of the following imaging modalities is the most appropriate for this patient?&lt;/div&gt;&lt;/div&gt;&lt;blockquote style=&quot;border: none; margin: 0px 0px 0px 40px; padding: 0px; text-align: left;&quot;&gt;&lt;div&gt;&lt;div&gt;A. Ultrasound&lt;/div&gt;&lt;/div&gt;&lt;div&gt;&lt;div&gt;B. PET scan&lt;/div&gt;&lt;/div&gt;&lt;div&gt;&lt;div&gt;C. Arthrogram&lt;/div&gt;&lt;/div&gt;&lt;div&gt;&lt;div&gt;D. Additional plain radiographs&lt;/div&gt;&lt;/div&gt;&lt;/blockquote&gt;&lt;div&gt;&lt;br /&gt;&lt;/div&gt;&lt;div&gt;&lt;br /&gt;&lt;/div&gt;&lt;div&gt;&lt;b&gt;&lt;span style=&quot;color: #017087; font-size: medium;&quot;&gt;&lt;u&gt;ANSWERS&lt;/u&gt;&lt;/span&gt;&lt;/b&gt;&lt;/div&gt;&lt;div&gt;&lt;br /&gt;&lt;/div&gt;&lt;div&gt;&lt;div&gt;21.1 &lt;b&gt;D.&lt;/b&gt; A positive lift-off test is consistent with a subscapularis injury. In this maneuver, the patient’s arm is internally rotated, the elbow flexed to 90 degrees, and hand held posteriorly at the waist. If the patient experiences pain or weakness as he tries to move his arm away from his body against resistance, the test is positive.&lt;/div&gt;&lt;/div&gt;&lt;div&gt;&lt;br /&gt;&lt;/div&gt;&lt;div&gt;&lt;div&gt;21.2 &lt;b&gt;B.&lt;/b&gt; Forty percent of patients older than 60 years with a shoulder dislocation will have a concomitant RCT. However, it is also important to recognize that even in the absence of trauma, RCT is very common in older individuals. Sixty-five percent of patients older than 70 years have a full-thickness RCT.&lt;/div&gt;&lt;div&gt;&lt;br /&gt;&lt;/div&gt;&lt;div&gt;21.3 &lt;b&gt;C.&lt;/b&gt; Arthrograms are used when MRI is contraindicated. They improve sensitivity and specificity in the diagnosis of an RCT. It is a fluoroscopic examination of the shoulder joint after the introduction of contrast media into the joint. Extravasation of the dye from the joint would support the clinical diagnosis of an RCT. Although ultrasound is also an alternative imaging modality in the setting of a contraindication to an MRI, an arthrogram is a better choice. There is no role for PET scans in the diagnosis of RCT. Additional plain radiographs will likely be of no benefit unless the originals are of poor quality.&lt;/div&gt;&lt;/div&gt;&lt;div&gt;&lt;br /&gt;&lt;/div&gt;&lt;div&gt;&lt;b style=&quot;background-color: #4093a5;&quot;&gt;&amp;nbsp; &amp;nbsp;&lt;span style=&quot;font-size: medium;&quot;&gt;&amp;nbsp;&lt;/span&gt;&lt;span style=&quot;color: white;&quot;&gt;&lt;span style=&quot;font-size: medium;&quot;&gt;CLINICAL PEARLS&lt;/span&gt;&lt;span&gt;&amp;nbsp; &amp;nbsp;&amp;nbsp;&lt;/span&gt;&lt;/span&gt;&lt;/b&gt;&lt;/div&gt;&lt;div&gt;&lt;table border=&quot;1&quot; cellpadding=&quot;0&quot; cellspacing=&quot;0&quot; class=&quot;MsoTableGrid&quot; style=&quot;border-collapse: collapse; border: none; mso-border-alt: solid windowtext .5pt; mso-padding-alt: 0in 5.4pt 0in 5.4pt; mso-yfti-tbllook: 1184;&quot;&gt;
  314. &lt;tbody&gt;&lt;tr style=&quot;height: 19.15pt; mso-yfti-firstrow: yes; mso-yfti-irow: 0; mso-yfti-lastrow: yes;&quot;&gt;
  315.  &lt;td style=&quot;background: rgb(221, 237, 239); border-left: solid #017087 1.0pt; border: 1pt solid rgb(64, 147, 165); height: 19.15pt; padding: 0in 5.4pt; width: 433.7pt;&quot; valign=&quot;top&quot; width=&quot;578&quot;&gt;
  316.  &lt;div style=&quot;line-height: normal; margin-bottom: 0in;&quot;&gt;&lt;span style=&quot;color: #017087;&quot;&gt;&lt;br /&gt;&lt;/span&gt;&lt;/div&gt;&lt;div style=&quot;line-height: normal; margin-bottom: 0in;&quot;&gt;&lt;span style=&quot;color: #017087;&quot;&gt;►&lt;/span&gt;&amp;nbsp;The rotator cuff is an important dynamic stabilizer of the shoulder and provides humeral head depression, rotation, shoulder abduction, and GH joint compression.&lt;/div&gt;&lt;div style=&quot;line-height: normal; margin-bottom: 0in;&quot;&gt;&lt;span style=&quot;color: #017087;&quot;&gt;&lt;br /&gt;&lt;/span&gt;&lt;/div&gt;&lt;div style=&quot;line-height: normal; margin-bottom: 0in;&quot;&gt;&lt;span style=&quot;color: #017087;&quot;&gt;►&lt;/span&gt;&amp;nbsp;The supraspinatus is the most commonly torn tendon.&lt;/div&gt;&lt;div style=&quot;line-height: normal; margin-bottom: 0in;&quot;&gt;&lt;span style=&quot;color: #017087;&quot;&gt;&lt;br /&gt;&lt;/span&gt;&lt;/div&gt;&lt;div style=&quot;line-height: normal; margin-bottom: 0in;&quot;&gt;&lt;span style=&quot;color: #017087;&quot;&gt;►&lt;/span&gt;&amp;nbsp;MRI is extremely accurate in detecting full-thickness tears of the rotator cuff and can evaluate tear size, tendon retraction, muscle atrophy, and related intraarticular pathology.&lt;/div&gt;&lt;div style=&quot;line-height: normal; margin-bottom: 0in;&quot;&gt;&lt;span style=&quot;color: #017087;&quot;&gt;&lt;br /&gt;&lt;/span&gt;&lt;/div&gt;&lt;div style=&quot;line-height: normal; margin-bottom: 0in;&quot;&gt;&lt;span style=&quot;color: #017087;&quot;&gt;►&lt;/span&gt;&amp;nbsp;Young, active patients presenting with an acute tear and a primary complaint of weakness are best treated with early surgical repair of a rotator cuff tear.&lt;/div&gt;&lt;div style=&quot;line-height: normal; margin-bottom: 0in;&quot;&gt;&lt;span style=&quot;color: #017087;&quot;&gt;&lt;br /&gt;&lt;/span&gt;&lt;/div&gt;&lt;div style=&quot;line-height: normal; margin-bottom: 0in;&quot;&gt;&lt;span style=&quot;color: #017087;&quot;&gt;►&lt;/span&gt;&amp;nbsp;Open and arthroscopic techniques are used to repair the rotator cuff.&lt;/div&gt;&lt;div style=&quot;line-height: normal; margin-bottom: 0in;&quot;&gt;&lt;br /&gt;&lt;/div&gt;
  317.  &lt;/td&gt;
  318. &lt;/tr&gt;
  319. &lt;/tbody&gt;&lt;/table&gt;&lt;br /&gt;&lt;/div&gt;
  320.  
  321. &lt;details close=&quot;&quot;&gt;
  322.  &lt;summary&gt;&lt;b&gt;&lt;span style=&quot;color: #00788b;&quot;&gt;REFERENCES&lt;/span&gt;&lt;/b&gt;&lt;/summary&gt;
  323.  &lt;p&gt;Bassett RW, Cofield RH. Acute tears of the rotator cuff: the timing of surgical repair. Clin Orthop.
  324. 1983;175:18-24.&amp;nbsp;&lt;/p&gt;&lt;p&gt;McMahon P. Current Diagnosis and Treatment in Sports Medicine . New York: McGraw-Hill; 2007:120-155.&amp;nbsp;&lt;/p&gt;&lt;p&gt;Rockwood CA, Matsen FA. The Shoulder . Philadelphia: WB Saunders; 1998:755-795.&amp;nbsp;&lt;/p&gt;&lt;p&gt;Sperling JW, Cofield RH. Rotator cuff repair in patients fifty years of age and younger. J Bone Joint Surg
  325. Am . 2004;86:2212-2215.&amp;nbsp;&lt;/p&gt;&lt;p&gt;Vaccaro AR. Orthopaedic Knowledge Update 8 . Washington, DC: American Academy of Orthopaedic
  326. Surgeons; 2005:257-350.&amp;nbsp;&lt;/p&gt;&lt;p&gt;Wilk KE, Reinold MM, Andrews JR. The Athlete’s Shoulder . New York: McGraw-Hill; 2008:25-60.&lt;/p&gt;
  327. &lt;/details&gt;&lt;/div&gt;</content><link rel='replies' type='application/atom+xml' href='https://medical-phd.blogspot.com/feeds/483106974847837394/comments/default' title='Post Comments'/><link rel='replies' type='text/html' href='https://medical-phd.blogspot.com/2022/03/rotator-cuff-injury-case-file.html#comment-form' title='0 Comments'/><link rel='edit' type='application/atom+xml' href='https://www.blogger.com/feeds/3302710210518459416/posts/default/483106974847837394'/><link rel='self' type='application/atom+xml' href='https://www.blogger.com/feeds/3302710210518459416/posts/default/483106974847837394'/><link rel='alternate' type='text/html' href='https://medical-phd.blogspot.com/2022/03/rotator-cuff-injury-case-file.html' title='Rotator Cuff Injury Case File'/><author><name>Medical Group</name><uri>http://www.blogger.com/profile/01780656800578397393</uri><email>noreply@blogger.com</email><gd:image rel='http://schemas.google.com/g/2005#thumbnail' width='32' height='32' src='//blogger.googleusercontent.com/img/b/R29vZ2xl/AVvXsEiMuROkBDv6yw4SmxGqRA5BPQhOXSwFld-Gufmy4kUqfpoAGQRl7cv-3naew6wjAnCjRFSe_FKX8z7CdE6cFKxxBpnqBuwUTSr-15xd3mFGcr_wJOX2y73G_zJCGND98Q/s220/Final-logo.jpg'/></author><media:thumbnail xmlns:media="http://search.yahoo.com/mrss/" url="https://blogger.googleusercontent.com/img/b/R29vZ2xl/AVvXsEhJMD9EDdnNS7c_BthHZb4OrKduqx_kNw8426tXdXHs5lnTRcOARAxdJGLG3ZyZsIGC4dZfJso5QM3jIYEY0Pip7jWbyDCo8LiAzLbZp11TF-hNjQoEkX8ELBWMvE3SqNs5M4t-ojkk-4saEirPkcZSlqt8F1UZ7ZA_nnNPb7ZRHUSkDKx0924vpyVcDg/s72-w640-h518-c/evaluate-specific-muscles-of-the-rotator-cuff.jpg" height="72" width="72"/><thr:total>0</thr:total></entry><entry><id>tag:blogger.com,1999:blog-3302710210518459416.post-1923706066753798097</id><published>2022-03-24T15:32:00.007-07:00</published><updated>2022-03-24T15:32:44.044-07:00</updated><category scheme="http://www.blogger.com/atom/ns#" term="Case Files"/><category scheme="http://www.blogger.com/atom/ns#" term="Orthopaedic Surgery Case File"/><title type='text'>Osteomyelitis Case File</title><content type='html'>&lt;div class=&quot;scrollbox&quot;&gt;
  328.  
  329. &lt;div style=&quot;text-align: left;&quot;&gt;&lt;b&gt;&lt;span style=&quot;font-size: large;&quot;&gt;Osteomyelitis Case File&lt;/span&gt;&lt;/b&gt;&lt;/div&gt;&lt;div&gt;Eugene C. Toy, MD, Andrew J. Rosenbaum, MD, Timothy T. Roberts, MD, Joshua S. Dines, MD&lt;/div&gt;&lt;div&gt;&lt;br /&gt;&lt;/div&gt;&lt;div&gt;&lt;b&gt;&lt;span style=&quot;color: #017087; font-size: medium;&quot;&gt;CASE 20&lt;/span&gt;&lt;/b&gt;&lt;/div&gt;&lt;div&gt;&lt;div&gt;A 62-year-old female is referred to your office for a 3-week history of right foot pain. The patient denies recent trauma to the area, but states she had her second toe amputated by her podiatrist 6 months ago, secondary to an infection she contracted “from a pebble in [her] shoe.” Although her foot had healed, she has begun experiencing a recurrence of pain in this area, despite a change in footwear and diligent monitoring of her feet for ulcers. She admits to mild fevers over the past 10 days, but denies other symptoms. On further history, the patient has type 2 diabetes mellitus. She admits that she stopped seeing her primary care physician last year since he started prescribing “painful insulin shots” for her. On physical exam, erythema is noted at the plantar-medial aspect of her right forefoot. This region is warm to the touch, mildly edematous, and firm, but without any obvious fluctuance or drainage. She has chronic neuropathies in her toes and feet, bilaterally. Recent laboratory results from her primary care provider’s office are remarkable for a white blood cell (WBC) count of 7.4 thousand/μL, erythrocyte sedimentation rate (ESR) of 55 mm/hr, and C-reactive protein (CRP) of 65 mg/L. An anteroposterior (AP) x-ray and a T2-weighted magnetic resonance imaging (MRI) of the right foot are shown in Figures 20–1 and 20–2 .&lt;/div&gt;&lt;div&gt;&lt;span style=&quot;color: #017087;&quot;&gt;&lt;br /&gt;&lt;/span&gt;&lt;/div&gt;&lt;div&gt;&lt;span style=&quot;color: #017087;&quot;&gt;►&lt;/span&gt;&amp;nbsp;What are the radiologic findings?&lt;/div&gt;&lt;div&gt;&lt;span style=&quot;color: #017087;&quot;&gt;►&lt;/span&gt;&amp;nbsp;What is the most likely diagnosis?&lt;/div&gt;&lt;div&gt;&lt;span style=&quot;color: #017087;&quot;&gt;►&lt;/span&gt;&amp;nbsp;What risk factors predispose her to such a diagnosis?&lt;/div&gt;&lt;/div&gt;&lt;div&gt;&lt;br /&gt;&lt;/div&gt;&lt;div class=&quot;separator&quot; style=&quot;clear: both; text-align: center;&quot;&gt;&lt;a href=&quot;https://blogger.googleusercontent.com/img/b/R29vZ2xl/AVvXsEgONM_robY4_OnJBo-44NU554n8Jq02popS-ReIAr6_iOj8J2O7D9BMDxQGZ1m2YggVPr_qwQSIRTz3JYl1PfqdoL663yrM2qB6171d_w1wAso4Jnv-BRDYcrcGTI6nsFxms39p7zRljz5Socjjfjjv0gnzqEwd3QqqRphP_3UV_ur7t3eYEBUhHBGOog/s562/AP-radiograph-of-the-right-foot.jpg&quot; style=&quot;margin-left: 1em; margin-right: 1em;&quot;&gt;&lt;img alt=&quot;AP radiograph of the right foot&quot; border=&quot;0&quot; data-original-height=&quot;562&quot; data-original-width=&quot;301&quot; height=&quot;640&quot; src=&quot;https://blogger.googleusercontent.com/img/b/R29vZ2xl/AVvXsEgONM_robY4_OnJBo-44NU554n8Jq02popS-ReIAr6_iOj8J2O7D9BMDxQGZ1m2YggVPr_qwQSIRTz3JYl1PfqdoL663yrM2qB6171d_w1wAso4Jnv-BRDYcrcGTI6nsFxms39p7zRljz5Socjjfjjv0gnzqEwd3QqqRphP_3UV_ur7t3eYEBUhHBGOog/w342-h640/AP-radiograph-of-the-right-foot.jpg&quot; title=&quot;AP radiograph of the right foot&quot; width=&quot;342&quot; /&gt;&lt;/a&gt;&lt;/div&gt;&lt;div&gt;&lt;b&gt;&lt;span style=&quot;color: #017087;&quot;&gt;Figure 20–1.&lt;/span&gt;&lt;/b&gt; AP radiograph of the right foot, status post second ray resection.&lt;/div&gt;&lt;div&gt;&lt;br /&gt;&lt;/div&gt;&lt;div class=&quot;separator&quot; style=&quot;clear: both; text-align: center;&quot;&gt;&lt;a href=&quot;https://blogger.googleusercontent.com/img/b/R29vZ2xl/AVvXsEgn2xbp1Ek3jnC4qOvqYsLSTyKmSK3RCo0nsx0Sk8cfokDH_XetthDf_zRzcpTBEJ5EeZuUiqrjjc_HLqoMuXS3FzCHsRw5BrMa97aKlQxhU8UDEMZXi7BRFXwd_x9yY_K3Ux-wrewNg3QLuP8t9bX8NQ2RyD47W8eUS-I1w2Hs3RD-oE9J-A2Nwh3KuA/s499/MRI-slice-of-the-right-foot.jpg&quot; style=&quot;margin-left: 1em; margin-right: 1em;&quot;&gt;&lt;img alt=&quot;MRI slice of the right foot&quot; border=&quot;0&quot; data-original-height=&quot;499&quot; data-original-width=&quot;417&quot; height=&quot;400&quot; src=&quot;https://blogger.googleusercontent.com/img/b/R29vZ2xl/AVvXsEgn2xbp1Ek3jnC4qOvqYsLSTyKmSK3RCo0nsx0Sk8cfokDH_XetthDf_zRzcpTBEJ5EeZuUiqrjjc_HLqoMuXS3FzCHsRw5BrMa97aKlQxhU8UDEMZXi7BRFXwd_x9yY_K3Ux-wrewNg3QLuP8t9bX8NQ2RyD47W8eUS-I1w2Hs3RD-oE9J-A2Nwh3KuA/w334-h400/MRI-slice-of-the-right-foot.jpg&quot; title=&quot;MRI slice of the right foot&quot; width=&quot;334&quot; /&gt;&lt;/a&gt;&lt;/div&gt;&lt;div&gt;&lt;b&gt;&lt;span style=&quot;color: #017087;&quot;&gt;Figure 20–2.&lt;/span&gt;&lt;/b&gt; T2-weighted axial MRI slice of the right foot.&lt;/div&gt;&lt;div&gt;&lt;br /&gt;&lt;/div&gt;&lt;div&gt;&lt;br /&gt;&lt;/div&gt;&lt;div&gt;&lt;div&gt;&lt;b&gt;&lt;span style=&quot;font-size: medium;&quot;&gt;ANSWER TO CASE 20:&lt;/span&gt;&lt;/b&gt;&lt;/div&gt;&lt;div&gt;&lt;span style=&quot;background-color: #daebf0; font-size: medium;&quot;&gt;&lt;span&gt;&lt;span style=&quot;background-color: transparent;&quot;&gt;&lt;span style=&quot;color: #017087;&quot;&gt;&lt;b&gt;Osteomyelitis&lt;/b&gt;&lt;/span&gt;&lt;/span&gt;&lt;b&gt;&amp;nbsp;&lt;/b&gt;&lt;/span&gt;&lt;span style=&quot;font-weight: bold;&quot;&gt;&amp;nbsp;&amp;nbsp; &amp;nbsp;&lt;/span&gt;&lt;span style=&quot;font-weight: bold;&quot;&gt;&amp;nbsp;&amp;nbsp; &amp;nbsp;&lt;/span&gt;&lt;span style=&quot;font-weight: bold;&quot;&gt;&amp;nbsp;&amp;nbsp; &amp;nbsp;&lt;/span&gt;&lt;span style=&quot;font-weight: bold;&quot;&gt;&amp;nbsp;&amp;nbsp; &amp;nbsp;&lt;/span&gt;&lt;span style=&quot;font-weight: bold;&quot;&gt;&amp;nbsp;&amp;nbsp; &amp;nbsp;&lt;/span&gt;&lt;span style=&quot;font-weight: bold;&quot;&gt;&amp;nbsp;&amp;nbsp; &amp;nbsp;&lt;/span&gt;&lt;span style=&quot;font-weight: bold;&quot;&gt;&amp;nbsp; &amp;nbsp;&amp;nbsp;&lt;/span&gt;&lt;/span&gt;&lt;/div&gt;&lt;/div&gt;&lt;div&gt;&lt;br /&gt;&lt;/div&gt;&lt;div&gt;&lt;div&gt;&lt;i&gt;Summary&lt;/i&gt;: A 62-year-old woman with uncontrolled diabetes and peripheral neuropathy presents with atraumatic, insidious right foot pain, 3 weeks in duration. She has mild systemic symptoms (fever) and has experienced similar symptoms in the past for which she underwent an amputation of her second toe.&lt;/div&gt;&lt;div&gt;&lt;ul style=&quot;text-align: left;&quot;&gt;&lt;li&gt;&lt;b&gt;Radiologic findings:&lt;/b&gt; This AP radiograph of the right foot shows chronic bone and soft tissue loss in the second ray, consistent with the patient’s history of osteomyelitis and toe amputation. There is significant soft tissue swelling over the head of the first metatarsal and base of the great toe. There is also significant bony erosion in this region, with a loss of normal cortical bone, periosteal elevation, and lytic changes consistent with osteomyelitis. On T2-weighted MRI, the head and distal shaft of the first metatarsal are hyper-resonant, consistent with edema and inflammation in the region. There is also adjacent hyperintensity of the soft tissues medial to the great toe, consistent with soft tissue inflammation and possibly an abscess collection.&lt;/li&gt;&lt;li&gt;&lt;b&gt;Most likely diagnosis:&lt;/b&gt; Osteomyelitis of the first ray.&lt;/li&gt;&lt;li&gt;&lt;b&gt;Predisposing factors:&lt;/b&gt; Poorly controlled type 2 diabetes mellitus, peripheral neuropathy, and history of previous osteomyelitis.&lt;/li&gt;&lt;/ul&gt;&lt;/div&gt;&lt;/div&gt;&lt;div&gt;&lt;br /&gt;&lt;/div&gt;&lt;div&gt;&lt;br /&gt;&lt;/div&gt;&lt;div&gt;&lt;div&gt;&lt;b&gt;&lt;span style=&quot;color: #017087; font-size: medium;&quot;&gt;ANALYSIS&lt;/span&gt;&lt;/b&gt;&lt;/div&gt;&lt;div&gt;&lt;b&gt;&lt;span style=&quot;font-size: medium;&quot;&gt;Objectives&lt;/span&gt;&lt;/b&gt;&lt;/div&gt;&lt;/div&gt;&lt;div&gt;&lt;div&gt;&lt;ol style=&quot;text-align: left;&quot;&gt;&lt;li&gt;Understand the pathogenesis of osteomyelitis and recognize its clinical and radiographic presentation.&lt;/li&gt;&lt;li&gt;Be familiar with the differential diagnoses for, and diagnostic workup of, osteomyelitis.&lt;/li&gt;&lt;li&gt;Be familiar with basic treatment options for osteomyelitis, both operative and nonoperative.&lt;/li&gt;&lt;/ol&gt;&lt;/div&gt;&lt;/div&gt;&lt;div&gt;&lt;br /&gt;&lt;/div&gt;&lt;div&gt;&lt;br /&gt;&lt;/div&gt;&lt;div&gt;&lt;div&gt;&lt;i&gt;&lt;span style=&quot;font-size: medium;&quot;&gt;Considerations&lt;/span&gt;&lt;/i&gt;&lt;/div&gt;&lt;div&gt;&lt;br /&gt;&lt;/div&gt;&lt;div&gt;This patient presents with a soft tissue infection that has progressed to involve the underlying metatarsal bone. She has a history of uncontrolled diabetes and neuropathy. As a risk factor for osteomyelitis, neuropathies predispose patients to microtrauma because the body’s pain-protective response to skin-damaging repetitive wear is lost. Previously, this patient’s neuropathy left her susceptible to skin ulceration from a pebble in her shoe, which wore through the protective dermal barrier and introduced bacteria into the soft tissues overlying the second metatarsal. Poorly controlled diabetes impedes the body’s wound healing and infection-fighting capabilities, rendering diabetics susceptible to bacterial colonization, particularly in their feet. Radiographic studies should be used to further investigate and diagnose osteomyelitis: plain radiographs, computed tomography (CT) scans, MRI, WBC scans, and/or bone scans are used to make the diagnosis. In this patient’s case, plain x-rays clearly show erosive bony changes in the first ray, while T2-weighted MRI shows edematous changes in the bone and surrounding soft tissues consistent with osteomyelitis. This patient has an elevated ESR and CRP, which are elevated in more than 90% of cases of osteomyelitis. An elevated WBC count, although positive in many types of infections and inflammatory states, is only positive in approximately 40% of cases.&lt;/div&gt;&lt;/div&gt;&lt;div&gt;&lt;br /&gt;&lt;/div&gt;&lt;div&gt;&lt;div&gt;&lt;span&gt;&amp;nbsp;&amp;nbsp; &amp;nbsp;&lt;/span&gt;Once the diagnosis is made, the patient should be admitted to the hospital and intravenous antibiotic therapy should commence. If deep tissue or bone biopsy can be easily obtained, and if the patient remains stable, antibiotic therapy may be delayed until biopsies or aspirations are acquired. This is especially true in patients for whom osteomyelitis represents one in several possible diagnoses or when&lt;/div&gt;&lt;div&gt;osteomyelitis is occurring in particularly difficult to debride regions such as vertebral bodies. Biopsy before antibiotic administration optimizes the chance that an adequate sample of bacteria will be acquired for culture and that an appropriate, targeted antibiotic agent may be selected. Unstable patients with evidence of sepsis, however, should receive prompt systemic antibiotic treatment, regardless of whether biopsy can be performed. An additional caveat involves the culturing of superficial tissues, as these cultures tend to grow out benign, typical skin flora, which sometimes leads to the unnecessary and misdirected treatment.&lt;/div&gt;&lt;/div&gt;&lt;div&gt;&lt;br /&gt;&lt;/div&gt;&lt;div&gt;&lt;div&gt;&lt;span&gt;&amp;nbsp;&amp;nbsp; &amp;nbsp;&lt;/span&gt;For this patient, with extensive involvement of deep tissues, surgical drainage and debridement should be performed. Amputation may also be considered. Intravenous antibiotic therapy typically continues for 4 to 6 weeks, with infectious disease consultation useful for assistance with antibiotic selection and management.&lt;/div&gt;&lt;/div&gt;&lt;div&gt;&lt;br /&gt;&lt;/div&gt;&lt;div&gt;&lt;br /&gt;&lt;/div&gt;&lt;div&gt;&lt;div&gt;&lt;div&gt;&lt;b&gt;&lt;span style=&quot;font-size: medium;&quot;&gt;APPROACH TO:&lt;/span&gt;&lt;/b&gt;&lt;/div&gt;&lt;div&gt;&lt;span style=&quot;background-color: #daebf0; color: #017087; font-size: medium;&quot;&gt;&lt;span&gt;&lt;span style=&quot;background-color: transparent;&quot;&gt;&lt;b&gt;Osteomyelitis&lt;/b&gt;&lt;/span&gt;&lt;b&gt;&amp;nbsp;&lt;/b&gt;&lt;/span&gt;&lt;span style=&quot;font-weight: bold;&quot;&gt;&amp;nbsp;&amp;nbsp; &amp;nbsp;&lt;/span&gt;&lt;span style=&quot;font-weight: bold;&quot;&gt;&amp;nbsp;&amp;nbsp; &amp;nbsp;&lt;/span&gt;&lt;span style=&quot;font-weight: bold;&quot;&gt;&amp;nbsp;&amp;nbsp; &amp;nbsp;&lt;/span&gt;&lt;span style=&quot;font-weight: bold;&quot;&gt;&amp;nbsp;&amp;nbsp; &amp;nbsp;&lt;/span&gt;&lt;span style=&quot;font-weight: bold;&quot;&gt;&amp;nbsp;&amp;nbsp; &amp;nbsp;&lt;/span&gt;&lt;span style=&quot;font-weight: bold;&quot;&gt;&amp;nbsp;&amp;nbsp; &amp;nbsp;&lt;/span&gt;&lt;span style=&quot;font-weight: bold;&quot;&gt;&amp;nbsp;&amp;nbsp; &amp;nbsp;&lt;/span&gt;&lt;span style=&quot;font-weight: bold;&quot;&gt;&amp;nbsp;&amp;nbsp; &amp;nbsp;&lt;/span&gt;&lt;span style=&quot;font-weight: bold;&quot;&gt;&amp;nbsp;&amp;nbsp; &amp;nbsp;&lt;/span&gt;&lt;span style=&quot;font-weight: bold;&quot;&gt;&amp;nbsp;&amp;nbsp;&lt;/span&gt;&lt;/span&gt;&lt;/div&gt;&lt;/div&gt;&lt;div&gt;&lt;br /&gt;&lt;/div&gt;&lt;div&gt;&lt;b&gt;&lt;span style=&quot;color: #017087; font-size: medium;&quot;&gt;DEFINITIONS&lt;/span&gt;&lt;/b&gt;&lt;/div&gt;&lt;/div&gt;&lt;div&gt;&lt;br /&gt;&lt;/div&gt;&lt;div&gt;&lt;div&gt;&lt;b&gt;ACUTE OSTEOMYELITIS:&lt;/b&gt; Presence of bacteria or infectious pathogens in bone or bone marrow without the chronic histologic changes described below. Defined in some literature as a bone infection lasting less than 6 weeks. Acute osteomyelitis most commonly occurs in children.&lt;/div&gt;&lt;div&gt;&lt;b&gt;&lt;br /&gt;&lt;/b&gt;&lt;/div&gt;&lt;div&gt;&lt;b&gt;CHRONIC OSTEOMYELITIS:&lt;/b&gt; May be defined by histologic changes such as the presence of a sequestrum or an involucrum, or by duration of infection greater than 6 weeks. Chronic osteomyelitis is characterized by the presence of bacterial biofilm and bacterial resistance to host defenses and antibiotic agents and often requires surgical debridement and long-term systemic antibiotic therapy.&lt;/div&gt;&lt;div&gt;&lt;b&gt;&lt;br /&gt;&lt;/b&gt;&lt;/div&gt;&lt;div&gt;&lt;b&gt;BIOFILM:&lt;/b&gt; An aggregate of microorganisms embedded within an adherent polysaccharide matrix that provides bacterial immunity to host cellular defenses and antiobiotic agents.&lt;/div&gt;&lt;div&gt;&lt;b&gt;&lt;br /&gt;&lt;/b&gt;&lt;/div&gt;&lt;div&gt;&lt;b&gt;SEQUESTRUM:&lt;/b&gt; A region of devitalized bone that serves as a nidus for continual infection; a hallmark of chronic osteomyelitis.&lt;/div&gt;&lt;/div&gt;&lt;div&gt;&lt;br /&gt;&lt;/div&gt;&lt;div&gt;&lt;div&gt;&lt;b&gt;INVOLUCRUM:&lt;/b&gt; The formation of new, reactive bone around an area of septic necrosis; another hallmark of chronic osteomyelitis.&lt;/div&gt;&lt;/div&gt;&lt;div&gt;&lt;br /&gt;&lt;/div&gt;&lt;div&gt;&lt;br /&gt;&lt;/div&gt;&lt;div&gt;&lt;b&gt;&lt;span style=&quot;color: #017087; font-size: medium;&quot;&gt;CLINICAL APPROACH&lt;/span&gt;&lt;/b&gt;&lt;/div&gt;&lt;div&gt;&lt;div&gt;&lt;br /&gt;&lt;/div&gt;&lt;div&gt;&lt;i&gt;&lt;span style=&quot;font-size: medium;&quot;&gt;Pathophysiology&lt;/span&gt;&lt;/i&gt;&lt;/div&gt;&lt;div&gt;&lt;br /&gt;&lt;/div&gt;&lt;div&gt;Osteomyelitis is an infection of &lt;b&gt;bone or bone marrow.&lt;/b&gt; This highly variable disease may be classified as acute versus chronic, hematogenous versus contiguous, and with or without vascular compromise (Lee and Waldvogel classification). Osteomyelitis may also be classified by its location and extent (Cierny and Mader classification).&lt;/div&gt;&lt;div&gt;&lt;br /&gt;&lt;/div&gt;&lt;div&gt;Osteomyelitis can result from a &lt;b&gt;continuous spread&lt;/b&gt; of an infection from soft tissue or joints, trauma causing &lt;b&gt;direct bacterial inoculation&lt;/b&gt; into bone or adjacent soft tissues, or &lt;b&gt;hematogenously spread&lt;/b&gt; from an infection elsewhere in the body.&lt;/div&gt;&lt;div&gt;&lt;br /&gt;&lt;/div&gt;&lt;div&gt;Patients with &lt;b&gt;vascular diseases, poorly controlled diabetes, and those in states&amp;nbsp;&lt;/b&gt;&lt;b&gt;of immunocompromise are at elevated risk&lt;/b&gt; for developing acute osteomyelitis. Immunocompromised individuals are also at risk for developing chronic osteomyelitis. Other risk factors for &lt;b&gt;chronic osteomyelitis&lt;/b&gt; include history of &lt;b&gt;intravenous drug&amp;nbsp;&lt;/b&gt;&lt;b&gt;abuse &lt;/b&gt;and&lt;b&gt; failure of treatment of acute osteomyelitis.&lt;/b&gt;&lt;/div&gt;&lt;div&gt;&lt;br /&gt;&lt;/div&gt;&lt;div&gt;Osteomyelitis can be caused by bacteria, fungi, and viruses. By a great majority, the &lt;b&gt;most common causative agents are bacterial,&lt;/b&gt; with &lt;i&gt;Staphylococcus&lt;/i&gt; &lt;i&gt;aureus&amp;nbsp;&lt;/i&gt;being the most commonly identified offending organism. Other common inoculants include &lt;i&gt;Pseudomonas aeruginosa, Escherichia coli&lt;/i&gt;, coagulase-negative staphylococci, enterococci, and propionibacteria. &lt;i&gt;Salmonella&lt;/i&gt; species are commonly identified pathogens in patients with sickle-cell anemia; however, staphylococcal species are still most common in these patients.&lt;/div&gt;&lt;/div&gt;&lt;div&gt;&lt;br /&gt;&lt;/div&gt;&lt;div&gt;&lt;br /&gt;&lt;/div&gt;&lt;div&gt;&lt;div&gt;&lt;i&gt;&lt;span style=&quot;font-size: medium;&quot;&gt;Diagnosis&lt;/span&gt;&lt;/i&gt;&lt;/div&gt;&lt;div&gt;&lt;br /&gt;&lt;/div&gt;&lt;div&gt;Osteomyelitis may present as an occult infection in patients of all ages. Its most common presentation is in patients with &lt;b&gt;deep bone pain and without significant systemic&amp;nbsp;&lt;/b&gt;&lt;b&gt;symptoms.&lt;/b&gt; When systemic symptoms are present, fever and chills predominate. &lt;b&gt;Soft&amp;nbsp;&lt;/b&gt;&lt;b&gt;tissues surrounding an infection may be edematous, warm, and erythematous.&amp;nbsp;&lt;/b&gt;Patients may present with a history of recent trauma or infection or with a history of an invasive procedure. Osteomyelitis should be included in the differential diagnosis of other musculoskeletal pathologies such as gout, ischemia, arthritis, neuropathy, and degenerative disc disease in the spine. Delay in diagnosis and treatment of acute infection may lead to chronic osteomyelitis, a relapsing and remitting indolent infection characterized by intermittent pain attacks and systemic symptoms.&lt;/div&gt;&lt;div&gt;&lt;br /&gt;&lt;/div&gt;&lt;div&gt;&lt;br /&gt;&lt;/div&gt;&lt;div&gt;&lt;i&gt;&lt;span style=&quot;font-size: medium;&quot;&gt;Radiographic Evaluation&lt;/span&gt;&lt;/i&gt;&lt;/div&gt;&lt;div&gt;&lt;br /&gt;&lt;/div&gt;&lt;div&gt;Plain radiographs should be obtained before more advanced imaging is considered. On plain films, acute osteomyelitis may be identified by cortical bone destruction with a surrounding area of periosteal elevation. Acute osteomyelitis will typically have a lytic, or radiolucent, appearance, whereas chronic infections may be distinguished by the presence of a mixed lytic and blastic, or radio-opaque, appearance. Radiographs are not sensitive for osteomyelitis, however, as up to &lt;b&gt;30% to&amp;nbsp;&lt;/b&gt;&lt;b&gt;40% of bone destruction must occur before osteomyelitic changes are visible on&amp;nbsp;&lt;/b&gt;&lt;b&gt;plain x-rays. &lt;/b&gt;Symptoms of the disease typically are present before x-ray changes are apparent.&lt;/div&gt;&lt;/div&gt;&lt;div&gt;&lt;br /&gt;&lt;/div&gt;&lt;div&gt;&lt;div&gt;&lt;span&gt;&amp;nbsp;&amp;nbsp; &amp;nbsp;&lt;/span&gt;MRI with intravenous (IV) gadolinium contrast is far more sensitive for diagnosing osteomyelitis. MRI offers a clear view of soft tissue pathology and is currently the gold standard in the diagnosis of osteomyelitis. Be wary that in the presence of metallic implants, the sensitive/specificity of MRI is decreased; in such situations, IV contrast CT scans may be helpful in elucidating bony pathology.&lt;/div&gt;&lt;div&gt;&lt;br /&gt;&lt;/div&gt;&lt;div&gt;&lt;span&gt;&amp;nbsp;&amp;nbsp; &amp;nbsp;&lt;/span&gt;Finally, radioisotope tagged WBC scans are very specific for osteomyelitis and are often used to follow treatment progression. Bone scans may be used in a similar fashion, but are less specific.&lt;/div&gt;&lt;/div&gt;&lt;div&gt;&lt;br /&gt;&lt;/div&gt;&lt;div&gt;&lt;br /&gt;&lt;/div&gt;&lt;div&gt;&lt;b style=&quot;color: #017087;&quot;&gt;&lt;span style=&quot;font-size: medium;&quot;&gt;TREATMENT&lt;/span&gt;&lt;/b&gt;&lt;/div&gt;&lt;div&gt;&lt;br /&gt;&lt;/div&gt;&lt;div&gt;&lt;div&gt;Acute osteomyelitis, typically presenting in children and IV drug abusers with erythematous and painful limbs, requires prompt antibiotic therapy, often 4 to 6 weeks’ duration. Such patients may not require surgical debridement, but should be followed closely to ensure successful therapy.&lt;/div&gt;&lt;div&gt;&lt;br /&gt;&lt;/div&gt;&lt;div&gt;&lt;span&gt;&amp;nbsp;&amp;nbsp; &amp;nbsp;&lt;/span&gt;Chronic osteomyelitis also requires long-term IV drug therapy, but typically requires additional open surgical procedures to debride necrotic tissue, drain abscesses, and remove sequestra or nonviable bone serving as nidi for bacteria.&lt;/div&gt;&lt;div&gt;&lt;br /&gt;&lt;/div&gt;&lt;div&gt;&lt;span&gt;&amp;nbsp;&amp;nbsp; &amp;nbsp;&lt;/span&gt;In chronic osteomyelitis, IV antibiotic is typically administered for 4 to 6 weeks and may be administered in an outpatient setting once a patient has been stabilized. In patients with extensive disease or immunocompromised states, longer courses of IV drug therapy may be indicated. Initial antibiotic coverage should be broad, with tailored antibiotic selection once, and only if, reliable bacterial cultures and drug sensitivities are available. Initial antibiotic selection must cover S. aureus , the most commonly identified pathogen in osteomyelitis. In patients with a history of recent hospitalization, initial therapy must cover methicillin-resistant&amp;nbsp;&lt;i&gt;S. aureus&lt;/i&gt;, a prevalent nosocomial infection. Serial CRP and ESR studies, although not especially specific for osteomyelitis, are sometimes followed as markers for successful treatment.&lt;/div&gt;&lt;/div&gt;&lt;div&gt;&lt;br /&gt;&lt;/div&gt;&lt;div&gt;&lt;div&gt;&lt;span&gt;&amp;nbsp;&amp;nbsp; &amp;nbsp;&lt;/span&gt;In addition to a thorough surgical debridement, several adjunctive surgical treatments may be indicated, including the placement of antibiotic-eluting polymethyl- methacrylate (PMMA) beads. PMMA, the same bone “cement” used in the implantation of joint prostheses, may be mixed with antibiotic agents such as gentamycin, molded into round beads or fashioned into an antibiotic spacer, and placed into the debrided cavity. After resolution of infection, the beads are typically removed.&lt;/div&gt;&lt;/div&gt;&lt;div&gt;&lt;br /&gt;&lt;/div&gt;&lt;div&gt;&lt;div&gt;&lt;span&gt;&amp;nbsp;&amp;nbsp; &amp;nbsp;&lt;/span&gt;The placement of orthopaedic implants such as screws, plates, rods, and prostheses pose a special challenge to the orthopaedist, as artificial implants may serve as a nidus for chronic infection. In such cases, the surgeon must weight the benefits of removing infected hardware with the risks of losing stability or function. As a general rule, infected hardware should be removed when possible.&lt;/div&gt;&lt;/div&gt;&lt;div&gt;&lt;br /&gt;&lt;/div&gt;&lt;div&gt;&lt;br /&gt;&lt;/div&gt;&lt;div&gt;&lt;div&gt;&lt;i&gt;&lt;span style=&quot;font-size: medium;&quot;&gt;Outcomes and Complications&lt;/span&gt;&lt;/i&gt;&lt;/div&gt;&lt;div&gt;&lt;br /&gt;&lt;/div&gt;&lt;div&gt;Musculoskeletal infection that has been treated with antibiotics and surgical debridement can lead to functional deficit in the given extremity. This is dependent on the amount of tissue debrided and if vital structures have been violated and destabilized. When this occurs, it is usually in an attempt to perform a limb-salvage procedure, as both patients and surgeons try to avoid amputation if at all possible. However, in the setting of severe and recurrent musculoskeletal infections, amputation may be the only means by which to permanently eradicate the infection.&lt;/div&gt;&lt;/div&gt;&lt;div&gt;&lt;br /&gt;&lt;/div&gt;&lt;div&gt;&lt;br /&gt;&lt;/div&gt;&lt;div&gt;&lt;b&gt;&lt;span style=&quot;color: #017087; font-size: medium;&quot;&gt;&lt;u&gt;COMPREHENSION QUESTIONS&lt;/u&gt;&lt;/span&gt;&lt;/b&gt;&lt;/div&gt;&lt;div&gt;&lt;br /&gt;&lt;/div&gt;&lt;div&gt;&lt;div&gt;20.1 A 34-year-old, HIV-positive male presents to the emergency department (ED) with complaints of several weeks of neck pain. The patient admits to using recreational drugs, but does not willingly discuss what substances he uses. He has recently begun feeling numbness in his bilateral hands and states he is now unable to properly write with a pen. On exam, he has point tenderness along the lateral aspect of his neck. An MRI is obtained, showing edema and prevertebral swelling in his C3 and C4 vertebrae. There is T2-weighted hyperresonance of the C3-4 disc space, with swelling posterior to C3 that is impinging on the cord. What is the best treatment for this patient?&lt;/div&gt;&lt;/div&gt;&lt;blockquote style=&quot;border: none; margin: 0px 0px 0px 40px; padding: 0px; text-align: left;&quot;&gt;&lt;div&gt;&lt;div&gt;A. Inform the patient that he should follow up as an outpatient for further workup.&lt;/div&gt;&lt;/div&gt;&lt;div&gt;&lt;div&gt;B. Physical therapy and warm compresses, as the patient has no symptoms of myelopathy.&lt;/div&gt;&lt;/div&gt;&lt;div&gt;&lt;div&gt;C. Admit to the hospital promptly for antibiotic therapy and evaluation for surgical intervention.&lt;/div&gt;&lt;/div&gt;&lt;div&gt;&lt;div&gt;D. Bone scan and CT scanning of the chest abdomen and pelvis to evaluate for other lesions.&lt;/div&gt;&lt;/div&gt;&lt;div&gt;&lt;div&gt;E. Biopsies to classify the lesions and determine their source.&lt;/div&gt;&lt;/div&gt;&lt;/blockquote&gt;&lt;div&gt;&lt;div&gt;&lt;br /&gt;&lt;/div&gt;&lt;div&gt;20.2 A 44-year-old truck driver is almost 1 year in recovery from knife wound to his left distal thigh after an altercation at a local bar. He had initially been treated with debridement in the ED and wound closure and was treated with a short course of oral antibiotics. He was neurovascularly intact after the injury. He presents now to your clinic with serous drainage through a sinus tract on the medial aspect of his distal thigh. The area is erythematous and edematous and is mildly tender to palpation. He has been able to bear weight on it, but states that it is causing increasing discomfort while trying to drive his truck. Plain radiograph films are obtained and are equivocal. An MRI is also obtained and shows a region of hyperintensity in the metaphysis of the distal left femur surrounding an area of hypointense bone. What is the best treatment for this patient?&lt;/div&gt;&lt;/div&gt;&lt;blockquote style=&quot;border: none; margin: 0px 0px 0px 40px; padding: 0px; text-align: left;&quot;&gt;&lt;div&gt;&lt;div&gt;A. Long (14-day) course of oral antibiotics with close follow-up&lt;/div&gt;&lt;/div&gt;&lt;div&gt;&lt;div&gt;B. Needle biopsy and targeted oral antibiotics&lt;/div&gt;&lt;/div&gt;&lt;div&gt;&lt;div&gt;C. CT scan of the affected extremity&lt;/div&gt;&lt;/div&gt;&lt;div&gt;&lt;div&gt;D. Needle biopsy and IV antibiotics&lt;/div&gt;&lt;/div&gt;&lt;div&gt;&lt;div&gt;E. Surgical debridement and IV antibiotics&lt;/div&gt;&lt;/div&gt;&lt;/blockquote&gt;&lt;div&gt;&lt;br /&gt;&lt;/div&gt;&lt;div&gt;&lt;div&gt;20.3 A 12-year-old African American male with a history of sickle cell disease is admitted to the hospital with severe left upper extremity pain and fevers. Blood cultures are obtained, and broad-spectrum IV antibiotics and fluid resuscitation are initiated. What is the most likely causative organism in this patient?&lt;/div&gt;&lt;/div&gt;&lt;blockquote style=&quot;border: none; margin: 0px 0px 0px 40px; padding: 0px; text-align: left;&quot;&gt;&lt;div&gt;&lt;div&gt;A. Viral infection&lt;/div&gt;&lt;/div&gt;&lt;div&gt;&lt;div&gt;B. &lt;i&gt;Salmonella&lt;/i&gt;&lt;/div&gt;&lt;/div&gt;&lt;div&gt;&lt;div&gt;C. &lt;i&gt;Streptococcus&lt;/i&gt; species&lt;/div&gt;&lt;/div&gt;&lt;div&gt;&lt;div&gt;D. &lt;i&gt;Staphylococcus aureus&lt;/i&gt;&lt;/div&gt;&lt;/div&gt;&lt;div&gt;&lt;div&gt;E. &lt;i&gt;Escherichia coli&lt;/i&gt;&lt;/div&gt;&lt;/div&gt;&lt;/blockquote&gt;&lt;div&gt;&lt;br /&gt;&lt;/div&gt;&lt;div&gt;&lt;br /&gt;&lt;/div&gt;&lt;div&gt;&lt;b&gt;&lt;span style=&quot;color: #017087; font-size: medium;&quot;&gt;&lt;u&gt;ANSWERS&lt;/u&gt;&lt;/span&gt;&lt;/b&gt;&lt;/div&gt;&lt;div&gt;&lt;br /&gt;&lt;/div&gt;&lt;div&gt;&lt;div&gt;20.1 &lt;b&gt;C.&lt;/b&gt; This patient has osteomyelitis of the cervical spine with discitis at C3-4, and evidence of epidural abscess. Given the patient’s myelopathic state, the patient should be admitted immediately to the hospital and IV antibiotic therapy should begin promptly, ideally after a biopsy is obtained. If symptoms do not remit on broad-spectrum antibiotic treatment, or if myelopathy worsens, prompt surgical evacuation of the abscess should be performed.&lt;/div&gt;&lt;div&gt;&lt;br /&gt;&lt;/div&gt;&lt;div&gt;20.2 &lt;b&gt;E&lt;/b&gt;. This patient has history and radiographic findings consistent with a chronic osteomyelitis. The region of bone involved on MRI is consistent with a sequestrum that must be surgically debrided. After culture, broad-spectrum IV antibiotics should be initiated and targeted according to susceptibility testing of the cultures. Although needle biopsy may help target the appropriate organism, it is insufficient to effectively eradicate the infection. Similarly antibiotics alone, whether oral or broad-spectrum IV, are insufficient without a thorough surgical debridement.&lt;/div&gt;&lt;div&gt;&lt;br /&gt;&lt;/div&gt;&lt;div&gt;20.3 &lt;b&gt;D&lt;/b&gt;. Although &lt;i&gt;Salmonella&lt;/i&gt; species are commonly associated with sickle cell patients with osteomyelitis, the most common causative agent is actually S. &lt;i&gt;aureus. Escherichia coli&lt;/i&gt; and streptococcal species infections also occur in these populations; however, they are less common. Viral osteomyelitis is exceedingly rare.&lt;/div&gt;&lt;/div&gt;&lt;div&gt;&lt;br /&gt;&lt;/div&gt;&lt;div&gt;&lt;b style=&quot;background-color: #4093a5;&quot;&gt;&amp;nbsp; &amp;nbsp;&lt;span style=&quot;font-size: medium;&quot;&gt;&amp;nbsp;&lt;/span&gt;&lt;span style=&quot;color: white;&quot;&gt;&lt;span style=&quot;font-size: medium;&quot;&gt;CLINICAL PEARLS&lt;/span&gt;&lt;span&gt;&amp;nbsp; &amp;nbsp;&amp;nbsp;&lt;/span&gt;&lt;/span&gt;&lt;/b&gt;&lt;/div&gt;&lt;div&gt;&lt;table border=&quot;1&quot; cellpadding=&quot;0&quot; cellspacing=&quot;0&quot; class=&quot;MsoTableGrid&quot; style=&quot;border-collapse: collapse; border: none; mso-border-alt: solid windowtext .5pt; mso-padding-alt: 0in 5.4pt 0in 5.4pt; mso-yfti-tbllook: 1184;&quot;&gt;
  330. &lt;tbody&gt;&lt;tr style=&quot;height: 19.15pt; mso-yfti-firstrow: yes; mso-yfti-irow: 0; mso-yfti-lastrow: yes;&quot;&gt;
  331.  &lt;td style=&quot;background: rgb(221, 237, 239); border-left: solid #017087 1.0pt; border: 1pt solid rgb(64, 147, 165); height: 19.15pt; padding: 0in 5.4pt; width: 433.7pt;&quot; valign=&quot;top&quot; width=&quot;578&quot;&gt;&lt;div style=&quot;line-height: normal; margin-bottom: 0in;&quot;&gt;&lt;span style=&quot;color: #017087;&quot;&gt;&lt;br /&gt;&lt;/span&gt;&lt;/div&gt;&lt;div style=&quot;line-height: normal; margin-bottom: 0in;&quot;&gt;&lt;span style=&quot;color: #017087;&quot;&gt;►&amp;nbsp;&lt;/span&gt;Osteomyelitis is sometimes referred to as the “great imitator” because it can radiographically mimic almost any neoplastic or inflammatory process.&lt;/div&gt;&lt;div style=&quot;line-height: normal; margin-bottom: 0in;&quot;&gt;&lt;span style=&quot;color: #017087;&quot;&gt;&lt;br /&gt;&lt;/span&gt;&lt;/div&gt;&lt;div style=&quot;line-height: normal; margin-bottom: 0in;&quot;&gt;&lt;span style=&quot;color: #017087;&quot;&gt;►&lt;/span&gt;&amp;nbsp;Osteomyelitis must destroy 30% to 40% of a given region of bone before it is evident on plain x-ray.&lt;/div&gt;&lt;div style=&quot;line-height: normal; margin-bottom: 0in;&quot;&gt;&lt;span style=&quot;color: #017087;&quot;&gt;&lt;br /&gt;&lt;/span&gt;&lt;/div&gt;&lt;div style=&quot;line-height: normal; margin-bottom: 0in;&quot;&gt;&lt;span style=&quot;color: #017087;&quot;&gt;►&lt;/span&gt;&amp;nbsp;Elevated WBC counts are found in only 40% of patients presenting with chronic osteomyelitis; elevated CRP and ESR are found in approximately 90%.&lt;/div&gt;&lt;div style=&quot;line-height: normal; margin-bottom: 0in;&quot;&gt;&lt;span style=&quot;color: #017087;&quot;&gt;&lt;br /&gt;&lt;/span&gt;&lt;/div&gt;&lt;div style=&quot;line-height: normal; margin-bottom: 0in;&quot;&gt;&lt;span style=&quot;color: #017087;&quot;&gt;►&lt;/span&gt;&amp;nbsp;Acute osteomyelitis usually requires prolonged IV antibiotic therapy. Chronic osteomyelitis generally requires surgical debridement as well as prolonged antibiotics.&lt;/div&gt;&lt;div style=&quot;line-height: normal; margin-bottom: 0in;&quot;&gt;&lt;br /&gt;&lt;/div&gt;
  332.  &lt;/td&gt;
  333. &lt;/tr&gt;
  334. &lt;/tbody&gt;&lt;/table&gt;&lt;br /&gt;&lt;/div&gt;
  335.  
  336. &lt;details close=&quot;&quot;&gt;
  337.  &lt;summary&gt;&lt;b&gt;&lt;span style=&quot;color: #00788b;&quot;&gt;REFERENCES&lt;/span&gt;&lt;/b&gt;&lt;/summary&gt;
  338.  &lt;p&gt;Forsberg JA, Potter BK, Cierny III G, Webb L. Diagnosis and management of chronic infection. J Am
  339. Acad Orthop Surg. 2011;19(suppl 1):S8-S19.&amp;nbsp;&lt;/p&gt;&lt;p&gt;Frassica FJ, Frassica DA, McCarthy EF. Orthopaedic pathology. In: Miller MD, ed. Review of Orthopaedics .
  340. 5th ed. Philadelphia, PA: Saunders Elsevier; 2008.&amp;nbsp;&lt;/p&gt;&lt;p&gt;Srinivasan RC, Tolhurst S, Vanderhave KL. Orthopedic surgery. In: Doherty GM, ed. Current Diagnosis
  341. &amp;amp; Treatment: Surgery . 13th ed. New York: McGraw-Hill; 2010.&amp;nbsp;&lt;/p&gt;&lt;p&gt;Tice AD. Osteomyelitis. In: Longo DL, Fauci AS, Kasper DL, Hauser SL, Jameson JL, Loscalzo J, eds.
  342. Harrison’s Principles of Internal Medicine . 18th ed. New York: McGraw-Hill; 2012.&lt;/p&gt;
  343. &lt;/details&gt;&lt;/div&gt;</content><link rel='replies' type='application/atom+xml' href='https://medical-phd.blogspot.com/feeds/1923706066753798097/comments/default' title='Post Comments'/><link rel='replies' type='text/html' href='https://medical-phd.blogspot.com/2022/03/osteomyelitis-case-file.html#comment-form' title='0 Comments'/><link rel='edit' type='application/atom+xml' href='https://www.blogger.com/feeds/3302710210518459416/posts/default/1923706066753798097'/><link rel='self' type='application/atom+xml' href='https://www.blogger.com/feeds/3302710210518459416/posts/default/1923706066753798097'/><link rel='alternate' type='text/html' href='https://medical-phd.blogspot.com/2022/03/osteomyelitis-case-file.html' title='Osteomyelitis Case File'/><author><name>Medical Group</name><uri>http://www.blogger.com/profile/01780656800578397393</uri><email>noreply@blogger.com</email><gd:image rel='http://schemas.google.com/g/2005#thumbnail' width='32' height='32' src='//blogger.googleusercontent.com/img/b/R29vZ2xl/AVvXsEiMuROkBDv6yw4SmxGqRA5BPQhOXSwFld-Gufmy4kUqfpoAGQRl7cv-3naew6wjAnCjRFSe_FKX8z7CdE6cFKxxBpnqBuwUTSr-15xd3mFGcr_wJOX2y73G_zJCGND98Q/s220/Final-logo.jpg'/></author><media:thumbnail xmlns:media="http://search.yahoo.com/mrss/" url="https://blogger.googleusercontent.com/img/b/R29vZ2xl/AVvXsEgONM_robY4_OnJBo-44NU554n8Jq02popS-ReIAr6_iOj8J2O7D9BMDxQGZ1m2YggVPr_qwQSIRTz3JYl1PfqdoL663yrM2qB6171d_w1wAso4Jnv-BRDYcrcGTI6nsFxms39p7zRljz5Socjjfjjv0gnzqEwd3QqqRphP_3UV_ur7t3eYEBUhHBGOog/s72-w342-h640-c/AP-radiograph-of-the-right-foot.jpg" height="72" width="72"/><thr:total>0</thr:total></entry><entry><id>tag:blogger.com,1999:blog-3302710210518459416.post-7780823323538198488</id><published>2022-03-24T14:13:00.002-07:00</published><updated>2022-03-24T14:13:17.654-07:00</updated><category scheme="http://www.blogger.com/atom/ns#" term="Case Files"/><category scheme="http://www.blogger.com/atom/ns#" term="Orthopaedic Surgery Case File"/><title type='text'>Septic Knee Case File</title><content type='html'>&lt;div class=&quot;scrollbox&quot;&gt;
  344.  
  345. &lt;div style=&quot;text-align: left;&quot;&gt;&lt;b&gt;&lt;span style=&quot;font-size: large;&quot;&gt;Septic Knee Case File&lt;/span&gt;&lt;/b&gt;&lt;/div&gt;&lt;div&gt;Eugene C. Toy, MD, Andrew J. Rosenbaum, MD, Timothy T. Roberts, MD, Joshua S. Dines, MD&lt;/div&gt;&lt;div&gt;&lt;br /&gt;&lt;/div&gt;&lt;div&gt;&lt;b&gt;&lt;span style=&quot;color: #017087; font-size: medium;&quot;&gt;CASE 19&lt;/span&gt;&lt;/b&gt;&lt;/div&gt;&lt;div&gt;&lt;div&gt;A 56-year-old obese man presents to the emergency department for a 2-day history of progressive pain, swelling, and inability to bear weight on his right knee. The patient denies a history of recent trauma, prior knee surgery, sick contacts, or similar symptoms in the past. The patient states he has had fevers and chills over this time. His past medical history is notable for poorly controlled non– insulin-dependent diabetes and alcohol abuse. He is febrile with a temperature of 102.7°F and has an elevated heart rate of 110 beats/minute. His blood pressure is 160/95 mmHg, respiratory rate is 16 breaths/min, and oxygen saturation is 100% on room air. Examination of the knee demonstrates a swollen, erythematous, and diffusely tender right knee that is warm to palpation with painful and limited passive range of motion. Radiographs of the knee are negative for an acute fracture or dislocation but do show an effusion. The patient’s laboratory values include a white blood cell (WBC) count of 19,100 cells/μL, erythrocyte sedimentation rate (ESR) of 48 mm/hr, and C-reactive protein (CRP) of 20 mg/L. Blood cultures are pending.&lt;/div&gt;&lt;div&gt;&lt;br /&gt;&lt;/div&gt;&lt;div&gt;&lt;span style=&quot;color: #017087;&quot;&gt;►&lt;/span&gt;&amp;nbsp;What is the most likely diagnosis?&lt;/div&gt;&lt;div&gt;&lt;span style=&quot;color: #017087;&quot;&gt;►&lt;/span&gt;&amp;nbsp;What is your next diagnostic step?&lt;/div&gt;&lt;div&gt;&lt;span style=&quot;color: #017087;&quot;&gt;►&lt;/span&gt;&amp;nbsp;What is the most appropriate initial treatment for this patient?&lt;/div&gt;&lt;/div&gt;&lt;div&gt;&lt;br /&gt;&lt;/div&gt;&lt;div&gt;&lt;br /&gt;&lt;/div&gt;&lt;div&gt;&lt;div&gt;&lt;b&gt;&lt;span style=&quot;font-size: medium;&quot;&gt;ANSWER TO CASE 19:&lt;/span&gt;&lt;/b&gt;&lt;/div&gt;&lt;div&gt;&lt;span style=&quot;background-color: #daebf0; font-size: medium;&quot;&gt;&lt;span&gt;&lt;span style=&quot;background-color: transparent;&quot;&gt;&lt;span style=&quot;color: #017087;&quot;&gt;&lt;b&gt;Septic Knee&lt;/b&gt;&lt;/span&gt;&lt;/span&gt;&lt;b&gt;&amp;nbsp;&amp;nbsp;&lt;/b&gt;&lt;/span&gt;&lt;span style=&quot;font-weight: bold;&quot;&gt;&amp;nbsp;&amp;nbsp; &amp;nbsp;&lt;/span&gt;&lt;span style=&quot;font-weight: bold;&quot;&gt;&amp;nbsp;&amp;nbsp; &amp;nbsp;&lt;/span&gt;&lt;span style=&quot;font-weight: bold;&quot;&gt;&amp;nbsp;&amp;nbsp; &amp;nbsp;&lt;/span&gt;&lt;span style=&quot;font-weight: bold;&quot;&gt;&amp;nbsp;&amp;nbsp; &amp;nbsp;&lt;/span&gt;&lt;span style=&quot;font-weight: bold;&quot;&gt;&amp;nbsp;&amp;nbsp; &amp;nbsp;&lt;/span&gt;&lt;span style=&quot;font-weight: bold;&quot;&gt;&amp;nbsp;&amp;nbsp; &amp;nbsp;&lt;/span&gt;&lt;span style=&quot;font-weight: bold;&quot;&gt;&amp;nbsp; &amp;nbsp;&amp;nbsp;&lt;/span&gt;&lt;/span&gt;&lt;/div&gt;&lt;/div&gt;&lt;div&gt;&lt;br /&gt;&lt;/div&gt;&lt;div&gt;&lt;div&gt;&lt;i&gt;Summary&lt;/i&gt;: A 56-year-old man with medical history significant for obesity, diabetes, and alcohol abuse presents to the emergency department febrile and tachycardic with a 2-day history of right knee pain, swelling, and an inability to bear weight. He denies any trauma or previous right knee surgery. Radiographs show an effusion but no fracture or dislocation of his right knee. Additionally, his WBC, ESR, and CRP are all elevated. Blood cultures are pending.&lt;/div&gt;&lt;div&gt;&lt;ul style=&quot;text-align: left;&quot;&gt;&lt;li&gt;&lt;b&gt;Most likely diagnosis:&lt;/b&gt; Septic arthritis. However, the differential diagnosis also includes crystalline arthropathies (ie, gout, pseudogout) inflammatory arthridites (ie, rheumatoid, psoriatic, and reactive arthritis), tumor, occult fracture, Lyme disease, and osteoarthritis.&lt;/li&gt;&lt;li&gt;&lt;b&gt;Next diagnostic step:&lt;/b&gt; Obtaining joint fluid aspirate for analysis, including a cell count with differential, Gram stain, glucose level, and crystal analysis.&lt;/li&gt;&lt;li&gt;&lt;b&gt;Initial treatment:&lt;/b&gt; Intravenous antibiotic therapy (after obtaining blood cultures and joint fluid aspirate) and operative irrigation and drainage.&lt;/li&gt;&lt;/ul&gt;&lt;/div&gt;&lt;/div&gt;&lt;div&gt;&lt;br /&gt;&lt;/div&gt;&lt;div&gt;&lt;br /&gt;&lt;/div&gt;&lt;div&gt;&lt;div&gt;&lt;b&gt;&lt;span style=&quot;color: #017087; font-size: medium;&quot;&gt;ANALYSIS&lt;/span&gt;&lt;/b&gt;&lt;/div&gt;&lt;div&gt;&lt;b&gt;&lt;span style=&quot;font-size: medium;&quot;&gt;Objectives&lt;/span&gt;&lt;/b&gt;&lt;/div&gt;&lt;/div&gt;&lt;div&gt;&lt;div&gt;&lt;ol style=&quot;text-align: left;&quot;&gt;&lt;li&gt;Be familiar with the multitude of diseases and conditions that can present similar to a septic joint.&lt;/li&gt;&lt;li&gt;Understand the appropriate workup for septic joint.&lt;/li&gt;&lt;li&gt;Know the treatment for a septic joint.&lt;/li&gt;&lt;/ol&gt;&lt;/div&gt;&lt;/div&gt;&lt;div&gt;&lt;br /&gt;&lt;/div&gt;&lt;div&gt;&lt;br /&gt;&lt;/div&gt;&lt;div&gt;&lt;i&gt;&lt;span style=&quot;font-size: medium;&quot;&gt;Considerations&lt;/span&gt;&lt;/i&gt;&lt;/div&gt;&lt;div&gt;&lt;br /&gt;&lt;/div&gt;&lt;div&gt;&lt;div&gt;This 56-year-old male developed a grossly swollen, erythematous, and painful right knee with an inability to bear weight over the prior 48 hours. He is also febrile to 102.7°F and tachycardic. His laboratory values are abnormal and are representative of an acute infectious or inflammatory process. Furthermore, the patient’s history of poorly controlled diabetes makes him vulnerable to systemic infection. The suspicion for a septic right knee in this patient must be high. Given the history of alcohol abuse, gout is a possibility. However, his fevers and chills again favor an infectious cause of his knee pain and swelling. The patient has no history of inflammatory arthridities, such as rheumatoid arthritis, and this diagnosis is unlikely. A diagnosis of Lyme disease should always be considered in the differential for septic arthritis, but is more likely in the setting of a positive tick exposure history where the disease is endemic. Synovial fluid aspirate from this patient’s right knee will likely be consistent with septic arthritis ( Table 19–1 ), at which time intravenous antibiotics must be started and operative irrigation and drainage performed.&lt;/div&gt;&lt;/div&gt;&lt;div&gt;&lt;br /&gt;&lt;/div&gt;&lt;div class=&quot;separator&quot; style=&quot;clear: both; text-align: center;&quot;&gt;&lt;a href=&quot;https://blogger.googleusercontent.com/img/b/R29vZ2xl/AVvXsEiZIFlDxEooZsjbEbHwsxhD2vrnbiM9xUDaD5biXP9YKD1zAh5DTzb8Y2ChsDaj5ihrXIpSRFUTffvYVLKUWQ0MxclGSJ421zVyCeWxfimerHkRXDe8gsAiRwZkGDo9vJr4ICdDumTWP6UYpQ97mnyhB0GVGEMbxjFFdZgB2D9ldj6Qkhv68p1ZoWZBTA/s773/classification-of-synovial-fluid-in-an-adult-knee-joint.jpg&quot; style=&quot;margin-left: 1em; margin-right: 1em;&quot;&gt;&lt;img alt=&quot;classification of synovial fluid in an adult knee joint&quot; border=&quot;0&quot; data-original-height=&quot;476&quot; data-original-width=&quot;773&quot; height=&quot;394&quot; src=&quot;https://blogger.googleusercontent.com/img/b/R29vZ2xl/AVvXsEiZIFlDxEooZsjbEbHwsxhD2vrnbiM9xUDaD5biXP9YKD1zAh5DTzb8Y2ChsDaj5ihrXIpSRFUTffvYVLKUWQ0MxclGSJ421zVyCeWxfimerHkRXDe8gsAiRwZkGDo9vJr4ICdDumTWP6UYpQ97mnyhB0GVGEMbxjFFdZgB2D9ldj6Qkhv68p1ZoWZBTA/w640-h394/classification-of-synovial-fluid-in-an-adult-knee-joint.jpg&quot; title=&quot;classification of synovial fluid in an adult knee joint&quot; width=&quot;640&quot; /&gt;&lt;/a&gt;&lt;/div&gt;&lt;br /&gt;&lt;div&gt;&lt;br /&gt;&lt;/div&gt;&lt;div&gt;&lt;div&gt;&lt;div&gt;&lt;b&gt;&lt;span style=&quot;font-size: medium;&quot;&gt;APPROACH TO:&lt;/span&gt;&lt;/b&gt;&lt;/div&gt;&lt;div&gt;&lt;span style=&quot;background-color: #daebf0; color: #017087; font-size: medium;&quot;&gt;&lt;span&gt;&lt;span style=&quot;background-color: transparent;&quot;&gt;&lt;b&gt;Septic Knee&lt;/b&gt;&lt;/span&gt;&lt;b&gt;&amp;nbsp;&lt;/b&gt;&lt;/span&gt;&lt;span style=&quot;font-weight: bold;&quot;&gt;&amp;nbsp;&amp;nbsp; &amp;nbsp;&lt;/span&gt;&lt;span style=&quot;font-weight: bold;&quot;&gt;&amp;nbsp;&amp;nbsp; &amp;nbsp;&lt;/span&gt;&lt;span style=&quot;font-weight: bold;&quot;&gt;&amp;nbsp;&amp;nbsp; &amp;nbsp;&lt;/span&gt;&lt;span style=&quot;font-weight: bold;&quot;&gt;&amp;nbsp;&amp;nbsp; &amp;nbsp;&lt;/span&gt;&lt;span style=&quot;font-weight: bold;&quot;&gt;&amp;nbsp;&amp;nbsp; &amp;nbsp;&lt;/span&gt;&lt;span style=&quot;font-weight: bold;&quot;&gt;&amp;nbsp;&amp;nbsp; &amp;nbsp;&lt;/span&gt;&lt;span style=&quot;font-weight: bold;&quot;&gt;&amp;nbsp;&amp;nbsp; &amp;nbsp;&lt;/span&gt;&lt;span style=&quot;font-weight: bold;&quot;&gt;&amp;nbsp;&amp;nbsp;&lt;/span&gt;&lt;span style=&quot;font-weight: bold;&quot;&gt;&amp;nbsp;&lt;/span&gt;&lt;/span&gt;&lt;/div&gt;&lt;/div&gt;&lt;div&gt;&lt;br /&gt;&lt;/div&gt;&lt;div&gt;&lt;b&gt;&lt;span style=&quot;color: #017087; font-size: medium;&quot;&gt;DEFINITIONS&lt;/span&gt;&lt;/b&gt;&lt;/div&gt;&lt;/div&gt;&lt;div&gt;&lt;br /&gt;&lt;/div&gt;&lt;div&gt;&lt;div&gt;&lt;b&gt;SEPTIC ARTHRITIS:&lt;/b&gt; An orthopaedic surgical emergency most commonly caused by bacterial seeding of a joint.&lt;/div&gt;&lt;div&gt;&lt;b&gt;&lt;br /&gt;&lt;/b&gt;&lt;/div&gt;&lt;div&gt;&lt;b&gt;OSTEOMYELITIS:&lt;/b&gt; An infection of bone that can cause a septic arthritis or occur as a sequela of it.&lt;/div&gt;&lt;div&gt;&lt;b&gt;&lt;br /&gt;&lt;/b&gt;&lt;/div&gt;&lt;div&gt;&lt;b&gt;ARTHROCENTESIS:&lt;/b&gt; Term describing the process by which synovial fluid is obtained; also referred to as joint aspiration.&lt;/div&gt;&lt;/div&gt;&lt;div&gt;&lt;br /&gt;&lt;/div&gt;&lt;div&gt;&lt;br /&gt;&lt;/div&gt;&lt;div&gt;&lt;div&gt;&lt;b&gt;&lt;span style=&quot;color: #017087; font-size: medium;&quot;&gt;CLINICAL APPROACH&lt;/span&gt;&lt;/b&gt;&lt;/div&gt;&lt;div&gt;&lt;br /&gt;&lt;/div&gt;&lt;div&gt;&lt;i&gt;&lt;span style=&quot;font-size: medium;&quot;&gt;Pathophysiology&lt;/span&gt;&lt;/i&gt;&lt;/div&gt;&lt;div&gt;&lt;br /&gt;&lt;/div&gt;&lt;div&gt;Septic arthritis most commonly occurs in the knee, followed in frequency by the hip, elbow, ankle, and, least commonly, the sternoclavicular joint. It typically occurs after bacterial seeding of the joint, but can also be caused by fungi and viruses. The 3 predominant ways by which this occurs are through direct inoculation from trauma or surgery, contiguous spread from an adjacent osteomyelitis, or in the setting of bacteremia, which is more likely to arise in immunocompromised and hospitalized individuals who have recently undergone invasive procedures.&lt;/div&gt;&lt;div&gt;&lt;br /&gt;&lt;/div&gt;&lt;div&gt;&lt;span&gt;&amp;nbsp;&amp;nbsp; &amp;nbsp;&lt;/span&gt;Septic arthritis causes rapid and irreversible articular hyaline cartilage destruction in the involved joint due to the release of proteolytic enzymes from the bacteria as well as host synovial cells, chondrocytes, and inflammatory cells.&amp;nbsp;&lt;b&gt;Cartilage damage occurs within 8 hours of bacterial inoculation of the joint.&amp;nbsp;&lt;/b&gt;Certain bacteria produce proteolytic enzymes such as collagenase, elastase, hyaluronidase, lipase, and lipoproteinase, which lead to cartilage destruction. Bacterial virulence factors also contribute to cartilage destruction. An example is the coagulase produced by &lt;i&gt;Staphylococcus aureus&lt;/i&gt;, the most common pathogen responsible for bacterial septic arthritis ( Table 19–2 ). It is believed that the coagulase impairs intracapsular vascular supply due to small vessel thrombosis. The subsequent increased intracapsular pressure impedes an effective host immune response.&lt;/div&gt;&lt;/div&gt;&lt;div&gt;&lt;br /&gt;&lt;/div&gt;&lt;div class=&quot;separator&quot; style=&quot;clear: both; text-align: center;&quot;&gt;&lt;a href=&quot;https://blogger.googleusercontent.com/img/b/R29vZ2xl/AVvXsEhKMyPq9NPCLlX9_ebxl9Zqq7oNxlT9IHIvoD3LV_O7IuxQZPD0Uf7S_HkTz8dXhVI_yjPgJSAUvytIBRJOx5ZiYAvvX-UyZR_U1rU_ikbfTLioSwZOhu61I5BK-9F3aPCg1F8fQJXlPgTks29QhXNYUv3xMqPsPh0iSMpHhYkE5ekd2VO5vx-9e9Z6kg/s773/common-organisms-associated-with-septic-arthritis.jpg&quot; style=&quot;margin-left: 1em; margin-right: 1em;&quot;&gt;&lt;img alt=&quot;common organisms associated with septic arthritis&quot; border=&quot;0&quot; data-original-height=&quot;385&quot; data-original-width=&quot;773&quot; height=&quot;318&quot; src=&quot;https://blogger.googleusercontent.com/img/b/R29vZ2xl/AVvXsEhKMyPq9NPCLlX9_ebxl9Zqq7oNxlT9IHIvoD3LV_O7IuxQZPD0Uf7S_HkTz8dXhVI_yjPgJSAUvytIBRJOx5ZiYAvvX-UyZR_U1rU_ikbfTLioSwZOhu61I5BK-9F3aPCg1F8fQJXlPgTks29QhXNYUv3xMqPsPh0iSMpHhYkE5ekd2VO5vx-9e9Z6kg/w640-h318/common-organisms-associated-with-septic-arthritis.jpg&quot; title=&quot;common organisms associated with septic arthritis&quot; width=&quot;640&quot; /&gt;&lt;/a&gt;&lt;/div&gt;&lt;br /&gt;&lt;div&gt;&lt;div&gt;&lt;br /&gt;&lt;/div&gt;&lt;div&gt;&lt;i&gt;&lt;span style=&quot;font-size: medium;&quot;&gt;Clinical Presentation and Workup&lt;/span&gt;&lt;/i&gt;&lt;/div&gt;&lt;div&gt;&lt;br /&gt;&lt;/div&gt;&lt;div&gt;Most patient’s with septic arthritis present with a relatively acute onset of a hot, swollen, and tender joint with restricted and limited range of motion and weightbearing. Systemic signs, such as fevers and chills, may also be present. In patients with concurrent bacteremia, they may appear toxic.&lt;/div&gt;&lt;div&gt;&lt;br /&gt;&lt;/div&gt;&lt;div&gt;&lt;span&gt;&amp;nbsp;&amp;nbsp; &amp;nbsp;&lt;/span&gt;Radiographs are useful for ruling out other acute processes that can cause joint effusions, such as fractures and dislocation. Ultrasound is useful for confirming effusions in large joints such as the hip and for guiding aspirations. Magnetic resonance imaging will also show joint effusions and can also detect adjacent osteomyelitis.&lt;/div&gt;&lt;div&gt;&lt;br /&gt;&lt;/div&gt;&lt;div&gt;&lt;span&gt;&amp;nbsp;&amp;nbsp; &amp;nbsp;&lt;/span&gt;Laboratory findings consistent with a septic joint include an elevated WBC with a left shift, ESR &amp;gt;30 mm/hr, and a CRP &amp;gt;5 mg/L. Although an elevated peripheral WBC count with increased number and percentage of polymorphonuclear leukocytes is &lt;i&gt;indicative&lt;/i&gt; of infection, &lt;b&gt;the absence of an elevated WBC does not rule out&amp;nbsp;&lt;/b&gt;&lt;b&gt;infection.&lt;/b&gt; Both ESR and CRP are acute-phase response markers that are elevated in the presence of infection and/or inflammation. CRP rises within a few hours of infection, reaching values up to 400 mg/L within 36 to 50 hours. It normalizes within 1 week after initiation of appropriate treatment. ESR rises within 2 days of the onset of infection and continues to rise for 3 to 5 days after appropriate antibiotic treatment is instituted. It normalizes after 3 to 4 weeks. Blood cultures should also be obtained, as bacteremia can cause septic arthritis.&lt;/div&gt;&lt;/div&gt;&lt;div&gt;&lt;br /&gt;&lt;/div&gt;&lt;div&gt;&lt;div&gt;&lt;span&gt;&amp;nbsp;&amp;nbsp; &amp;nbsp;&lt;/span&gt;Joint fluid aspiration is the gold standard for diagnosing a septic joint and helps guide antibiotic treatment and determine the need for operative intervention. &lt;b&gt;It&amp;nbsp;&lt;/b&gt;&lt;b&gt;should be analyzed for cell count with a differential, Gram stain, culture, glucose&amp;nbsp;&lt;/b&gt;&lt;b&gt;level, and crystal analysis ( Table 19–1 ).&lt;/b&gt;&lt;/div&gt;&lt;/div&gt;&lt;div&gt;&lt;br /&gt;&lt;/div&gt;&lt;div&gt;&lt;br /&gt;&lt;/div&gt;&lt;div&gt;&lt;b style=&quot;color: #017087;&quot;&gt;&lt;span style=&quot;font-size: medium;&quot;&gt;TREATMENT AND OTHER CONSIDERATIONS&lt;/span&gt;&lt;/b&gt;&lt;/div&gt;&lt;div&gt;&lt;br /&gt;&lt;/div&gt;&lt;div&gt;&lt;div&gt;Only after obtaining blood cultures and a joint aspirate should broad-spectrum, empiric antibiotic therapy commence. With organism speciation and antibiotic sensitivities, more focused antibiotic treatment and duration can be determined. Depending on institutional availability, consultation with infectious disease specialists often proves invaluable in assisting with the management of these oftentimes complicated clinical cases. &lt;b&gt;Because of the rapid and irreversible cartilage&amp;nbsp;&lt;/b&gt;&lt;b&gt;damage, the diagnosis of septic arthritis requires emergent operative irrigation&amp;nbsp;&lt;/b&gt;&lt;b&gt;and drainage.&lt;/b&gt;&lt;/div&gt;&lt;div&gt;&lt;br /&gt;&lt;/div&gt;&lt;div&gt;&lt;span&gt;&amp;nbsp;&amp;nbsp; &amp;nbsp;&lt;/span&gt;A septic joint can quickly lead to bacteremia or a systemic inflammatory response syndrome (SIRS), particularly in the immunocompromised patient. Thus, beyond orthopaedic evaluation and intervention, it is essential as a practitioner to recognize the signs of systemic compromise (ie, tachycardia, tachypnea, hypotension, end organ damage) when evaluating a patient for a septic joint.&lt;/div&gt;&lt;div&gt;&lt;br /&gt;&lt;/div&gt;&lt;div&gt;&lt;span&gt;&amp;nbsp;&amp;nbsp; &amp;nbsp;&lt;/span&gt;Bacterial septic arthritis has a reported mortality of approximately 10%. Patient outcome depends not only on the type of organism involved and the general health of the individual, but also on the speed and decisiveness with which the diagnosis is made.&lt;/div&gt;&lt;/div&gt;&lt;div&gt;&lt;br /&gt;&lt;/div&gt;&lt;div&gt;&lt;br /&gt;&lt;/div&gt;&lt;div&gt;&lt;b&gt;&lt;span style=&quot;color: #017087; font-size: medium;&quot;&gt;&lt;u&gt;COMPREHENSION QUESTIONS&lt;/u&gt;&lt;/span&gt;&lt;/b&gt;&lt;/div&gt;&lt;div&gt;&lt;br /&gt;&lt;/div&gt;&lt;div&gt;&lt;div&gt;19.1 A 70-year-old man with a history of osteoarthritis presents to the emergency department with a 2-day history of right knee pain and swelling. He has no history of similar symptoms. He denies trauma, fever, or pain in any other joints. On examination, he is afebrile. The right knee is swollen, erythematous, and painful with passive range of motion. The remainder of the examination is unremarkable. An arthrocentesis is performed, which shows purulent fluid and a leukocyte count of 90,000/μL with 90% neutrophils. No crystals are seen on polarized microscopy. Gram stain and culture are pending. What is the most likely cause of this patient’s symptoms?&lt;/div&gt;&lt;/div&gt;&lt;blockquote style=&quot;border: none; margin: 0px 0px 0px 40px; padding: 0px; text-align: left;&quot;&gt;&lt;div&gt;&lt;div&gt;A. Gout&lt;/div&gt;&lt;/div&gt;&lt;div&gt;&lt;div&gt;B. Pseudogout&lt;/div&gt;&lt;/div&gt;&lt;div&gt;&lt;div&gt;C. Rheumatoid arthritis&lt;/div&gt;&lt;/div&gt;&lt;div&gt;&lt;div&gt;D. Septic arthritis&lt;/div&gt;&lt;/div&gt;&lt;/blockquote&gt;&lt;div&gt;&lt;br /&gt;&lt;/div&gt;&lt;div&gt;&lt;div&gt;19.2 A 40-year-old intravenous drug user presents to the emergency department with a 2-day history of right knee pain with associated swelling and erythema. The patient is febrile with a holosystolic murmur at the right lower sternal border. Complete blood count reveals a leukocytosis. Blood and synovial fluid cultures are sent, and broad-spectrum antibiotics are started. Synovial fluid analysis is pending. What is the most likely pathogen causing these symptoms?&lt;/div&gt;&lt;/div&gt;&lt;blockquote style=&quot;border: none; margin: 0px 0px 0px 40px; padding: 0px; text-align: left;&quot;&gt;&lt;div&gt;&lt;div&gt;A. &lt;i&gt;Neisseria gonorrhoeae&lt;/i&gt;&lt;/div&gt;&lt;/div&gt;&lt;div&gt;&lt;div&gt;B. &lt;i&gt;Staphylococcus aureus&lt;/i&gt;&lt;/div&gt;&lt;/div&gt;&lt;div&gt;&lt;div&gt;C. &lt;i&gt;Pseudomonas aeruginosa&lt;/i&gt;&lt;/div&gt;&lt;/div&gt;&lt;div&gt;&lt;div&gt;D. &lt;i&gt;Borrelia burgdorferi&lt;/i&gt;&lt;/div&gt;&lt;/div&gt;&lt;/blockquote&gt;&lt;div&gt;&lt;div&gt;&lt;br /&gt;&lt;/div&gt;&lt;div&gt;19.3 A 50-year-old woman diagnosed with rheumatoid arthritis many years ago presents to your clinic with acute on chronic left knee pain. Her knee pain has been well controlled with anti–tumor necrosis factor therapy. She denies recent trauma, fevers, or rash. She is afebrile, and examination of the knee reveals swelling and pain with passive range of motion. What is the next most appropriate step in managing this patient’s symptoms?&lt;/div&gt;&lt;/div&gt;&lt;blockquote style=&quot;border: none; margin: 0px 0px 0px 40px; padding: 0px; text-align: left;&quot;&gt;&lt;div&gt;&lt;div&gt;A. Intraarticular steroidal injection&lt;/div&gt;&lt;/div&gt;&lt;div&gt;&lt;div&gt;B. Add methotrexate&lt;/div&gt;&lt;/div&gt;&lt;div&gt;&lt;div&gt;C. Add nonsteroidal anti-inflammatory drugs (NSAIDs)&lt;/div&gt;&lt;/div&gt;&lt;div&gt;&lt;div&gt;D. Arthrocentesis&lt;/div&gt;&lt;/div&gt;&lt;/blockquote&gt;&lt;div&gt;&lt;br /&gt;&lt;/div&gt;&lt;div&gt;&lt;br /&gt;&lt;/div&gt;&lt;div&gt;&lt;b&gt;&lt;span style=&quot;color: #017087; font-size: medium;&quot;&gt;&lt;u&gt;ANSWERS&lt;/u&gt;&lt;/span&gt;&lt;/b&gt;&lt;/div&gt;&lt;div&gt;&lt;br /&gt;&lt;/div&gt;&lt;div&gt;&lt;div&gt;19.1 &lt;b&gt;D.&lt;/b&gt; Although the patient’s clinical presentation is consistent with a septic joint, one must consider other causes of a joint effusion in the early stages of the workup. However, the arthrocentesis, which yielded gross pus, makes a septic knee the most likely of the answer choices.&lt;/div&gt;&lt;div&gt;&lt;br /&gt;&lt;/div&gt;&lt;div&gt;19.2 &lt;b&gt;B.&lt;/b&gt; Gram-positive bacteria remain the most common cause of septic arthritis.&amp;nbsp;&lt;i&gt;Staphylococcus&lt;/i&gt; aureus accounts for the majority of culture-positive septic arthritis, especially within certain patient subgroups such as hemodialysis patients and intravenous drug abusers. The predominance of S. aureus in septic arthritis has remained unchanged for many years.&lt;/div&gt;&lt;div&gt;&lt;br /&gt;&lt;/div&gt;&lt;div&gt;19.3 &lt;b&gt;D.&lt;/b&gt; The most appropriate next step would be arthrocentesis to rule out a joint infection. Patients with rheumatoid arthritis have consistently accounted for a high percentage of patients with septic arthritis. These patients may be predisposed to septic arthritis because of poor clearance of bacteria from abnormal joints or because of phagocytic defects acquired secondary to drugs or disease.&lt;/div&gt;&lt;/div&gt;&lt;div&gt;&lt;br /&gt;&lt;/div&gt;&lt;div&gt;&lt;b style=&quot;background-color: #4093a5;&quot;&gt;&amp;nbsp; &amp;nbsp;&lt;span style=&quot;font-size: medium;&quot;&gt;&amp;nbsp;&lt;/span&gt;&lt;span style=&quot;color: white;&quot;&gt;&lt;span style=&quot;font-size: medium;&quot;&gt;CLINICAL PEARLS&lt;/span&gt;&lt;span&gt;&amp;nbsp; &amp;nbsp;&amp;nbsp;&lt;/span&gt;&lt;/span&gt;&lt;/b&gt;&lt;/div&gt;&lt;div&gt;&lt;table border=&quot;1&quot; cellpadding=&quot;0&quot; cellspacing=&quot;0&quot; class=&quot;MsoTableGrid&quot; style=&quot;border-collapse: collapse; border: none; mso-border-alt: solid windowtext .5pt; mso-padding-alt: 0in 5.4pt 0in 5.4pt; mso-yfti-tbllook: 1184;&quot;&gt;
  346. &lt;tbody&gt;&lt;tr style=&quot;height: 19.15pt; mso-yfti-firstrow: yes; mso-yfti-irow: 0; mso-yfti-lastrow: yes;&quot;&gt;
  347.  &lt;td style=&quot;background: rgb(221, 237, 239); border-left: solid #017087 1.0pt; border: 1pt solid rgb(64, 147, 165); height: 19.15pt; padding: 0in 5.4pt; width: 433.7pt;&quot; valign=&quot;top&quot; width=&quot;578&quot;&gt;
  348.  &lt;div style=&quot;line-height: normal; margin-bottom: 0in;&quot;&gt;&lt;div style=&quot;line-height: normal; margin-bottom: 0in;&quot;&gt;&lt;br /&gt;&lt;/div&gt;&lt;div style=&quot;line-height: normal; margin-bottom: 0in;&quot;&gt;&lt;span style=&quot;color: #017087;&quot;&gt;►&lt;/span&gt;&amp;nbsp;There is considerable overlap in the clinical presentation of the most common etiologies of acute joint inflammation.&lt;/div&gt;&lt;div style=&quot;line-height: normal; margin-bottom: 0in;&quot;&gt;&lt;span style=&quot;color: #017087;&quot;&gt;&lt;br /&gt;&lt;/span&gt;&lt;/div&gt;&lt;div style=&quot;line-height: normal; margin-bottom: 0in;&quot;&gt;&lt;span style=&quot;color: #017087;&quot;&gt;►&lt;/span&gt;&amp;nbsp;A detailed history and physical exam is vital to eliciting risk factors for septic arthritis.&lt;/div&gt;&lt;div style=&quot;line-height: normal; margin-bottom: 0in;&quot;&gt;&lt;span style=&quot;color: #017087;&quot;&gt;&lt;br /&gt;&lt;/span&gt;&lt;/div&gt;&lt;div style=&quot;line-height: normal; margin-bottom: 0in;&quot;&gt;&lt;span style=&quot;color: #017087;&quot;&gt;►&lt;/span&gt;&amp;nbsp;Always investigate a source of infection in suspected septic arthritis, since the pathophysiology most often involves hematogenous spread to the joint.&lt;/div&gt;&lt;div style=&quot;line-height: normal; margin-bottom: 0in;&quot;&gt;&lt;span style=&quot;color: #017087;&quot;&gt;&lt;br /&gt;&lt;/span&gt;&lt;/div&gt;&lt;div style=&quot;line-height: normal; margin-bottom: 0in;&quot;&gt;&lt;span style=&quot;color: #017087;&quot;&gt;►&lt;/span&gt;&amp;nbsp;Do not depend on serum markers to rule out septic arthritis.&lt;/div&gt;&lt;div style=&quot;line-height: normal; margin-bottom: 0in;&quot;&gt;&lt;span style=&quot;color: #017087;&quot;&gt;&lt;br /&gt;&lt;/span&gt;&lt;/div&gt;&lt;div style=&quot;line-height: normal; margin-bottom: 0in;&quot;&gt;&lt;span style=&quot;color: #017087;&quot;&gt;►&lt;/span&gt;&amp;nbsp;Arthrocentesis is vital to determining the etiology of an acutely erythematous, swollen, and painful joint.&lt;/div&gt;&lt;div style=&quot;line-height: normal; margin-bottom: 0in;&quot;&gt;&lt;span style=&quot;color: #017087;&quot;&gt;&lt;br /&gt;&lt;/span&gt;&lt;/div&gt;&lt;div style=&quot;line-height: normal; margin-bottom: 0in;&quot;&gt;&lt;span style=&quot;color: #017087;&quot;&gt;►&lt;/span&gt;&amp;nbsp;Do not initiate empiric antibiotic treatment without first having obtained both blood and synovial fluid specimens.&lt;/div&gt;&lt;div style=&quot;line-height: normal; margin-bottom: 0in;&quot;&gt;&lt;br /&gt;&lt;/div&gt;&lt;/div&gt;
  349.  &lt;/td&gt;
  350. &lt;/tr&gt;
  351. &lt;/tbody&gt;&lt;/table&gt;&lt;br /&gt;&lt;/div&gt;
  352.  
  353. &lt;details close=&quot;&quot;&gt;
  354.  &lt;summary&gt;&lt;b&gt;&lt;span style=&quot;color: #00788b;&quot;&gt;REFERENCES&lt;/span&gt;&lt;/b&gt;&lt;/summary&gt;
  355.  &lt;p&gt;Ateschrang A, Albrecht D, Schroeter S, Weise K, Dolderer J. Current concepts review: septic arthritis of
  356. the knee pathophysiology, diagnostics, and therapy. Central Eur J Med. 2011;123:191-197.&amp;nbsp;&lt;/p&gt;&lt;p&gt;Carpenter C, Schuur J, Everett W, Pines J. Evidence-based diagnostics: adult septic arthritis. Acad Emerg
  357. Med . 2011;18:781-796.&amp;nbsp;&lt;/p&gt;&lt;p&gt;Lieberman JR, ed. American Academy of Orthopaedic Surgeons Comprehensive Orthopaedic Review.
  358. Rosemont, IL: American Academy of Orthopaedic Surgeons; 2009.&amp;nbsp;&lt;/p&gt;&lt;p&gt;Salter RB. Textbook of Disorders and Injuries of the Musculoskeletal System. 2nd ed. Baltimore, MD:
  359. Williams &amp;amp; Wilkins; 1983:178-181.&lt;/p&gt;
  360. &lt;/details&gt;&lt;/div&gt;</content><link rel='replies' type='application/atom+xml' href='https://medical-phd.blogspot.com/feeds/7780823323538198488/comments/default' title='Post Comments'/><link rel='replies' type='text/html' href='https://medical-phd.blogspot.com/2022/03/septic-knee-case-file.html#comment-form' title='0 Comments'/><link rel='edit' type='application/atom+xml' href='https://www.blogger.com/feeds/3302710210518459416/posts/default/7780823323538198488'/><link rel='self' type='application/atom+xml' href='https://www.blogger.com/feeds/3302710210518459416/posts/default/7780823323538198488'/><link rel='alternate' type='text/html' href='https://medical-phd.blogspot.com/2022/03/septic-knee-case-file.html' title='Septic Knee Case File'/><author><name>Medical Group</name><uri>http://www.blogger.com/profile/01780656800578397393</uri><email>noreply@blogger.com</email><gd:image rel='http://schemas.google.com/g/2005#thumbnail' width='32' height='32' src='//blogger.googleusercontent.com/img/b/R29vZ2xl/AVvXsEiMuROkBDv6yw4SmxGqRA5BPQhOXSwFld-Gufmy4kUqfpoAGQRl7cv-3naew6wjAnCjRFSe_FKX8z7CdE6cFKxxBpnqBuwUTSr-15xd3mFGcr_wJOX2y73G_zJCGND98Q/s220/Final-logo.jpg'/></author><media:thumbnail xmlns:media="http://search.yahoo.com/mrss/" url="https://blogger.googleusercontent.com/img/b/R29vZ2xl/AVvXsEiZIFlDxEooZsjbEbHwsxhD2vrnbiM9xUDaD5biXP9YKD1zAh5DTzb8Y2ChsDaj5ihrXIpSRFUTffvYVLKUWQ0MxclGSJ421zVyCeWxfimerHkRXDe8gsAiRwZkGDo9vJr4ICdDumTWP6UYpQ97mnyhB0GVGEMbxjFFdZgB2D9ldj6Qkhv68p1ZoWZBTA/s72-w640-h394-c/classification-of-synovial-fluid-in-an-adult-knee-joint.jpg" height="72" width="72"/><thr:total>0</thr:total></entry><entry><id>tag:blogger.com,1999:blog-3302710210518459416.post-712576940784628641</id><published>2022-03-24T13:31:00.002-07:00</published><updated>2022-03-24T13:31:15.745-07:00</updated><category scheme="http://www.blogger.com/atom/ns#" term="Case Files"/><category scheme="http://www.blogger.com/atom/ns#" term="Orthopaedic Surgery Case File"/><title type='text'>Developmental Dysplasia of the Hip Case File</title><content type='html'>&lt;div class=&quot;scrollbox&quot;&gt;
  361.  
  362. &lt;div style=&quot;text-align: left;&quot;&gt;&lt;b&gt;&lt;span style=&quot;font-size: large;&quot;&gt;Developmental Dysplasia of the Hip Case File&lt;/span&gt;&lt;/b&gt;&lt;/div&gt;&lt;div&gt;Eugene C. Toy, MD, Andrew J. Rosenbaum, MD, Timothy T. Roberts, MD, Joshua S. Dines, MD&lt;/div&gt;&lt;div&gt;&lt;br /&gt;&lt;/div&gt;&lt;div&gt;&lt;b&gt;&lt;span style=&quot;color: #017087; font-size: medium;&quot;&gt;CASE 18&lt;/span&gt;&lt;/b&gt;&lt;/div&gt;&lt;div&gt;&lt;div&gt;A young couple’s 14-month-old daughter is referred to the pediatric orthopaedic clinic by their general pediatrician because of a limp they noticed when their child ambulates. The parents cannot recall an inciting event or specific day that this began and deny any recent trauma. They state that their daughter does not appear to be in pain when she walks and has had no recent illnesses or sick contacts. The child is in good health and has met all developmental milestones up to this point, including taking her first steps at 9 months and walking independently at 12 months. Of note, this is the parent’s first child. Although the pregnancy was uncomplicated, the child was ultimately delivered at full term via cesarean section as a result of a breech presentation. The parents also state that ultrasonography for “some type of hip problem” was performed at 5 weeks of age and was normal. On exam, the left leg appears shorter than the right. Additionally, a greater degree of hip abduction is noted on the right versus the left.&lt;/div&gt;&lt;div&gt;&lt;span style=&quot;color: #017087;&quot;&gt;&lt;br /&gt;&lt;/span&gt;&lt;/div&gt;&lt;div&gt;&lt;span style=&quot;color: #017087;&quot;&gt;►&lt;/span&gt;&amp;nbsp;What is the most likely diagnosis?&lt;/div&gt;&lt;div&gt;&lt;span style=&quot;color: #017087;&quot;&gt;►&lt;/span&gt;&amp;nbsp;What is the next step in workup?&lt;/div&gt;&lt;div&gt;&lt;span style=&quot;color: #017087;&quot;&gt;►&lt;/span&gt;&amp;nbsp;What is the appropriate treatment of this condition?&lt;/div&gt;&lt;div&gt;&lt;span style=&quot;color: #017087;&quot;&gt;►&lt;/span&gt;&amp;nbsp;What physical exam maneuvers are used to diagnose this condition in newborns?&lt;/div&gt;&lt;/div&gt;&lt;div&gt;&lt;br /&gt;&lt;/div&gt;&lt;div&gt;&lt;br /&gt;&lt;/div&gt;&lt;div&gt;&lt;div&gt;&lt;div&gt;&lt;b&gt;&lt;span style=&quot;font-size: medium;&quot;&gt;ANSWER TO CASE 18:&lt;/span&gt;&lt;/b&gt;&lt;/div&gt;&lt;div&gt;&lt;span style=&quot;background-color: #daebf0; font-size: medium;&quot;&gt;&lt;span&gt;&lt;span style=&quot;background-color: transparent;&quot;&gt;&lt;span style=&quot;color: #017087;&quot;&gt;&lt;b&gt;Developmental Dysplasia of the Hip&lt;/b&gt;&lt;/span&gt;&lt;/span&gt;&lt;b&gt;&amp;nbsp;&amp;nbsp;&lt;/b&gt;&lt;/span&gt;&lt;span style=&quot;font-weight: bold;&quot;&gt;&amp;nbsp;&amp;nbsp; &amp;nbsp;&lt;/span&gt;&lt;span style=&quot;font-weight: bold;&quot;&gt;&amp;nbsp;&amp;nbsp; &amp;nbsp;&lt;/span&gt;&lt;span style=&quot;font-weight: bold;&quot;&gt;&amp;nbsp;&amp;nbsp; &amp;nbsp;&lt;/span&gt;&lt;span style=&quot;font-weight: bold;&quot;&gt;&amp;nbsp;&amp;nbsp; &amp;nbsp;&lt;/span&gt;&lt;span style=&quot;font-weight: bold;&quot;&gt;&amp;nbsp;&amp;nbsp; &amp;nbsp;&lt;/span&gt;&lt;span style=&quot;font-weight: bold;&quot;&gt;&amp;nbsp;&amp;nbsp; &amp;nbsp;&lt;/span&gt;&lt;span style=&quot;font-weight: bold;&quot;&gt;&amp;nbsp; &amp;nbsp;&amp;nbsp;&lt;/span&gt;&lt;/span&gt;&lt;/div&gt;&lt;/div&gt;&lt;div&gt;&lt;br /&gt;&lt;/div&gt;&lt;/div&gt;&lt;div&gt;&lt;div&gt;&lt;i&gt;Summary&lt;/i&gt;: An otherwise healthy 14-month-old girl is brought to the clinic by her parents due to a limp. On exam, the left hip has decreased abduction compared with the right, and the left leg is shorter than the right. The parents state that this is their first child and that she was born full term via cesarean section as a result of a breech presentation. She has met all developmental milestones to this point. They also recall an ultrasound for “some type of hip problem” performed at 5 weeks of age that was negative.&lt;/div&gt;&lt;div&gt;&lt;ul style=&quot;text-align: left;&quot;&gt;&lt;li&gt;&lt;b&gt;Most likely diagnosis:&lt;/b&gt; Developmental dysplasia of the left hip (DDH).&lt;/li&gt;&lt;li&gt;&lt;b&gt;Next step in workup:&lt;/b&gt; Anteroposterior (AP) radiograph of the pelvis.&lt;/li&gt;&lt;li&gt;&lt;b&gt;Treatment:&lt;/b&gt; Closed reduction of the left hip under general anesthesia followed by spica cast placement.&lt;/li&gt;&lt;li&gt;&lt;b&gt;Diagnostic maneuvers in the newborn:&lt;/b&gt; Ortolani and Barlow maneuvers.&lt;/li&gt;&lt;/ul&gt;&lt;/div&gt;&lt;/div&gt;&lt;div&gt;&lt;br /&gt;&lt;/div&gt;&lt;div&gt;&lt;br /&gt;&lt;/div&gt;&lt;div&gt;&lt;div&gt;&lt;b&gt;&lt;span style=&quot;color: #017087; font-size: medium;&quot;&gt;ANALYSIS&lt;/span&gt;&lt;/b&gt;&lt;/div&gt;&lt;div&gt;&lt;b&gt;&lt;span style=&quot;font-size: medium;&quot;&gt;Objectives&lt;/span&gt;&lt;/b&gt;&lt;/div&gt;&lt;/div&gt;&lt;div&gt;&lt;div&gt;&lt;ol style=&quot;text-align: left;&quot;&gt;&lt;li&gt;Understand the “classic” history and presentation of DDH.&lt;/li&gt;&lt;li&gt;Know the diagnostic approach to DDH.&lt;/li&gt;&lt;li&gt;Understand treatment and how it differs based on patient’s age.&lt;/li&gt;&lt;li&gt;Appreciate the complications associated with DDH.&lt;/li&gt;&lt;/ol&gt;&lt;/div&gt;&lt;/div&gt;&lt;div&gt;&lt;br /&gt;&lt;/div&gt;&lt;div&gt;&lt;br /&gt;&lt;/div&gt;&lt;div&gt;&lt;div&gt;&lt;i&gt;&lt;span style=&quot;font-size: medium;&quot;&gt;Considerations&lt;/span&gt;&lt;/i&gt;&lt;/div&gt;&lt;div&gt;&lt;br /&gt;&lt;/div&gt;&lt;div&gt;This 14-month-old child has a constellation of findings concerning for left hip pathology. The differential diagnosis of a limp in this age group must be considered here and includes both infectious and noninfectious causes. The former includes conditions such as a septic hip or osteomyelitis, whereas the latter includes transient synovitis, trauma, and DDH. Per the patient’s family, the patient has had no recent illness or been exposed to any sick contacts, making infectious causes and transient synovitis unlikely. The parents also deny any recent trauma. When entertaining the diagnosis of DDH in this patient, the birth history is significant, as DDH risk factors include female sex, breech presentation, and first-born status; this patient has all of these. The parents also state that an ultrasound done when their daughter was 5 weeks old, presumably to evaluate for DDH, was negative. This should not deter the orthopaedist from a diagnosis of DDH, as diagnosis can be delayed for multiple reasons, including late development of the pathologic changes associated with it. The same can be said for negative physical exam findings for DDH during the initial newborn screening, which is performed on all infants and includes the Ortolani and Barlow tests. Because of the significant long-term complications of DDH, a complete workup must ensue, with treatment initiated as soon as possible.&lt;/div&gt;&lt;/div&gt;&lt;div&gt;&lt;br /&gt;&lt;/div&gt;&lt;div&gt;&lt;br /&gt;&lt;/div&gt;&lt;div&gt;&lt;div&gt;&lt;div&gt;&lt;b&gt;&lt;span style=&quot;font-size: medium;&quot;&gt;APPROACH TO:&lt;/span&gt;&lt;/b&gt;&lt;/div&gt;&lt;div&gt;&lt;span style=&quot;background-color: #daebf0; color: #017087; font-size: medium;&quot;&gt;&lt;span&gt;&lt;span style=&quot;background-color: transparent;&quot;&gt;&lt;b&gt;Developmental Dysplasia of the Hip&lt;/b&gt;&lt;/span&gt;&lt;b&gt;&amp;nbsp;&lt;/b&gt;&lt;/span&gt;&lt;span style=&quot;font-weight: bold;&quot;&gt;&amp;nbsp;&amp;nbsp; &amp;nbsp;&lt;/span&gt;&lt;span style=&quot;font-weight: bold;&quot;&gt;&amp;nbsp;&amp;nbsp; &amp;nbsp;&lt;/span&gt;&lt;span style=&quot;font-weight: bold;&quot;&gt;&amp;nbsp;&amp;nbsp; &amp;nbsp;&lt;/span&gt;&lt;span style=&quot;font-weight: bold;&quot;&gt;&amp;nbsp;&amp;nbsp; &amp;nbsp;&lt;/span&gt;&lt;span style=&quot;font-weight: bold;&quot;&gt;&amp;nbsp;&amp;nbsp; &amp;nbsp;&lt;/span&gt;&lt;span style=&quot;font-weight: bold;&quot;&gt;&amp;nbsp;&amp;nbsp; &amp;nbsp;&lt;/span&gt;&lt;span style=&quot;font-weight: bold;&quot;&gt;&amp;nbsp;&amp;nbsp; &amp;nbsp;&lt;/span&gt;&lt;span style=&quot;font-weight: bold;&quot;&gt;&amp;nbsp;&amp;nbsp; &amp;nbsp;&lt;/span&gt;&lt;span style=&quot;font-weight: bold;&quot;&gt;&amp;nbsp;&amp;nbsp; &amp;nbsp;&lt;/span&gt;&lt;span style=&quot;font-weight: bold;&quot;&gt;&amp;nbsp;&amp;nbsp;&lt;/span&gt;&lt;/span&gt;&lt;/div&gt;&lt;/div&gt;&lt;div&gt;&lt;br /&gt;&lt;/div&gt;&lt;div&gt;&lt;b&gt;&lt;span style=&quot;color: #017087; font-size: medium;&quot;&gt;DEFINITIONS&lt;/span&gt;&lt;/b&gt;&lt;/div&gt;&lt;/div&gt;&lt;div&gt;&lt;br /&gt;&lt;/div&gt;&lt;div&gt;&lt;div&gt;&lt;b&gt;DEVELOPMENTAL DYSPLASIA OF THE HIP:&lt;/b&gt; Term describing a spectrum of developmental abnormalities of the hip joint that lead to subluxation and dislocation, predisposing patients to the development of early degenerative changes.&lt;/div&gt;&lt;div&gt;&lt;b&gt;&lt;br /&gt;&lt;/b&gt;&lt;/div&gt;&lt;div&gt;&lt;b&gt;TRENDELENBURG GAIT:&lt;/b&gt; An abnormal gait associated with hip abductor weakness (ie, gluteus medius). It is characterized by the dropping of the pelvis on the unaffected side of the body at the time of heelstrike on the affected side. This lasts until heelstrike on the unaffected side, at which time lateral protrusion of the affected hip occurs. This gait can be seen in children with DDH.&lt;/div&gt;&lt;div&gt;&lt;b&gt;&lt;br /&gt;&lt;/b&gt;&lt;/div&gt;&lt;div&gt;&lt;b&gt;GALEAZZI TEST:&lt;/b&gt; Test to evaluate for DDH that is performed by flexing the infant’s knees in the supine position so that the ankles touch the buttocks ( Figure 18–1 ). If the knees are not level, the test is considered positive and indicates a possible congenital hip malformation. The test is also known as the &lt;b&gt;Allis sign.&lt;/b&gt;&lt;/div&gt;&lt;/div&gt;&lt;div&gt;&lt;br /&gt;&lt;/div&gt;&lt;div class=&quot;separator&quot; style=&quot;clear: both; text-align: center;&quot;&gt;&lt;a href=&quot;https://blogger.googleusercontent.com/img/b/R29vZ2xl/AVvXsEhR36Gz5bm10kucI9AQraySLzAObsYDXiQAnBA9-1sGSwkYf3Id1Ys3ZXSERlDvdxEm3oh1zqgETAVerdxoEDaVo3eO_CeWEFG6xedF-Off1W7v_Sg-_uAibLHupjepQ31DrHmESPbVwtx4E5zBKfsrk_nO7jb2l7QDr90xbYlpZFPhzV9IdPBKhvxTeQ/s623/Clinical-examination-of-the-hip.jpg&quot; style=&quot;margin-left: 1em; margin-right: 1em;&quot;&gt;&lt;img alt=&quot;Clinical examination of the hip&quot; border=&quot;0&quot; data-original-height=&quot;623&quot; data-original-width=&quot;436&quot; height=&quot;640&quot; src=&quot;https://blogger.googleusercontent.com/img/b/R29vZ2xl/AVvXsEhR36Gz5bm10kucI9AQraySLzAObsYDXiQAnBA9-1sGSwkYf3Id1Ys3ZXSERlDvdxEm3oh1zqgETAVerdxoEDaVo3eO_CeWEFG6xedF-Off1W7v_Sg-_uAibLHupjepQ31DrHmESPbVwtx4E5zBKfsrk_nO7jb2l7QDr90xbYlpZFPhzV9IdPBKhvxTeQ/w448-h640/Clinical-examination-of-the-hip.jpg&quot; title=&quot;Clinical examination of the hip&quot; width=&quot;448&quot; /&gt;&lt;/a&gt;&lt;/div&gt;&lt;div&gt;&lt;div&gt;&lt;b&gt;&lt;span style=&quot;color: #017087;&quot;&gt;Figure 18–1.&lt;/span&gt;&lt;/b&gt; Clinical examination of developmental dislocation of the hip. In all pictures, the child’s&lt;/div&gt;&lt;div&gt;left hip is the abnormal side. ( &lt;b&gt;A&lt;/b&gt;) Asymmetric skin folds; ( &lt;b&gt;B&lt;/b&gt;) Galeazzi test; ( &lt;b&gt;C&lt;/b&gt;) limitation of abduction; (D, E, F ) Ortolani and Barlow tests (see text). (Reproduced, with permission, from Skinner HB. &lt;i&gt;Current Diagnosis &amp;amp; Treatment in Orthopedics &lt;/i&gt;. 4th ed. New York, NY: McGraw-Hill; 2006:Fig. 11-4.)&lt;/div&gt;&lt;/div&gt;&lt;div&gt;&lt;br /&gt;&lt;/div&gt;&lt;div&gt;&lt;br /&gt;&lt;/div&gt;&lt;div&gt;&lt;div&gt;&lt;b&gt;ORTOLANI AND BARLOW SIGNS:&lt;/b&gt; Two maneuvers used to evaluate for DDH. The Ortolani exam is a reduction maneuver that restores normal hip joint anatomy, whereas the Barlow exam is a provocative maneuver that detects an unstable hip ( Figure 18–1 ). These tests may be negative beginning at 3 months of age, despite DDH being present, due to the development of soft-tissue contractures.&lt;/div&gt;&lt;/div&gt;&lt;div&gt;&lt;br /&gt;&lt;/div&gt;&lt;div&gt;&lt;br /&gt;&lt;/div&gt;&lt;div&gt;&lt;b&gt;&lt;span style=&quot;color: #017087; font-size: medium;&quot;&gt;CLINICAL APPROACH&lt;/span&gt;&lt;/b&gt;&lt;/div&gt;&lt;div&gt;&lt;br /&gt;&lt;/div&gt;&lt;div&gt;&lt;div&gt;&lt;i&gt;&lt;span style=&quot;font-size: medium;&quot;&gt;Etiology&lt;/span&gt;&lt;/i&gt;&lt;/div&gt;&lt;div&gt;&lt;br /&gt;&lt;/div&gt;&lt;div&gt;DDH is the most common orthopaedic abnormality in newborns and is the result of a disruption in the normal relationship between the acetabulum and femoral head. The incidence of DDH is reported anywhere from 1 to more than 35 per 1000 live births. DDH is most common in Native Americans and is also seen in whites. It is rare in African Americans. &lt;b&gt;It is considered a multifactorial trait and is more common&amp;nbsp;&lt;/b&gt;&lt;b&gt;in females (females comprise ~80% of cases), firstborn infants, and those&amp;nbsp;&lt;/b&gt;&lt;b&gt;born in the breech presentation.&lt;/b&gt; The risk for DDH is increased 12-fold if a firstdegree relative has it.&lt;/div&gt;&lt;div&gt;&lt;br /&gt;&lt;/div&gt;&lt;div&gt;&lt;span&gt;&amp;nbsp;&amp;nbsp; &amp;nbsp;&lt;/span&gt;The neonatal hip is unstable because of the undeveloped muscle, ligamentous laxity, and easily deformed cartilaginous surfaces comprising the joint. This relative instability, coupled with any positioning in utero that may stretch the hip capsule (as seen with the excessive flexion and adduction in a breech presentation), will predispose the neonate to subluxation or dislocation. &lt;b&gt;The left hip is more frequently&amp;nbsp;&lt;/b&gt;&lt;b&gt;involved because the left femur is adducted against the mother’s lumbosacral&amp;nbsp;&lt;/b&gt;&lt;b&gt;spine when in the left occiput anterior (most common) intrauterine position.&amp;nbsp;&lt;/b&gt;&lt;b&gt;Instability results as less femoral epiphysis is contained by the acetabulum.&lt;/b&gt;&lt;/div&gt;&lt;div&gt;&lt;br /&gt;&lt;/div&gt;&lt;div&gt;&lt;span&gt;&amp;nbsp;&amp;nbsp; &amp;nbsp;&lt;/span&gt;The femoral head displaces posteriorly and superiorly because of the pull of the gluteal and hip flexor muscles. When subluxated, this causes an asymmetric pressure that leads to a dysplastic, progressive flattening of the posterior and superior acetabular rim and medial femoral head. When dislocated, the normal, concentric motion seen at the hip joint is lost, and the joint surfaces become dysplastic with a deformed and shallow configuration.&lt;/div&gt;&lt;/div&gt;&lt;div&gt;&lt;br /&gt;&lt;/div&gt;&lt;div&gt;&lt;br /&gt;&lt;/div&gt;&lt;div&gt;&lt;div&gt;&lt;i&gt;&lt;span style=&quot;font-size: medium;&quot;&gt;Diagnosis&lt;/span&gt;&lt;/i&gt;&lt;/div&gt;&lt;div&gt;&lt;br /&gt;&lt;/div&gt;&lt;div&gt;Physical exam and imaging are the mainstays of diagnosis. &lt;b&gt;In the newborn, the&amp;nbsp;&lt;/b&gt;&lt;b&gt;Ortolani and Barlow tests are performed as part of the standard newborn screens.&amp;nbsp;&lt;/b&gt;In the Ortolani maneuver ( Figure 18–1 ), the examiner will feel the dislocated hip reduce as the flexed hip is abducted while the greater trochanter is lifted anteriorly. A positive sign is a “clunk” that is often heard and felt as the femoral head reduces anteriorly into the acetabulum. In the Barlow maneuver ( Figure 18–1 ), the infant’s thigh is flexed and adducted as the examiner applies a posteriorly directed pressure. It is positive if the hip dislocates from this. Another diagnostic maneuver is the&amp;nbsp;&lt;b&gt;Galeazzi test,&lt;/b&gt; which identifies a dislocated hip via a difference in knee levels when the patient’s knees are flexed in the supine position so that the ankles touch the buttocks. This test can be used for diagnosis in older patients, unlike the Ortolani and Barlow maneuvers, which are rarely helpful after 3 months of age because of the development of soft-tissue contractures. Other physical exam findings consistent with DDH include asymmetric or limited abduction of the dysplastic hip (approximately &amp;lt; 70 degrees from the midline), asymmetric thigh folds and buttock creases, and leg length discrepancies in which the shorter leg is indicative of dysplasia. Once a child can walk, a Trendelenburg gait may be observed, as may a leg length discrepancy with asymmetric toe-walking on the affected side. A waddling, wide-based limp may suggest bilaterally dislocated hips.&lt;/div&gt;&lt;/div&gt;&lt;div&gt;&lt;br /&gt;&lt;/div&gt;&lt;div&gt;&lt;div&gt;&lt;span&gt;&amp;nbsp;&amp;nbsp; &amp;nbsp;&lt;/span&gt;Radiography and ultrasonography are used for confirming the diagnosis. &lt;b&gt;In the&amp;nbsp;&lt;/b&gt;&lt;b&gt;newborn, ultrasound is very sensitive and provides a dynamic view of the cartilaginous&amp;nbsp;&lt;/b&gt;&lt;b&gt;femoral head and acetabulum. However, it should be used only after 4 to&amp;nbsp;&lt;/b&gt;&lt;b&gt;6 weeks of age because of the initial hip laxity associated with birth and concern&amp;nbsp;&lt;/b&gt;&lt;b&gt;for overdiagnosis.&lt;/b&gt; Furthermore, imaging should only be used on infants with a risk factor or physical findings concerning for DDH. Universal screening of newborns with ultrasound is not cost effective and would also lead to overdiagnosis. &lt;b&gt;Plain&amp;nbsp;&lt;/b&gt;&lt;b&gt;radiographs cannot distinguish the cartilaginous components of the hip joint and&amp;nbsp;&lt;/b&gt;&lt;b&gt;should therefore only be performed when children are at least 4 to 6 months old,&amp;nbsp;&lt;/b&gt;&lt;b&gt;which is when the ossific nucleus of the femoral head can be seen on radiographs&amp;nbsp;&lt;/b&gt;&lt;b&gt;(it appears on ultrasonographic images at 12 weeks).&lt;/b&gt; Several lines and angles can be drawn on AP pelvic radiographs to identify DDH ( Figure 18–2 ).&lt;/div&gt;&lt;/div&gt;&lt;div&gt;&lt;br /&gt;&lt;/div&gt;&lt;div class=&quot;separator&quot; style=&quot;clear: both; text-align: center;&quot;&gt;&lt;a href=&quot;https://blogger.googleusercontent.com/img/b/R29vZ2xl/AVvXsEhJrb9yeCBDZMJdQf6hDszLmnpwqa8fmq34ERTCaOrBG4N-boBs-lXB-SfFzdwvASe9Du_W2jyfxAakkbl3ncxiJmk6cgm94T-N2fi8VrRpHt1P95q6UumLmxXWtYCuIXJKRDIHnQUk1xJ16bmpszNjELIWPA6y9fetF-V3auiN79hyZxZgazbdQjp-2w/s432/developmental-dysplasia-of-the-hip.jpg&quot; style=&quot;margin-left: 1em; margin-right: 1em;&quot;&gt;&lt;img alt=&quot;developmental dysplasia of the hip&quot; border=&quot;0&quot; data-original-height=&quot;374&quot; data-original-width=&quot;432&quot; height=&quot;346&quot; src=&quot;https://blogger.googleusercontent.com/img/b/R29vZ2xl/AVvXsEhJrb9yeCBDZMJdQf6hDszLmnpwqa8fmq34ERTCaOrBG4N-boBs-lXB-SfFzdwvASe9Du_W2jyfxAakkbl3ncxiJmk6cgm94T-N2fi8VrRpHt1P95q6UumLmxXWtYCuIXJKRDIHnQUk1xJ16bmpszNjELIWPA6y9fetF-V3auiN79hyZxZgazbdQjp-2w/w400-h346/developmental-dysplasia-of-the-hip.jpg&quot; title=&quot;developmental dysplasia of the hip&quot; width=&quot;400&quot; /&gt;&lt;/a&gt;&lt;/div&gt;&lt;div&gt;&lt;div&gt;&lt;b&gt;&lt;span style=&quot;color: #017087;&quot;&gt;Figure 18–2.&lt;/span&gt;&lt;/b&gt; Lines drawn for measurement in developmental dysplasia of the hip. In the figure, the&lt;/div&gt;&lt;div&gt;patient’s left hip (on the right of the figure) is the subluxated one. (A) Hilgenreiner line is a horizontal&lt;/div&gt;&lt;div&gt;line of the pelvis, drawn between the triradiate cartilages. The proximal femoral ossification center&lt;/div&gt;&lt;div&gt;should be below this line. (B ) Perkins line is a vertical line (perpendicular to Hilgenreiner line) drawn&lt;/div&gt;&lt;div&gt;down from the lateral edge of the acetabulum. The femoral head ossification center, as well as the&lt;/div&gt;&lt;div&gt;medial beak of the proximal metaphysis, should fall medial to this line. (C) The acetabular index&lt;/div&gt;&lt;div&gt;is the angle between Hilgenreiner line and a line joining the acetabular center (triradiate) with the&lt;/div&gt;&lt;div&gt;acetabular edge as it intersects Perkins line. It measures acetabular depth and should be below&lt;/div&gt;&lt;div&gt;30 degrees by 1 year of age and below 25 degrees by 2 years of age. (D) The center-edge angle is the&lt;/div&gt;&lt;div&gt;angle between Perkins line and a line joining the lateral edge of the acetabulum with the center of&lt;/div&gt;&lt;div&gt;the femoral head. It is a measure of lateral subluxation that becomes smaller as the hip subluxates&lt;/div&gt;&lt;div&gt;laterally. Normal is 20 degrees or greater. (Reproduced, with permission, from Skinner HB. &lt;i&gt;Current&lt;/i&gt;&lt;/div&gt;&lt;div&gt;&lt;i&gt;Diagnosis &amp;amp; Treatment in Orthopedics&lt;/i&gt;. 4th ed. New York, NY: McGraw-Hill; 2006:Fig. 11-5.)&lt;/div&gt;&lt;/div&gt;&lt;div&gt;&lt;br /&gt;&lt;/div&gt;&lt;div&gt;&lt;br /&gt;&lt;/div&gt;&lt;div&gt;&lt;b style=&quot;color: #017087;&quot;&gt;&lt;span style=&quot;font-size: medium;&quot;&gt;TREATMENT&lt;/span&gt;&lt;/b&gt;&lt;/div&gt;&lt;div&gt;&lt;br /&gt;&lt;/div&gt;&lt;div&gt;&lt;div&gt;&lt;b&gt;The goal of treatment for DDH is to reestablish a concentric relationship between&amp;nbsp;&lt;/b&gt;&lt;b&gt;the femoral head and acetabulum.&lt;/b&gt; It must begin as soon as the diagnosis is made to decrease the risk of aseptic necrosis and permanent dysplastic changes and depends on the patient’s age. &lt;b&gt;From 0 to 6 months of age, treatment involves splinting&amp;nbsp;&lt;/b&gt;&lt;b&gt;the hips in flexion and abduction, which is achieved with a Pavlik harness&amp;nbsp;&lt;/b&gt;( Figure 18–3 ). In this position, a dislocated hip may spontaneously reduce over several weeks. The harness is used for 6 to 8 weeks or until normal hip anatomy develops. If concentric reduction is not achieved after 2 to 4 weeks, other treatment should be administered, such as closed reduction with spica casting.&lt;b&gt; Closed reduction&amp;nbsp;&lt;/b&gt;&lt;b&gt;under general anesthesia with spica cast application is a treatment option for&amp;nbsp;&lt;/b&gt;&lt;b&gt;ages 6 to 18 months,&lt;/b&gt; as the infants are too large and strong to be controlled by a Pavlik harness. An arthrogram is used during this procedure to confirm a concentric reduction before cast application. The spica cast typically remains on for 12 weeks. If unable to obtain a concentric reduction at this time, open reduction techniques may be necessary. &lt;b&gt;After 18 months of age, the femoral head and acetabulum will&amp;nbsp;&lt;/b&gt;&lt;b&gt;have developed abnormally, and open reduction is required to maintain reduction.&lt;/b&gt;&lt;/div&gt;&lt;/div&gt;&lt;div&gt;&lt;br /&gt;&lt;/div&gt;&lt;div class=&quot;separator&quot; style=&quot;clear: both; text-align: center;&quot;&gt;&lt;a href=&quot;https://blogger.googleusercontent.com/img/b/R29vZ2xl/AVvXsEjPUm-j1X8j6GsajDWaykG5t1jjlBKEGrq7q96FxB6RqspQQuW7RYZh5fpb0g098HbmXr4Bata1JRPdy25DAka22PPKjZXWKLaTbB8baEl0e2NUWSjXI_5BrHv2KIPiBEOXnnaCiTIuiNHriGthEkLhPJ3UgNxyZsh0BAxetcvHbCt80UeG1Wz-a6AnKA/s500/treatment-of-hip-dislocation.jpg&quot; style=&quot;margin-left: 1em; margin-right: 1em;&quot;&gt;&lt;img alt=&quot;treatment of hip dislocation&quot; border=&quot;0&quot; data-original-height=&quot;500&quot; data-original-width=&quot;339&quot; height=&quot;320&quot; src=&quot;https://blogger.googleusercontent.com/img/b/R29vZ2xl/AVvXsEjPUm-j1X8j6GsajDWaykG5t1jjlBKEGrq7q96FxB6RqspQQuW7RYZh5fpb0g098HbmXr4Bata1JRPdy25DAka22PPKjZXWKLaTbB8baEl0e2NUWSjXI_5BrHv2KIPiBEOXnnaCiTIuiNHriGthEkLhPJ3UgNxyZsh0BAxetcvHbCt80UeG1Wz-a6AnKA/w217-h320/treatment-of-hip-dislocation.jpg&quot; title=&quot;treatment of hip dislocation&quot; width=&quot;217&quot; /&gt;&lt;/a&gt;&lt;/div&gt;&lt;div&gt;&lt;div&gt;&lt;b&gt;&lt;span style=&quot;color: #017087;&quot;&gt;Figure 18–3. &lt;/span&gt;&lt;/b&gt;The Pavlik harness, a device used for treatment of hip dislocation, subluxation, and dysplasia. (Reproduced, with permission, from Skinner HB. &lt;i&gt;Current Diagnosis &amp;amp; Treatment in Orthopedics.&lt;/i&gt; 4th ed. New York, NY: McGraw-Hill; 2006:Fig. 11-6.)&lt;/div&gt;&lt;/div&gt;&lt;div&gt;&lt;br /&gt;&lt;/div&gt;&lt;div&gt;&lt;br /&gt;&lt;/div&gt;&lt;div&gt;&lt;div&gt;&lt;b&gt;When DDH is untreated after 2 years of age, bony procedures involving either&amp;nbsp;&lt;/b&gt;&lt;b&gt;the acetabulum (ie, Salter, Pemberton, and Dega osteotomies), femur (ie, femoral&amp;nbsp;&lt;/b&gt;&lt;b&gt;varus derotational osteotomy), or both are required&lt;/b&gt; and attempt to create an environment conducive to remodeling to a more normal joint.&lt;/div&gt;&lt;/div&gt;&lt;div&gt;&lt;br /&gt;&lt;/div&gt;&lt;div&gt;&lt;br /&gt;&lt;/div&gt;&lt;div&gt;&lt;div&gt;&lt;i&gt;&lt;span style=&quot;font-size: medium;&quot;&gt;Complications&lt;/span&gt;&lt;/i&gt;&lt;/div&gt;&lt;div&gt;&lt;br /&gt;&lt;/div&gt;&lt;div&gt;&lt;b&gt;Ischemic necrosis is the most severe complication of DDH&lt;/b&gt; and can occur despite treatment. It results from compression of the vasculature supplying the capital femoral epiphysis and extreme direct pressure on the femoral head. Increased rates of ischemic necrosis are seen with forceful or excessive abduction, repeat surgery, and multiple attempts at closed reduction. Diagnosis is made based on radiographic findings, including the failure of the ossific nucleus of the femoral head to develop within 1 year after reduction, broadening of the femoral neck 1 year after reduction, and an increased density and fragmentation of an ossified femoral head.&lt;/div&gt;&lt;/div&gt;&lt;div&gt;&lt;br /&gt;&lt;/div&gt;&lt;div&gt;&lt;br /&gt;&lt;/div&gt;&lt;div&gt;&lt;b&gt;&lt;span style=&quot;color: #017087; font-size: medium;&quot;&gt;&lt;u&gt;COMPREHENSION QUESTIONS&lt;/u&gt;&lt;/span&gt;&lt;/b&gt;&lt;/div&gt;&lt;div&gt;&lt;br /&gt;&lt;/div&gt;&lt;div&gt;&lt;div&gt;18.1 A 15-month-old girl presents to your office for evaluation of her right hip. Which of the following is most associated with a diagnosis of developmental hip dysplasia?&lt;/div&gt;&lt;/div&gt;&lt;blockquote style=&quot;border: none; margin: 0px 0px 0px 40px; padding: 0px; text-align: left;&quot;&gt;&lt;div&gt;&lt;div&gt;A. Negative Barlow maneuver&lt;/div&gt;&lt;/div&gt;&lt;div&gt;&lt;div&gt;B. Asymmetry of hip abduction&lt;/div&gt;&lt;/div&gt;&lt;div&gt;&lt;div&gt;C. Negative Ortolani maneuver&lt;/div&gt;&lt;/div&gt;&lt;div&gt;&lt;div&gt;D. Increased hip abduction&lt;/div&gt;&lt;/div&gt;&lt;/blockquote&gt;&lt;div&gt;&lt;div&gt;&lt;br /&gt;&lt;/div&gt;&lt;div&gt;18.2 Which of the following is associated with developmental dysplasia of the hip?&lt;/div&gt;&lt;/div&gt;&lt;blockquote style=&quot;border: none; margin: 0px 0px 0px 40px; padding: 0px; text-align: left;&quot;&gt;&lt;div&gt;&lt;div&gt;A. Male sex&lt;/div&gt;&lt;/div&gt;&lt;div&gt;&lt;div&gt;B. Vertex occiput posterior delivery&lt;/div&gt;&lt;/div&gt;&lt;div&gt;&lt;div&gt;C. Polydactyly&lt;/div&gt;&lt;/div&gt;&lt;div&gt;&lt;div&gt;D. First born&lt;/div&gt;&lt;/div&gt;&lt;/blockquote&gt;&lt;div&gt;&lt;div&gt;&lt;br /&gt;&lt;/div&gt;&lt;div&gt;18.3 An otherwise healthy 5-month-old infant diagnosed with developmental dysplasia of the right hip is treated in a Pavlik harness with the hips flexed to 90 degrees and abduction of 50 degrees. An AP radiograph of the pelvis at 4-week follow-up shows that the hip remains dislocated. What is your next step in management?&lt;/div&gt;&lt;/div&gt;&lt;blockquote style=&quot;border: none; margin: 0px 0px 0px 40px; padding: 0px; text-align: left;&quot;&gt;&lt;div&gt;&lt;div&gt;A. Continue treatment as is; the hip should reduce within the next 2 weeks&lt;/div&gt;&lt;/div&gt;&lt;div&gt;&lt;div&gt;B. Closed reduction with hip arthrogram and hip spica casting&lt;/div&gt;&lt;/div&gt;&lt;div&gt;&lt;div&gt;C. Open reduction with femoral shortening osteotomy&lt;/div&gt;&lt;/div&gt;&lt;div&gt;&lt;div&gt;D. Open reduction with pelvic acetabular osteotomy&lt;/div&gt;&lt;/div&gt;&lt;/blockquote&gt;&lt;div&gt;&lt;br /&gt;&lt;/div&gt;&lt;div&gt;&lt;br /&gt;&lt;/div&gt;&lt;div&gt;&lt;b&gt;&lt;span style=&quot;color: #017087; font-size: medium;&quot;&gt;&lt;u&gt;ANSWERS&lt;/u&gt;&lt;/span&gt;&lt;/b&gt;&lt;/div&gt;&lt;div&gt;&lt;br /&gt;&lt;/div&gt;&lt;div&gt;&lt;div&gt;18.1 &lt;b&gt;B.&lt;/b&gt; There are many maneuvers used to diagnose DDH, including the Ortolani and Barlow tests. However, they are rarely helpful after 3 months of age because of the development of soft-tissue contractures. Therefore, in older children one must rely on other exam findings. This includes asymmetric and limited hip abduction on the affected side.&lt;/div&gt;&lt;div&gt;&lt;br /&gt;&lt;/div&gt;&lt;div&gt;18.2 &lt;b&gt;D.&lt;/b&gt; Polydactyly is not associated with DDH. However, being the firstborn, female, and/or breech at birth are.&lt;/div&gt;&lt;div&gt;&lt;br /&gt;&lt;/div&gt;&lt;div&gt;18.3 &lt;b&gt;B.&lt;/b&gt; A 6-month-old infant who has failed treatment of DDH with a Pavlik harness after 2 to 4 weeks should be treated with closed reduction of the hip and spica casting. Continued treatment with the harness may lead to posterior acetabular erosion, a severe and devastating consequence. Both femoral and pelvic osteotomies are unnecessary in this patient and are often reserved for those 2 years of age and older with untreated DDH.&lt;/div&gt;&lt;/div&gt;&lt;div&gt;&lt;br /&gt;&lt;/div&gt;&lt;div&gt;&lt;b style=&quot;background-color: #4093a5;&quot;&gt;&amp;nbsp; &amp;nbsp;&lt;span style=&quot;font-size: medium;&quot;&gt;&amp;nbsp;&lt;/span&gt;&lt;span style=&quot;color: white;&quot;&gt;&lt;span style=&quot;font-size: medium;&quot;&gt;CLINICAL PEARLS&lt;/span&gt;&lt;span&gt;&amp;nbsp; &amp;nbsp;&amp;nbsp;&lt;/span&gt;&lt;/span&gt;&lt;/b&gt;&lt;/div&gt;&lt;div&gt;&lt;table border=&quot;1&quot; cellpadding=&quot;0&quot; cellspacing=&quot;0&quot; class=&quot;MsoTableGrid&quot; style=&quot;border-collapse: collapse; border: none; mso-border-alt: solid windowtext .5pt; mso-padding-alt: 0in 5.4pt 0in 5.4pt; mso-yfti-tbllook: 1184;&quot;&gt;
  363. &lt;tbody&gt;&lt;tr style=&quot;height: 19.15pt; mso-yfti-firstrow: yes; mso-yfti-irow: 0; mso-yfti-lastrow: yes;&quot;&gt;
  364.  &lt;td style=&quot;background: rgb(221, 237, 239); border-left: solid #017087 1.0pt; border: 1pt solid rgb(64, 147, 165); height: 19.15pt; padding: 0in 5.4pt; width: 433.7pt;&quot; valign=&quot;top&quot; width=&quot;578&quot;&gt;
  365.  &lt;div style=&quot;line-height: normal; margin-bottom: 0in;&quot;&gt;&lt;span style=&quot;color: #017087;&quot;&gt;&lt;br /&gt;&lt;/span&gt;&lt;/div&gt;&lt;div style=&quot;line-height: normal; margin-bottom: 0in;&quot;&gt;&lt;span style=&quot;color: #017087;&quot;&gt;►&lt;/span&gt;&amp;nbsp;DDH is considered a multifactorial trait and is more common in females&lt;/div&gt;&lt;div style=&quot;line-height: normal; margin-bottom: 0in;&quot;&gt;(females comprise ~80% of cases), firstborn infants, and those born in&lt;/div&gt;&lt;div style=&quot;line-height: normal; margin-bottom: 0in;&quot;&gt;the breech position.&lt;/div&gt;&lt;div style=&quot;line-height: normal; margin-bottom: 0in;&quot;&gt;&lt;span style=&quot;color: #017087;&quot;&gt;&lt;br /&gt;&lt;/span&gt;&lt;/div&gt;&lt;div style=&quot;line-height: normal; margin-bottom: 0in;&quot;&gt;&lt;span style=&quot;color: #017087;&quot;&gt;►&lt;/span&gt;&amp;nbsp;The risk for DDH is increased 12-fold if a first-degree relative has it.&lt;/div&gt;&lt;div style=&quot;line-height: normal; margin-bottom: 0in;&quot;&gt;&lt;span style=&quot;color: #017087;&quot;&gt;&lt;br /&gt;&lt;/span&gt;&lt;/div&gt;&lt;div style=&quot;line-height: normal; margin-bottom: 0in;&quot;&gt;&lt;span style=&quot;color: #017087;&quot;&gt;►&amp;nbsp;&lt;/span&gt;Diagnosis involves several exam maneuvers, including the Ortolani, Barlow, and Galeazzi tests.&lt;/div&gt;&lt;div style=&quot;line-height: normal; margin-bottom: 0in;&quot;&gt;&lt;span style=&quot;color: #017087;&quot;&gt;&lt;br /&gt;&lt;/span&gt;&lt;/div&gt;&lt;div style=&quot;line-height: normal; margin-bottom: 0in;&quot;&gt;&lt;span style=&quot;color: #017087;&quot;&gt;►&lt;/span&gt;&amp;nbsp;Plain radiographs cannot distinguish the cartilaginous components of the hip joint and should therefore only be performed when children are at least 4 to 6 months old, which is when the ossific nucleus of the femoral head can be seen on radiographs (it appears on ultrasonographic images at 12 weeks).&lt;/div&gt;&lt;div style=&quot;line-height: normal; margin-bottom: 0in;&quot;&gt;&lt;span style=&quot;color: #017087;&quot;&gt;&lt;br /&gt;&lt;/span&gt;&lt;/div&gt;&lt;div style=&quot;line-height: normal; margin-bottom: 0in;&quot;&gt;&lt;span style=&quot;color: #017087;&quot;&gt;►&lt;/span&gt;&amp;nbsp;The goal of treatment for DDH is to reestablish a concentric relationship between the femoral head and acetabulum.&lt;/div&gt;&lt;div style=&quot;line-height: normal; margin-bottom: 0in;&quot;&gt;&lt;span style=&quot;color: #017087;&quot;&gt;&lt;br /&gt;&lt;/span&gt;&lt;/div&gt;&lt;div style=&quot;line-height: normal; margin-bottom: 0in;&quot;&gt;&lt;span style=&quot;color: #017087;&quot;&gt;►&lt;/span&gt;&amp;nbsp;Treatment is age-dependent and includes use of a Pavlik harness when patients are &amp;lt; 6 months old, closed reduction and spica casting when patients are 6 to 18 months old, and open reduction when patients are &amp;gt; 18 months of age. After 24 months of age, reconstructive osteotomies involving the femur and/or acetabulum are often required.&lt;/div&gt;&lt;div style=&quot;line-height: normal; margin-bottom: 0in;&quot;&gt;&lt;span style=&quot;color: #017087;&quot;&gt;&lt;br /&gt;&lt;/span&gt;&lt;/div&gt;&lt;div style=&quot;line-height: normal; margin-bottom: 0in;&quot;&gt;&lt;span style=&quot;color: #017087;&quot;&gt;►&lt;/span&gt;&amp;nbsp;Ischemic necrosis is the most severe complication of DDH and can occur despite treatment.&lt;/div&gt;&lt;div style=&quot;line-height: normal; margin-bottom: 0in;&quot;&gt;&lt;br /&gt;&lt;/div&gt;
  366.  &lt;/td&gt;
  367. &lt;/tr&gt;
  368. &lt;/tbody&gt;&lt;/table&gt;&lt;br /&gt;&lt;/div&gt;
  369.  
  370. &lt;details close=&quot;&quot;&gt;
  371.  &lt;summary&gt;&lt;b&gt;&lt;span style=&quot;color: #00788b;&quot;&gt;REFERENCES&lt;/span&gt;&lt;/b&gt;&lt;/summary&gt;
  372.  &lt;p&gt;Guille JT, Pizzutillo PD, MacEwen GD. Development dysplasia of the hip from birth to six months. J Am
  373. Acad Orthop Surg. 2000;8:232-242.&amp;nbsp;&lt;/p&gt;&lt;p&gt;Vitale MG, Skaggs DL. Developmental dysplasia of the hip from six months to four years of age. J Am
  374. Acad Orthop Surg . 2001;9:401-411.&lt;/p&gt;
  375. &lt;/details&gt;&lt;/div&gt;</content><link rel='replies' type='application/atom+xml' href='https://medical-phd.blogspot.com/feeds/712576940784628641/comments/default' title='Post Comments'/><link rel='replies' type='text/html' href='https://medical-phd.blogspot.com/2022/03/developmental-dysplasia-of-hip-case-file.html#comment-form' title='0 Comments'/><link rel='edit' type='application/atom+xml' href='https://www.blogger.com/feeds/3302710210518459416/posts/default/712576940784628641'/><link rel='self' type='application/atom+xml' href='https://www.blogger.com/feeds/3302710210518459416/posts/default/712576940784628641'/><link rel='alternate' type='text/html' href='https://medical-phd.blogspot.com/2022/03/developmental-dysplasia-of-hip-case-file.html' title='Developmental Dysplasia of the Hip Case File'/><author><name>Medical Group</name><uri>http://www.blogger.com/profile/01780656800578397393</uri><email>noreply@blogger.com</email><gd:image rel='http://schemas.google.com/g/2005#thumbnail' width='32' height='32' src='//blogger.googleusercontent.com/img/b/R29vZ2xl/AVvXsEiMuROkBDv6yw4SmxGqRA5BPQhOXSwFld-Gufmy4kUqfpoAGQRl7cv-3naew6wjAnCjRFSe_FKX8z7CdE6cFKxxBpnqBuwUTSr-15xd3mFGcr_wJOX2y73G_zJCGND98Q/s220/Final-logo.jpg'/></author><media:thumbnail xmlns:media="http://search.yahoo.com/mrss/" url="https://blogger.googleusercontent.com/img/b/R29vZ2xl/AVvXsEhR36Gz5bm10kucI9AQraySLzAObsYDXiQAnBA9-1sGSwkYf3Id1Ys3ZXSERlDvdxEm3oh1zqgETAVerdxoEDaVo3eO_CeWEFG6xedF-Off1W7v_Sg-_uAibLHupjepQ31DrHmESPbVwtx4E5zBKfsrk_nO7jb2l7QDr90xbYlpZFPhzV9IdPBKhvxTeQ/s72-w448-h640-c/Clinical-examination-of-the-hip.jpg" height="72" width="72"/><thr:total>0</thr:total></entry><entry><id>tag:blogger.com,1999:blog-3302710210518459416.post-5183420447322986566</id><published>2022-03-24T12:49:00.003-07:00</published><updated>2022-03-24T12:49:41.015-07:00</updated><category scheme="http://www.blogger.com/atom/ns#" term="Case Files"/><category scheme="http://www.blogger.com/atom/ns#" term="Orthopaedic Surgery Case File"/><title type='text'>Slipped Capital Femoral Epiphysis (SCFE) Case File</title><content type='html'>&lt;div class=&quot;scrollbox&quot;&gt;
  376.  
  377. &lt;div style=&quot;text-align: left;&quot;&gt;&lt;b&gt;&lt;span style=&quot;font-size: large;&quot;&gt;Slipped Capital Femoral Epiphysis (SCFE) Case File&lt;/span&gt;&lt;/b&gt;&lt;/div&gt;&lt;div&gt;Eugene C. Toy, MD, Andrew J. Rosenbaum, MD, Timothy T. Roberts, MD, Joshua S. Dines, MD&lt;/div&gt;&lt;div&gt;&lt;br /&gt;&lt;/div&gt;&lt;div&gt;&lt;b&gt;&lt;span style=&quot;color: #017087; font-size: medium;&quot;&gt;CASE 17&lt;/span&gt;&lt;/b&gt;&lt;/div&gt;&lt;div&gt;&lt;div&gt;A 13-year-old African American male patient presents to the emergency department with pain in his left knee after falling on his side during a soccer game. He is unable to ambulate and is in significant discomfort. His mother states that he has been experiencing several weeks of pain in his left knee before his fall, but x-rays and repeated exams of his knee at his primary care physician’s office had failed to demonstrate any pathology. The patient has no known past medical history. He denies fevers, chills, and recent illness, and he recalls no history of traumatic injury to his left lower extremity. On examination, the child is obese and is comfortable after administration of appropriate analgesia. His left lower extremity is held in slight external rotation. His knee is without effusion and has a full range of painless motion. Motion at the hip, however, is painful, especially with passive internal/external rotation. He is neurovascularly intact throughout his bilateral extremities. Exam of the right lower extremity is unremarkable. Laboratory studies are significant for a thyroid-stimulating hormone (TSH) level of 7.3 mIU/mL (normal 0.6-5.5 mIU/mL). An anteroposterior (AP) pelvis and bilateral frog-leg lateral views are shown in Figures 17–1 and 17–2 , respectively.&lt;/div&gt;&lt;/div&gt;&lt;div&gt;&lt;br /&gt;&lt;/div&gt;&lt;div&gt;&lt;div&gt;&lt;span style=&quot;color: #017087;&quot;&gt;►&lt;/span&gt;&amp;nbsp;What is the most likely diagnosis?&lt;/div&gt;&lt;div&gt;&lt;span style=&quot;color: #017087;&quot;&gt;►&lt;/span&gt;&amp;nbsp;What aspects of this patient’s history put him at risk for this injury?&lt;/div&gt;&lt;div&gt;&lt;span style=&quot;color: #017087;&quot;&gt;►&lt;/span&gt;&amp;nbsp;What is the next step in the management of this patient?&lt;/div&gt;&lt;/div&gt;&lt;div&gt;&lt;br /&gt;&lt;/div&gt;&lt;div class=&quot;separator&quot; style=&quot;clear: both; text-align: center;&quot;&gt;&lt;a href=&quot;https://blogger.googleusercontent.com/img/b/R29vZ2xl/AVvXsEjMXFsXkk_0cUuxjOSZ_Oks3AoZF8I_q4DgfJS9u9cr2J2-2ELFbSISUT7by3wJ3_mD8pODV5_sTR6cr1AQwwNz5aJOM5ERBVSGeTJDVTTe9JfiiIK7sOv38Deg1CzgfPRynyAU0TecAd5JdkNFNOg-nXnUCxAC9Y-s5fl2zB-93E_Nin4HG3Ma63QrPQ/s486/AP-radiograph-of-the-pelvis-1.jpg&quot; imageanchor=&quot;1&quot; style=&quot;margin-left: 1em; margin-right: 1em;&quot;&gt;&lt;img alt=&quot;AP radiograph of the pelvis&quot; border=&quot;0&quot; data-original-height=&quot;400&quot; data-original-width=&quot;486&quot; height=&quot;526&quot; src=&quot;https://blogger.googleusercontent.com/img/b/R29vZ2xl/AVvXsEjMXFsXkk_0cUuxjOSZ_Oks3AoZF8I_q4DgfJS9u9cr2J2-2ELFbSISUT7by3wJ3_mD8pODV5_sTR6cr1AQwwNz5aJOM5ERBVSGeTJDVTTe9JfiiIK7sOv38Deg1CzgfPRynyAU0TecAd5JdkNFNOg-nXnUCxAC9Y-s5fl2zB-93E_Nin4HG3Ma63QrPQ/w640-h526/AP-radiograph-of-the-pelvis-1.jpg&quot; title=&quot;AP radiograph of the pelvis&quot; width=&quot;640&quot; /&gt;&lt;/a&gt;&lt;/div&gt;&lt;div&gt;&lt;b&gt;&lt;span style=&quot;color: #017087;&quot;&gt;Figure 17–1.&lt;/span&gt;&lt;/b&gt; AP radiograph of the pelvis.&lt;/div&gt;&lt;div&gt;&lt;br /&gt;&lt;/div&gt;&lt;div&gt;&lt;br /&gt;&lt;/div&gt;&lt;div class=&quot;separator&quot; style=&quot;clear: both; text-align: center;&quot;&gt;&lt;a href=&quot;https://blogger.googleusercontent.com/img/b/R29vZ2xl/AVvXsEhkRWHpAWiR57T-kb2XKI25tijHcYsgCa4suTe0vBYbcpRKRrMrDw3O_Ne0mHlrICo1ikok70_s_9YfB5zELosSG9R76iF4Bym7Pe2_2zJ3_x64xTxMydbOXem0IP3Zi-EpDV3IZaCZs34sovcvGYckgnnsBodpE4FbmwtlQhqotgM0CBWVpH0_HakLjA/s482/Frog-leg-lateral-radiograph-of-the-pelvis.jpg&quot; imageanchor=&quot;1&quot; style=&quot;margin-left: 1em; margin-right: 1em;&quot;&gt;&lt;img alt=&quot;Frog-leg lateral radiograph of the pelvis&quot; border=&quot;0&quot; data-original-height=&quot;400&quot; data-original-width=&quot;482&quot; height=&quot;532&quot; src=&quot;https://blogger.googleusercontent.com/img/b/R29vZ2xl/AVvXsEhkRWHpAWiR57T-kb2XKI25tijHcYsgCa4suTe0vBYbcpRKRrMrDw3O_Ne0mHlrICo1ikok70_s_9YfB5zELosSG9R76iF4Bym7Pe2_2zJ3_x64xTxMydbOXem0IP3Zi-EpDV3IZaCZs34sovcvGYckgnnsBodpE4FbmwtlQhqotgM0CBWVpH0_HakLjA/w640-h532/Frog-leg-lateral-radiograph-of-the-pelvis.jpg&quot; title=&quot;Frog-leg lateral radiograph of the pelvis&quot; width=&quot;640&quot; /&gt;&lt;/a&gt;&lt;/div&gt;&lt;div&gt;&lt;b&gt;&lt;span style=&quot;color: #017087;&quot;&gt;Figure 17–2.&lt;/span&gt;&lt;/b&gt; Frog-leg lateral radiograph of the pelvis.&lt;/div&gt;&lt;div&gt;&lt;br /&gt;&lt;/div&gt;&lt;div&gt;&lt;br /&gt;&lt;/div&gt;&lt;div&gt;&lt;div&gt;&lt;b&gt;&lt;span style=&quot;font-size: medium;&quot;&gt;ANSWER TO CASE 17:&lt;/span&gt;&lt;/b&gt;&lt;/div&gt;&lt;div&gt;&lt;span style=&quot;background-color: #daebf0; font-size: medium;&quot;&gt;&lt;span&gt;&lt;span style=&quot;background-color: transparent;&quot;&gt;&lt;span style=&quot;color: #017087;&quot;&gt;&lt;b&gt;Slipped Capital Femoral Epiphysis (SCFE)&lt;/b&gt;&lt;/span&gt;&lt;/span&gt;&lt;b&gt;&amp;nbsp;&amp;nbsp;&lt;/b&gt;&lt;/span&gt;&lt;span style=&quot;font-weight: bold;&quot;&gt;&amp;nbsp;&amp;nbsp; &amp;nbsp;&lt;/span&gt;&lt;span style=&quot;font-weight: bold;&quot;&gt;&amp;nbsp;&amp;nbsp; &amp;nbsp;&lt;/span&gt;&lt;span style=&quot;font-weight: bold;&quot;&gt;&amp;nbsp;&amp;nbsp; &amp;nbsp;&lt;/span&gt;&lt;span style=&quot;font-weight: bold;&quot;&gt;&amp;nbsp;&amp;nbsp; &amp;nbsp;&lt;/span&gt;&lt;span style=&quot;font-weight: bold;&quot;&gt;&amp;nbsp;&amp;nbsp; &amp;nbsp;&lt;/span&gt;&lt;span style=&quot;font-weight: bold;&quot;&gt;&amp;nbsp;&amp;nbsp; &amp;nbsp;&lt;/span&gt;&lt;span style=&quot;font-weight: bold;&quot;&gt;&amp;nbsp; &amp;nbsp;&amp;nbsp;&lt;/span&gt;&lt;/span&gt;&lt;/div&gt;&lt;/div&gt;&lt;div&gt;&lt;br /&gt;&lt;/div&gt;&lt;div&gt;&lt;div&gt;&lt;i&gt;Summary&lt;/i&gt;: A 13-year-old obese, African American male presents with several weeks of left lower extremity pain, exacerbated by a recent fall. Exam demonstrates left hip pathology that is referred to the knee. Additionally, the patient has previously undiagnosed hypothyroidism.&lt;/div&gt;&lt;div&gt;&lt;ul style=&quot;text-align: left;&quot;&gt;&lt;li&gt;&lt;b&gt;Most likely diagnosis:&lt;/b&gt; Left slipped capital femoral epiphysis.&lt;/li&gt;&lt;li&gt;&lt;b&gt;Historical risk factors:&lt;/b&gt; Obesity, male, African American, adolescence, with concomitant endocrinopathy (hypothyroidism).&lt;/li&gt;&lt;li&gt;&lt;b&gt;Next appropriate step:&lt;/b&gt; Percutaneous screw fixation.&lt;/li&gt;&lt;/ul&gt;&lt;/div&gt;&lt;/div&gt;&lt;div&gt;&lt;br /&gt;&lt;/div&gt;&lt;div&gt;&lt;br /&gt;&lt;/div&gt;&lt;div&gt;&lt;div&gt;&lt;b&gt;&lt;span style=&quot;color: #017087; font-size: medium;&quot;&gt;ANALYSIS&lt;/span&gt;&lt;/b&gt;&lt;/div&gt;&lt;div&gt;&lt;b&gt;&lt;span style=&quot;font-size: medium;&quot;&gt;Objectives&lt;/span&gt;&lt;/b&gt;&lt;/div&gt;&lt;/div&gt;&lt;div&gt;&lt;div&gt;&lt;ol style=&quot;text-align: left;&quot;&gt;&lt;li&gt;Recognize the presentation of SCFE.&lt;/li&gt;&lt;li&gt;Understand the workup for SCFE and patient population.&lt;/li&gt;&lt;li&gt;Be familiar with the treatment for SCFE and its potential long-term complications.&lt;/li&gt;&lt;/ol&gt;&lt;/div&gt;&lt;/div&gt;&lt;div&gt;&lt;br /&gt;&lt;/div&gt;&lt;div&gt;&lt;br /&gt;&lt;/div&gt;&lt;div&gt;&lt;div&gt;&lt;i&gt;&lt;span style=&quot;font-size: medium;&quot;&gt;Considerations&lt;/span&gt;&lt;/i&gt;&lt;/div&gt;&lt;div&gt;&lt;br /&gt;&lt;/div&gt;&lt;div&gt;This is an overweight, African American male adolescent with a history of pain in his left knee. Up to 46% of patients with SCFE will present initially with complaints of distal thigh or knee pain. It is essential to recognize that hip pathology can often be referred to the distal thigh or knee. Complaints of knee pain should warrant complete and thorough physical exam, and typically radiographic evaluation, of the ipsilateral hip. Patients with SCFE typically present with an externally rotated, subtly foreshortened lower extremity, with a markedly decrease range of painful internal rotation.&lt;/div&gt;&lt;div&gt;&lt;br /&gt;&lt;/div&gt;&lt;div&gt;&lt;span&gt;&amp;nbsp;&amp;nbsp; &amp;nbsp;&lt;/span&gt;Hip x-rays should include, at a minimum, an AP view of the pelvis as well as lateral views of each femoral head and neck, typically achieved through a frog-leg lateral view. Although this patient’s SCFE is clearly visible on both AP and lateral films, early SCFE lesions are often subtle and are generally apparent on lateral views before they are obvious on the AP.&lt;/div&gt;&lt;div&gt;&lt;br /&gt;&lt;/div&gt;&lt;div&gt;&lt;span&gt;&amp;nbsp;&amp;nbsp; &amp;nbsp;&lt;/span&gt;Patients with newly diagnosed SCFE lesions should be immediately made nonweightbearing on the effected extremity. Complete workup for SCFE secondary to underlying medical conditions should be performed, including initial laboratory testing for TSH, a complete metabolic panel, and a complete blood count. Patients undergoing potential operative fixation should always receive blood type and screening and coagulation (prothrombin time/partial thromboplastin time) studies.&lt;/div&gt;&lt;/div&gt;&lt;div&gt;&lt;br /&gt;&lt;/div&gt;&lt;div&gt;&lt;br /&gt;&lt;/div&gt;&lt;div&gt;&lt;div&gt;&lt;div&gt;&lt;b&gt;&lt;span style=&quot;font-size: medium;&quot;&gt;APPROACH TO:&lt;/span&gt;&lt;/b&gt;&lt;/div&gt;&lt;div&gt;&lt;span style=&quot;background-color: #daebf0; color: #017087; font-size: medium;&quot;&gt;&lt;span&gt;&lt;span style=&quot;background-color: transparent;&quot;&gt;&lt;b&gt;Slipped Capital Femoral Epiphysis (SCFE)&lt;/b&gt;&lt;/span&gt;&lt;b&gt;&amp;nbsp;&lt;/b&gt;&lt;/span&gt;&lt;span style=&quot;font-weight: bold;&quot;&gt;&amp;nbsp;&amp;nbsp; &amp;nbsp;&lt;/span&gt;&lt;span style=&quot;font-weight: bold;&quot;&gt;&amp;nbsp;&amp;nbsp; &amp;nbsp;&lt;/span&gt;&lt;span style=&quot;font-weight: bold;&quot;&gt;&amp;nbsp;&amp;nbsp; &amp;nbsp;&lt;/span&gt;&lt;span style=&quot;font-weight: bold;&quot;&gt;&amp;nbsp;&amp;nbsp; &amp;nbsp;&lt;/span&gt;&lt;span style=&quot;font-weight: bold;&quot;&gt;&amp;nbsp;&amp;nbsp; &amp;nbsp;&lt;/span&gt;&lt;span style=&quot;font-weight: bold;&quot;&gt;&amp;nbsp;&amp;nbsp; &amp;nbsp;&lt;/span&gt;&lt;span style=&quot;font-weight: bold;&quot;&gt;&amp;nbsp;&amp;nbsp; &amp;nbsp;&lt;/span&gt;&lt;span style=&quot;font-weight: bold;&quot;&gt;&amp;nbsp;&amp;nbsp;&lt;/span&gt;&lt;/span&gt;&lt;/div&gt;&lt;/div&gt;&lt;div&gt;&lt;br /&gt;&lt;/div&gt;&lt;div&gt;&lt;b&gt;&lt;span style=&quot;color: #017087; font-size: medium;&quot;&gt;DEFINITIONS&lt;/span&gt;&lt;/b&gt;&lt;/div&gt;&lt;/div&gt;&lt;div&gt;&lt;br /&gt;&lt;/div&gt;&lt;div&gt;&lt;div&gt;&lt;b&gt;OSTEONECROSIS:&lt;/b&gt; The cellular death of bone, typically resulting from a prolonged disruption of blood supply.&lt;/div&gt;&lt;div&gt;&lt;b&gt;&lt;br /&gt;&lt;/b&gt;&lt;/div&gt;&lt;div&gt;&lt;b&gt;STABLE SCFE:&lt;/b&gt; Defined simply by the patient’s ability to ambulate, even with crutch assistance. Less than 10% of patients with a stable SCFE develop osteonecrosis.&lt;/div&gt;&lt;div&gt;&lt;b&gt;&lt;br /&gt;&lt;/b&gt;&lt;/div&gt;&lt;div&gt;&lt;b&gt;UNSTABLE SCFE:&lt;/b&gt; Defined as SCFE in patients who are unable to ambulate. These patients have a high incidence of osteonecrosis, upward of 50%.&lt;/div&gt;&lt;/div&gt;&lt;div&gt;&lt;br /&gt;&lt;/div&gt;&lt;div&gt;&lt;br /&gt;&lt;/div&gt;&lt;div&gt;&lt;b&gt;&lt;span style=&quot;color: #017087; font-size: medium;&quot;&gt;CLINICAL APPROACH&lt;/span&gt;&lt;/b&gt;&lt;/div&gt;&lt;div&gt;&lt;br /&gt;&lt;/div&gt;&lt;div&gt;&lt;div&gt;SCFE (often pronounced “skiffy”) is a disorder of adolescence in which a fracture— or, technically, a disruption—occurs through the growth plate of the femoral head. The epiphysis, or region of developing bone above the growth plate (physis), is therefore mobile and tends to “slip” from the neck of the femur under the repetitive loading of body weight. The “slipping” process is actually a misnomer, however, as&amp;nbsp;&lt;b&gt;it is not technically the epiphysis that slips from the femur, but rather the femur&amp;nbsp;&lt;/b&gt;&lt;b&gt;that displaces from the anatomically stable epiphysis.&lt;/b&gt;&lt;/div&gt;&lt;/div&gt;&lt;div&gt;&lt;br /&gt;&lt;/div&gt;&lt;div&gt;&lt;br /&gt;&lt;/div&gt;&lt;div&gt;&lt;div&gt;&lt;i&gt;&lt;span style=&quot;font-size: medium;&quot;&gt;Epidemiology&lt;/span&gt;&lt;/i&gt;&lt;/div&gt;&lt;div&gt;&lt;br /&gt;&lt;/div&gt;&lt;div&gt;SCFE typically affects children between &lt;b&gt;10 and 17 years of age,&lt;/b&gt; occurring at an average of 13.4 years for boys and 12.2 years for girls. Its prevalence in the United States is 10 in 100,000 people annually. SCFE has a slight male-to-female predominance of 3:2 and occurs at increased incidences of 2.2 in patients of African ancestry, 4.5 in those of Pacific Islander ancestry, and 0.1 for North African and Indian subcontinental ancestry, versus 1.0 for white controls. Racial differences are closely related to average adolescent body weights.&lt;/div&gt;&lt;/div&gt;&lt;div&gt;&lt;br /&gt;&lt;/div&gt;&lt;div&gt;&lt;br /&gt;&lt;/div&gt;&lt;div&gt;&lt;div&gt;&lt;i&gt;&lt;span style=&quot;font-size: medium;&quot;&gt;Pathogenesis&lt;/span&gt;&lt;/i&gt;&lt;/div&gt;&lt;div&gt;&lt;br /&gt;&lt;/div&gt;&lt;div&gt;As stated, SCFE is a failure of the physis, with separation of the epiphysis from the proximal femoral metaphysis. Biomechanical factors such as obesity, femoral retroversion (increased posterior angulation of the femoral neck), and increased physeal obliquity (an increasingly angulated growth plate that is vulnerable to shearing forces when body weight is loaded) all contribute to physeal weakening. In younger children, the physis is protected by a perichondral ring that resists shearing forces. This protective ring weakens in adolescence, however, and increases the risk of SCFE. SCFE occurs during puberty, when rapid cellular expansion is occurring at the&amp;nbsp;&lt;b&gt;physeal zone of hypertrophy.&lt;/b&gt; Failure is thought to occur at this slightly weakened zone of rapid, immature expansion. Finally, hormonal and endocrine changes are associated with SCFE, but the mechanisms by which they contribute to the disease process are not fully understood. There are data to show that &lt;b&gt;hypothyroidism,&amp;nbsp;&lt;/b&gt;&lt;b&gt;growth hormone supplementation, and hypogonadism increase one’s risk of SCFE.&lt;/b&gt;&lt;/div&gt;&lt;/div&gt;&lt;div&gt;&lt;br /&gt;&lt;/div&gt;&lt;div&gt;&lt;br /&gt;&lt;/div&gt;&lt;div&gt;&lt;div&gt;&lt;i&gt;&lt;span style=&quot;font-size: medium;&quot;&gt;Radiology&lt;/span&gt;&lt;/i&gt;&lt;/div&gt;&lt;div&gt;&lt;br /&gt;&lt;/div&gt;&lt;div&gt;The direction of a typical “slip” causes the femur to fall into &lt;b&gt;varus, extension, and&amp;nbsp;&lt;/b&gt;&lt;b&gt;external rotation.&lt;/b&gt; Typically, the epiphysis tends to move &lt;b&gt;posteriorly first,&lt;/b&gt; a translation that is &lt;b&gt;most apparent on frog-leg lateral views.&lt;/b&gt; With the femurs externally rotated, abducted, and flexed, an unobstructed lateral view of the femoral neck shows early posterior displacement of the epiphysis. For this reason, frog-leg lateral images are considered most sensitive for the diagnosis.&lt;/div&gt;&lt;div&gt;&lt;br /&gt;&lt;/div&gt;&lt;div&gt;&lt;span&gt;&amp;nbsp;&amp;nbsp; &amp;nbsp;&lt;/span&gt;Several radiographic measurements can be made to diagnose and grade SCFE. The Klein line, or a line drawn parallel to the superior femoral neck, should intersect the epiphysis in normal individuals. In patients with advanced slips, however, the Klein line contacts the edge of, or is superior to, the migrating epiphysis. The metaphyseal blanch sign of Steel is a blurring of the proximal femoral metaphysis that may be visualized on an AP pelvis film. This is caused by overlapping of the normal metaphysis with the posteriorly displaced epiphysis.&lt;/div&gt;&lt;/div&gt;&lt;div&gt;&lt;br /&gt;&lt;/div&gt;&lt;div&gt;&lt;br /&gt;&lt;/div&gt;&lt;div&gt;&lt;b style=&quot;color: #017087;&quot;&gt;&lt;span style=&quot;font-size: medium;&quot;&gt;CLASSIFICATIONS&lt;/span&gt;&lt;/b&gt;&lt;/div&gt;&lt;div&gt;&lt;br /&gt;&lt;/div&gt;&lt;div&gt;&lt;div&gt;Although several classification schemes exist, the most practical and prognostically relevant classification divides the disease into stable and unstable SCFE. SCFE stability is defined simply by whether or not the patient is able to tolerate weightbearing on the affected extremity. Stability includes those who are able to partially weight bear with crutches. Patients with unstable SCFE are so uncomfortable with movement of the hip that they refuse to ambulate. &lt;b&gt;Up to 50% of&amp;nbsp;&lt;/b&gt;&lt;b&gt;patients with unstable SCFE have an incidence of osteonecrosis of the femoral&amp;nbsp;&lt;/b&gt;&lt;b&gt;head.&lt;/b&gt;&lt;/div&gt;&lt;/div&gt;&lt;div&gt;&lt;br /&gt;&lt;/div&gt;&lt;div&gt;&lt;br /&gt;&lt;/div&gt;&lt;div&gt;&lt;b style=&quot;color: #017087;&quot;&gt;&lt;span style=&quot;font-size: medium;&quot;&gt;TREATMENT&lt;/span&gt;&lt;/b&gt;&lt;/div&gt;&lt;div&gt;&lt;br /&gt;&lt;/div&gt;&lt;div&gt;&lt;div&gt;Intervention should occur as soon as the diagnosis is made. For patients with mild or moderate stable disease, &lt;b&gt;in situ fixation is the method of choice.&lt;/b&gt; Attempts to forcefully reduce the deformity are not recommended; however, sometimes the slip will spontaneously reduce or improve when the patient is positioned for surgery. Regardless, the goal or treatment is to stabilize the slipped epiphysis, as remodeling often occurs and patients can tolerate a certain degree of residual external rotation.&amp;nbsp;&lt;b&gt;Single-screw percutaneous fixation is the most common mode of treatment,&amp;nbsp;&lt;/b&gt;although double-screw fixation techniques are sometimes performed. For singlescrew fixation, the goal is to place the screw through the middle of the epiphysis and perpendicular to the growth plate. Patients with stable slips are typically allowed to bear weight after fixation. Bilateral fixation may be indicated in patients with underlying endocrinopathies, even if the contralateral hip is asymptomatic and without radiographic evidence of the disease. &lt;b&gt;Although controversial, some&amp;nbsp;&lt;/b&gt;&lt;b&gt;authors also advocate for the prophylactic pinning of the contralateral hip in&amp;nbsp;&lt;/b&gt;&lt;b&gt;children less than 10 years of age with unilateral SCFE or in those with open&amp;nbsp;&lt;/b&gt;&lt;b&gt;triradiate cartilage.&lt;/b&gt; The impetus behind prophylactic fixation of the unaffected hip stems from the elevated rates at which young children and patients with endocrinopathies develop bilateral disease.&lt;/div&gt;&lt;/div&gt;&lt;div&gt;&lt;br /&gt;&lt;/div&gt;&lt;div&gt;&lt;div&gt;&lt;span&gt;&amp;nbsp;&amp;nbsp; &amp;nbsp;&lt;/span&gt;In patients with unstable SCFE, there is significant controversy over whether reduction manipulations should be employed versus in situ fixation, whether capsulotomy or arthrocentesis should be performed versus no joint decompression, and whether single- versus multiple-screw fixation techniques should be used. Most surgeons advocate relatively urgent treatment in these patients, as greater than 24 hours between acute injury and fixation may be associated with increased risk of osteonecrosis. A common treatment regimen for unstable SCFE involves single-screw fixation after joint aspiration to relieve intracapsular pressure and promote vascular perfusion. These patients are generally made nonweightbearing with crutches for 6 to 8 weeks postoperatively.&lt;/div&gt;&lt;/div&gt;&lt;div&gt;&lt;br /&gt;&lt;/div&gt;&lt;div&gt;&lt;br /&gt;&lt;/div&gt;&lt;div&gt;&lt;div&gt;&lt;i&gt;&lt;span style=&quot;font-size: medium;&quot;&gt;Complications&lt;/span&gt;&lt;/i&gt;&lt;/div&gt;&lt;div&gt;&lt;br /&gt;&lt;/div&gt;&lt;div&gt;&lt;b&gt;Osteonecrosis&lt;/b&gt; is a severe, debilitating complication, for which risk is increased with unstable SCFE, delayed surgical fixation of acute unstable SCFE, attempted reduction manipulations, and improper placement of pins, specifically in the posteriorsuperior femoral neck, leading to disruption of vasculature. Osteonecrosis is initially managed with nonweightbearing, nonsteroidal anti-inflammatory drugs, and gentle range-of-motion exercises. When severe, however, reconstructive intervention may be necessary. Slip progression after initial fixation is another potential complication that, fortunately, occurs in only 1% to 2% of patients after single-screw fixation. Although double-screw fixation may theoretically reduce this complication, elevated potential risks of osteonecrosis with multiple-screw fixation favors singlescrew techniques for most surgeons.&lt;/div&gt;&lt;/div&gt;&lt;div&gt;&lt;br /&gt;&lt;/div&gt;&lt;div&gt;&lt;br /&gt;&lt;/div&gt;&lt;div&gt;&lt;b&gt;&lt;span style=&quot;color: #017087; font-size: medium;&quot;&gt;&lt;u&gt;COMPREHENSION QUESTIONS&lt;/u&gt;&lt;/span&gt;&lt;/b&gt;&lt;/div&gt;&lt;div&gt;&lt;br /&gt;&lt;/div&gt;&lt;div&gt;&lt;div&gt;17.1 A 13-year-old boy is referred to your office with a right-sided SCFE after workup over several weeks by his pediatrician for insidious right knee pain. On exam, he has obligatory external rotation with hip flexion and is refusing to bear weight. His TSH is 8.5 mIU/mL. Which of the following is the most appropriate treatment for this patient?&lt;/div&gt;&lt;/div&gt;&lt;blockquote style=&quot;border: none; margin: 0px 0px 0px 40px; padding: 0px; text-align: left;&quot;&gt;&lt;div&gt;&lt;div&gt;A. Percutaneous pinning of the right hip&lt;/div&gt;&lt;/div&gt;&lt;div&gt;&lt;div&gt;B. Open reduction and capsulotomy of the right hip with plate and screw fixation&lt;/div&gt;&lt;/div&gt;&lt;div&gt;&lt;div&gt;C. Percutaneous pinning of the left hip&lt;/div&gt;&lt;/div&gt;&lt;div&gt;&lt;div&gt;D. Percutaneous pinning of bilateral hips&lt;/div&gt;&lt;/div&gt;&lt;div&gt;&lt;div&gt;E. Administration of levothyroxine with follow-up TSH levels in outpatient setting and bed rest until resolution of pain&lt;/div&gt;&lt;/div&gt;&lt;/blockquote&gt;&lt;div&gt;&lt;br /&gt;&lt;/div&gt;&lt;div&gt;&lt;div&gt;17.2 Through which of the following physeal zones does the disruption in SCFE typically occur?&lt;/div&gt;&lt;/div&gt;&lt;blockquote style=&quot;border: none; margin: 0px 0px 0px 40px; padding: 0px; text-align: left;&quot;&gt;&lt;div&gt;&lt;div&gt;A. Reserve zone&lt;/div&gt;&lt;/div&gt;&lt;div&gt;&lt;div&gt;B. Proliferative zone&lt;/div&gt;&lt;/div&gt;&lt;div&gt;&lt;div&gt;C. Hypertrophic zone&lt;/div&gt;&lt;/div&gt;&lt;div&gt;&lt;div&gt;D. Zone of provisional calcification&lt;/div&gt;&lt;/div&gt;&lt;/blockquote&gt;&lt;div&gt;&lt;br /&gt;&lt;/div&gt;&lt;div&gt;&lt;div&gt;17.3 An 8-year-old obese girl of Pacific Island ancestry is referred to your office for a right-sided unstable SCFE lesion after a fall while playing kick-ball. She had no previous pain in this extremity. On examination, she has an externally rotated right lower extremity that is severely painful with passive range of motion. Frog-leg lateral x-rays show a displaced right SCFE lesion with a normal-appearing left femoral head. Which of the following factors is indication to prophylactically fix this patient’s left hip?&lt;/div&gt;&lt;/div&gt;&lt;blockquote style=&quot;border: none; margin: 0px 0px 0px 40px; padding: 0px; text-align: left;&quot;&gt;&lt;div&gt;&lt;div&gt;A. Obesity&lt;/div&gt;&lt;/div&gt;&lt;div&gt;&lt;div&gt;B. Age&lt;/div&gt;&lt;/div&gt;&lt;div&gt;&lt;div&gt;C. Unstable nature of the right-sided SCFE&lt;/div&gt;&lt;/div&gt;&lt;div&gt;&lt;div&gt;D. Acute, traumatic right-sided SCFE without preexisting symptoms&lt;/div&gt;&lt;/div&gt;&lt;div&gt;&lt;div&gt;E. Pacific Island ancestry&lt;/div&gt;&lt;/div&gt;&lt;/blockquote&gt;&lt;div&gt;&lt;br /&gt;&lt;/div&gt;&lt;div&gt;&lt;br /&gt;&lt;/div&gt;&lt;div&gt;&lt;b&gt;&lt;span style=&quot;color: #017087; font-size: medium;&quot;&gt;&lt;u&gt;ANSWERS&lt;/u&gt;&lt;/span&gt;&lt;/b&gt;&lt;/div&gt;&lt;div&gt;&lt;br /&gt;&lt;/div&gt;&lt;div&gt;&lt;div&gt;17.1 &lt;b&gt;D.&lt;/b&gt; Bilateral percutaneous screw fixation is appropriate in this patient due to the patient’s history of undiagnosed hypothyroidism. His right hip, most urgently, requires fixation, but the presence of endocrinopathy is a generally accepted indication for additional fixation of the unaffected contralateral side. Management of hypothyroidism is important in this patient, but bed rest is generally an unsuitable treatment for SCFE.&lt;/div&gt;&lt;div&gt;&lt;br /&gt;&lt;/div&gt;&lt;div&gt;17.2 &lt;b&gt;C.&lt;/b&gt; SCFE classically occurs through the zone of hypertrophy, a region of rapid cellular expansion during the adolescent growth spurt that is especially vulnerable to shearing injury.&lt;/div&gt;&lt;div&gt;&lt;br /&gt;&lt;/div&gt;&lt;div&gt;17.3 &lt;b&gt;B.&lt;/b&gt; Indications to prophylactically fix the contralateral hip in a patient with a unilateral SCFE lesion are limited to patients with obvious endocrinopathies and those younger than 10 years of age or with open triradiate cartilage. This patient’s age is reason enough to strongly consider preemptively pinning the left hip. Her obesity and ethnicity are risk factors for SCFE; however, they are not indications for prophylactic fixation. The unstable and acute nature of her SCFE lesions does not necessarily correlate with a need for prophylactic intervention.&lt;/div&gt;&lt;/div&gt;&lt;div&gt;&lt;br /&gt;&lt;/div&gt;&lt;div&gt;&lt;b style=&quot;background-color: #4093a5;&quot;&gt;&amp;nbsp; &amp;nbsp;&lt;span style=&quot;font-size: medium;&quot;&gt;&amp;nbsp;&lt;/span&gt;&lt;span style=&quot;color: white;&quot;&gt;&lt;span style=&quot;font-size: medium;&quot;&gt;CLINICAL PEARLS&lt;/span&gt;&lt;span&gt;&amp;nbsp; &amp;nbsp;&amp;nbsp;&lt;/span&gt;&lt;/span&gt;&lt;/b&gt;&lt;/div&gt;&lt;div&gt;&lt;table border=&quot;1&quot; cellpadding=&quot;0&quot; cellspacing=&quot;0&quot; class=&quot;MsoTableGrid&quot; style=&quot;border-collapse: collapse; border: none; mso-border-alt: solid windowtext .5pt; mso-padding-alt: 0in 5.4pt 0in 5.4pt; mso-yfti-tbllook: 1184;&quot;&gt;
  378. &lt;tbody&gt;&lt;tr style=&quot;height: 19.15pt; mso-yfti-firstrow: yes; mso-yfti-irow: 0; mso-yfti-lastrow: yes;&quot;&gt;
  379.  &lt;td style=&quot;background: #DDEDEF; border-left: solid #017087 1.0pt; border: solid #4093A5 1.0pt; height: 19.15pt; padding: 0in 5.4pt 0in 5.4pt; width: 433.7pt;&quot; valign=&quot;top&quot; width=&quot;578&quot;&gt;
  380.  &lt;div style=&quot;line-height: normal; margin-bottom: .0001pt; margin-bottom: 0in;&quot;&gt;&lt;span style=&quot;color: #017087;&quot;&gt;&lt;br /&gt;&lt;/span&gt;&lt;/div&gt;&lt;div style=&quot;line-height: normal; margin-bottom: .0001pt; margin-bottom: 0in;&quot;&gt;&lt;span style=&quot;color: #017087;&quot;&gt;►&lt;/span&gt;&amp;nbsp;Remember that hip pathology can often be referred to the distal thigh or knee and thus a complete exam of both joints is essential. Almost 50% of SCFE patients present with distal thigh or knee pain.&lt;/div&gt;&lt;div style=&quot;line-height: normal; margin-bottom: .0001pt; margin-bottom: 0in;&quot;&gt;&lt;span style=&quot;color: #017087;&quot;&gt;&lt;br /&gt;&lt;/span&gt;&lt;/div&gt;&lt;div style=&quot;line-height: normal; margin-bottom: .0001pt; margin-bottom: 0in;&quot;&gt;&lt;span style=&quot;color: #017087;&quot;&gt;►&lt;/span&gt;&amp;nbsp;Percutaneous single-screw in situ fixation is the treatment of choice for SCFE.&lt;/div&gt;&lt;div style=&quot;line-height: normal; margin-bottom: .0001pt; margin-bottom: 0in;&quot;&gt;&lt;span style=&quot;color: #017087;&quot;&gt;&lt;br /&gt;&lt;/span&gt;&lt;/div&gt;&lt;div style=&quot;line-height: normal; margin-bottom: .0001pt; margin-bottom: 0in;&quot;&gt;&lt;span style=&quot;color: #017087;&quot;&gt;►&lt;/span&gt;&amp;nbsp;Prophylactic fixation of the contralateral hip, regardless of whether or not it is symptomatic, is generally advocated in patients with SCFE secondary to endocrinopathies or in SCFE patients younger than 10 years at presentation. Admittedly, this is controversial.&lt;/div&gt;&lt;div style=&quot;line-height: normal; margin-bottom: .0001pt; margin-bottom: 0in;&quot;&gt;&lt;br /&gt;&lt;/div&gt;
  381.  &lt;/td&gt;
  382. &lt;/tr&gt;
  383. &lt;/tbody&gt;&lt;/table&gt;&lt;/div&gt;
  384.  
  385. &lt;details&gt;
  386.  &lt;summary&gt;&lt;b&gt;&lt;span style=&quot;color: #00788b;&quot;&gt;REFERENCES&lt;/span&gt;&lt;/b&gt;&lt;/summary&gt;
  387.  &lt;p&gt;Aronsson DD, Loder RT, Breur GJ, Weinstein SL. Slipped capital femoral epiphysis: current concepts.
  388. J Am Acad Orthop Surg. 2006;14:666-679.
  389. Flynn, JM, ed. Hip, pelvis, and femur disorders: pediatrics. In: Orthopaedic Knowledge Update: Ten.
  390. Rosemont, IL: American Academy of Orthopaedic Surgeons; 2011:739-752.&lt;/p&gt;
  391. &lt;/details&gt;&lt;/div&gt;</content><link rel='replies' type='application/atom+xml' href='https://medical-phd.blogspot.com/feeds/5183420447322986566/comments/default' title='Post Comments'/><link rel='replies' type='text/html' href='https://medical-phd.blogspot.com/2022/03/slipped-capital-femoral-epiphysis-scfe.html#comment-form' title='0 Comments'/><link rel='edit' type='application/atom+xml' href='https://www.blogger.com/feeds/3302710210518459416/posts/default/5183420447322986566'/><link rel='self' type='application/atom+xml' href='https://www.blogger.com/feeds/3302710210518459416/posts/default/5183420447322986566'/><link rel='alternate' type='text/html' href='https://medical-phd.blogspot.com/2022/03/slipped-capital-femoral-epiphysis-scfe.html' title='Slipped Capital Femoral Epiphysis (SCFE) Case File'/><author><name>Medical Group</name><uri>http://www.blogger.com/profile/01780656800578397393</uri><email>noreply@blogger.com</email><gd:image rel='http://schemas.google.com/g/2005#thumbnail' width='32' height='32' src='//blogger.googleusercontent.com/img/b/R29vZ2xl/AVvXsEiMuROkBDv6yw4SmxGqRA5BPQhOXSwFld-Gufmy4kUqfpoAGQRl7cv-3naew6wjAnCjRFSe_FKX8z7CdE6cFKxxBpnqBuwUTSr-15xd3mFGcr_wJOX2y73G_zJCGND98Q/s220/Final-logo.jpg'/></author><media:thumbnail xmlns:media="http://search.yahoo.com/mrss/" url="https://blogger.googleusercontent.com/img/b/R29vZ2xl/AVvXsEjMXFsXkk_0cUuxjOSZ_Oks3AoZF8I_q4DgfJS9u9cr2J2-2ELFbSISUT7by3wJ3_mD8pODV5_sTR6cr1AQwwNz5aJOM5ERBVSGeTJDVTTe9JfiiIK7sOv38Deg1CzgfPRynyAU0TecAd5JdkNFNOg-nXnUCxAC9Y-s5fl2zB-93E_Nin4HG3Ma63QrPQ/s72-w640-h526-c/AP-radiograph-of-the-pelvis-1.jpg" height="72" width="72"/><thr:total>0</thr:total></entry><entry><id>tag:blogger.com,1999:blog-3302710210518459416.post-5088681108440372635</id><published>2022-03-24T12:08:00.004-07:00</published><updated>2022-03-24T12:09:25.452-07:00</updated><category scheme="http://www.blogger.com/atom/ns#" term="Case Files"/><category scheme="http://www.blogger.com/atom/ns#" term="Orthopaedic Surgery Case File"/><title type='text'>Child Abuse Case File</title><content type='html'>&lt;div class=&quot;scrollbox&quot;&gt;
  392.  
  393. &lt;div style=&quot;text-align: left;&quot;&gt;&lt;b&gt;&lt;span style=&quot;font-size: large;&quot;&gt;Child Abuse Case File&lt;/span&gt;&lt;/b&gt;&lt;/div&gt;&lt;div&gt;Eugene C. Toy, MD, Andrew J. Rosenbaum, MD, Timothy T. Roberts, MD, Joshua S. Dines, MD&lt;/div&gt;&lt;div&gt;&lt;br /&gt;&lt;/div&gt;&lt;div&gt;&lt;b&gt;&lt;span style=&quot;color: #017087; font-size: medium;&quot;&gt;CASE&amp;nbsp;&lt;/span&gt;&lt;/b&gt;&lt;b&gt;&lt;span style=&quot;color: #017087; font-size: medium;&quot;&gt;16&lt;/span&gt;&lt;/b&gt;&lt;/div&gt;&lt;div&gt;&lt;div&gt;An 8-month-old boy is brought to the emergency department by his parents with a 48-hour history of left thigh swelling and irritability. They also state that the child has been barely moving his left lower extremity over this time. The parents do not recall any injury or recent trauma. After further discussion, the mother does recall that approximately 2 days ago, the child fell from the changing table (~3 feet off the ground) onto the carpeted floor of their apartment. On examination, the child is developmentally appropriate but irritable. His left thigh is swollen and exquisitely tender. He will not move his leg. You also note several other bruises on the child’s right arm. Radiographs are taken ( Figure 16–1 ).&lt;/div&gt;&lt;/div&gt;&lt;div&gt;&lt;br /&gt;&lt;/div&gt;&lt;div class=&quot;separator&quot; style=&quot;clear: both; text-align: center;&quot;&gt;&lt;a href=&quot;https://blogger.googleusercontent.com/img/b/R29vZ2xl/AVvXsEhHmr1g96XxdHyl6TAw2L_yc6w_ArnQnRqII8K99115SFFDE8epCwTjYhfps5xpYImGzTsQgQ0fSgJlK8hWUU3zx9dkC8epjDz2E4wL76flto46hd3Vqrn2bPw89nXo9fDDs7JrZO3s7Ibq0m_KshoYFF3IRoY3xo-Lg8fi6j1qD6DxbpCE5SmUMBmyCg/s562/proximal-tibia-fracture.jpg&quot; style=&quot;margin-left: 1em; margin-right: 1em;&quot;&gt;&lt;img alt=&quot;proximal tibia fracture&quot; border=&quot;0&quot; data-original-height=&quot;562&quot; data-original-width=&quot;334&quot; height=&quot;640&quot; src=&quot;https://blogger.googleusercontent.com/img/b/R29vZ2xl/AVvXsEhHmr1g96XxdHyl6TAw2L_yc6w_ArnQnRqII8K99115SFFDE8epCwTjYhfps5xpYImGzTsQgQ0fSgJlK8hWUU3zx9dkC8epjDz2E4wL76flto46hd3Vqrn2bPw89nXo9fDDs7JrZO3s7Ibq0m_KshoYFF3IRoY3xo-Lg8fi6j1qD6DxbpCE5SmUMBmyCg/w380-h640/proximal-tibia-fracture.jpg&quot; title=&quot;proximal tibia fracture&quot; width=&quot;380&quot; /&gt;&lt;/a&gt;&lt;/div&gt;&lt;div&gt;&lt;div&gt;&lt;b&gt;&lt;span style=&quot;color: #017087;&quot;&gt;Figure 16–1.&lt;/span&gt;&lt;/b&gt; Spiral femur and proximal tibia fracture. Note displaced spiral femur fracture with faint&lt;/div&gt;&lt;div&gt;callus formation and more solid (older) periosteal reaction of the proximal tibia. Child abuse is likely&lt;/div&gt;&lt;div&gt;because there are 2 injuries that occurred at different times and no treatment was obtained. (Reproduced, with permission, from Knoop KJ, Stack LB, Storrow AB, et al. &lt;i&gt;Atlas of Emergency Medicine&lt;/i&gt;. 3rd ed. New York, NY: McGraw-Hill; 2009:Fig. 15-26. Photo contributor: Alan E. Oestreich, MD.)&lt;/div&gt;&lt;/div&gt;&lt;div&gt;&lt;br /&gt;&lt;/div&gt;&lt;div&gt;&lt;div&gt;&lt;span style=&quot;color: #017087;&quot;&gt;►&lt;/span&gt;&amp;nbsp;What is the most likely diagnosis?&lt;/div&gt;&lt;div&gt;&lt;span style=&quot;color: #017087;&quot;&gt;►&lt;/span&gt;&amp;nbsp;What is your next diagnostic step?&lt;/div&gt;&lt;/div&gt;&lt;div&gt;&lt;br /&gt;&lt;/div&gt;&lt;div&gt;&lt;br /&gt;&lt;/div&gt;&lt;div&gt;&lt;div&gt;&lt;div&gt;&lt;b&gt;&lt;span style=&quot;font-size: medium;&quot;&gt;ANSWER TO CASE 16:&lt;/span&gt;&lt;/b&gt;&lt;/div&gt;&lt;div&gt;&lt;span style=&quot;background-color: #daebf0; font-size: medium;&quot;&gt;&lt;span&gt;&lt;span style=&quot;background-color: transparent;&quot;&gt;&lt;span style=&quot;color: #017087;&quot;&gt;&lt;b&gt;Child Abuse&lt;/b&gt;&lt;/span&gt;&lt;/span&gt;&lt;b&gt;&amp;nbsp;&lt;/b&gt;&lt;/span&gt;&lt;span style=&quot;font-weight: bold;&quot;&gt;&amp;nbsp;&amp;nbsp; &amp;nbsp;&lt;/span&gt;&lt;span style=&quot;font-weight: bold;&quot;&gt;&amp;nbsp;&amp;nbsp; &amp;nbsp;&lt;/span&gt;&lt;span style=&quot;font-weight: bold;&quot;&gt;&amp;nbsp;&amp;nbsp; &amp;nbsp;&lt;/span&gt;&lt;span style=&quot;font-weight: bold;&quot;&gt;&amp;nbsp;&amp;nbsp; &amp;nbsp;&lt;/span&gt;&lt;span style=&quot;font-weight: bold;&quot;&gt;&amp;nbsp;&amp;nbsp; &amp;nbsp;&lt;/span&gt;&lt;span style=&quot;font-weight: bold;&quot;&gt;&amp;nbsp;&amp;nbsp; &amp;nbsp;&lt;/span&gt;&lt;span style=&quot;font-weight: bold;&quot;&gt;&amp;nbsp; &amp;nbsp;&amp;nbsp;&lt;/span&gt;&lt;/span&gt;&lt;/div&gt;&lt;/div&gt;&lt;div&gt;&lt;br /&gt;&lt;/div&gt;&lt;/div&gt;&lt;div&gt;&lt;div&gt;&lt;i&gt;Summary&lt;/i&gt;: An 8-month-old boy presents with a 48-hour history of irritability and left thigh swelling. The parents at first deny any recent trauma but then recall the child falling from his changing table 2 days prior. On examination, the thigh is swollen and the child is irritable. You also note bruising of the right upper extremity. Radiographs of the left femur reveal a spiral fracture and an older proximal tibia fracture.&lt;/div&gt;&lt;/div&gt;&lt;div&gt;&lt;div&gt;&lt;ul style=&quot;text-align: left;&quot;&gt;&lt;li&gt;&lt;b&gt;Most likely diagnosis:&lt;/b&gt; Physical abuse.&lt;/li&gt;&lt;li&gt;&lt;b&gt;Next step:&lt;/b&gt; Skeletal survey.&lt;/li&gt;&lt;/ul&gt;&lt;/div&gt;&lt;/div&gt;&lt;div&gt;&lt;br /&gt;&lt;/div&gt;&lt;div&gt;&lt;br /&gt;&lt;/div&gt;&lt;div&gt;&lt;div&gt;&lt;b&gt;&lt;span style=&quot;color: #017087; font-size: medium;&quot;&gt;ANALYSIS&lt;/span&gt;&lt;/b&gt;&lt;/div&gt;&lt;div&gt;&lt;b&gt;&lt;span style=&quot;font-size: medium;&quot;&gt;Objectives&lt;/span&gt;&lt;/b&gt;&lt;/div&gt;&lt;/div&gt;&lt;div&gt;&lt;div&gt;&lt;ol style=&quot;text-align: left;&quot;&gt;&lt;li&gt;Recognize orthopaedic injuries associated with child abuse.&lt;/li&gt;&lt;li&gt;Know the proper workup for suspected child maltreatment.&lt;/li&gt;&lt;li&gt;Understand the importance of reporting suspected child abuse.&lt;/li&gt;&lt;/ol&gt;&lt;/div&gt;&lt;/div&gt;&lt;div&gt;&lt;br /&gt;&lt;/div&gt;&lt;div&gt;&lt;br /&gt;&lt;/div&gt;&lt;div&gt;&lt;i&gt;&lt;span style=&quot;font-size: medium;&quot;&gt;Considerations&lt;/span&gt;&lt;/i&gt;&lt;/div&gt;&lt;div&gt;&lt;div&gt;&lt;br /&gt;&lt;/div&gt;&lt;div&gt;The proposed mechanism of injury, a fall from a changing table only 3 feet off the ground, is unlikely to cause a femur fracture. Furthermore, the &lt;b&gt;spiral nature&lt;/b&gt; of the fracture is inconsistent with the mother’s story, as these fractures typically result from twisting injuries and not simple falls. Additional concerns include the bruising observed on the child’s right upper extremity, the older proximal tibia fracture, and the mother’s delay in seeking medical attention for 2 days from the initial accident. &lt;b&gt;Many pediatricians who frequently encounter child abuse&amp;nbsp;&lt;/b&gt;&lt;b&gt;advocate for clinicians to assume that any long bone fracture in a child under&amp;nbsp;&lt;/b&gt;&lt;b&gt;3 years of age is child abuse until proven otherwise.&lt;/b&gt; Therefore, it is imperative that after a complete skeletal survey of this child to assess for other bony injuries, this case be reported to children’s protective services for suspected abuse.&lt;/div&gt;&lt;/div&gt;&lt;div&gt;&lt;br /&gt;&lt;/div&gt;&lt;div&gt;&lt;br /&gt;&lt;/div&gt;&lt;div&gt;&lt;div&gt;&lt;div&gt;&lt;b&gt;&lt;span style=&quot;font-size: medium;&quot;&gt;APPROACH TO:&lt;/span&gt;&lt;/b&gt;&lt;/div&gt;&lt;div&gt;&lt;span style=&quot;background-color: #daebf0; color: #017087; font-size: medium;&quot;&gt;&lt;span&gt;&lt;span style=&quot;background-color: transparent;&quot;&gt;&lt;b&gt;Child Abuse&lt;/b&gt;&lt;/span&gt;&lt;b&gt;&amp;nbsp;&lt;/b&gt;&lt;/span&gt;&lt;span style=&quot;font-weight: bold;&quot;&gt;&amp;nbsp;&amp;nbsp; &amp;nbsp;&lt;/span&gt;&lt;span style=&quot;font-weight: bold;&quot;&gt;&amp;nbsp;&amp;nbsp; &amp;nbsp;&lt;/span&gt;&lt;span style=&quot;font-weight: bold;&quot;&gt;&amp;nbsp;&amp;nbsp; &amp;nbsp;&lt;/span&gt;&lt;span style=&quot;font-weight: bold;&quot;&gt;&amp;nbsp;&amp;nbsp; &amp;nbsp;&lt;/span&gt;&lt;span style=&quot;font-weight: bold;&quot;&gt;&amp;nbsp;&amp;nbsp; &amp;nbsp;&lt;/span&gt;&lt;span style=&quot;font-weight: bold;&quot;&gt;&amp;nbsp;&amp;nbsp; &amp;nbsp;&lt;/span&gt;&lt;span style=&quot;font-weight: bold;&quot;&gt;&amp;nbsp;&amp;nbsp; &amp;nbsp;&lt;/span&gt;&lt;span style=&quot;font-weight: bold;&quot;&gt;&amp;nbsp;&amp;nbsp; &amp;nbsp;&lt;/span&gt;&lt;span style=&quot;font-weight: bold;&quot;&gt;&amp;nbsp;&amp;nbsp; &amp;nbsp;&lt;/span&gt;&lt;span style=&quot;font-weight: bold;&quot;&gt;&amp;nbsp;&amp;nbsp;&lt;/span&gt;&lt;/span&gt;&lt;/div&gt;&lt;/div&gt;&lt;div&gt;&lt;br /&gt;&lt;/div&gt;&lt;div&gt;&lt;b&gt;&lt;span style=&quot;color: #017087; font-size: medium;&quot;&gt;DEFINITIONS&lt;/span&gt;&lt;/b&gt;&lt;/div&gt;&lt;/div&gt;&lt;div&gt;&lt;br /&gt;&lt;/div&gt;&lt;div&gt;&lt;div&gt;&lt;b&gt;SKELETAL SURVEY:&lt;/b&gt; Dedicated radiographs of every anatomic region of the body used to look for signs of previous and/or acute injuries when abuse is suspected. Anteroposterior (AP) and lateral images of the axial skeleton and frontal projections of each extremity are included.&lt;/div&gt;&lt;/div&gt;&lt;div&gt;&lt;br /&gt;&lt;/div&gt;&lt;div&gt;&lt;div&gt;&lt;b&gt;CHILDREN’S PROTECTIVE SERVICES:&lt;/b&gt; Local governmental agency responsible for conducting civil investigations of children alleged to have been abused or neglected.&lt;/div&gt;&lt;div&gt;&lt;b&gt;&lt;br /&gt;&lt;/b&gt;&lt;/div&gt;&lt;div&gt;&lt;b&gt;THE CHILD ABUSE PREVENTION AND TREATMENT ACT:&lt;/b&gt; Defines child abuse as “at a minimum, any act or failure to act resulting in imminent risk of serious harm, death, serious physical or emotional harm, sexual abuse, or exploitation of a child by a parent or caretaker who is responsible for the child’s welfare.”&lt;/div&gt;&lt;/div&gt;&lt;div&gt;&lt;br /&gt;&lt;/div&gt;&lt;div&gt;&lt;br /&gt;&lt;/div&gt;&lt;div&gt;&lt;b&gt;&lt;span style=&quot;color: #017087; font-size: medium;&quot;&gt;CLINICAL APPROACH&lt;/span&gt;&lt;/b&gt;&lt;/div&gt;&lt;div&gt;&lt;br /&gt;&lt;/div&gt;&lt;div&gt;&lt;div&gt;Each year more than 1 million children are the victims of abuse or neglect, and more than 1200 of them die as a result of the abuse. Although health care providers are mandated to report all cases of suspected abuse as dictated by civil statutes, the diagnosis is rarely straightforward. It often requires consideration of sociobehavioral factors and clinical findings. Fractures are the second most common presentation of physical abuse after skin lesions. It is therefore pivotal that the orthopaedist understand the signs of nonaccidental trauma to increase the likelihood of recognition and proper management.&lt;/div&gt;&lt;/div&gt;&lt;div&gt;&lt;br /&gt;&lt;/div&gt;&lt;div&gt;&lt;br /&gt;&lt;/div&gt;&lt;div&gt;&lt;div&gt;&lt;i&gt;&lt;span style=&quot;font-size: medium;&quot;&gt;General Considerations&lt;/span&gt;&lt;/i&gt;&lt;/div&gt;&lt;div&gt;&lt;br /&gt;&lt;/div&gt;&lt;div&gt;A thorough history and complete general and orthopaedic examination is necessary in cases of suspected child abuse. An abused child may be either &lt;b&gt;overly&amp;nbsp;&lt;/b&gt;&lt;b&gt;passive&lt;/b&gt; or &lt;b&gt;overly aggressive&lt;/b&gt;. Irritability, hyperactivity, and destructive behavior may all be observed on exam. Age-appropriate questions must be asked. Leading questions should be avoided. The parents’ behavior may also provide clues regarding whether this is a case of abuse. &lt;b&gt;Red flags&lt;/b&gt; include a vague history that lacks detail, providing an injury mechanism inconsistent with the physical findings, or parents who are hostile or too casual during questioning ( Table 16–1 ).&lt;/div&gt;&lt;/div&gt;&lt;div&gt;&lt;br /&gt;&lt;/div&gt;&lt;div&gt;&lt;table border=&quot;1&quot; cellpadding=&quot;0&quot; cellspacing=&quot;0&quot; class=&quot;MsoTableGrid&quot; style=&quot;border-collapse: collapse; border: none; mso-border-alt: solid windowtext .5pt; mso-padding-alt: 0in 5.4pt 0in 5.4pt; mso-yfti-tbllook: 1184;&quot;&gt;
  394. &lt;tbody&gt;&lt;tr style=&quot;height: 7.05pt; mso-yfti-firstrow: yes; mso-yfti-irow: 0;&quot;&gt;
  395.  &lt;td style=&quot;background: rgb(0, 120, 138); border-bottom: 1pt solid rgb(0, 120, 138); border-left: 1pt solid rgb(1, 112, 135); border-right: 1pt solid rgb(64, 147, 165); border-style: solid; border-top: 1pt rgb(64, 147, 165); height: 7.05pt; mso-border-bottom-alt: #00788A .5pt; mso-border-left-alt: #017087 1.0pt; mso-border-right-alt: #4093A5 1.0pt; mso-border-style-alt: solid; mso-border-top-alt: #4093A5 1.0pt; padding: 0in 5.4pt; width: 433.7pt;&quot; valign=&quot;top&quot; width=&quot;578&quot;&gt;
  396.  &lt;div style=&quot;line-height: normal; margin-bottom: 0in; mso-layout-grid-align: none; text-autospace: none;&quot;&gt;&lt;b&gt;&lt;span face=&quot;ScalaSansLF-Bold&quot; style=&quot;color: white;&quot;&gt;Table 16–1 • CONCERNING FEATURES THAT MAY BE
  397.  ASSOCIATED WITH&amp;nbsp;&lt;/span&gt;&lt;/b&gt;&lt;b&gt;&lt;span face=&quot;ScalaSansLF-Bold&quot; style=&quot;color: white;&quot;&gt;CHILD ABUSE&lt;/span&gt;&lt;/b&gt;&lt;/div&gt;
  398.  &lt;/td&gt;
  399. &lt;/tr&gt;
  400. &lt;tr style=&quot;height: 11.95pt; mso-yfti-irow: 1; mso-yfti-lastrow: yes;&quot;&gt;
  401.  &lt;td style=&quot;border-bottom: 1pt solid rgb(64, 147, 165); border-left: 1pt solid rgb(1, 112, 135); border-right: 1pt solid rgb(64, 147, 165); border-top: none; height: 11.95pt; mso-border-top-alt: solid #00788A .5pt; padding: 0in 5.4pt; width: 433.7pt;&quot; valign=&quot;top&quot; width=&quot;578&quot;&gt;
  402.  &lt;div style=&quot;line-height: normal; margin-bottom: 0in;&quot;&gt;&lt;div style=&quot;line-height: normal; margin-bottom: 0in;&quot;&gt;&lt;br /&gt;&lt;/div&gt;&lt;div style=&quot;line-height: normal; margin-bottom: 0in;&quot;&gt;Evolving or absent history about injury&lt;/div&gt;&lt;div style=&quot;line-height: normal; margin-bottom: 0in;&quot;&gt;Delay in seeking care for concerning condition&lt;/div&gt;&lt;div style=&quot;line-height: normal; margin-bottom: 0in;&quot;&gt;Unusual interactions between child and parent&lt;/div&gt;&lt;div style=&quot;line-height: normal; margin-bottom: 0in;&quot;&gt;Overly compliant child with painful medical procedures&lt;/div&gt;&lt;div style=&quot;line-height: normal; margin-bottom: 0in;&quot;&gt;Overly affectionate behavior from child to medical staff&lt;/div&gt;&lt;div style=&quot;line-height: normal; margin-bottom: 0in;&quot;&gt;Protective of abusing parent&lt;/div&gt;&lt;div style=&quot;line-height: normal; margin-bottom: 0in;&quot;&gt;Parental substance abuse or intoxication&lt;/div&gt;&lt;div style=&quot;line-height: normal; margin-bottom: 0in;&quot;&gt;Poor self-esteem in parent&lt;/div&gt;&lt;div style=&quot;line-height: normal; margin-bottom: 0in;&quot;&gt;History of abuse in parent’s childhood&lt;/div&gt;&lt;div style=&quot;line-height: normal; margin-bottom: 0in;&quot;&gt;History of domestic violence&lt;/div&gt;&lt;div style=&quot;line-height: normal; margin-bottom: 0in;&quot;&gt;Loss of control of parent triggered by child’s behavior&lt;/div&gt;&lt;div style=&quot;line-height: normal; margin-bottom: 0in;&quot;&gt;&lt;br /&gt;&lt;/div&gt;&lt;/div&gt;
  403.  &lt;/td&gt;
  404. &lt;/tr&gt;
  405. &lt;/tbody&gt;&lt;/table&gt;&lt;/div&gt;&lt;div&gt;&lt;br /&gt;&lt;/div&gt;&lt;div class=&quot;separator&quot; style=&quot;clear: both; text-align: center;&quot;&gt;&lt;a href=&quot;https://blogger.googleusercontent.com/img/b/R29vZ2xl/AVvXsEiLSGDdgyNi79c28SqkPnJXZgHBmsKbkajoKH9ZXy1NyxdAXtuCIToxMbvYCzzxcFmKgoh6P5CNtKmWGTKruARv0hX5fMQENcloio97puUq0855ADIu5KDpH2LHIAYo7io1OZrqTUmWBmhYJke6TTxY-_kQpseETmeQlpZnzDiL7HPT0w2d53B8VsbNmw/s503/Classic-metaphyseal-lesions.jpg&quot; style=&quot;margin-left: 1em; margin-right: 1em;&quot;&gt;&lt;img alt=&quot;Classic metaphyseal lesions&quot; border=&quot;0&quot; data-original-height=&quot;503&quot; data-original-width=&quot;500&quot; height=&quot;640&quot; src=&quot;https://blogger.googleusercontent.com/img/b/R29vZ2xl/AVvXsEiLSGDdgyNi79c28SqkPnJXZgHBmsKbkajoKH9ZXy1NyxdAXtuCIToxMbvYCzzxcFmKgoh6P5CNtKmWGTKruARv0hX5fMQENcloio97puUq0855ADIu5KDpH2LHIAYo7io1OZrqTUmWBmhYJke6TTxY-_kQpseETmeQlpZnzDiL7HPT0w2d53B8VsbNmw/w636-h640/Classic-metaphyseal-lesions.jpg&quot; title=&quot;Classic metaphyseal lesions&quot; width=&quot;636&quot; /&gt;&lt;/a&gt;&lt;/div&gt;&lt;div&gt;&lt;div&gt;&lt;b&gt;&lt;span style=&quot;color: #017087;&quot;&gt;Figure 16–2.&lt;/span&gt;&lt;/b&gt; Classic metaphyseal lesions of the distal left femur and proximal left tibia in a 27-monthold child. (Reproduced, with permission, from Tintinalli J, et al. &lt;i&gt;Tintinalli’s Emergency Medicine: A Comprehensive Study Guide&lt;/i&gt;. 7th ed. New York, NY: McGraw-Hill; 2010:Fig. 290-4.)&lt;/div&gt;&lt;/div&gt;&lt;div&gt;&lt;br /&gt;&lt;/div&gt;&lt;div&gt;&lt;br /&gt;&lt;/div&gt;&lt;div&gt;&lt;div&gt;&lt;i&gt;&lt;span style=&quot;font-size: medium;&quot;&gt;Musculoskeletal Features&lt;/span&gt;&lt;/i&gt;&lt;/div&gt;&lt;div&gt;&lt;br /&gt;&lt;/div&gt;&lt;div&gt;Approximately one third of children who are victims of nonaccidental trauma require orthopaedic care, with fractures most common in younger children due to the vulnerability of their developing skeletons and defenselessness. Although multiple fractures in various stages of healing are very common in abused children, many will present with only 1 fracture.&lt;/div&gt;&lt;div&gt;&lt;br /&gt;&lt;/div&gt;&lt;div&gt;&lt;span&gt;&amp;nbsp;&amp;nbsp; &amp;nbsp;&lt;/span&gt;Long bone (femur or humerus) fractures are the most commonly fractured bones in child abuse. Certain types, such as the midshaft spiral femur fracture, were once thought of as indicative of nonaccidental trauma. &lt;b&gt;However, studies have found&amp;nbsp;&lt;/b&gt;&lt;b&gt;that no specific fracture pattern in the femur is pathognomonic of abuse and that&amp;nbsp;&lt;/b&gt;&lt;b&gt;all patterns can be observed.&lt;/b&gt; Fractures of the hands, feet, and clavicle are uncommon in abuse, as are physeal fractures. The lone exception is transphyseal fractures of the distal humerus in children less than 1 year old. Metaphyseal fractures are less common than diaphyseal injuries, but are very concerning for physical abuse ( Figure 16–2 ).&lt;/div&gt;&lt;/div&gt;&lt;div&gt;&lt;br /&gt;&lt;/div&gt;&lt;div&gt;&lt;br /&gt;&lt;/div&gt;&lt;div&gt;&lt;div&gt;&lt;span style=&quot;color: #017087; font-size: medium;&quot;&gt;&lt;b&gt;DIFFERENTIAL DIAGNOSIS AND WORKUP OF&amp;nbsp;&lt;/b&gt;&lt;/span&gt;&lt;b style=&quot;color: #017087; font-size: large;&quot;&gt;SUSPECTED ABUSE&lt;/b&gt;&lt;/div&gt;&lt;/div&gt;&lt;div&gt;&lt;br /&gt;&lt;/div&gt;&lt;div&gt;&lt;div&gt;Any condition that can lead to bruising, fracture, or periosteal changes must be considered in the differential diagnosis of child abuse. Biomechanical testing for metabolic markers for bone disease (ie, calcium, phosphate, alkaline phosphate, copper, parathyroid hormone, and 25-hydroxyvitamin D) should be performed.&amp;nbsp;&lt;b&gt;Osteogenesis imperfecta&lt;/b&gt; may present like nonaccidental trauma and should also be investigated with genetic testing and biochemical analysis. When bruising is present, a complete blood count, differential blood count, and coagulation studies must be performed. Occasionally, children with &lt;b&gt;leukemia, hemophilia,&amp;nbsp;&lt;/b&gt;or other hematologic disorders are brought in for suspected abuse due to multiple bruises. Other rare pediatric conditions that can present like abuse include&amp;nbsp;&lt;b&gt;Caffey disease, rickets, &lt;/b&gt;and&lt;b&gt; congenital syphilis.&lt;/b&gt; It is important to note that the practitioner must also always consider accidental trauma and normal radiographic variants as potential causes of the clinical presentation of a patient for whom abuse is suspected.&lt;/div&gt;&lt;/div&gt;&lt;div&gt;&lt;br /&gt;&lt;/div&gt;&lt;div&gt;&lt;div&gt;&lt;span&gt;&amp;nbsp;&amp;nbsp; &amp;nbsp;&lt;/span&gt;A skeletal survey should be used in addition to imaging of obvious deformities to detect other acute or healed fractures in potentially abused children. &lt;b&gt;The American&amp;nbsp;&lt;/b&gt;&lt;b&gt;Academy of Pediatrics Section on Radiology recommends a mandatory survey in&amp;nbsp;&lt;/b&gt;&lt;b&gt;all cases of suspected abuse in children less than 2 years of age. In those between&amp;nbsp;&lt;/b&gt;&lt;b&gt;2 and 5 years of age, a skeletal survey should be done based on clinical indications.&amp;nbsp;&lt;/b&gt;&lt;b&gt;In children older than 5 years, the skeletal survey has minimal value.&amp;nbsp;&lt;/b&gt;Radionuclide bone scanning for detection of physical abuse is controversial and should be reserved for use when skeletal surveys are negative despite high suspicion for abuse. Additionally, a repeat skeletal survey 2 weeks after initial presentation may reveal injuries not evident in the initial survey in cases with high suspicion for abuse.&lt;/div&gt;&lt;/div&gt;&lt;div&gt;&lt;br /&gt;&lt;/div&gt;&lt;div&gt;&lt;br /&gt;&lt;/div&gt;&lt;div&gt;&lt;b style=&quot;color: #017087;&quot;&gt;&lt;span style=&quot;font-size: medium;&quot;&gt;MANAGEMENT&lt;/span&gt;&lt;/b&gt;&lt;/div&gt;&lt;div&gt;&lt;br /&gt;&lt;/div&gt;&lt;div&gt;&lt;div&gt;In most states, physicians are required to report suspected cases of child abuse based on reasonable suspicion of nonaccidental trauma or maltreatment. &lt;b&gt;Reporters are&amp;nbsp;&lt;/b&gt;&lt;b&gt;immune from civil and criminal liability, even if it is ultimately determined that&amp;nbsp;&lt;/b&gt;&lt;b&gt;no abuse has occurred.&lt;/b&gt;&lt;/div&gt;&lt;div&gt;&lt;br /&gt;&lt;/div&gt;&lt;div&gt;&lt;span&gt;&amp;nbsp;&amp;nbsp; &amp;nbsp;&lt;/span&gt;The treatment of abuse fractures is identical to those incurred accidentally and are typically amenable to closed, nonoperative treatment. However, the orthopaedic management of the abused child is only one component of treatment, and a teamoriented approach is essential. This includes pediatricians, other subspecialists, and social workers.&lt;/div&gt;&lt;div&gt;&lt;br /&gt;&lt;/div&gt;&lt;div&gt;&lt;span&gt;&amp;nbsp;&amp;nbsp; &amp;nbsp;&lt;/span&gt;When the orthopaedist is the first physician to see a potentially abused child, communication with the nearby emergency department or local child abuse agency must be swiftly initiated. Hospital admission is often required to care for the acute injuries and to provide a safe environment where a full workup can be conducted.&lt;/div&gt;&lt;/div&gt;&lt;div&gt;&lt;br /&gt;&lt;/div&gt;&lt;div&gt;&lt;br /&gt;&lt;/div&gt;&lt;div&gt;&lt;b&gt;&lt;span style=&quot;color: #017087; font-size: medium;&quot;&gt;&lt;u&gt;COMPREHENSION QUESTIONS&lt;/u&gt;&lt;/span&gt;&lt;/b&gt;&lt;/div&gt;&lt;div&gt;&lt;br /&gt;&lt;/div&gt;&lt;div&gt;&lt;div&gt;16.1 A 3-year-old girl presents shortly after a right arm injury. The patient’s father reports that he was playing with his daughter by swinging the girl in circles while holding on to her arms when she suddenly complained of pain in the left arm. The patient immediately held the injured arm close to her body with the help of her noninjured arm. Which of the following is most accurate regarding this injury?&lt;/div&gt;&lt;/div&gt;&lt;blockquote style=&quot;border: none; margin: 0px 0px 0px 40px; padding: 0px; text-align: left;&quot;&gt;&lt;div&gt;&lt;div&gt;A. Represents child abuse&lt;/div&gt;&lt;/div&gt;&lt;div&gt;&lt;div&gt;B. Requires a skeletal survey&lt;/div&gt;&lt;/div&gt;&lt;div&gt;&lt;div&gt;C. Is a common accidental injury&lt;/div&gt;&lt;/div&gt;&lt;div&gt;&lt;div&gt;D. Is inconsistent with the described mechanism of injury&lt;/div&gt;&lt;/div&gt;&lt;/blockquote&gt;&lt;div&gt;&lt;div&gt;&lt;br /&gt;&lt;/div&gt;&lt;div&gt;16.2 Osteogenesis imperfecta (OI) may present similarly to child maltreatment. However, there are features of OI that may help differentiate the 2, including blue sclera and osteopenia. Which of the following is another distinguishing feature?&lt;/div&gt;&lt;/div&gt;&lt;blockquote style=&quot;border: none; margin: 0px 0px 0px 40px; padding: 0px; text-align: left;&quot;&gt;&lt;div&gt;&lt;div&gt;A. Polydactyly&lt;/div&gt;&lt;/div&gt;&lt;div&gt;&lt;div&gt;B. Patent foramen ovale&lt;/div&gt;&lt;/div&gt;&lt;div&gt;&lt;div&gt;C. Multiple fractures in the setting of minimal trauma&lt;/div&gt;&lt;/div&gt;&lt;div&gt;&lt;div&gt;D. Dental involvement&lt;/div&gt;&lt;/div&gt;&lt;/blockquote&gt;&lt;div&gt;&lt;div&gt;&lt;br /&gt;&lt;/div&gt;&lt;div&gt;16.3 The pediatrician of a 2-year-old girl orders a skeletal survey because of suspected abuse. The imaging identifies multiple fractures, including the right humerus, left tibia, and right femur. Because it is important to distinguish between those that are acute versus those that are older and already healing, the pediatrician calls her colleagues in the Orthopaedics Department, who reviews with her the age-based features of fracture healing. At what point does new periosteal bone formation become apparent on plain radiographs?&lt;/div&gt;&lt;/div&gt;&lt;blockquote style=&quot;border: none; margin: 0px 0px 0px 40px; padding: 0px; text-align: left;&quot;&gt;&lt;div&gt;&lt;div&gt;A. 4 to 14 days&lt;/div&gt;&lt;/div&gt;&lt;div&gt;&lt;div&gt;B. 1 to 2 months&lt;/div&gt;&lt;/div&gt;&lt;div&gt;&lt;div&gt;C. 1 year&lt;/div&gt;&lt;/div&gt;&lt;div&gt;&lt;div&gt;D. Within hours&lt;/div&gt;&lt;/div&gt;&lt;/blockquote&gt;&lt;div&gt;&lt;br /&gt;&lt;/div&gt;&lt;div&gt;&lt;br /&gt;&lt;/div&gt;&lt;div&gt;&lt;b&gt;&lt;span style=&quot;color: #017087; font-size: medium;&quot;&gt;&lt;u&gt;ANSWERS&lt;/u&gt;&lt;/span&gt;&lt;/b&gt;&lt;/div&gt;&lt;div&gt;&lt;br /&gt;&lt;/div&gt;&lt;div&gt;&lt;div&gt;16.1 C. This case represents a nursemaid’s elbow, which is caused by a traction force on an outstretched arm. This is a common accidental injury in which the patient presents with a slightly flexed and pronated arm. The injury is reduced by flexion and supination of the patient’s forearm with concurrent pressure over the radial head. A successful reduction should return the patient to full use of the arm immediately.&lt;/div&gt;&lt;/div&gt;&lt;div&gt;&lt;br /&gt;&lt;/div&gt;&lt;div&gt;&lt;div&gt;16.2 D. Multiple medical conditions can present similarly to nonaccidental trauma, including OI. This autosomal dominant genetic disease often presents with multiple fractures in the setting of minimal trauma. However, distinguishing features include osteopenia, blue sclera, family history, and dental involvement. Polydactyly and a patent foramen ovale are not typical features of OI.&lt;/div&gt;&lt;div&gt;&lt;br /&gt;&lt;/div&gt;&lt;div&gt;16.3 A. To estimate the age of a fracture, it is important to know the age-dependent radiographic features of healing fractures. Listed below is a timeline for the appearance of various radiographic features:&lt;/div&gt;&lt;div&gt;Days 2 to 10: Soft tissue swelling subsides&lt;/div&gt;&lt;div&gt;Days 4 to 14: New periosteal bone formation becomes apparent&lt;/div&gt;&lt;div&gt;Days 10 to 21: Loss of definition of fracture line, presence of soft callus&lt;/div&gt;&lt;div&gt;Days 14 to 42: Presence of hard callus&lt;/div&gt;&lt;div&gt;Months 3 to 12: Fracture remodeling&lt;/div&gt;&lt;/div&gt;&lt;div&gt;&lt;br /&gt;&lt;/div&gt;&lt;div&gt;&lt;b style=&quot;background-color: #4093a5;&quot;&gt;&amp;nbsp; &amp;nbsp;&lt;span style=&quot;font-size: medium;&quot;&gt;&amp;nbsp;&lt;/span&gt;&lt;span style=&quot;color: white;&quot;&gt;&lt;span style=&quot;font-size: medium;&quot;&gt;CLINICAL PEARLS&lt;/span&gt;&lt;span&gt;&amp;nbsp; &amp;nbsp;&amp;nbsp;&lt;/span&gt;&lt;/span&gt;&lt;/b&gt;&lt;/div&gt;&lt;div&gt;&lt;table border=&quot;1&quot; cellpadding=&quot;0&quot; cellspacing=&quot;0&quot; class=&quot;MsoTableGrid&quot; style=&quot;border-collapse: collapse; border: none; mso-border-alt: solid windowtext .5pt; mso-padding-alt: 0in 5.4pt 0in 5.4pt; mso-yfti-tbllook: 1184;&quot;&gt;
  406. &lt;tbody&gt;&lt;tr style=&quot;height: 19.15pt; mso-yfti-firstrow: yes; mso-yfti-irow: 0; mso-yfti-lastrow: yes;&quot;&gt;
  407.  &lt;td style=&quot;background: rgb(221, 237, 239); border-left: solid #017087 1.0pt; border: 1pt solid rgb(64, 147, 165); height: 19.15pt; padding: 0in 5.4pt; width: 433.7pt;&quot; valign=&quot;top&quot; width=&quot;578&quot;&gt;
  408.  &lt;div style=&quot;line-height: normal; margin-bottom: 0in;&quot;&gt;&lt;div style=&quot;line-height: normal; margin-bottom: 0in;&quot;&gt;&lt;span style=&quot;color: #017087;&quot;&gt;&lt;br /&gt;&lt;/span&gt;&lt;/div&gt;&lt;div style=&quot;line-height: normal; margin-bottom: 0in;&quot;&gt;&lt;span style=&quot;color: #017087;&quot;&gt;►&lt;/span&gt;&amp;nbsp;Although multiple fractures in various stages of healing are very common in abused children, many will present with only 1 fracture.&lt;/div&gt;&lt;div style=&quot;line-height: normal; margin-bottom: 0in;&quot;&gt;&lt;span style=&quot;color: #017087;&quot;&gt;&lt;br /&gt;&lt;/span&gt;&lt;/div&gt;&lt;div style=&quot;line-height: normal; margin-bottom: 0in;&quot;&gt;&lt;span style=&quot;color: #017087;&quot;&gt;►&lt;/span&gt;&amp;nbsp;Any condition that can lead to bruising, fracture, or periosteal changes must be considered in the differential diagnosis of child abuse.&lt;/div&gt;&lt;div style=&quot;line-height: normal; margin-bottom: 0in;&quot;&gt;&lt;span style=&quot;color: #017087;&quot;&gt;&lt;br /&gt;&lt;/span&gt;&lt;/div&gt;&lt;div style=&quot;line-height: normal; margin-bottom: 0in;&quot;&gt;&lt;span style=&quot;color: #017087;&quot;&gt;►&lt;/span&gt;&amp;nbsp;The American Academy of Pediatrics Section on Radiology recommends a mandatory survey in all cases of suspected abuse in children less than 2 years of age.&lt;/div&gt;&lt;div style=&quot;line-height: normal; margin-bottom: 0in;&quot;&gt;&lt;span style=&quot;color: #017087;&quot;&gt;&lt;br /&gt;&lt;/span&gt;&lt;/div&gt;&lt;div style=&quot;line-height: normal; margin-bottom: 0in;&quot;&gt;&lt;span style=&quot;color: #017087;&quot;&gt;►&lt;/span&gt;&amp;nbsp;A team-oriented approach to the abused child is crucial and involves pediatricians, subspecialists, and social workers.&lt;/div&gt;&lt;div style=&quot;line-height: normal; margin-bottom: 0in;&quot;&gt;&lt;br /&gt;&lt;/div&gt;&lt;/div&gt;
  409.  &lt;/td&gt;
  410. &lt;/tr&gt;
  411. &lt;/tbody&gt;&lt;/table&gt;&lt;br /&gt;&lt;/div&gt;
  412.  
  413. &lt;details close=&quot;&quot;&gt;
  414.  &lt;summary&gt;&lt;b&gt;&lt;span style=&quot;color: #00788b;&quot;&gt;REFERENCES&lt;/span&gt;&lt;/b&gt;&lt;/summary&gt;
  415.  &lt;p&gt;Jayakumar P, Ramachandran M. Orthopaedic aspects of paediatric non-accidental injury. J Bone Joint Surg
  416. Br. 2010;92:189-195.&amp;nbsp;&lt;/p&gt;&lt;p&gt;Kocher MS, Kasser JA. Orthopaedic aspects of child abuse. J Am Acad Orthop Surg . 2000;8:10-20.&lt;/p&gt;
  417. &lt;/details&gt;&lt;/div&gt;  </content><link rel='replies' type='application/atom+xml' href='https://medical-phd.blogspot.com/feeds/5088681108440372635/comments/default' title='Post Comments'/><link rel='replies' type='text/html' href='https://medical-phd.blogspot.com/2022/03/child-abuse-case-file.html#comment-form' title='0 Comments'/><link rel='edit' type='application/atom+xml' href='https://www.blogger.com/feeds/3302710210518459416/posts/default/5088681108440372635'/><link rel='self' type='application/atom+xml' href='https://www.blogger.com/feeds/3302710210518459416/posts/default/5088681108440372635'/><link rel='alternate' type='text/html' href='https://medical-phd.blogspot.com/2022/03/child-abuse-case-file.html' title='Child Abuse Case File'/><author><name>Medical Group</name><uri>http://www.blogger.com/profile/01780656800578397393</uri><email>noreply@blogger.com</email><gd:image rel='http://schemas.google.com/g/2005#thumbnail' width='32' height='32' src='//blogger.googleusercontent.com/img/b/R29vZ2xl/AVvXsEiMuROkBDv6yw4SmxGqRA5BPQhOXSwFld-Gufmy4kUqfpoAGQRl7cv-3naew6wjAnCjRFSe_FKX8z7CdE6cFKxxBpnqBuwUTSr-15xd3mFGcr_wJOX2y73G_zJCGND98Q/s220/Final-logo.jpg'/></author><media:thumbnail xmlns:media="http://search.yahoo.com/mrss/" url="https://blogger.googleusercontent.com/img/b/R29vZ2xl/AVvXsEhHmr1g96XxdHyl6TAw2L_yc6w_ArnQnRqII8K99115SFFDE8epCwTjYhfps5xpYImGzTsQgQ0fSgJlK8hWUU3zx9dkC8epjDz2E4wL76flto46hd3Vqrn2bPw89nXo9fDDs7JrZO3s7Ibq0m_KshoYFF3IRoY3xo-Lg8fi6j1qD6DxbpCE5SmUMBmyCg/s72-w380-h640-c/proximal-tibia-fracture.jpg" height="72" width="72"/><thr:total>0</thr:total></entry></feed>
Copyright © 2002-9 Sam Ruby, Mark Pilgrim, Joseph Walton, and Phil Ringnalda